Printable Version of Topic
Click here to view this topic in its original format
თბილისის ფორუმი > ჯანმრთელობა და მედიცინა > შეკითხვები ინტერნისტებს(თერაპევტებს) Vol. II


Posted by: blblbl 28 Dec 2010, 23:19
ფილტვების ძლიერი ანთების დროს,წყალიც იდგა ფილტვებში,სისხლი შეიძლება ამოყვეს ნახველს?

Posted by: gramatikikina 31 Dec 2010, 12:05
აუჰ, რამდენი კითხვებია ბოლო გვერდზე და არც ერთი პასუხი, მარა მაინც დავწერ




მეზიმფორტე რომ ვსვა(სანამ ექიმთან მივალ), რამე ხომ არ დაშავდება? სავარაუდოდ კუჭი მაწუხებს. ღვიძლი და ნაღვლის ბუშტი გამოვიკვლიე, ანუ არაა ეგენი. კუჭი ვერ მოვასწარი fig.gif

Posted by: WOO 1 Jan 2011, 16:01
ამ თემაში არავინ პასუხოს კითხვებს ტყუილად ვწერთ მგონი mo.gif

blblbl
QUOTE
ფილტვების ძლიერი ანთების დროს,წყალიც იდგა ფილტვებში,სისხლი შეიძლება ამოყვეს ნახველს?

კი შეიძლება.



მეც ფილტვების ანთება მაქ მგონი.
ორშაბათს მივდივარ ექიმთან იმედია მანამდე ვიცოცხლებ givi.gif

აქამდე ტელეფონით დავურეკე და ანტიბიოტიკი დამინიშნა 3 დღიანი ხოდა მოვრჩი უკვე მაგის სმას მარა დიდ გაუმჯობესებას ვერაფერს ვამჩნევ ჯერ mo.gif

Posted by: buub 2 Jan 2011, 23:40
ორი კვირის წინ თავის ტკივილი დამეწყო ტკივილი აუტანელი იყო ვერაფრით ვერ ვიყუჩებდი ეხლა ოდნავ დაძაბულობაზე მტკივდება . თავის სახადასხვა მიდამოში ისე მწარედ გამკრავს ხოლმე, მერე თავის შუა ნაწილი ზოლად მტკივდება თავის უკან გადაწევაზეც კეფა მტკივდება.მოკლედ ტკივილის შეგრძნება სულ არის რისი ბრალი შეიძლება იყოს? რომელ ექიმს მივმართო


Posted by: Natuka NGN 3 Jan 2011, 01:01
blblbl
შეიძლება. შესაძლოა უბრალოდ კაპილარი კასკდეს დატვირთვისას და შესაძლოა ბრონქებიდან მოდიოდეს. ერთი სიტყვით, იგივე სითხის ციტოლოგიური კვლევა უნდა გაკეთდეს, სიტხეში სისხლი ხომ არ იყო.
gramatikikina
მეზიმ-ფორტე ფერმენტია და უფრი კუჭის მოქმედებას აწესრიგებს თუ ფერმენტული ნაკლებობაა. დაშავებიტ არაფერს დააშავებს, მაგრამ თუ არასწორად იღებ, ნებისმიერმა წამალმა შესაძლოა ზუსტი დიაგნოსტირებას შეუშალოს ხელი, ანამნეზი შეცვალოს.
WOO
სადმღიანი ანტიბიოტიკოთერაპია პნევმონიის დროს არ შეიძლება. მეორეც, გააჩნია რა ტიპის პნევმონიაა, თუ მიკოპლაზმისგან, მაგას სხვაგვარი მკურნალობა უნდა საერთოდ, სპეციფიკური ანტიბიოტიკებით.
buub
უპირველესად ნევროლოგთან. მაგრამ მანამდე გაისინჯეთ არტერიული წნევა როგორია. მსგავსი ტკივილები არა მხოლდო მაღალმა, დაბალმა წნევამაც კი შეიძლება გმოიწვიოს.

Posted by: buub 3 Jan 2011, 01:13
Natuka NGN
სად მივიდე ვისთან თუ შეგიძლიათ მირჩიეთ კარგი ნევროლოგი თორე აღარ შემიძლია. მადლობააა

Posted by: Natuka NGN 3 Jan 2011, 01:22
buub
ოი, მაგას ხვალ დაგიზუსტებთ, ორივე რესპუბლიკურში ვიცი. უბრალოდ უნდა დავეკითხო ჩემს მეგობარ ექიმს.

Posted by: WOO 3 Jan 2011, 01:23
Natuka NGN
QUOTE
სადმღიანი ანტიბიოტიკოთერაპია პნევმონიის დროს არ შეიძლება. მეორეც, გააჩნია რა ტიპის პნევმონიაა, თუ მიკოპლაზმისგან, მაგას სხვაგვარი მკურნალობა უნდა საერთოდ, სპეციფიკური ანტიბიოტიკებით.

mo.gif

რავიცი ხვალ მივდივარ ექიმთან და ვნახავ რას მეტყვის. ზუსტად არც ვიცი პნევმონია მაქვს თუ არა. მგონი Streptococcal pharyngitis ეს მაქვს. ინგლისურად იგივე strep throat

ისე მახველებდა და ეგ ანტიბიოტიკები რაც მოვრჩი ის 3 დღიანი იმის მერე თითქმის აღარ მახველებს.
მარა ყელი მაქ წითელი და რაღაცეები და ენაზე წერტილები ცოტათი.

Posted by: Natuka NGN 3 Jan 2011, 01:36
WOO
ააა, დიაგნოზი თავად დაისვი smile.gif შესაძლოა მართლაც ყელი გტკიოდა, ან ბრონქიტია. ერთი სიტყვით ხველებისას ყელში აგხველებდა და ბრონქებში, ეს თავად უნდა განსაზრვრო. ენაზე წერტილები ვერ მივხვდი... თეთრი ნადებია? ყელში არის ჩIრქის საცობები? ერთი სიტყვით, მიიდ ექიმთან და რას გეტყვის გვითხარი, ისე, არც სტრეპროკოკული ანგინისთვისაა საკმარისი 3 დღე ანტიბიოტიკი. სამწუხაროდ, ძლიან გაძლიერდნენ შტამები და აქეთ გვიტევენ.

Posted by: WOO 3 Jan 2011, 01:45
Natuka NGN
ხო მე თვითონ დავისვი დიაგნოზი biggrin.gif მარა მერე ექიმს დავურეკე და იმან რა ანტიბიოტიკიც დამინიშნა ვიცოდი რო ეგ იმას შველოდა რასაც მე ვფიქრობდი რომ მქონდა და დავლიე
ხო ჩირქის საცობები არის მგონი. ენაზე ნადები მგონი არა

ენაზე ამაზე ვამბობდი. ოღონდ ამას ძაან აქვს, მე ცოტათი მაქვს
http://67pics.com/view.php?q=Pictures%20Bumps%20On%20Tongue&url=http://sitemaker.umich.edu/mc7/files/white_strawberry_tongue__deb.jpg


ჰო მივალ ხვალ და რას მეტყვის დავწერ

Posted by: Natuka NGN 3 Jan 2011, 02:11
WOO
ეს რძიანაა ჩემო კარგო, იგივე სოკო. ანტიბიოტიკების შედეგიც შესაძლოა იყოს. ამიტომ თუ კვლავ გაგიოგრძელა ანტიბიოტიკი, ფუნგიციდიც დაანიშნინე.

Posted by: Summer_Rain 3 Jan 2011, 12:34
სალამი!!

ცუდია, რომ არაა თემა "კითხვა გასტროლოგს"
მარტო ჩემს გარშემო იმდენი ვიცი, რომლებსაც აწუხებს კუჭი..


იქნებ აქ მაინც მიპასუხოთ ჩემს გასაჭირზე და პრობლემებზე.


დავიწყებ თავიდან.
ივლისის ბოლოს დამეწყო კუჭში სპაზმების მსგავსი, 2-3 დღე გამიგრძელდა, ნოშპამ და მცირე დიეტამ მიშველა.
აგვისტოს ბოლოს კი პამიდორმა მომწამლა,
მეორე დღეს ვიდიეტე
მესამე დღეს ხინკალი ვჭამე ისიც არ შემერგგო
მეოთხე დღეს ხაჭაპური, ისევ პირღებინება
შემდეგ 1 კვირიანი დიეტა..
შემდეგ აჯაფსანდალი მივირთვი, მაინც ვერ გადაამუშავა კუჭმა.

დავამატებ იმასაც, რომ როგორც კი გულისრევის შეგრძნებას ვიგრძნობდი ეგრევე ნერვებს ვერ ვთოკავდი
მთელი სხეული მეჭიმებოდა. და ასანამ არ მოვიცილებდი არასასურველ საკვებს კუჭიდან განუწყვეტლივ მაკანკალებდა..
შემდეგ რის ვაი ვაგლახით ვახერხებდი ჩაძინებას.. sad.gif

გადავწყვიტე გასტროლოგთან მისვლა, ზონდი არ გადამაყლაპა, მითხრა ვეგეტო-ნევროზი გაქვსო, რომელიც გულის რევითაა გამოხატულიო და გამიშვა ნევროპათოლოგთან.
ნევროპათოლოგმაც ნევროზის ნიშნებიაო და გამომიწერა წამლები, ერთმა მადა დამიკარგა, მეორემ გული ამიჩქარა. ისევ სარაჯიშვილის სიროფი ავიღე და ვსვამდი საღამოობით.

სექტემბრიდან ჩემით დიეტა დავიწყე სერიოზული, ოქტომბრის ბოლომდე ვიდიეტე, თითქოს უკეთ ვგრძნობდი თავს, მაგრამ კუჭში სიმძიმე ისევ მქონდა და საღამოობით გულისრევის შეგრძნება ისევ მაწუხებდა.

ამიტომ გადავწყვიტე ზონდი გადამეყლაპა, იქ სადაც ვარ დაზღვეული, აისი გრუპის სადაზღვევოში, გავიკეთე გასტროფიბროსკოპია.
ექიმმა კუჭის ანთებაო, ნაღვლსი მომატებული რაოდენობაო და მაღალი მჟავიანობაო დამისვა დიაგნოზად. ზედაპირული გასტროდუოდენტი (მგონი სწორად მახსოვს)

ნოემბრის დასაწყისიდან დავიწყე დიეტა, კვება გრაფიკით, დილით ულსეპანს ვღებულობდი, სიმალგელის სიროფს ჭამის წინ, 40 დღის მერე ლანსოლით ჩავანაცვლე ულსეპანი. ბევრად უკეთესად ვგრძნობდი თავს, თუ არ გავითვალისწინებთ იმას რო კვირაში ერთხელ მაინც მაწუხებდა საღამოობით გულისრევის შეგრძნება. და ამავდროულად ძალიან მოვიკელი.












მივედი კითხვამდე
მოვრჩი მკურნალობას.
გავიდა 2 თვე
და 30-ში ღამით მაინც ცუდად გავხდი
არადიეტური არაფერი მიჭამია არადა
ისევ კუჭში სიმძიმე მაქვს
წვა დამეწყო
თითქოს რაღაც მიჭერს კუჭზე და მექაჩება




ძალიან ვნერვიულობ
ნერვებსაც ვეღარ ვთოკავ
მგონია რაღაც საშინელება მჭირს, რატომ ვერ გამოვჯანმრთელდი sad.gif
ოდნავ ცუდად ვიგრძნო თავი და სულ ცუდე ვფიქრობ.





მაინტერესებს
ასეთი ჩივილებით რას მირჩევთ
მივიდე სხვა გასტროლოგთან
კიდევ ნევროპათოლოგთან?
თუ რავქნა?


ძალიან განვიცდი უკვე
დეპრესიის ნიშნებიც მაქვს
ეს 3-4 დღეა წესიერად არაფერი მიჭამია, იმდენად მეშინია რო შემაწუხებს
ისედაც მეწვის და მექაჩება კუჭი და რაღააც რო ვჭამო მგონია უარესად მაწყენს.

მოკლედ გელოდებით კომპეტენტურებო
მირჩიეთ რამე.

Natuka NGN

მგონი შენ უფრო შეგიძლია კითხვაზე მიპასუხო
იქნებ კარგი გასტროლოგის ან ნევროპათოლოგის კოორდინატები მომცე და
მირჩიო როგორ მოვიქცე.


მადლობა მას, ვინც ამდენი წაიკითხა smile.gif

Posted by: WOO 3 Jan 2011, 15:58
Natuka NGN
ვიყავი და მოვედი ხოდა რავი არაფერი აღარ დაუნიშნია ფილტვებიც კარგად გაქო bis.gif
mo.gif

Posted by: aranormaluri 3 Jan 2011, 18:54
Natuka NGN
გაიხარე დიდი მადლობა რჩევისთვის.
ამჯერად გადავრჩი და კარგად ვარ. მერე მივალ ექიმთან. wink.gif
ისე რა უცნაურია, რომ გგონია სიცხე გაქვს, არადა პირიქით 35 გაქვს.

Posted by: blblbl 3 Jan 2011, 18:58
Summer_Rain
მე მგონი ფსიქოლოგი გჭირდება,მე მგონი კი არა დარწმუნებული ვარ.

Posted by: aranormaluri 3 Jan 2011, 19:00
Summer_Rain
კუჭის წვა არ მქონდა მაგრამ ერთხელ რაღაცით მოვიწამლე და გული ამერია. იმის მერე წელიწადი გულისრევის შეგრძნება არ მასვენებდა, როგორც კი შევჭამდი მეჩვენებოდა რომ გული უნდა ამრეოდა. არც ნაღველზე და არც კუჭზე არაფერი არ მჭირდა. სულ ცელოფნით ხელში დავდიოდი ლექციებზე. მერე დიდი მარხვა რომ ჩამთავრდა გაზაფხულზე, მას მერე აღარ მქონია. (შეიძლება ხორცი, კვერცხი და რძის პროდუქტის გამორიცხვით შენც გეშველოს. სულ მცირე გულის რევის შეგრძნებას მაინც დაკარგავ. კუჭის წვასაც რამე ეშველება)

Posted by: Summer_Rain 3 Jan 2011, 20:35
blblbl

QUOTE
მე მგონი ფსიქოლოგი გჭირდება,მე მგონი კი არა დარწმუნებული ვარ.



დარწმუნებული ხარ რომ მხოლოდ ფსიქოლოგი მომიგვარებს ამ პრობბლემებს?

კუჭის წვას და გულისრევის შეგრძნებებს, ტკივილს?

შეიძლება ნერვული დაძაბულობა მომიხსნას მაგრამ ეს ზემოთ ჩამოთვლილი ჩივილები... ?!?!

იცი ვინმე კარგი ფსიქოლოგი?
* * *
aranormaluri

QUOTE
კუჭის წვა არ მქონდა მაგრამ ერთხელ რაღაცით მოვიწამლე და გული ამერია. იმის მერე წელიწადი გულისრევის შეგრძნება არ მასვენებდა, როგორც კი შევჭამდი მეჩვენებოდა რომ გული უნდა ამრეოდა. არც ნაღველზე და არც კუჭზე არაფერი არ მჭირდა. სულ ცელოფნით ხელში დავდიოდი ლექციებზე. მერე დიდი მარხვა რომ ჩამთავრდა გაზაფხულზე, მას მერე აღარ მქონია. (შეიძლება ხორცი, კვერცხი და რძის პროდუქტის გამორიცხვით შენც გეშველოს. სულ მცირე გულის რევის შეგრძნებას მაინც დაკარგავ. კუჭის წვასაც რამე ეშველება)


კიდევ ერთი წელი ასე რო გამიგრძელოს გავგიჟდები biggrin.gif:D:D


Posted by: aranormaluri 3 Jan 2011, 22:29
Summer_Rain
QUOTE
კიდევ ერთი წელი ასე რო გამიგრძელოს გავგიჟდები biggrin.gif:D


როგორც კი შეეჩვევი მაგ მდგომარეობას ეგრევე გაგივლის biggrin.gif ანუ დაიჯერე რომ ეს ასეა, შეგრძნება გაქვს, თან შეიძლება აღებინო კიდეც. მერე ისე გაქრება ვერც გაიგებ.

Posted by: Bo-BuBu 4 Jan 2011, 10:20
არ ვიცი მოკლედ მისამართი მეშლება თუ არა
თუ შემეშალა გხოვთ გადამამისამართეთ.

რამდენიმე დღეა თავი მტკივა კოშმარულად.
უფრო სწორად ტავის ცალი მხარე. თან თითქოს ყურიც მაგ მხარეს. დლიერად სუნთქვის დროსაც მტკივა თავი. ზოგჯერ ჭამის დროს კბილის ნერვი.

მოკლედ მგონი რარაც ვირუსი ან ინფექცია ან ყურის ანთება ხომ არაა?
ან იქნებ დაახლოებით მითხრათ რას გავს და რა მოვუხერხო?

Posted by: Masked 4 Jan 2011, 18:02
Summer_Rain

გაკეტე გამოკვლევა ჰელიკობაკტა პილორი ს!
გაკეტე გამოკვლევა ფარის. ჯირკვალიზე, დარღვეული არის დეიოდიიღუნგ.
გაკეტე გამოკვლევა ხოლეცისტიტის. ნაწლავის ფლოა გადატვირტული არის.
მიიღე ნოირო ინიეელ რაც რეგულირებს ვეგეტატ. ნერფიული სისტემას.

Posted by: Summer_Rain 4 Jan 2011, 18:47
Masked

QUOTE
გაკეტე გამოკვლევა ჰელიკობაკტა პილორი ს!


ჰელიკობაკტა პილორი რა არის?
რომელ ექიმს უნდა მივმართო ამაზე?

QUOTE
გამოკვლევა ფარის. ჯირკვალიზე


ფარისებური ოდნავ მაქვს შემცირებული

QUOTE
დარღვეული არის დეიოდიიღუნგ


დეიოდიიღუნგ? :|
ეს რაღაა?

QUOTE
გამოკვლევა ხოლეცისტიტის. ნაწლავის ფლოა გადატვირტული არის


ისევ გასტროენტეროლოგამდე მივდივართ?

QUOTE
მიიღე ნოირო ინიეელ რაც რეგულირებს ვეგეტატ. ნერფიული სისტემას


ამ წამლის მიღება რეცეპტის გარეშე შეიძლება?

Posted by: Masked 4 Jan 2011, 19:11
Summer_Rain

პილორის გამოკვლევას აკეტებს გასტროენტ. ოგე, სისხლის ანალიზიტ არის ანტისხეულები პოზიტიც, არის ჰელიკობაკტა ტესტი. არ ვიცი არის ტუ არა ეს გამოკვლება საქ. შI.

ფარის. ჯირკვალი ანალიზების პასუხი დაწერე, და ეხოსკ. პასუხი.
დეიოდიიღუნგ დაკავშირებული არის ფარი.ს ჯირკ. ჰორკონების დარღვევასტან, ესე იგი ეფტე4 და ეფტე3 დამოკიდებულება არის დარღვეული.
კონკ. მაღალი ეფტე4 იწვევს შინაგანი დაძაბულობას. ასევე "ნევფიული კუჭს".
ნოირო ინიეელ შეიძლება მიღება, მაგრამ ჯერ ფარის. ჯირკ. ანალიზები კონტროლლე გაკეტე.


Posted by: Kublicha 4 Jan 2011, 19:27
არ ვიცი, ამ თემაში უნდა ვიკითხო თU სად, მაგრამ აქ ვიკითხავ.

ბრმა ნაწლავის ოპერაციისწინა სიმპტომები რა არის? ანუ სანამ გასკდება თუ რაც ქვია.
მარჯვენა გვერდში არცთუ ძლიერი ტკივილი, სხვა რა შეიძლება იყოს?
შეიძლება რამდენიმე დღიან ქეიფს გამოეწვია და გაიაროს თავისით?
+ბოლო ხუთუ დღეა სურდო და გრიპიც მაქვს.

რა დიაგნოზს დამისვამთ?

Posted by: shunshura 7 Jan 2011, 21:00
ფეხში ჟანგიანი ლურსმანი შემერჭო. სპირტით და პერეკისით დავიმუშავე. აცრა ძალიან არ მინდა .როგორ მოვიქცე?

Posted by: buub 7 Jan 2011, 23:56
shunshura
მგონი არ ღირს ჯანმრთელობასთან და სიცოცხლესთან თამაში ან გაგიმართლებს ან არა ყოველთვის ჯობია თავი დაიზღვიო და რაც შეიძლება მალე აცრა გაიკეთო იმიტომ რო ვერავინ ვერ გეტყვის დაზუსტებით დაგემართება თუ არა.. http://forum.ge/?f=43&showtopic=33775031&st=0

Posted by: ane mekeshi 8 Jan 2011, 00:52
ვინმემ მიშვლეთ რამე, რა, სანამ ექიმამდე მივაღწევ sad.gif
რამდენიმე დღის წინ მარჯვენა ხელის მტევანზე საშინელი წვა ვიგრძენი (სავარაუდოდ, კაპილარი გამისკდა) და დღემდე მახსენებს თავს ეს ტკივილი... დღეს კიდევ რამდენიმე ადგილას ვიგრძენი ეს ოხერი წვა და ტკივილს აღარ ვჩივი უკვე, შეშინებული ვარ სერიოზულად... საერთოდ, არანორმალურად მეშინია მსგავსი რამეების...
და თუ ვინმე ერკვევა ამ ყველაფერში, ამიხსენით, რა, რა ხდება ან როგორ უნდა მოვიქცე, სანამ ეს დღესასწაულები ჩაივლის და ექიმის ნახვას ვეღირსები...

Posted by: krasusi_92 8 Jan 2011, 03:02
მაგრა მევასება სამედიცინო ფორუმი , ჩემი მეორე ჰობია , ნეტა აკ ჩამებარებინა, მაგრამ რას ვიზამთ ჩემი პროფესიაც მიყვარს, მედიცინა კი ჰობია თუ რაგაჩ ჯანდაბა :დ

Posted by: METEORA 8 Jan 2011, 15:07

მაინტერესებს თუ არსებობს ისეთი წამალი, რომლის დალევასაც ფატალური შედეგი შეიძლება მოჰყვეს, ან რომლის დიდი დოზით დალევასაც?

Posted by: ლუკასა და ნიკოს მამა 11 Jan 2011, 16:40
ჯაბა ექიმის რჩევები სად ვნახო? spy.gif spy.gif

Posted by: კახა251 11 Jan 2011, 19:54
METEORA
ყველა წამლის დიდი დოზით დალევას შეიძლება მოყვეს ფატ. შედეგი! ! ! ამას მედიცინა იძახის ჩემი სიტყვები არაა! პ.ს თავს არაფერი აუტეხო :X

Posted by: rock in rose 12 Jan 2011, 17:02
Lukas & Nikos Mama

QUOTE
ჯაბა ექიმის რჩევები სად ვნახო?


გაზეთ "ასავალ-დასავალში". ფორუმზე არაა



METEORA
QUOTE
მაინტერესებს თუ არსებობს ისეთი წამალი, რომლის დალევასაც ფატალური შედეგი შეიძლება მოჰყვეს, ან რომლის დიდი დოზით დალევასაც?


ძალიან ბევრია, თითქმის ყველა. ფრთხილად იყავი smile.gif


Posted by: drifters wife 15 Jan 2011, 00:25
ხალხო ეს ჯაბა ექიმი ვინაა საერთოდ?

Posted by: Ia-forumze 17 Jan 2011, 23:15
არ ვიცი ამ თემას ეხება თუ არა,
მშობიარობიდან 2-3 თვის მერე
ზურგზე, ბეჭების ძვლები რომ არის (არ ვიცი როგორ ავხსნა, აი რომ მოიხრები ზურგშI და კუზებივით ამოდის smile.gif))) )
მანდ შევამჩნიე ცხიმგროვის მსგავსი პატარა ბურთულა.
რომ ვიმართები მხრებში თითქოს უფრო შიგნით შედის, ნაკლებად იგრძნობა და პირიქით.
ტკივილით არ მტკივა, ზომაში მგონი არ იზრდება, ვიზუალურადაც არ ჩანს.

სანამ ექიმთან წავიდოდე იქნებ დამაკვალიანოთ, რა შეიძლება იყოს და ვის მივმართო?

Posted by: კახა251 18 Jan 2011, 00:21
Ia-forumze
მე მაქვს ცხიმგროვა უკვე 6-7 წელი იქნება მეტიც შეიძლება იღლიის დაბლა... არც ჩემსას ჭირს რამე ჯერჯერობით... მიაკითხე მიანც თერაპევტს და აჩვენე... იქნებ ცხიმგროვა არაა... : )

Posted by: φιλόσοφος 19 Jan 2011, 20:30
ვინმე დამეხმარება? .

გაციებული ვიყავი, ყელის ტკივილი, სურდო, ყველაფერი ერთად მჭირდა, საკმაოდ მწვავედ.
ცოტა გამოვკეთდი და ახლა სახის მარცხენა მხარეს ყვიმრალის ძვალი მტკივა ძალიან, თან პერიოდულად კბილებიც მტკივდება მარცხენა მხარეს.
შეიძლება რამე ნერვის ანთება ან მსგავსი იყოს? და რითი ვუშველო? ..

Posted by: baksona1980 21 Jan 2011, 08:24
ხველება ძნელად ამოსაღები,წებოვანი ნახველით...თეთრი ფერის,ტემპერატურა დილას ნორმალური,დღის გან-ში მატულობს,საღამოს 38.5.საერთო სისუსტე,ძვლებში ტეხვის შეგრძნება განს-ით საღამოს როცა ტემპ-ა იწევს...სითხეებით დატვირთვა,ც ვიტამინი 100 მგ,ვირატონი,კოლდრექსის ჩაიც,აზიმაკი 500 მგ..მაგრამ შედეგი ნოუუ! ისევ ცეფამედი და აუგმენტინი დამჭირდება ალბადsad.gif
* * *
აბა თქვენი მოსაზრებები ექიმებო!!! smile.gif

Posted by: baksona1980 22 Jan 2011, 18:49
φιλόσοφος
არასტეროიდი დასცხე....ექსისტენსი მაგალითად(მელოქსიკამი)

Posted by: ravi_shen 27 Jan 2011, 09:22
მიშველეტ რა რამე :

სამი დღე სიცხე მქონდა
ექიმთან ვიყავი და არაფერი სერიოზული ვირუსი გაქვს ო..
ფილტვებშიც მომისმინა და არაფერი იყო..
გავიდა სამი დღე და დამიტოვა ხველა..
თან ისეთი ხველა რომ ვიხრჩობი.. აშკარად ბრონხებიდან ვახველებ
და ორ ყლუპ წყალს დავლევ თუ არა ეგრევე გული მერევა
არა მხოლოდ წყალზე ნებისმიერ რამეზე..
ახლაცგულის რევის შეგრძნეაბ მაქვს რისი ბრალი შეიძLბეა იყოს..
ხოდა რაც მთავარია მთელი ღამე ტუალეტთან ჩახუტებულმა გავატარე და აღარ შემიძლია
რა ვუშველო :\

Posted by: SPC 28 Jan 2011, 01:48
როდესაც ღრმად ჩავისუნთქავ მარცხენა მხარეას მტკივა რაღაცა (დაახლოებით ფილტვი ნეკნების მიდამოებში) ანუ ეს ფილტვია? ან რა შეიძლება იყოს და რატომ მტკივა მხოლოდ ღრმა ჩასუნთქვისას? მადლობა

Posted by: harvi 28 Jan 2011, 20:07
გამარჯობათ
AსეთI შEმთხვევაა: 15 წლის ასაკშI პაციენტი მკურნალობდბდა რევმატიზმზე.ამ ხანგრძLIვ პროცესშI,დაუშვა შეცდომა პრეპარატის მიღEბის დროს კერზოდ,ბიცილინის გაკეთემამდე არ მიღOო ალერგიის საწინააღმდეგო წამალი.ამის შEმდეგ ბიცილინმა მისცა რეაქცია,დაეწყო ქავილი საზარდულზე აგრეთვე კანის აწითLება.მდგომარეობა ვერ შEფასდა სათანადოდ არაკომპეტენტური რჩEვით დაზიანებულ ადგილებზე წასივა ნიტროფუნგინის ხსნარი მიუხედავად საშIნელი ტკივილისა,მშOბლების დაძAლებით 10ი ოდე Dღის მანძIლზე გააგრძELა ასეთO ფომით კურნალობა.რამაც რათქმაუნდა არ უშVEლა თუმცა რამოდენიმე დროის შEMდეგ ქავილი შEჩერდა. ეს რაც შEEხება სავარაუდო მიზეზს ეხლა აქვს სისველის შEGრძNება საზარდულის კანზე და ისევ აწითლება კანის.მკვეთრი მგრძნობელობა სიცივეზე რამაც მიიყვანა იმ მგომარეობამდე რო დაეწყო პროსტატიტთან დაკავშXIრებული მპობლემები გაცივება და ასშ.. რა შEიძლება ეშველოს
გაზეთშI სტატიას კი გავს მარა...

Posted by: ghjm 29 Jan 2011, 22:37
მაინტერესებს ერთი რამ თუ შეგიძლიათ მიპასუხოთ ავიღე სისხლის საერთო ანალიზი ვემზადები ოპერაციისთვის და მაინტერესებს ლეიკოცოტები მაქვს 7.0 ხოლო ლიმფოციტები მაქვს 44 ანუ ნორმაზე მეტი და აქვე წერია რომ :აღინიშნება ლიმფოციტებში ფართოციტოპლაზმიანი ფორმები: თუ შეგიდძლიათ მითხრათ რას ნიშნავს და შეიძლება თუ არა ამ შემთხვევაში ოპერაციის გაკეთება მადლობა წინასწარ.

Posted by: utaridi 30 Jan 2011, 20:59
ადამიანის ფსიქიკურ მდგომარეობას დიდი მნიშვნელობა აქვს სცოვრებაში...არის ეგეთდ "დაავადება" რასაც ჰქვია უმართავი შიში, ნერვოზი...ასეთ მდგომარეობაში მყოფ პირებს ეცვენებათ, თითქოს ისინი დაავადებული არიან სხვადასხვა სნეულებებით, მათ აწუხებთ სხვადასხვა ორგანოების ტკივილები და ასე შემდეგ !!! ამ ყოველივეს ვწერ საკუთარი გამოცდილებიდან smile.gif ყველაზე დიდი ჯანმრთელობა და წამალი ყველაფრის არის სპორტი smile.gif რა ჯობია იმას როდესაც იხარჯები ბოლომდე და თავიდან აღგიდგება ენერგია ორმაგი მუხტით...სცადეთ შეძლებისდაგვარად ვარჯიში და აღმოაცენთ რომ სრულიად ჯანმრთელი ყოფილხართ !!! ცოტა ბუდდას მოძღვრებასავით გამომივიდა... smile.gif

Posted by: nikaa 30 Jan 2011, 22:26
harvi
აუცილებლად უნდა ინახოს ვიზულაურად რა არის, ესე პასუხის გაცემა ძალიან ძნელია, დერმატოლოგს აჩვენოთ ჯერ ჯობია. პროსტატიტთან დაკავშირებით რა პრობლემები დაეწყო უფრო კონკრეტულად
* * *
SPC
ფილვი გტკიოდეს ეს შეუძლებელია, ფილტვს ნერვები არ აქვს ამიტომ ფილტვის არ გტკივა, სავარაუდოდ ნერვოზს გავს.
ravi_shen
ხველებისთვის არაფერი დაგინიშნეს? მშრალი ხველა გაქვს ჯობია ისევ შეიარო თერაპევთან, თუ ნახველით მაშინ შეგიძლია აცც დალიო გაგითხიერებს ნახველს და მოგეშვება.გულის რევას რაც შეეხება დააკვირდი ხველებასთან ხომ არ არის დაკავშირებული

Posted by: ravi_shen 30 Jan 2011, 23:00
nikaa
ხვალ მიდივარ ..
ხველებასთან არის დაკავშირებული და ამდენმა გუილს რევამ კუჭი გამიღიზიანა და ახლა ჰერკულესის და სუხრის მეტს ვერაფერს ვჭამ.
შევჭამ თუ არა ვერ ვირგებ იმ წამსვე უკან ვიღებ..
ეგ აცც უნდა ვიყიდო ერთი ხვალ რა .. ექიმს არ გამოუწერია მარა ბევრმა მითხრა . ვკვდები ხველით ვიხრჩობი. . დღეს 5 დღიანი გარეთ გაუსვლელობის შემდეგ პირველად გავედი და ისე სუსტად ვიგრძენი თავი რომ გადავიფიქრე საბოლოოდ მოძლიერებამდე გარეთ გასვლა. : (

Posted by: Natuka NGN 30 Jan 2011, 23:06
nikaa
QUOTE
ფილვი გტკიოდეს ეს შეუძლებელია, ფილტვს ნერვები არ აქვს ამიტომ ფილტვის არ გტკივა, სავარაუდოდ ნერვოზს გავს.

ფილტვს არა, მაგრან აქვს პლევრას და შეიძლება რაAღც ხახუნი ხდება და შესაძლოა არც ეჩვენება.

Posted by: expert4ge 30 Jan 2011, 23:26
QUOTE (utaridi @ 30 Jan 2011, 20:59 )
ადამიანის ფსიქიკურ მდგომარეობას დიდი მნიშვნელობა აქვს სცოვრებაში...არის ეგეთდ "დაავადება" რასაც ჰქვია უმართავი შიში, ნერვოზი...ასეთ მდგომარეობაში მყოფ პირებს ეცვენებათ, თითქოს ისინი დაავადებული არიან სხვადასხვა სნეულებებით, მათ აწუხებთ სხვადასხვა ორგანოების ტკივილები და ასე შემდეგ !!! ამ ყოველივეს ვწერ საკუთარი გამოცდილებიდან smile.gif ყველაზე დიდი ჯანმრთელობა და წამალი ყველაფრის არის სპორტი smile.gif რა ჯობია იმას როდესაც იხარჯები ბოლომდე და თავიდან აღგიდგება ენერგია ორმაგი მუხტით...სცადეთ შეძლებისდაგვარად ვარჯიში და აღმოაცენთ რომ სრულიად ჯანმრთელი ყოფილხართ !!! ცოტა ბუდდას მოძღვრებასავით გამომივიდა... smile.gif

უტარიდი მარტალი ხარ შენ ბოქსი ყველაზე დიდი საშუალებაა რომ ენერგია გამოჰყო და საკუტარ თავს დაუმტკიცო რომ შენ შეგიძლია ეს ეგრე არ არის? smile.gif

Posted by: harvi 31 Jan 2011, 15:41
nikaa
ხშირი შარდვა,წვის შეგრძნება. უროლოგთან იყო და მკურნალობას შEდეგი არ აქვს მხოლოდ ცოტა შეუმსუბუქა მდომარეობა,მარა ამ სიცივეების პერიოდში ისევ დაუბრუნდა იგივე წინა მდგომარეობა.ps მე ვიფიქრე ალერგოლოგის საქმეათქო და კანის ექიმის არის? ამაზე მიტხარი მეც რო მქონდეს სათქმელად...
დიდი მადლობა ყურადღებისთვის

Posted by: nikaa 31 Jan 2011, 19:50
harvi
ასეთ ასაკში პროსტატიტი ცოტა საეჭვოა, უფრო აღმავალ ინფექციას გავს, შარდის ბიოქიმიური ანალიზი გაიკეთოს და ანტიბოტიკოგრამა, რომ ზუსტად ის დალიოს რაც სჭირდება და ზედმეტად არ იწვალოს. ჩემი აზრით კანის ექიმთან ჯობია და თუ საჭიროს ჩათვლის თვითონ გადაამისამართებს სხვა სპეციალისტთან. შემიძლია ძალიან კარგი დერმატოლოგი მიგასწავლოთ

Posted by: Natuka NGN 31 Jan 2011, 20:29
drifters wife
QUOTE
ხალხო ეს ჯაბა ექიმი ვინაა საერთოდ

ფორუმელი Roufus-სია smile.gif

Posted by: harvi 31 Jan 2011, 23:02
ეგ იმ ექიმსაც უთქვამს ანალიზიო თუმცა გადამდები ინFექციის ანალიზზე...ისე შარდის საერთO ანალიზი აქვს განაკეთები მარა არაფერი ამოუვიდა,კიდე ეხო გადაუღეს და 50 მლ გრ შარდი რჩEბაო ეგ რას ნიშნავს ამდენი აღარ ვიცი..არ აცდება ბოდიალი ექიმებთან მოკლედ?

Posted by: expert4ge 31 Jan 2011, 23:28
როდესაც შარდში ქალის შემთხვევაში ე.წ. ცისტიტი გაცივების შედეგად "საკვერცხე" ცივდება და შემდეგ შედეგი ტკივილი და ა.შ ეს არ ვგულისხმობ ქვასა თუ კენჭს მამაკაცების კიდევ განსხვავებულია და შარდის ანალიზი აუცილებელია რადგანაც ვაკონტროლოთ კრისტალური წარმონაქმნები რაც თირკმელის პრობლემს მიმანისნებელია

Posted by: harvi 1 Feb 2011, 18:53
nikaa
შEნი რჩევა რა იქნება?

Posted by: allie kraft 1 Feb 2011, 20:52
რა მაინტერესებს იცით–აი ჩემს გარშემო და მათ შორის მეც ზამთრის პერიოდში საკვერცხეები მქაჩავს,მერე გამივლის,მერე ისევ,ნუ ძალიან შემაწუხებლად არა.ჩემმა ძმამ თქვა იმ დღეს ის საზარდულის მიდამო თუ რაც ქვია მქაჩავსო.17 წლისაა.მოკლედ რაში მდგომარეობს ჩემი კითხვა–ასეთ გარდამავალ ქაჩვაგამოქაჩვებს,რომელიც ძალიან არ გაწუხებს და სავარაუდოდ გაციების და სეზონის ბრალია,უნდა ან ანტიბიოტიკოთერაპიები ან ექიმებთან სირბილი?ნეტა?

Posted by: nikaa 1 Feb 2011, 22:51
harvi
ნუ რამდენად უნდა ვენდოთ ლაბორატორიებს ეგ ვაფშე სხვა საკითხია, და მთელი თემა არ მეყოფა მაგათ სალანძღავათ, ბევრი ისეთი ლაბორატორიაა ჩაის და შარდს ერთმანეთისადან ვერ არჩევენ.რამდენად სანდოა ის რომ არაფერი არ ამოვიდა, ცოტა საეჭვოა. რო გადაამოწმოს შარდი არ უნდა? ან მივიდეს კარგ ექიმთან, სადღაც მაქვს ერთი ძალიან ძლიერი ნეფროლოგის ნომერი თუ გაინტერესებს მოგწერ, იმასთან ჯობია მივიდეს, ყველაერი მოუყვეს და სწორად დააკვალიანებს დარწმუნებული ვარ.
QUOTE
50 მლ გრ შარდი რჩEბაო ეგ რას ნიშნავს ამდენი აღარ ვიცი.

ესეთი რამე არ გამიგია, არ ვიცი
QUOTE
შEნი რჩევა რა იქნება?

მადლობ ნდობისთვის 2kiss.gif

allie kraft
QUOTE
მეც ზამთრის პერიოდში საკვერცხეები მქაჩავს,მერე გამივლის,მერე ისევ,ნუ ძალიან შემაწუხებლად არა

ერთი მიზეზი ის რომ ცივდები და უფრო თბილათ უნდა ჩაიცვა
მეორე თუ რეპროდუქციულ ასაკში ხარ ოვულაციის დროს იცის საკვერცხეების ქაცვა და წიწკვნა biggrin.gif
QUOTE
ანტიბიოტიკოთერაპიები ან ექიმებთან სირბილი

ანტიბიოტიკოთერაპია არავითარ შემთხვევასი, ანტიბიოტიკების უპრაგონოთ მოხმარება კატეგორიულად აკრძალულია.
კარგ ქვეყანაში რომ ვცხოვრობდეთ ურიგო არ იქნებოდა, მაგრამ საქართველოში იშვიატი ექიმი რომელიც დაგინდობს და არ გაგატყავებს, არ დაგინიშნავს არასაჭირო წამლებს და არ დაგაგლიჯავს ნერვებს. ასე რომ ჯობია თბილატ ჩაიცვათ

Posted by: Mr Adam 4 Feb 2011, 00:43
შარდვისას სისხლდენა ქალებში რამ შეიძლება გამოიწვიოს?..

Posted by: მაჰათმა განდი 4 Feb 2011, 13:22
კითხვა:

ცისტონესა და კიდფლეიმის მიღების დროს, რამდენად დაშვებულია ალკოჰოლის ( მაგალითად ~ 500 გრამი ლუდი) მიღEბა? რამდენად გამოიწვევს ეს რაოდენობა ალკოჰოლისა გვერდით მოვლენებს?



პატივისცემით,
განდი.

Posted by: buub 5 Feb 2011, 00:24
მითხარით რა ამჟამად აფთიაქებში რომელიმე ანტიდეპრესანტი იყიდება ურეცეპტოდ?

Posted by: nikaa 5 Feb 2011, 01:30
Mr Adam
ყველაზე ხშირად შარდ გამომგყოფი სისტემის დაავადებებმა, თუმცა შეიძლება გინეკოლოგიურიც უყოს. მარტო შარდვის დროს არის სისხლიანი გამონადენი? შარდვის დროს თუ აქვს დისკომფორტი (წვა, ტკივილი, ხშირი შარდვა)
mahatma-gandhi
ზოგადად არც ერთ მედიკამენტთან ერთად არაა სასურველი ალკოჰოლის მიღება, მითუმეტეს ლუდის მიღება ამ პრეპარატებთან ერთად.
buub
QUOTE
მითხარით რა ამჟამად აფთიაქებში რომელიმე ანტიდეპრესანტი იყიდება ურეცეპტოდ?

რამდენადაც მე ვიცი არა, მაგრამ არის მცენარეული დამამშვიდებლები ურეცეპტოდ და ძალიან კარგი ეფექტი აქვთ დეპრესიის დროს, თან არ იწვევენ მიცვევას ანტიდეპრესანტებისგან განსხვავებით.

Posted by: მაჰათმა განდი 5 Feb 2011, 02:28
nikaa
QUOTE
mahatma-gandhi
ზოგადად არც ერთ მედიკამენტთან ერთად არაა სასურველი ალკოჰოლის მიღება, მითუმეტეს ლუდის მიღება ამ პრეპარატებთან ერთად.


მადლობა გამოხმაურებისთვის.



პატივისცემით,
განდი.

Posted by: Mr Adam 5 Feb 2011, 19:29
nikaa
QUOTE
მარტო შარდვის დროს არის სისხლიანი გამონადენი?


კიი..

QUOTE
შარდვის დროს თუ აქვს დისკომფორტი (წვა, ტკივილი, ხშირი შარდვა)


არა, ისე კი აწუხებს ტკვილები..

Posted by: umbr_ella 6 Feb 2011, 23:28
ელენთის ტკივილი რისგან შეიძლება იყოს გამოწვეული?

Posted by: utaridi 8 Feb 2011, 20:55
ცჰემო მეგობრებო ვაპირებ გლანდების ოპერაციის გაკეთებას და რას მირჩევთ, ყელის ექიმმა დაიმიდგინა ტონზილიტი...და ეხლა პრინციპში არ მაწუხებს...მარა მაინც მინდა გაკეთება !!! ბევრისგან მსმენია ტუ როგორ პათოლოგიებს იწვევს ადამიანის ორგანიზმში ანგინა და განსაკუტრებით ქრონიკული ტონზელიტი...მაინტერესებს თქვენი აზრი !!!

Posted by: rock in rose 8 Feb 2011, 20:56
utaridi

ექიმმა თუ გირჩია ტონზილექტომია? (გლანდების ოპერაცია)

Posted by: utaridi 8 Feb 2011, 21:22
კი მირჩია...და კიდევ ისიც მითხრა რაც უფრო ადრე გაიკეთებ მით უკეტესიო !!!

Posted by: expert4ge 8 Feb 2011, 21:25
ერთს გირჩევ ჩვენო დიადო მეგობარო აუცილებლად გლანდებს რომ ამოიჭრი მიირთვი ბევრი ნაყინი და რძე ასე მეუბნებოდნენ ბავშვობაში და როცა ამოვიჭერი ვის ახსოვდა ნაყინი smile.gif aba.gif

Posted by: baksona1980 9 Feb 2011, 03:12
არა რა უბედურებაა,უკვე მე 20 დღეა რაა ასე ვარ!ხველეა,დაბალი სიცხეები,სისხლის ანალიზი ნორმალური,ნუ არც ედს არის მომატებული და არც ლეიკოციტარული ფორმულა მარცხნივ გადახრილი.რენტგენოგრამაზეე არის ფილტვის ფესვის კარის გაფართოება და ინფილფრაცი,გაძლიერება ბრონქული სურათის...ხო მომკლა ამ დაბალმა სიცხეებმა,ცოტა ცივ რამეზე მახველებს

Posted by: rock in rose 10 Feb 2011, 20:49
utaridi
QUOTE
კი მირჩია...და კიდევ ისიც მითხრა რაც უფრო ადრე გაიკეთებ მით უკეტესიო !!!


მაშინ უნდა გაიკეთო smile.gif ფორუმზე მაგის კითხვა უაზრობაა

baksona1980

და აქ კონკრეტულად რას კითხულობ? smile.gif

Posted by: mia-maria 11 Feb 2011, 21:08
მითხარით რა ვქნა ძალიან დავიბენი და ყველა ექიმთან სირბილმა დამღალა.

გადატანილი მაქვს ძლიერი ნერვული სტრესი, ცხვირის ოპერაცია და ზლიერი მოწამვა მწვავე ალერგიული ფონით. ეს ყველაფერი ამ უახლოესი წლის განმავლობაში. რამდენიმე თვეა მაქვს მარჯვენა ყურში წრიპინი და დამაკლდა ყურს, კიდევ ალერგიული ფონისგან (მაშინ ერთიანად გავსივდი და გადასხმები გამიკეთეს) თითქმის ყოველ საღამოს მიშუპდება ფეხები და თითქოს მიბუჟდება, მაგრამ უფრო კუნთოვანი ნაწილი მაგრდება და ქვასავით ხდება და ხშირად მაქვს შეგრძნება რომ სადაცაა გული წამივა და დავეცემი ან რომ აღარაფერი მესმის ან ვერაფერს ვხედავ, მიჭირს ზოგჯერ კონცენტრაცია. რომელს ექიმს მივმართო აღარ ვიცი ყურის ექიმთან და ნერვოპათოლოგთან ვიყავი ახლა ვაპირებ ანალიზები გავიკეთო და სისხლის მიმოქცევა გამოვიკვლიო თუ რა ჯანდაბა ვქნა აღარ ვიცი sad.gif

მირჩიეთ რამე, ან ჩამდეთ პირდაპირ სამარეში დავისვენებ მაინც. sad.gif

Roufus
rock in rose
nikaa

მირჩიეთ რა რამე.

Posted by: rock in rose 12 Feb 2011, 17:57
mia-maria
QUOTE
გადატანილი მაქვს ძლიერი ნერვული სტრესი, ცხვირის ოპერაცია და ზლიერი მოწამვა მწვავე ალერგიული ფონით. ეს ყველაფერი ამ უახლოესი წლის განმავლობაში. რამდენიმე თვეა მაქვს მარჯვენა ყურში წრიპინი და დამაკლდა ყურს, კიდევ ალერგიული ფონისგან (მაშინ ერთიანად გავსივდი და გადასხმები გამიკეთეს) თითქმის ყოველ საღამოს მიშუპდება ფეხები და თითქოს მიბუჟდება, მაგრამ უფრო კუნთოვანი ნაწილი მაგრდება და ქვასავით ხდება და ხშირად მაქვს შეგრძნება რომ სადაცაა გული წამივა და დავეცემი ან რომ აღარაფერი მესმის ან ვერაფერს ვხედავ, მიჭირს ზოგჯერ კონცენტრაცია. რომელს ექიმს მივმართო აღარ ვიცი ყურის ექიმთან და ნერვოპათოლოგთან ვიყავი ახლა ვაპირებ ანალიზები გავიკეთო და სისხლის მიმოქცევა გამოვიკვლიო თუ რა ჯანდაბა ვქნა აღარ ვიცი

მირჩიეთ რამე, ან ჩამდეთ პირდაპირ სამარეში დავისვენებ მაინც.


პირველ რიგში დამშვიდდი, არ ინერვიულო smile.gif

ანგიოლოგთან უნდა მიხვიდე. დოპლეროგრაფია და ანგიოგრაფია ჩაიტარო, ულტრაბგერითი გამოკლვლევაა, არტერიათა მდგომარეობის შესასწავლად ტარდება smile.gif

Posted by: V_S 14 Feb 2011, 11:56
ნეფროლოგს ვის გვირჩევთ? მეგობარს სჭირდება და შევპირდი, ფორუმზე ვიკითხავ, ეცოდინებათ მეთქი, არ გამაწბილოთ smile.gif

Posted by: NikolOzzI 14 Feb 2011, 13:00
არ ვიცი სად უნდა მეკითხა და აქ ვიკითხავ.. ხშირად მეყინება მარჯვენა ხელი და მარცხენა მაქვს თბილი რისი ბრალი შეიძლება იყოს? ერთნაირ პირობებში აფსოლიტურად.. ჰა ვკვტები?

Posted by: la cioccolata calda 15 Feb 2011, 18:32
ჯანმრთელობის ცნობა სად ავიღო? sad.gif
ნუ ვიცი რომ პოლიკნინიკასი,მაგრამ ძვირი დამიჯდება თუ ოჯახის ექიმის კონსულტაციას გადავიხდი მხოლოდ?

Posted by: expert4ge 16 Feb 2011, 13:11
ხალხნო იქნებ იცოდეთ იქნებ იცით რა?! ხველა მაქვს მაგრამ არ სიცხე არც გაცივებული ვარ უბრალოდ ესე ხაველებდა ერთიც და მეც გადმომედო როგორც სცანს იქნებ იცოდეთ რა სიმპტომია ე.წ. "ყრუ ხველება" დამტანჯა ექიმთან ვერ მოვახერხე გასვლა იქნებ ამ ღორის გრიპის სიმპტომია მაგრამ სიცხე რატომ არ მაქვს თუ ეგრეა რავი გაურკვევლობაში ვარ! sad.gif

Posted by: misss 17 Feb 2011, 22:48
არ ვიცი, სწორედ ვკითხულობ თუ არა, მაგრამ სისხლის ანალიზში ერთი კომპონენტი მაინტერესებს :

თრომბოციტების განაწილების საზღვრები (pdw) : ნორმა : 15.3-17,3 და მე მაქვს 10,5 და ეს რას ნიშნავს ?
დანარჩენი ნორმაში ზის ყველაფერი smile.gif

Posted by: KAM_M_MILA 18 Feb 2011, 13:35
ჩემს ახლობელს აქვს საყლაპავი მილის თიაქარი და უხვი რაოდენობის ლორწო (მგონი რეფლუქსი ქვია). იღებს ზანტაკს მუდმივად. ამ პრეპარატის ასე მუდმივად მირება შეიძლება? წინასწარ გიხდით მადლობას.

Posted by: natukat 18 Feb 2011, 22:40
გამარჯობათ,არ ვიცი აქ უნდა ვიკითხო თუ არა,მოკლედ 43წლის ქალბატონს ხშირად ეჩვენება თამბაქოს სუნი,როცა რეალურად სუნი არ დგას, ღამით ხელები უბუჟდება და ამბობს რომ სუნთქვა უჭირს,თითქოს ფილტვები ეკუმშება. დაახლოებით რისი სიმპტომები შეიძლება იყოს? დიდი მადლობა.

Posted by: ATJAZZ 20 Feb 2011, 14:21
მოგესალმებით ჩემ ძააან ახლო მეგობარს გოგოს აქვს B ჰეპატიტი (ქრონიკული)

მაინტერესებს განკურნებადია თუ არა (მცენარეული მედიკამენტებით)

გავიგე ვიღაც დავით ვაშაძე კურნავსო .იქნებ ისიც მითხრათ ვინ არის ვაშაძე და ქონია თუ არა რომელიმე თქვენგანს მასთან შეხება

წინასწარ მადლობ smile.gif

Posted by: pleita 23 Feb 2011, 18:45
ხალხო, ნახევარი საათია თვალებში ჭრელ წრეებს ვხედავ , აი ასეთებს
http://www.radikal.ru
biggrin.gif

ტრიალებს ილუზიასავით, არ ჩერდება, ვძაბავ თვალს, მაინც გრძელდება, თავიც ამტკივდა უკან, ზემოთ, ვერ ვიხრები, წნევაა ვითომ? მაღალი თუ დაბალი? რით დავარეგულირო? ეს საზიზღარი აპარატი ხან რას მიჩვენებს, ხან რას, ვეღარ გავიგე help.gif

Posted by: Unico 23 Feb 2011, 22:53
ნეკა და არათითი გამისივდა და ეს შეიძლება ალერგიის ბრალი იყოს?
ისეთი არაფერი მიჭამია, თავბრუ არ მეხვევა, ვერ გამიგია რა მჭირს.
გასივებული მაქ თითები, ოდნავ შეწითლებულია და მექავება, სახსრის ადგილას კი ოდნავ მტკივა... ხოდა რისი სიმპტომებია ეს? sad.gif აქამდე არ მქონია და .. spy.gif სახლის პირობებში რითი შემიძლია დავეხმარო ჩემს ორ თითს? ხელები გავითბო თუ ყინულის საფენები დავიდო?

Posted by: KAM_M_MILA 23 Feb 2011, 23:01
QUOTE
ჩემს ახლობელს აქვს საყლაპავი მილის თიაქარი და უხვი რაოდენობის ლორწო (მგონი რეფლუქსი ქვია). იღებს ზანტაკს მუდმივად. ამ პრეპარატის ასე მუდმივად მირება შეიძლება? წინასწარ გიხდით მადლობას.

ნეტა აქ კითხვებს არავინ პასუხობს?

Posted by: Masked 24 Feb 2011, 04:24
mia-maria

შტრესსი ტუ მიIღე უნდა გამოკვლევა თირკმ. ჯირკვალის და ფარის. ჯირკვალის.


natuka.t
შენი მეგობარი ქალი აქვს ასაკი როცა შეიძლება დაწკებული არის მენოპაუზა. ეს იწვევს მტელი რიგი ჰორმონალური ცვლილებები.
კველა შემძხვევაში უნდა გაკეტოს ფარის. ჯირკვალის ანალიზები, როცა ღამე აქვს გართულება ეს არის ფარის. ჯირკვალის ფუნკ. დარღვევა, სუნი გრძნობა დარღვევას იწვევს ჰიპოტჰირეოზე.



Posted by: Killuminater 25 Feb 2011, 22:04
მუხლები მტკია და რისი ბრალი შეიძლება იყოს ?

ფეხბურთის ბრალი არის ჩემ იაზრით არადა მსუბუქად ვიტვირთები და რა ვუშველო ? რითი გავიძლირო მუხლები ? rolleyes.gif rolleyes.gif

Posted by: K_rati 25 Feb 2011, 22:21
QUOTE
Killuminater

რამდენი წლის ხარ?

Posted by: xatia2525 26 Feb 2011, 16:47
თუ შეგიძლიათ რომ მიპასუხოთ, რა დაავადება შეიძლება იყოს სცორი ნაწლავიდან სისხლის გამოყოფა კუჭის მოქმედების დროს?

Posted by: K_rati 26 Feb 2011, 17:46
xatia2525
ზოგადად ბევრმა ნოზოლოგიამ შეიძლება გამოიწვიოს კუჭ-ნაწლავის ტრაქტიდან სისხლდენა,ძნელია ასე უბრალოდ თქმა.პაციენტის ასაკს აქვს დიდი მნიშვნელობა,40-50 წელს გადაცილებული თუ არის, მეტი შანსია სერიოზული პრობლემის შედეგი იყოს და შესაბამისად მეტ ყურადღებას საჭიროებს. ბევრ ფაქტორია გასათვალისწინებელი: რამდენი ხნის წინ დაიწყო სისხლდენა?სისხლი რა ფერისაა?მოწითალოა თუ უფრო მოშავო?სისხლიანი გამონადენი ფეკალურ მასებს ახლავს თუ დეფეკაციას მოსდევს,ოჯახური ანამნეზი როგორია,ტკივილები თუ აქვს,ან სხვა რაიმე არასპეციფიკური სიმპტომი?და კიდევ სხვა მრავალი დეტალია საჭირო სავარაუდო დიაგნოზის დასამელად

Posted by: xatia2525 26 Feb 2011, 18:28
დიდი მადლობა განმარტებისთვის, ახალგაზრდა ვარ, 34 წლის, ახლავს ფეკალურ მასებს, სისხლისფერია. სხვა უსიამოვნო სიმპტომები არ აქვს. ოჯახური ანამნეზი რას ნიშნავს? არც ნოზოლოგია ვიცი რა არის.

Posted by: rock in rose 26 Feb 2011, 18:37
K_rati

ძალიან კარგია ხალხის კითხვებს რომ პასუხობთ და საკმაოდ კომპეტენტურადაც, მაგრამ ერთი შენიშვნა მექნება smile.gif ისინი ჩვეულებრივი მოქალაქეები არიან, არასამედიცინო განათლებით და მათთვის გასაგებ ენაზე თუ ახსნით, ყოველგვარი სამედიცინო ტერმინოლოგიის გარეშე, მთლად გადასარევი იქნება smile.gif

აი ასეთი კითხვები რომ აღარ იყოს მერე smile.gif

QUOTE
ოჯახური ანამნეზი რას ნიშნავს?

Posted by: K_rati 26 Feb 2011, 18:53
xatia2525
თქვენს მიერ ჩამოთვლილი სიმპტომების მიხედვით,სისხლდენა შესაძლოა გამოწვეული იყოს ჰემოროიდული ვენების გაგანიერებით, ე.წ. ბუასილი.ყოველ შემთხვევაში საჭიროებს ექიმთან ვიზიტს.თუ გნებავთ P.M-ში მოგწერთ სად მიბრძანდეთ და ვისთან.
ოჯახური ანამნეზი ნიშნავს ვინმეს თუ ჰქონდა ან აქვს ოჯახში კუჭ-ნაწლავის მხრივ მსგავსი პრობლემა ან სხვა დაავადება.ნოზოლოგია, ასე ვთქვათ, დაავადებათა სიაა.
* * *
rock in rose
გეთანხმებით.მართალი ხართ აბსოლუტურად,ვითვალისწინებ თქვენს შენიშვნას და რჩევას.
smile.gif

Posted by: xatia2525 26 Feb 2011, 18:59
კი, ტუ შეიძლება მომწერეთ. პროფილაქტიკის მიზნით რა შეიძლება გამოვიყენო?

Posted by: rock in rose 26 Feb 2011, 19:02
K_rati

QUOTE
გეთანხმებით.მართალი ხართ აბსოლუტურად,ვითვალისწინებ თქვენს შენიშვნას და რჩევას.


გმადლობთ გაგებისთვის smile.gif კარგ საქმეს აკეთებთ yes.gif

Posted by: K_rati 26 Feb 2011, 19:11
rock in rose
მადლობა თქვენsmile.gif

xatia2525
P.M-ში მოგწერეთ

Posted by: Mr Adam 27 Feb 2011, 16:56
რამე მიშველეთ, ჯირკვლები მაქვს დასიებული ყელის არეში და საერთოდ ვერ ვამოძრავებ თავს.. cry.gif

Posted by: Killuminater 28 Feb 2011, 22:51
K_rati

15 ის : ) 16-ში გადადგმული მაქვს დიდი ნაბიჯიც : D

Posted by: saturngirl 28 Feb 2011, 23:57
http://www.radikal.ru
უკვე მეორეთვეა აი ამ ადგილას დაახლოებით სადაც ფოტოზეა მონიშნული, ვგრძნობ ტკივილს. ტკივილი არის მოვლითი. ჭვალივით უცნაური ტკივილია, თითქოს სიღრმეში რაღაცას გაჰყვება. არ ვიცი ვერ ვხსნი. რისი ბრალი შეიძლება იყოს?
კიდევ არ ვიცი ეს რამდენადაა ზემოთხსენებულთან კავშირში, მაგრამ ყოველ საღამოს (ეს 3 დღეა) მიწევს ტემპერატურა, 37.2-37.4 გაციებული არ ვარ..... არც გრიპი მაქვს........... ახლაც ასეთი ტემპერატურა მაქვს.

Posted by: BloodElf 1 Mar 2011, 12:46
ჯანმრთელობის მხრივ არანაირი სერიოზული პრობლემა არ მაქვს.. უამრავი გამოკვლევა მაქვს ჩატარებული და თითქოს ყველაფერი რიგზე მაქვს.. ნაღველა მაწუხებს ხანდახან მაგრამ ვმკურნალობ.. მაგრამ ამ ბოლო დროს ძალიან ცუდი მადა მაქვს .. გულისრევის შეგრძნება და სისუსტე მაწუხებს ხშირად .. მალე ვიღლები sad.gif( ჰოდა იქნება რამე რჩევა მომცეთ რით ვუშველო თავს ? sad.gif(

Posted by: K_rati 1 Mar 2011, 15:21
Killuminater
შეგიძლია,რომ სურათი დადო და მიანიშნო რა ადგილას გტკივა?ტკივილი მუდმივია?დატვირთვებზე ხო არ იმატებს(განსაკუთრებით კიბეზე ასვლა-ჩამოსვლის,ჩაკუზვის,სირბილის დროს) ან შეშუპებული ხო არ გაქვს?


http://images.supersport.co.za/Osgood1.jpg
ამ ადგილზე ხომ არ გტკივა?

Posted by: ATJAZZ 2 Mar 2011, 19:07
QUOTE
მოგესალმებით ჩემ ძააან ახლო მეგობარს გოგოს აქვს B ჰეპატიტი (ქრონიკული)

მაინტერესებს განკურნებადია თუ არა (მცენარეული მედიკამენტებით)

გავიგე ვიღაც დავით ვაშაძე კურნავსო .იქნებ ისიც მითხრათ ვინ არის ვაშაძე და ქონია თუ არა რომელიმე თქვენგანს მასთან შეხება

წინასწარ მადლობ



არ ვიკბინები smile.gif

Posted by: Killuminater 5 Mar 2011, 11:46
K_rati

არა შეშუპებული არ მაქვს, გარეგნობით კარგია, ნუ დატვირთვაზე უბრალოდ მეღლებასავით თითქოს ძვალი მტკივა

როცა ფეხბურთს ვთამაშOბ და ფინტს რომ ვაკეთებ მგონია რომ გადავტყები ხოლმე ანუ სისწრაფის დროს

რომ ვივარჯიშებ გადავუებს სურათს დადავდებ ზუსტად რა ადგილზე მეტკინება down1.gif

Posted by: donkosta 5 Mar 2011, 23:54
ეხო იმუსავებს ხვალ სადმე?

სად შეიძლება იყოს?

Posted by: kriketi-1985 7 Mar 2011, 02:03
აღელვებისა და ნერვიულობის დროს ყელში მაქვს პულსაციის შეგრძნება, თითქოს სპაზმები, კუმშვებიც შეინიშნება, რას მირჩევთ? biggrin.gif გმადლობთ წინასწარ...

Posted by: ბინლადენი 7 Mar 2011, 22:51
სად ვიკითხო არ ვიცი,მოკლედ ყელზე წვერს სადაც ვიპარსავ შიგნით პატარა ბურთივით გამიჩნდა (პრიშიკი არაა და არც ძირმაგარა )
არამგონია რო თმა მქონდეს ჩაბრუნებული,იმიტო რო არაფერი არ ჩანს და თავიდანაც არ ჩანდა და რამე მაზი მირჩიეთ რა რო ეს ყველაფერი ჩამიცხროს ,ან დამაკვალიანეთ რა როგორ მოვიქცე

Posted by: umbr_ella 9 Mar 2011, 16:13
იმედია სწორ ადგილზე ვსვავ კითხვას.
ამ ბოლო დროს ცისტიტი მაწუხებს საშინლად. ვიმკურნალე დაახლოებით 2 კვირის წინ და დავამთავრე. მერე ანალიზიც ავიღე შარდის და ყველაფერი წესრიგში მქონდა. ახლა ისევ დამეწყო, ოღონდ ისე მწვავედ არა.. ალბათ ქრონიკულია უკვე ხო? sad.gif არავინ იცის რამე, რითიც შეიძლება ბოლო მოვუღო ამ ავადმყოფობას? აშკარად ნერვებზე მოქმედებს sad.gif(((((((((((((((((
* * *
ამ თემაში არავინ არ შემოდის? sad.gif

Posted by: K_rati 14 Mar 2011, 20:20
umbr_ella
წინა შემთხვევა რა წამლებით იმკურნალეთ?და რამდენი დღე იღებდით?

Posted by: umbr_ella 15 Mar 2011, 03:40
K_rati
QUOTE
წინა შემთხვევა რა წამლებით იმკურნალეთ?

ვაიმე sad.gif 2 წამალი იყო... აუ არ მახსოვს სახელი. 1 ანტიბიოტიკი იყო. ხვალ შემიძლია დავწერო, დედა რომ სამსახურიდან მოვა, მან იცის...
QUOTE
რამდენი დღე იღებდით?

3 დღე :|

Posted by: pteranouka 16 Mar 2011, 19:51
თავის მტკივა, ქრონიკულად, ალბათ უკვე ორი თვეა, კეფა

რომელ ექიმთან უნდა მივიდე, ხომ ვერ მირჩევთ?

Posted by: lucia 17 Mar 2011, 00:25
არ ვიცი აქ თუ შემოდის ვინმე მაგრამ იქნებ მირჩიოთ ვინმემ:

ხელზე მაქვს კანი აგლეჯილი, საკმაოდ სქელი ფენა და შუაში გაჭრილიცაა, მაგრამ აი თავისით არ მძვრება. გათეთრებულია და პერიოდულად ოდნავ სითხე გამოდის. დაახლოებით 5-10 თეთრიანის ფართობზეა, მაგრამ მოხრის ადგილია და მაწუხებს gigi.gif არის შანსი რომ ეს კანი ისევ შემიხორცდეს, თU აუცილებლად უნდა ავიჭრა? თუ ასაჭრელია მე ალბათ ვერ ავიჭრი ხო და ექიმთან უნდა მივიდე? მადლობა წინასწარ

Posted by: Doloress 17 Mar 2011, 11:47
ე.ი. ორი დღეა მამტვრევს. წელი და სახსრები მტკივა ისე, რომ ვერ დავდივარ. ზუსტად ისე, როგორც გაციების წინ იცის ხოლმე. მაგრამ გაციებული არ ვარ, არანაირი სიმფტომი არ მაქვს.
რაღაცამ მაწყინა და შეიძლება ამის გამო მამტვრევდეს? თუ შევშფოთდე და ექიმთა წავიდე?


Posted by: pteranouka 17 Mar 2011, 13:35
lucia

ექიმთან მიდი, თუ სითხე გამოდის ე.ი. ინფექციაა. წყალბადის ზეჟანგით უნდა დაგემუშავებინა თავიდანვე.


Posted by: lucia 17 Mar 2011, 14:12
pteranouka
არა, ინფექცია არ არის. აი კანის სითხე გამოდის რაღაც... საშინელებებს ვწერ ცოტა მაპატიეთ

Posted by: K_rati 17 Mar 2011, 16:16
lucia
თუ გარშემო კანი შეწითლებული არ არის და გამონადენი სითხე არ არის ჩირქიანი,მაშინ ეგ სითხე ლიმფაა,უფერო ან ოდნავ მოყვითალო ფერის და ინფექციის გამო არ არის.კანი თუ გათეთრებულია და მგრძნობელობაც დაკარგული აქვს,ე.ი ცოცხალი აღარაა და აღარ შეხოცდება, მისი მოცილებაა საჭირო.ექიმთან მისვლა არამგონია აუცილებელი იყოს,თქვენვე შეგიძლიათ გააკეთოთ.ჭრილობა წინასწარ წყალბადის ზეჟანგით(პერეკისი) და ბეტადინით დაიმუშავეთ,შემდეგ როდესაც მკვდარ კანს მოაცილებთ ლევომიკოლის მალამოს საფენი დაიდეთ და შეიხვიეთ.

Posted by: lucia 17 Mar 2011, 23:47
K_rati
დიდი მადლობა. ხო ლიმფაა, ვერ გავიხსენე ეგ სიტყვა gigi.gif დღეს საშინებელა ჩავიდინე და ავიგლიჯე ნახევარი კანი და ეს ვარდისფერი ხორცი რომ მიყურებს შიგნიდან გულში მირტყავს. მაგის გამო დარჩენილ ნაწილს ჯერ ხელს არ ვკიდებ. ბრრრრრრრრრრ რამდენ ხანში ამომივა ახალი კანი? ვერ დავაჩქარებ სწრაფად ამოვიდეს? sad.gif

Posted by: K_rati 18 Mar 2011, 17:33
lucia

QUOTE
დღეს საშინებელა ჩავიდინე და ავიგლიჯე ნახევარი კანი და ეს ვარდისფერი ხორცი რომ მიყურებს შიგნიდან გულში მირტყავს. მაგის გამო დარჩენილ ნაწილს ჯერ ხელს არ ვკიდებ


მკვდარი ნაწილი რაც არის უნდა მოცილდეს მთლიანად,მაგრამ არა აგლეჯვით,მკვდარი უბნები მტკინვეული აღარ არის და შეგიძლიათ წინასწარი სადეზინფექციო დამუშავების შენდეგ მაკრატლით აიჭრათ(ცოტა ცუდად ჟღერს smile.gif ).შეხორცება და კანის მთლიანობის აღდგენის დრო დამოკიდებულია ჭრილობის სიღრმეზე.თუ მხოლოდ კანის ზედა ფენების დაზიანებაა,საშუალოდ ერთ კვირაში მთლიანად უნდა აღდგეს.ეცადეთ ჭრილობა მაქსიმალურად სტერილურად გქონდეთ და გამოიყენეთ ასევე ლევომიკოლის მალამო,რომელიც შეხორცებას დაჩქარებს.მალე გამოჯანმრთელებას გისურვებთsmile.gif

Posted by: lucia 18 Mar 2011, 18:47
K_rati
დიდი მადლობა. ხო ვიცი, კულტურულად უნდა მექნა, მაგრამ ექსტრემალურ სიტუაციაში ამძვრა ნაწილი და ბარემ დანარჩენიც მივაყოლე. რაღაც ჰიჩკოკში ვცხოვრობ მთელი კვირაა, ნეტა შაბათ-კვირას რას ვიზამ gigi.gif

Posted by: shilaida 19 Mar 2011, 12:31
მაინტერესებს,ქატო ქრონიკული გასტრიტისას თუ შეიძლება ???

Posted by: tikope 22 Mar 2011, 21:36
K_rati
ხველება მაწუხებს უკვე მეოთხე კვირაა, ღამე უფრო მეტად, სპაზმური,ღებინებით. ყელი არ მტკივა, არც ბრონხებია და არც ფილტვები, ექიმმა გამსინჯა, დავასკვენით , რომ ესაა ალერგიული , თუმცა 37 წლამდე ალერგია არასდროს მქონია, ერთი კვირა ალერტეკს ვსვამდი, მაგრამ უშედეგოდ, ახლა უკვე დღეც მახველებს, ყელში გამუდმებით დაგროვილი მაქვს ლორწო. . გარდა ალერგიისა , რა შეიძლება იწვევდეს ასეთ სპაზმურ ხველებას?
პ.ს.მწეველი არა ვარ

Posted by: Natuka NGN 22 Mar 2011, 22:11
tikope
ენაზე ხომ არ აგქვს თეთრი ნადები? და გაძლიერებული ნერწყვის დენა? ყელის სიმშრალე გაქვს?

Posted by: tikope 22 Mar 2011, 22:18
Natuka NGN
QUOTE
ენაზე ხომ არ აგქვს თეთრი ნადები? და გაძლიერებული ნერწყვის დენა? ყელის სიმშრალე გაქვს?

ენაზე არ ვიცი , არა მგონია.
ნერწყვის დენა არა მაქვს
ყელის სიმშრალე , აი, როგორ გითხრა, მშრალად ვახველებ, მაგრამ ყელში გამუდმებით ბლანტი რაგაც მაქვს და ნერწყვიც ასეთივე ბლანტია, რა ვიცი რაღაც უცნაურად, თან ამეღიტინება და მეწყება მერე ხველა, ისევ და ისევ

Posted by: Natuka NGN 22 Mar 2011, 22:32
tikope
ხველება საიდან მოდის ბრონქებიდან თუ ყელშია ლოკალიზებული. შესაძლოა იყოს ალერგია ანდა შესაძლოა იყოს რაიმე ვირუსით გამოწვეული ფარიგნგიტი, სულაც არაა საჭირო ყელი წითელი იყოს. ერთი სიოტყვიტ, მარტივ რაღაცას გეტყვი, ვაშლის ძმრიოს სავლები გამოივლე და ნახე ოდნავ მოეგშვება თუ არა.

Posted by: tikope 22 Mar 2011, 22:35
Natuka NGN
QUOTE
ხველება საიდან მოდის ბრონქებიდან თუ ყელშია ლოკალიზებული.

ყელშია ლოკალიზებული. ვაშლის ძმარზე მეც ვიფიქრე. გამოვივლებ ალბათ. ყელიც არაა წითელი. არც მიავადმყოფია, აი, სიცხეებსა და სვხვა ვირუსულ დაავადებებს ვგულისხმობ. მოკლედ რომ გახანგრძლივდა, იმიტომ შევშფოთდი....

Posted by: K_rati 22 Mar 2011, 23:08
tikope

QUOTE
ღამე უფრო მეტად,

დაწოლის შემდეგ? გულძმარვა ხომ არ გაქვთ? წვის შეგრძნება გულმკერდის მიდამოში?ხმა ჩახლეჩილი გაქვთ?

ხველება ამგვარი ხომ არ არის:ამოსუნთქვის დროს რამდენიმე(5-10) ერთმანეთზე გადაბმულ ხველის ციკლს მოსდევს გახანგრძლივებული ჩასუნთქვა და "ყივილის" მაგვარი ხმიანობა?

ამოღებულ მასას მოუნელებელი საჭმლის ნარჩენები ხომ არ მოყვება?

ქრონიკული ხველა შეიძლება ასევე იყოს გადატანილი ზედა სასუნთქი გზების ინფექციის შემდეგ, რამდენიმე კვირა და თვეც შეიძლება გაგრძელდეს,მაგრამ როგორც გავიგე მსგავსს არაფერს შეუწუხებიხართ

Posted by: tikope 23 Mar 2011, 08:35
QUOTE
დაწოლის შემდეგ? გულძმარვა ხომ არ გაქვთ? წვის შეგრძნება გულმკერდის მიდამოში?ხმა ჩახლეჩილი გაქვთ?

ჩახლეჩილი ხმა მაქვს მხოლოდ , დანარჩენი არაფერი
QUOTE
ხველება ამგვარი ხომ არ არის:ამოსუნთქვის დროს რამდენიმე(5-10) ერთმანეთზე გადაბმულ ხველის ციკლს მოსდევს გახანგრძლივებული ჩასუნთქვა და "ყივილის" მაგვარი ხმიანობა?

არა მსგავსი რამ არა მჭირს
QUOTE
ამოღებულ მასას მოუნელებელი საჭმლის ნარჩენები ხომ არ მოყვება?

დზალიან დიდი რაოდებოით ლორწო ამომყვა, რომ ვწვები ხოლმე პირი მევსება ამ რაღაც ლორწოთი, cry.gif cry.gif

Posted by: shilaida 23 Mar 2011, 09:30
QUOTE
მაინტერესებს,ქატო ქრონიკული გასტრიტისას თუ შეიძლება ???

უფრო ხომ არ გაამწვავებს მდგომარეობას?

იქნებ მეც მიპასუხოთ,პლზ

Posted by: Howl 23 Mar 2011, 11:25
გაუმარჯოს
ყველაზე ძლიერი ტკივილგამაყუჩებელი რა არის საქართველოს აფთიაქებში??? smile.gif

Posted by: Julika Stiller-Tschudy 23 Mar 2011, 17:03
გაციებული ვიყვი, ყელი მტკიოდა , სიცხეც მქონდა და საშინლად ვოფლიანობდი, 3 დღე ვიწექი. უკვე გამოვჯანმრთელდი, ე.ი. სიცხე აღარა მაქვს, აღარსც ყელი მტკივა, არც სურდო მაქვს, უბრალოდ სასუნთქი გზააცოტა გაღიზიანბული. თუმცა თავს მაინც სუსტად ვგრძნობ და გასაგებია ავადმყოფობის შემდეგ.

მაგრამ აი, ორი დღეა მტკივა წელი, თეძოები და საჯდომიც, ჯდომისას და წოლისას მაწუხებს, მოუსვენრადა ვარ, თუ დავდივარ მაშინ არა მიშავს. სულ გაფუთნული ვარ. მეზარება ექიმთან ამის გამო წასვლა, თან მგონია დიდად ვერ დამეხმარება. რა შეიძლება იყოს, ნერვი გაცივდა?

Posted by: K_rati 23 Mar 2011, 17:53
tikope
ლორწოს განსხვავებული ფერი ხომ არ აქვს? რენტგენი ხომ არ გაქვთ გადაღებული?


shilaida
ქატომ არ უნდა გამოიწვიოს წესით გამწვავება. მიზეზი დადგენილი გაქვთ ქრონიკული გასტრიტის? მკურნალობთ?

Julika Stiller-Tschudy
შესაძლოა ეს ნერვის ბრალი იყოს. წელში მოხრის დროს,გვერდზე გადახრის ან წამოდგომის დროს ხომ არ ძლიერდება ტკივილები? ეს ტკივილები ასევე შეიძლება იყოს გაციებისშემდგომი მიალგია,რომლის დროსაც კუნთები არის მტკინვეული.არასტეროიდული ანთების საწინაარმდეგო მალამო შეგიძლიათ ცადოთ,მაგ. ვოლტარენ ემულგელი. შეიზილეთ კარგად მტკინვეულ ადგილებზე და თბილად შეიხვიეთ.

Posted by: gryffindor 23 Mar 2011, 18:05
მოგესალმებით!

ვარ 23 წლის, 4 თვე მაქვს სიცხეები 37.2-37.4 დღის მეორე ნახევარში, დილით ადრე და გვიან საღამოს სიცხე არ მაქვს. გავიკეთე სისიხლის საერთო ანალიზი და ერითროციტები მქონდა დაბალი -3,7 და ე.დ.ს -3.
გავიკეთე რენდგენი და ფილტვები მაქვს სუქთა.თავს ვგრძნობ კარგად. ერთადერთი ყელი მაქვს სუსტი და ძალიან ხშირად მტკივა ,არანაირი წამალი და გამოვლება ჩემ ყელს არ შველის ,გლანდების ოპერაცია მაქვს გაკეთებული ბავშობაში, გავიკეთე ბაქტერიოლოგიური გამოკვლევა და დამიდგინდა სტაფილოკოკები და ენტეროკოკები. შეიძლება ეს იყოს სიცხის მიზეზი? ან რა გამოკვლევა უნდა ჩავიტარო? რა იწვევს სიცხეს?

წინასწარ გიხდით მადლობას

Posted by: MALI 24 Mar 2011, 06:36

gryffindor
მეც მაინტერესებს ექქიმის მოსაზრება

QUOTE
გავიკეთე ბაქტერიოლოგიური გამოკვლევა და დამიდგინდა სტაფილოკოკები და ენტეროკოკები. შეიძლება ეს იყოს სიცხის მიზეზი? ან რა გამოკვლევა უნდა ჩავიტარო? რა იწვევს სიცხეს?

კი, შეიძლება მაგის ბრალიც იყოს, ზუსთად ალბათ ექიმი გაგვცემს პასუხს.
სტაფილოკოკების და სტრეპტოკოკების გახსენება აღარ მინდა ვერ მოვირჩინე, ახალი ნაცხის აღებისას სხვადასხვა ტიპის სტრეპტოკოკები და სტაფილოკოკები მიდგინდება, ვიმკურნალობ ნაცხის მიხედვით ანტიბიოტიკებით , ფაგებითაც მიმკურნალია. მაგრამ უშედეგოდ, გლანდები არ მაქვს ამოჭრილი, მაგრამ არც მაწუხებს..'ერთადერთი ვენების უკმარისობა მაქვს და წითელი ქარი მემართება.

ჩემი კითხვა ასეთია: პირის ღრუდან ნაცხის აღება ნიშნავს რომ მთელ ორგანიზმშია კოკები, თუ მხოლოდ პირის ღრუში? და კიდევ გადამდებია თუ არა ?
მადლობა წინასწარ

Posted by: shilaida 24 Mar 2011, 09:57
K_rati
QUOTE
მიზეზი დადგენილი გაქვთ ქრონიკული გასტრიტის? მკურნალობთ?

დიაგნოზი ქრონიკული გასროდუოდენიტი გამწვავბის ფაზაში.
არადა ვიცი,რომ მეტაბილიზმის ხელშემწყობია.

Posted by: Julika Stiller-Tschudy 24 Mar 2011, 15:55
K_rati

მადლობა, კი ვოლტარენი გამოვიყენე. up.gif

Posted by: K_rati 24 Mar 2011, 16:06
Julika Stiller-Tschudy


არაფრისsmile.gif

Posted by: tikope 25 Mar 2011, 18:31
K_rati
QUOTE
ლორწოს განსხვავებული ფერი ხომ არ აქვს? რენტგენი ხომ არ გაქვთ გადაღებული?

არა ჩვეულებრივი გამჭვირვალეა თეთრი, რენდგენიც არ გადამიღია.
წუხელ დავლიე დამამშვიდებლების მთელი ბუკლეტი, მერე სუპრასტინი და ორი აბი ნოშპა და ცოტა მეძინა.სადღაც 5 წუთი მახველა მხოლოდ.

Posted by: ქეთუ 29 Mar 2011, 00:35
http://mymarket.ge/product_info.php?type_id=1&product_id=1232225

ვნახე ეს შემთხვევით და ვიფიქრე შევიძენთქო და რამდენად რეალურია?


Posted by: Tommy Guerrero 29 Mar 2011, 16:56
ეს ოფთალმოლოგი და ასე შემდეგ რას ნიშნავს არ ვიცი

ფეხი გადამიბრუნდა და კოჭთან გამისივდა
რო ვადგამ მტკივა მაგრა
ვერ დავდივარ კაროჩე

თუ მოტეხილი არაა 2 დღეში გამივლის?

Posted by: K_rati 29 Mar 2011, 17:15
Tommy Guererro

ვადებს ვერ იტყვი რამდენ ხანში გაგივლის ტკივილი. შეიძლება მოტეხილი არ იყოს,მაგრამ იოგოვანი აპარატი იყოს დაზიანებული, გადაბრუნების დროს ხშირად ზიანდება ე.წ ტიბიოფიბულარული სინდესმოსი,რომელიც კოჭის სახსრის სიმტკიცეს უზრუნველყოფს. ამ დროს ტკივილები უფრო მალე ქრება, მაგრამ თუ შესაბამისი მკურნალობა არ ჩატარდა სახსარი არასტაბილური რჩება და უმნიშვნელო მოძრაობებზეც ადვილად გადაგიბრუნდება ფეხი. სასურველია ყველა ვარიანტში ტრავმატოლოგს მიმართო და რენტგენი გადაიღო,იქამდე კი ყინულიანი კომპრესები დაიდე, შედარებით დააცხრობს შესიებას

Posted by: Tommy Guerrero 29 Mar 2011, 18:02
K_rati
გაიხარე ძმა
--------------

Posted by: gvelis_wiwila 2 Apr 2011, 19:56
მოკლედ მაქვს ბოლო თვეა ესეთი სიმპტომი - ვერ გამოვხატავ სიტყვებით წესიერად მაგრამ ისეთი შეგრძნებაა თითქოს ცხელი სითხე მოძრაობს გულთან ახლოს. ოღონდ მოძრაობს ნელა ძალიან და ხანდახან მახსენებს თავს. საკმაოდ უსიამო შეგრძნებაა user.gif

თან გულზე გავესინჯე და არაფერი არ მაქვს. ვერავინ ვერ მეუბნება რის გამო შეიძლება იყოს, არადა ადრე ამის მსგავსიც არაფერი არ მქონდა spy.gif

ფილტვების ანთება მქონდა და ანტიბიოტიკებს რომ ვიკეთებდი მაშინ გაიარა, მაგრამ ეხლა ისევ მაქვს და რა ჯანდაბაა ვინმემ გამაგებინოთ ეგებ gigi.gif

შეიძლება სითხე არცაა, შეგრძნებაა ესეთი უბრალოდ.
ეგებ გამარკვიოთ

Posted by: utaridi 3 Apr 2011, 01:18
QUOTE (gvelis_wiwila @ 2 Apr 2011, 19:56 )
მოკლედ მაქვს ბოლო თვეა ესეთი სიმპტომი - ვერ გამოვხატავ სიტყვებით წესიერად მაგრამ ისეთი შეგრძნებაა თითქოს ცხელი სითხე მოძრაობს გულთან ახლოს. ოღონდ მოძრაობს ნელა ძალიან და ხანდახან მახსენებს თავს. საკმაოდ უსიამო შეგრძნებაა user.gif

თან გულზე გავესინჯე და არაფერი არ მაქვს. ვერავინ ვერ მეუბნება რის გამო შეიძლება იყოს, არადა ადრე ამის მსგავსიც არაფერი არ მქონდა spy.gif

ფილტვების ანთება მქონდა და ანტიბიოტიკებს რომ ვიკეთებდი მაშინ გაიარა, მაგრამ ეხლა ისევ მაქვს და რა ჯანდაბაა ვინმემ გამაგებინოთ ეგებ gigi.gif

შეიძლება სითხე არცაა, შეგრძნებაა ესეთი უბრალოდ.
ეგებ გამარკვიოთ

ალბათ ნერვოზია !!! ნერვოზ ცჰვევია ეგეტი რაგაცეები...ტიტკოს და გეცჰვენება რომ რაგაც გცჰირს ან კიდე ბევრი ფიქრი მოგიწია იმაზე რომ თითქოს რაგაც გემართება...
რავი ალბათ ცოტა აგრესიული უნდა გახდე და კატეგორიული რაგაც საკითხებთან მიმართებაში...ცოტაც ხმას მოუმატე და იყვირე smile.gif ეს ყველაფერი ჩაგაყენებს ფორმაში...დამიჯერე ამას საკუთარი გამოცდილებიდან გეუბნები !!! არ მიცე საკუთარ თავს ისეთ თემებზე იფიქრო როგორიცაა ავადმყოფობა და დაავადებები...საერთოდ მედიცინა ძალიან მიყვარს...მაგრამ მომენტები მქონია რომ აღარ ვეკარებოდი სამედიცინო ლიტერატურას...იმიტომ ამ სამყაროში იმდენი დაავადებაა და ოხრობა, რომ ცხოვრება არ მოგინდება smile.gif შემდგომ ფსიქოლოგიურად რომ დაძლევ რიგ ბარიერებს მერე უნდა შეუდგე დაავადებების შესწავლას და გეცოდინება რა და როგორ...მე ესე ვქენი !!! და თუ შესაძლებლობა გექნება შეძლებისგადვარად სპორტს მიყევი ხელი...მართალია მედალს ორი მხარე აქვს !!! თუ აგრესიული ხარ ცოტათი მოდუნდი და შეეცადე, რომ სიმშვიდეს და ჰარმონიას მიაღციო...მე ვფიქრობ, რომ ნერვოზია !!! შეიძლება ვცდები...

Posted by: gvelis_wiwila 3 Apr 2011, 15:12
utaridi

ნევროზი გასაგებია, მაგრამ ნევროზი 2 წელია მაქვს და ყველა სიმპტომს შეგუებული ვარ.

ეს კიდევ უშუალოდ გრიპის მერე დამეწყო. მანამდე არ მქონია ესეთი sad.gif

Posted by: Natuka NGN 3 Apr 2011, 17:23
gvelis_wiwila
გააშუქე ფილტვები... ადვილია ნევროზისატვის ყვეალფრის დაბარლება. თან ფილტვებზე პრობლემა გქონია.

Posted by: ვივიანა 3 Apr 2011, 23:20
ივნისის ბოლოს მქონდა წნევები 140, 150, 160იც + თავი მტკიოდა. ანალიზი გავიკეთე შარდის და ნორმალურად იყო, ვარ 19 წლის, გადაღლილობას დავაბრალეთ და დავისვენე მასწავლებლებისგან..
უკვე 3-4 კვირაა თავბრუც მეხვევა და წნევები ისევ მაქვს.. თავი აღარ მტკივა..
რა შეიძლება ვქნა??

Posted by: Kate789 4 Apr 2011, 02:08
არ ვიცი სწორ თემაში ვპოსტავ თუ არა, მაგრამ იქნებ ვინმემ მიპასუხოთ.

რამდენიმე კვირაა არათითის ფრჩხილი მაქვს სხვებისგან განსხვავებული.
ანუ ერთგვაროვანი არ არის, ოთითქოს ჯოხივით მადევს რაღაც და ეგ "ჯოხი" ამოწეულია.
როგორც კი ნაპირისკენ მოემართება ანუ აი უკვე მოჭრას რომ ვიპირებ, ფრჩხილი იყოფა.
იდეაში რაღაც ხაზივითაა. ვერ გავიგე, რა არის. არაფრისთვის მიმირტყამს და რა ხდება, იქნებ ვინმემ გამარკვიოს.
კლიბითაც ვიქლიბავ, მაგრამ მაინც ისეა ხოლმე. ახალი ფრჩხილი რომ მოდის, გაყოფილია.

Posted by: Beck 4 Apr 2011, 02:39
QUOTE (Summer_Rain @ 3 Jan 2011, 12:34 )
სალამი!!

ცუდია, რომ არაა თემა "კითხვა გასტროლოგს"
მარტო ჩემს გარშემო იმდენი ვიცი, რომლებსაც აწუხებს კუჭი..


იქნებ აქ მაინც მიპასუხოთ ჩემს გასაჭირზე და პრობლემებზე.


დავიწყებ თავიდან.
ივლისის ბოლოს დამეწყო კუჭში სპაზმების მსგავსი, 2-3 დღე გამიგრძელდა, ნოშპამ და მცირე დიეტამ მიშველა.
აგვისტოს ბოლოს კი პამიდორმა მომწამლა,
მეორე დღეს ვიდიეტე
მესამე დღეს ხინკალი ვჭამე ისიც არ შემერგგო
მეოთხე დღეს ხაჭაპური, ისევ პირღებინება
შემდეგ 1 კვირიანი დიეტა..
შემდეგ აჯაფსანდალი მივირთვი, მაინც ვერ გადაამუშავა კუჭმა.

დავამატებ იმასაც, რომ როგორც კი გულისრევის შეგრძნებას ვიგრძნობდი ეგრევე ნერვებს ვერ ვთოკავდი
მთელი სხეული მეჭიმებოდა. და ასანამ არ მოვიცილებდი არასასურველ საკვებს კუჭიდან განუწყვეტლივ მაკანკალებდა..
შემდეგ რის ვაი ვაგლახით ვახერხებდი ჩაძინებას.. sad.gif

გადავწყვიტე გასტროლოგთან მისვლა, ზონდი არ გადამაყლაპა, მითხრა ვეგეტო-ნევროზი გაქვსო, რომელიც გულის რევითაა გამოხატულიო და გამიშვა ნევროპათოლოგთან.
ნევროპათოლოგმაც ნევროზის ნიშნებიაო და გამომიწერა წამლები, ერთმა მადა დამიკარგა, მეორემ გული ამიჩქარა. ისევ სარაჯიშვილის სიროფი ავიღე და ვსვამდი საღამოობით.

სექტემბრიდან ჩემით დიეტა დავიწყე სერიოზული, ოქტომბრის ბოლომდე ვიდიეტე, თითქოს უკეთ ვგრძნობდი თავს, მაგრამ კუჭში სიმძიმე ისევ მქონდა და საღამოობით გულისრევის შეგრძნება ისევ მაწუხებდა.

ამიტომ გადავწყვიტე ზონდი გადამეყლაპა, იქ სადაც ვარ დაზღვეული, აისი გრუპის სადაზღვევოში, გავიკეთე გასტროფიბროსკოპია.
ექიმმა კუჭის ანთებაო, ნაღვლსი მომატებული რაოდენობაო და მაღალი მჟავიანობაო დამისვა დიაგნოზად. ზედაპირული გასტროდუოდენტი (მგონი სწორად მახსოვს)

ნოემბრის დასაწყისიდან დავიწყე დიეტა, კვება გრაფიკით, დილით ულსეპანს ვღებულობდი, სიმალგელის სიროფს ჭამის წინ, 40 დღის მერე ლანსოლით ჩავანაცვლე ულსეპანი. ბევრად უკეთესად ვგრძნობდი თავს, თუ არ გავითვალისწინებთ იმას რო კვირაში ერთხელ მაინც მაწუხებდა საღამოობით გულისრევის შეგრძნება. და ამავდროულად ძალიან მოვიკელი.












მივედი კითხვამდე
მოვრჩი მკურნალობას.
გავიდა 2 თვე
და 30-ში ღამით მაინც ცუდად გავხდი
არადიეტური არაფერი მიჭამია არადა
ისევ კუჭში სიმძიმე მაქვს
წვა დამეწყო
თითქოს რაღაც მიჭერს კუჭზე და მექაჩება




ძალიან ვნერვიულობ
ნერვებსაც ვეღარ ვთოკავ
მგონია რაღაც საშინელება მჭირს, რატომ ვერ გამოვჯანმრთელდი sad.gif
ოდნავ ცუდად ვიგრძნო თავი და სულ ცუდე ვფიქრობ.





მაინტერესებს
ასეთი ჩივილებით რას მირჩევთ
მივიდე სხვა გასტროლოგთან
კიდევ ნევროპათოლოგთან?
თუ რავქნა?


ძალიან განვიცდი უკვე
დეპრესიის ნიშნებიც მაქვს
ეს 3-4 დღეა წესიერად არაფერი მიჭამია, იმდენად მეშინია რო შემაწუხებს
ისედაც მეწვის და მექაჩება კუჭი და რაღააც რო ვჭამო მგონია უარესად მაწყენს.

მოკლედ გელოდებით კომპეტენტურებო
მირჩიეთ რამე.

Natuka NGN

მგონი შენ უფრო შეგიძლია კითხვაზე მიპასუხო
იქნებ კარგი გასტროლოგის ან ნევროპათოლოგის კოორდინატები მომცე და
მირჩიო როგორ მოვიქცე.


მადლობა მას, ვინც ამდენი წაიკითხა smile.gif

ჩემი დაწერილი პოსტი მეგონა და ტირილი დავიწყე შუა კითხვისას cry.gif

მეც ნევროპათოლოგთან გამიშვეს არადა კუჭშია აშკარად პრობლემა, ზუსტად იგივე სიმპტომები მაქვს... მაგრამ ზონდის გადაყლაპვას სიკდვილი მირჩევნია user.gif მშიშარა ვარ

ყოველთვის პრობლემა მაქვს საჭმელზე, ისედაც სუსტი ვარ, კვება არც თუ ისე კარგი მქონდა. ჩიფსზე და კოკა-კოლაზე ვიყავი მთელი გასული ზაფხული...

ასე მგონია რაღაც მიჭერს და ქვემოთ მექაჩებათქო user.gif თან ჰაერის უკმარისობასაც იწვევს და გულსაც საშინლად მიჩქარებს.

ჰიპოქსია თუ რაღაც მითხრა კარდიოლოგმა და წამლებსაც ვსვამდი ერთი თვე მაგრამ რა აზრი აქვს კუჭს თუ არ მივხედე სხვა ყველაფერი აქედან არის გამოწვეული და არც ისე კრიტიკულ ფაზაშია სავარაუდოდ.

ვცდილობ დავიკიდო და სხვა რამეზე გადავიტანო ყურადღება, ძირითადად გამომდის კიდეც მაგრამ როცა ძალიან შემაწუხებს უკვე მოწყვეტილი ვარ გარესამყაროს, მეშინია და ათას სისულელეზე ვფიქრობ მაგ დროს cry.gif

მე რა მეშველება არ ვიცი მაგრამ ვისაც ოდნავ მაინც კი აწუხებს რამე თავს მიხედოს ყველას ვურჩევ სულ არ გაირთულოთ საქმე cry.gif ნეტა მეც მიმეხედა დროზე, ჯანმრთელობის გარდა არაფერი მინატრია ცხოვრებაში

ამაღამ თუ გადავრჩი შემოვირბენ ხვალ და იმედია რამე რჩევას მომცემთ cry.gif ცივია მტკვარი თან ცურვაც ვიცი...

Posted by: gvelis_wiwila 12 Apr 2011, 13:06
ესეიგი მაქვს რევმატიზმი სავარუადოდ ჩირქიანი გლანდების მერე და ვმკურნალობ ბიცილინით.


გუშინ გავიკეთე და თითქოს ოდნავ გამიჭირდა სუნთქვა (კაცმა არ იცის შეიძლება ნევროზულიც იყო) და ეგრევე დექსამეტაზონი და სუპრასტინის ნემესები დამარჭეს.

იმის მერე ძლივს გავედი საავადმყოფოდან მაბჟუალებდა და გათიშვის შეგრძNება მქონდა და მთელი დღეც კაიფში ვიყავისავით.

ჰოდა აი დღეს ავდექი და ისევ მაქვს ეს ბჟუალი და გათიშვის პონტი და შეიძლება რომ დღესაც ქონდეს ეფექტი? ხომ გაივლის მალე? biggrin.gif

თან მაკანკალებს უაზროდ ხელები განსაკუთრებით

და ალერგია ვეღარ მექნება ხო რახან გუშინ გამიკეთეს ესენი?

Posted by: inna82 12 Apr 2011, 14:12
ხველების საწინააღმდეგოდ გამოგვიწერეს (ბავშვი 2 წლის) სოლტუქსი, რა შეიძლება მივცე მაგის შემცვლელი. ნოტაცია რომ წავიკიტხე განსაკუტრებული არაფერია ფასი კი იცოცხლე... 34,5 ლარი. ჩვეულებრივი მუკოლიტიკია. ხველა არის ოდნავ სველი, ცოტა სპაზმურს ჰგავს, სიცხე არააქვს, ხშირი არაა. როგორც გვითხრა ექიმმა უფრო ალერგიული ფონი აქვს. მირჩიეთ რამე ანალოგიური
მოქმედების მედიკამენტი.

და კიდევ შეკიტხვა, უფროს გოგოს ხველებისას ვასმევ ლიბექსინს და ძალიან უხდება და ახლა პატარას რომ მივცე ასეთი ხველების სიმპტომების შემთხვევაში არის ლიბექსინი თუ არა შესაბამისი მედიკამენტი?

Posted by: Megi-meghan 14 Apr 2011, 13:52
დაახლოებით ერთი თვეა რაც სიცხე მაქვს.თამაშობს 36.8-სა და 37.5-ს შორის და თან თავი მტკივა.არც ყელი მტკივა,არც მახველებს.უბრალოდ ცხვირი მეწვის ხოლმე.თუ შეგიძლიათ მითხრათ რისი ბრალი შეიძლება იყოს ეს?წინასწარ დიდი მადლობა

Posted by: Night_Princess 16 Apr 2011, 21:45
კარგი იქნება ვინმე თუ მიპასუხებს.

ყური მტკივა ცოტა, თითქოს გავლითი ტკივლივითაა, გაჩერდება, ისევ იწყებს,
წყალი არ ჩამსვლია და არაფერი, უბრალოდ ცოტა სურდო მქონდა, და რა შეიძლება რო ჩავიწვეთო?

Posted by: Hana Arendtiii 16 Apr 2011, 22:02
გთხოვთ ვინმე გამომეხმაურეთ, ვისაც ცემნაირი სიმპტომები გაქვთ(გქონდათ).
ერთი კვირა მტკიოდა ყელი მხოლოდ დილით, შემდეგ გამიარა. ორი დღის წინ ისევ დამეწყო ყელის ტკივილი, ამჟამად ძალიან მაგრად მახველებს(რომ მახველებს მტკივა გულის არეში)და მაქვს სურდო. სიცხე დაბალი ან საერთოდ არ მაქვს. ეხლა დავიწყე აზიმაკისდალევა smile.gif

Posted by: Megi-meghan 17 Apr 2011, 14:28
დაახლოებით ერთი თვეა რაც სიცხე მაქვს.თამაშობს 36.8-სა და 37.5-ს შორის და თან თავი მტკივა.არც ყელი მტკივა,არც მახველებს.უბრალოდ ცხვირი მეწვის ხოლმე.თუ შეგიძლიათ მითხრათ რისი ბრალი შეიძლება იყოს ეს?წინასწარ დიდი მადლობა

Posted by: almada 17 Apr 2011, 20:54
http://www.radikal.ru
ხელი რომ უდევს იმ ადგილას ხელის მირტყმისას ტკივილს ვგრძნობ შიგნიდან, და რა შეიძლება იყოოს??

ბეჭების და მთლიანად ზურგის ტკივილიც მაწუხებს, მაგრამ ეგ სხვა საკითხია მგონი

ძალიანაც არ შემაშინოთ

Posted by: almada 18 Apr 2011, 21:21
მიპასუხეთ რაა ...........user.gif
* * *
QUOTE
მოკლედ მაქვს ბოლო თვეა ესეთი სიმპტომი - ვერ გამოვხატავ სიტყვებით წესიერად მაგრამ ისეთი შეგრძნებაა თითქოს ცხელი სითხე მოძრაობს გულთან ახლოს. ოღონდ მოძრაობს ნელა ძალიან და ხანდახან მახსენებს თავს. საკმაოდ უსიამო შეგრძნებაა user.gif

აუ ეს ბოლო დღეებია მეც ეგეთი შეგრძნება მაქვს
დღეს შემეშინდა ლექციაზე გული არ წამსვლოდა
QUOTE
ალბათ ნერვოზია !!! ნერვოზ ცჰვევია ეგეტი რაგაცეები...ტიტკოს და გეცჰვენება რომ რაგაც გცჰირს ან კიდე ბევრი ფიქრი მოგიწია იმაზე რომ თითქოს რაგაც გემართება...

მეც ეგ დავასკვენი

Posted by: Megi-meghan 20 Apr 2011, 17:58
დაახლოებით ერთი თვეა რაც სიცხე მაქვს.თამაშობს 36.8-სა და 37.5-ს შორის და თან თავი მტკივა.არც ყელი მტკივა,არც მახველებს.უბრალოდ ცხვირი მეწვის ხოლმე.თუ შეგიძლიათ მითხრათ რისი ბრალი შეიძლება იყოს ეს?წინასწარ დიდი მადლობა

Posted by: K_rati 21 Apr 2011, 16:31
Megi-meghan
გამონადენი გააქვთ ცხვირიდან? ალერგიული ხართ?

Posted by: Megi-meghan 21 Apr 2011, 20:44
არა გამონადენი არ მაქვს.დიახ ალერგიული ვარ.შეიძლება რომ ჯირკვლების ბრალი იყოს?

Posted by: K_rati 21 Apr 2011, 21:42
Megi-meghan
უფრო ალერგიული რინიტის ბრალი უნდა იყოს. სავარაუდოდ რაღაც გარემო ფაქტორზე გაქვთ ალერგია, რომელიც გაზაფხულის დადგომასთან უნდა იყოს კავშირში, ვინაიდან დაახლოებით ერთი თვეა რაც დაგეწყოთ. ადრე არ გქონიათ მსგავსი შემთხვევა?

Posted by: gramatikikina 21 Apr 2011, 23:28
ხელის მტევნეში და თითებში გაბუჟების შეგრძნება, თუ გრძნობის დაკარგვა რისი ბრალი უნდა იყოს?
ადრე სიცივეში ემართებოდა და ეხლა თბილოდა, თან უფრო ძლიერად და დიდ ხანს ჰქონდა ვიდრე ადრე.

Posted by: K_rati 21 Apr 2011, 23:35
gramatikikina
სიცივეზე დაბუჟება შეიძლება რეინოს სინდრომის ბრალი იყოს, ყურებიც ან ცხვირიც ხომ არ უცივდება ხოლმე?
რა პროფესიის არის? ხომ არ გადატვირთა ხელი?

Posted by: gramatikikina 22 Apr 2011, 00:52
K_rati
QUOTE
ხომ არ გადატვირთა ხელი?

არა ეგეთი არაფერი. ორივე ხელის თითები ორი დღეა ეგრე აქვს. ძალიან ბევრ ლუდს სვამს და მაგის ბრალი ხომ არ იქნება?
ყურები და ცხვირი არ ვიცი.

Posted by: Natuka NGN 22 Apr 2011, 02:25
gvelis_wiwila
ახლა ვნახე. შეიძლება გქონდეს ალერგია - ბიცილინი არის გახანგრძლივებული მოქმედების პენიცილინი, ამდენად მისი მოქმედების ვადა როგორც ვიცი ერთიი თევა. ამიტომ ნუ შეგეშინდება. იცის მასე.

Posted by: gvelis_wiwila 22 Apr 2011, 16:19
QUOTE
ამდენად მისი მოქმედების ვადა როგორც ვიცი ერთიი თევა. ამიტომ ნუ შეგეშინდება. იცის მასე.


ოკ smile.gif


თან პახმელიაზე ვიყავი რომ გავიკეთე და ალბათ ეგეც იყო smile.gif

Posted by: Tess 22 Apr 2011, 17:18
გამარჯობა

ძალიან გთხოვთ დამეხმარეთ
ჩემს მეგობარს სავარაუდოდ თმა ჩაუბრუნდა, თანაც უხერხულ ადგილას
დაახლოებით უკანა ტანზე ოღონდ ღრმად, როგორც გავიგე ასეთი რამ მამაკაცებს ემართებათ ხოლმე.
სიმაგრე აქვს და თავი არ აქვს გაკეთებული
ტკივილები აქვს სიარულის დროს
რა შეიძლება იყოს ან რა უნდა იხმაროს ან რომელ ექიმთან მივიდეს ?

ვინმემ თუ იცით რამე მითხარით გთხოვთ
მადლობა წინასწარ

Posted by: Natuka NGN 23 Apr 2011, 02:15
Tess
http://forum.ge/?showtopic=34077539&hl=%E1%83%97%E1%83%9B%E1%83%98%E1%83%A1+%E1%83%A9%E1%83%90%E1%83%91%E1%83%A0%E1%83%A3%E1%83%9C%E1%83%94%E1%83%91%E1%83%90+%E1%83%99%E1%83%A3%E1%83%93%E1%83%A3%E1%83%A1%E1%83%A3%E1%83%9C%E1%83%98%E1%83%A1+%E1%83%AE%E1%83%95%E1%83%A0%E1%83%94%E1%83%9A%E1%83%A8%E1%83%98
თემაა მთელი, გაადხედე

Posted by: TETISA 23 Apr 2011, 16:21
ჩემს შვილს მეორე დღეა შუბლი სტკივა. კერძოდ, მარჯვენა მხარე, დაახლოებით თვალის შიდა კუთხის თავზე. სიამოვნებს ცივის მიდება, მე სულ ცივი მაქვს ხელები და მთხოვს მიდებას. თვითონ არ ამობობს, მაგრამ მგონი წარბის ქვემოთაც სტკივა. რატომღაც შუადღეს ეწყება ტკივილი. გუშინ დაიძინა და გადაუარა, დღეს კი სადღაც 15 წუთში ნელ-ნელა გაუქრა.
5 წლის ბიჭია.
ჰაიმორიტი ხომ არ შეიძლება იყოს? მაგრამ მგონი ეგ ჯირკვალი ერთი მთლიანია შუბლის შუა ნაწილში და შეიძლება, რომ ნახევარი სტკიოდეს? 2-3 დღის წინ ავადაც იყო: სიცხეები ქონდა 38-38,5. ორი თუ სამი დღის მანძილზე გამოკეთდა და ცხვოროდან დარჩა ცოტა გამონადენი და ოდნავი ხველება.
როგორ ფიქრობთ, რა შეიძლება აწუხებდეს? რამდენადაა ეს ჯირკვალი? სხვა რა არის მაგ იდამოებში?

Posted by: rock in rose 23 Apr 2011, 16:25
TETISA

QUOTE
ჰაიმორიტი ხომ არ შეიძლება იყოს?


ჰაიმორიტი არა, ფრონტიტი. შუბლის წიაღის ანთებას ფრონტიტი ქვია

QUOTE
რამდენადაა ეს ჯირკვალი?


შუბლის მიდამოში ჯირკვალი არ არის არაფერი

თავს რომ ხრის ქვემოთ, შუბლზე დაწოლის შეგრძნება ხომ არ აქვს?

Posted by: TETISA 23 Apr 2011, 22:23
rock in rose
ხო, ფრონტიტია, ამერია. და ფრონტიტი რისი ანთებაა?
QUOTE
თავს რომ ხრის ქვემოთ, შუბლზე დაწოლის შეგრძნება ხომ არ აქვს?

არამგონია. ათასნაირ სალტოებს აკეთებს.
რაღაც მომენტში ასტკივდება ძალიან, და ცოტა ხანში მთლიანად გადასდის როგორც ჩანს ტკივილი, იმიტომ, რომ ჩვეულებრივად აგრძელებს ყველაფრის კეთებას: დახტის, დაძვრება და ა.შ.

Posted by: ooo,girl,girl 23 Apr 2011, 22:39
აუ, 2 კვირაა ყელი მტკივა ნერწყვის გადაყლაპვვისას, თაფლიან რძეს ვსვამ,ტყლაპს, ჩაის,ყელის ტკივილის საწუწნი კამფეტივით რომ არის იმას ვწუწნი, ისევ ისე ვარ, ხან თითქოს გამივლის,მარა .... სასა ხომ არის შუაში,ხოდა მე ცალ მხარეს მაქვს ცუდი შეგრძნება ნერწყვის გადაყლაპვისას, ზუსტად ვერ აღვწერ, ხან თითქოს სიმშრალის,ხან რავიცი მოკლედ ვერ ვამბობ. ვინმეს გქონიათ მსგავსი? ამდენი ხნით არასდროს მტკიებია და ცოტა შემეშინდა sad.gif

Posted by: rock in rose 24 Apr 2011, 14:02
TETISA
QUOTE
ფრონტიტი რისი ანთებაა?


შუბლის წიაღის ანთებაა

QUOTE
არამგონია. ათასნაირ სალტოებს აკეთებს.
რაღაც მომენტში ასტკივდება ძალიან, და ცოტა ხანში მთლიანად გადასდის როგორც ჩანს ტკივილი, იმიტომ, რომ ჩვეულებრივად აგრძელებს ყველაფრის კეთებას: დახტის, დაძვრება და ა.შ.


რასაც აღწერთ ფრონტიტს გავს, მაგრამ ექიმს აჩვენეთ smile.gif

პ.ს გილოცავთ აღდგომას 2kiss.gif

Posted by: kudobakan 24 Apr 2011, 14:15
გამარჯობა
მაინტერესებს რისი ბრალი შეიძლება იყოს ის, რომ სარკეში ჩახედვისას ერთი თვალი მეორეზე დიდი ჩანს (ეს განსხვავება არ არის პატარა, საკმაოდ დიდია ერთი თვალი მეორეზე) და რეალურად, როცა ვინმეს ვთხოვ რომ შემომხედოს და მითხრას მართლა ასეა თუ არა, განსხვავებას ვერ ამჩნევენ.. ეს უბრალოდ ვიზუალური ეფექტის ბრალია თუ თვალებზე რამე მჩირს? biggrin.gif

Posted by: TETISA 24 Apr 2011, 16:27
rock in rose
დიდი მადლობა. დავაკვირდები და თუ არ გაუარა, მივიყვან.
წარმატებები

Posted by: Tess 25 Apr 2011, 19:11
Natuka NGN
მადლობა
ვნახე ეგ თემა უბრალოდ ვერ მივხვდი ის ადგილია თუ არა რაც მე მაინტერესებდა smile.gif

Posted by: audrey-girl 29 Apr 2011, 11:04
გამარჯობა.
გუშინ დილით საშინელმა ყელის ტკივილმა გამაღვიძა, უფრო სწორად ყელი კი არა მარტო ტონზილები მტკიოდა, მერე ვნახე რომ საშინლად დაწითლებული მაქვს. დღეს კიდევ თითქმის მთელი ყელი მტკივა და რამე წამალს ხომ ვერ მირჩევთ. მე საწუწნი აბები მაქვს მარტო მაგრამ ეგენი აშკარად არ მშველის, და იქნებ რამე შესასხურებელი წამალი მეცადა. :შ

Posted by: leopardibuba 29 Apr 2011, 12:00
ყავას ან ჩაის რომ ვსვამ უზმოზე მსხვილი ნაწლავი მარჯვნივ,ზევით ცოტა მტკივდება. რატომ ხომ ვერ მეტყვით?

Posted by: K_rati 29 Apr 2011, 15:52
audrey-girl
ტონზილები მარტო შეწითლებულია თუ თეთრი ნადებიც აქვს? ცხელება გააქვთ?

Posted by: Neo777 30 Apr 2011, 00:35
K_rati
QUOTE
ტონზილები მარტო შეწითლებულია თუ თეთრი ნადებიც აქვს? ცხელება გააქვთ?

ანგინა არ არის, არც თეთრი ნადები აქვს და არც მაღალი ტემპერატურა.
აქვს სიწითლე და საშინელი ტკივილი.

პ.ს მე ვწერ მის მაგიერ.

Posted by: gvelis_wiwila 30 Apr 2011, 00:39
მეგობრებო როგორ გავარჩიო ნაღვლის ბუშტი მტკივა თუ ბრმანაწლავი ?

Posted by: rock in rose 30 Apr 2011, 19:43
gvelis_wiwila

სხვადასხვა ადგილასაა ორივე, თან მკვეთრად

http://pix.ge/

ქვემოთა წითელ ხაზზე ბრმანაწლავია, ხოლო ზემოთა წითელ ხაზზე კი ნაღვლის ბუშტი

Posted by: nodo77 1 May 2011, 15:53
გამარჯობათ,

ერთი თვეა მაწუხებს ლიმფური ჯირკვლების ყრუ ტკივილები , ყელში და იღლიების მიდამოებში, სიმპტომამდე ვიყავი გაციებული, სწორედ ამ პერიედის შემდეგ დამეწყო ჩივილები, რა შეიძლება იყოს და ვის ეხება ეს საკითხი, მირჩიეთ რამე!?

Posted by: leopardibuba 1 May 2011, 15:55
გუშინ ნაღვლის ბუშტი მებლატავებოდა და რა ვქმნა, მიშველეთ რამე. help.gif help.gif help.gif help.gif help.gif help.gif help.gif help.gif help.gif weep.gif

Posted by: Neo777 3 May 2011, 02:20
QUOTE (audrey-girl @ 29 Apr 2011, 11:04 )
გამარჯობა.
გუშინ დილით საშინელმა ყელის ტკივილმა გამაღვიძა, უფრო სწორად ყელი კი არა მარტო ტონზილები მტკიოდა, მერე ვნახე რომ საშინლად დაწითლებული მაქვს. დღეს კიდევ თითქმის მთელი ყელი მტკივა და რამე წამალს ხომ ვერ მირჩევთ. მე საწუწნი აბები მაქვს მარტო მაგრამ ეგენი აშკარად არ მშველის, და იქნებ რამე შესასხურებელი წამალი მეცადა. :შ

help.gif help.gif help.gif help.gif help.gif help.gif help.gif

Posted by: ravi_shen 3 May 2011, 10:00
ფსიქიატრიული სად გადაიტანეს ხომარავინ იცით
სერიოზულად მჭირდება ყოველგვარი ღადავის გარეშე

Posted by: MALI 4 May 2011, 09:50
გამრჯობა
პაციენტს ფაღარათი დაეწყო ტემპერატურის მატებით, რომელიც გაგრძელდა 3 დღე მხოლოდ, ფაღარათი კი დღემდე გრძელდება .
განავლის ანალიზში აღმოჩნდა ლეიკოციტები
დიაგნოზია ბაქტერიული დიარია.
დანიშნულეა ასეთი იყო:
5 დღე ციპრო, 14 დღე მეტრონიდაზოლი და დიეტა.

დღეს უკვე მე-8 დღეა და ფაღარათი კი აღარ არის, მაგრამ მუცლის არეში ტკივილი და გვრემის მოვლენები აღინიშნება ისევ..
მაინტერესებს ასე დიდხანს იცის გაგრძელება ამის მკურნალობამ?
თუ, შესაძლებელია რომ მკურნალობის კურსი მცდარია?
მადლობა

Posted by: loveless 4 May 2011, 20:01
ფეხმძიმე ვარ და შარდში ლეიკოციტები აღმომაჩნდა 22, ნაცხში გრ(+)კოკი -დიდი რაოდენობით.
აუცილებელია თუ არა დამატებითი ბაქტერიოლოგიური გამოკვლევა?

Posted by: audrey-girl 4 May 2011, 22:32
სისხლიანი ნახველი შეიძლება ტონზილების გამო იყოს? იქნებ ვინმემ მიპასუხოს რაა. sad.gif

Posted by: მეტივე 5 May 2011, 19:19
მინდა გამოვიკვლიო ნაწლავთა სისტემა, არ ვიცი სამედიცინო ენაზე რა ჰქვია ამას. კუჭი არა, იმენა ნაწლავები.
მაქვს მეტეორიზმი და ფაღარათი.

ხოოდა:
1) რა ჰქვია ამის ექიმს? რა უნდა ვიკითხო კლინიკებში? smile.gif
2) ხომ არ იცით ვინმე კარგი ექიმი ამ სფეროდან?
2) დაახლოებით რა ღირს ექიმთან კონსულტაცია და განავლის ანალიზი? user.gif

Posted by: Lacrimosa_ 5 May 2011, 19:39
მეგობრებო, უცნაური რაღაც მჭირს, 3 დღის განმავლობაში მქონდა სურდო, მეოთხე დღეს უცებ აღარ მქონდა სურდო და მას მერე ყნოსვა დავკარგე ნაწილობრივ sad.gif
მირჩიეთ რამე რა.

Posted by: audrey-girl 5 May 2011, 20:26
მეტივე
მე გირჩევ თაქვიდან თერაპევტთან მიხვიდე, შემდეგ გაგაზავნის გამოკველევებზე, ან ანამნეზის შეკრებით დაგინიშნავს მკურნალობას.
თუ არ გინდა თერაფევტთან ვიზიტი მაშინ მიდი გასტრო-ენტეროლოგთან. მე თოდუაში გირჩევდი გურგენიძეს. ცოტა უხეში ექიმია მაგრამ კარგი სპეციალისტია.
კონსულტაცია 25 ლარი ღირს მე როგორც ვიცი. აი რაც შეეხება განავლის ანალიზს არამგონია საჭირო იყოს. და არც ვიცი რა ჯდება გამოკვლევა.

* * *
Lacrimosa_
სურდოს დროს ყნოსვის დაკარგვა არ არის გასაკვირი, გააჩნია რამდენი ხანია რაც ეს გრძელდება.
რას ნიშნავს ნაწილობრივ დაკარგე ყნოსვა?

Posted by: Lacrimosa_ 5 May 2011, 21:39
audrey-girl
წინა ხუთშაბათიდან საერთოდ არა მაქვს სურდო, და ნაწილობრივ არც ყნოსვა.
ნაწილობრივ დავკარგე რას ნიშნავს და კონკრეტულად რაიმეს რომ დავსუნავ, მაშინ კი ვგრძნობ სურნელს, მაგრამ ზოგადად უსუნო გარემოში ვარ თითქოს ისე ვგრძნობ თავს.
ერთი მხრივ, დავისვენე, იმიტომ, რომ საერთოდ ძალიან არ მიყვარს სუნები, სუნამოს სუნიც კი smile.gif

Posted by: almada 5 May 2011, 21:56
http://forum.ge/?f=43&showtopic=34203346&st=135
მიპასუხეთ რაა

Posted by: audrey-girl 5 May 2011, 21:57
Lacrimosa_
ყნოსვასთან ერთად გემოვნების დაქვეითებაც ხომ არ გაქვს?
QUOTE
სუნამოს სუნიც კი

love.gif
* * *
almada
ტკივილის გარდა სხვა რაიმე ჩივილი გააქვს?

Posted by: rock in rose 5 May 2011, 22:14
almada

ხერხემლის პრობლემა შეიძლება იყოს

Posted by: audrey-girl 5 May 2011, 22:17
rock in rose
QUOTE
ხერხემლის პრობლემა შეიძლება იყოს


მე თირკმელს ვფიქრობ, ხერხემალის პრობლემის დროს მანდ კი არა შუაში უნდა იყოს ტკივილი.
almada
შარდის მხრივ რაიმე ცვლილებები ხომ არ შეგინიშნავს? ფერი, სუნი.... გამოყოფის სიხშირე? არტერიული წნევა როგორ გაქვს?

Posted by: rock in rose 5 May 2011, 22:25
audrey-girl

QUOTE
მე თირკმელს ვფიქრობ, ხერხემალის პრობლემის დროს მანდ კი არა შუაში უნდა იყოს ტკივილი.


თირკმელი უფრო ქვევითაა, თან ხელი ზუსტად შუაში უდევს

Posted by: Hektorinia 6 May 2011, 00:50
მჭირდება რევმატოლოგის კონსულტაცია. ვიცი, რომ არსებობს რევმატოლოგიის ცენტრი, მაგრამ იქნებ მირჩიოთ კონკრეტულად რომელ ექიმთან მივიდე.

მადლობა წინასწარ

Posted by: Lacrimosa_ 6 May 2011, 20:21
audrey-girl
QUOTE
ყნოსვასთან ერთად გემოვნების დაქვეითებაც ხომ არ გაქვს?

არა, გემოვნებას ნამდვილად არ ვუჩივი.
არც დღეს დამიბრუნდა ჩემი ყნოსვა, რა ჯანდაბა დამემართა ვერ ვხვდები

Posted by: almada 6 May 2011, 21:31
audrey-girl
rock in rose
მადლობა გამოხმაურებისთვის smile.gif
არა შარდზე ცვლილებებს ვერ ვატყობ
ბეჭების და ზურგის ტკივილები ბავშვობიდან მაწუხებს , შარშან რევმატოლოგთანაც ვიყავი,(მარცხენა ხელ-ფეხის გამუდმებული ყრუ რევმატიული ტკივილიც მაწუხებს) რენდგენი გადამიღო,მაგრამ ისეთი სერიოზული არაფერი არ უთქვამს
რევმატიზმი და რამე ძვლოვან ცვლილებები არ გაქვსო, მყესი გაქვს გრძელი და ჰიპერმობილი ხარო. წამლები გამომიწერა და არ მიშველა საერთოდ, წელს, ამ ბოლო თვეებში უკვე გარეგნულადაც მტკივა ზურგი, მაგალითად მაგარ სკამს რომ ვეყუდები.
მოკლედ დამტანჯა ტკივილებმა vis.gif

p.s მოძრაობაში ტკივილი ხელს არ მიშლის , პირიქით შეიძლება დილას უფრო მტკიოდეს

ხო იმ კვირას ისევ რევმატოლოგთან ვაპირებ მისვლას

Posted by: audrey-girl 6 May 2011, 23:04
Lacrimosa_
საერთოდ გრიპის დროს ყნოსვის რეცეპტორების ფუნქცია ქვეითდება ხოლმე, და საგანგაშო არ არის. ადრეც ხომ არ გქონია მსგავსი ჩივილი?
იმ კვირამდე ნახე როგორ გექნება და მერე ეწვიე ოტოლარინგოლოგს. მეტს ვერაფერს გირჩევ მე პირადად. თან არც ეგ სპეციალობა გამივლია ჯერ ამიტომ მაგ მხრივ ჩემი ცოდნა თითქმის ნოლის ტოლია.

* * *
almada

QUOTE
მყესი გაქვს გრძელი და ჰიპერმობილი ხარო

რომელი მყესი? სად?
მე ევრაფერს გირჩევ იქნებ rock in rose-მა გირჩიოს რამე.
ისე ექიმთან რომ მიხვალ თუ არ დაგეზარება დაწერე რას გეტყვის smile.gif

Posted by: Hektorinia 7 May 2011, 23:24
almada
QUOTE
შარშან რევმატოლოგთანაც ვიყავი,(მარცხენა ხელ-ფეხის გამუდმებული ყრუ რევმატიული ტკივილიც მაწუხებს)

კონკრეტულად ვიშტან იყავი და სად? შეგიძლია მითხრა?

რახანია, ვეძებ კარგ რევმატოლოგს და ჯერ ვერ ვიპოვე

Posted by: almada 8 May 2011, 12:35
Hektorinia
QUOTE
კონკრეტულად ვიშტან იყავი და სად? შეგიძლია მითხრა?

რევმატოლოგიურ ცენტრში ვიყავი, ქეთევან ონიანთან ,
თუმცა რისი ბრალია არ ვიცი, ,მაგრამ ტკივილები საერთოდ არ გამყუჩებია
ამიტომაც ვაპირებ ისევ მისვლას

Posted by: Hermione Jean Granger 8 May 2011, 15:56
QUOTE (gramatikikina @ 31 Dec 2010, 12:05 )
აუჰ, რამდენი კითხვებია ბოლო გვერდზე და არც ერთი პასუხი, მარა მაინც დავწერ




მეზიმფორტე რომ ვსვა(სანამ ექიმთან მივალ), რამე ხომ არ დაშავდება? სავარაუდოდ კუჭი მაწუხებს. ღვიძლი და ნაღვლის ბუშტი გამოვიკვლიე, ანუ არაა ეგენი. კუჭი ვერ მოვასწარი fig.gif

მეზიმფორტე არის ფერმენტი რომელლიც დაგეხმარება მონელებაში . კუჭის სამკურნალოდ არ გამოგადგება მაგრამ ზიანსაც არ მიიღებ>

* * *
QUOTE (blblbl @ 28 Dec 2010, 23:19 )
ფილტვების ძლიერი ანთების დროს,წყალიც იდგა ფილტვებში,სისხლი შეიძლება ამოყვეს ნახველს?

რაფერია სისხლი? თუ ალისფერია შეიძლება კაპილარი გასკდა . მაგრამ მაინც აუცილებლად გადაიღე რენდგენი და მიმართე ფტიზიატრს ტუბ-ის გამოსარიცხად

Posted by: nit123 10 May 2011, 11:55
ხალხნო, ჩემს მეგობარს მშობიარობის მერე, აქვს წნევები, 180/120-ზე....
არადა ორსულობის დროს საერთოდ არ ქონდა წნევა, პირიქით, დაბალ წნევას ატარებს.
ყოველს საღამოს, 6-ის მერე უწევს წნევა, წამალს ვერ სვამს ბავშVის გამოპ, მარა რასაც სვმას, (ნოშPა, პაპაზოლი და .აშ>) არ შველის
გამოიკვლიეს ყველაფერი, გული, თირკმელი, რავიცი მარა ვსო ოკ
რისი ბრალია ნეტა???
ექიმები მხრებს იჩეჩავენ (როგორც ყოველთვის) ალბათ ჰორმონალურიაო...
ან ენრვოზულიო
ნუ მოკელდ, გოგო გიჟადაა ფაქტია

Posted by: almada 11 May 2011, 20:32
ვიყავი რევმატოლოგთან და იგივე მითხრა(ანუ სერიოზულ არაფერიო, თუმცა მცირე ნერვის ანთება , მცირე სქოლიოზი,ჰიპერმობილობა ) ცურვა გამომიწერა smile.gif ანალიზები აღარ ამიღია.. ანოდინ დუალი და რამდენიმე წამალი გამომიწერა და ამანაც თუ არ მიშველა და ცოტათი მაინც არ გამიყუჩა ტკივილებიი vis.gif

პ.ს ოლიფენი ხომ არ გაგიგიათ წამალი? ავერსში არ ჰქონდათ და ვერც გუგლმა მაპოვნინა

Posted by: GIO BIO 11 May 2011, 22:52
შეკითხვა მაქვს ექიმთან: ძილად ვარ მივარდნილი ( მაგარი გამოთქმაა არა ?) მაგრამ მართლა ასეა. სულ მინდა რომ მეძინოს. თან ეს ძილი რაგაცნაერად არაჯანსაღი ძილია. რომ მეღვიძება პირი მაქვს გამომშრალი და საშინელი გემო მაქვს პირში.სანამ რაგაცას არ შევჭამ ტკბილს და წვნიანს ენერგია არ მაქვს საერთოდ. თუ რაღაცას ვაკეტებ ანუ თუ დავდივარ ან რაიმეს ვაკეთებ ფიზიკურად მაშინ ვერ ვგრძნობ,დავჯდები ტუ არა ეგრევე მეძინება.ბებერი ახალგაზრდა ვარ მგონი.ენერგია არ მაქვს სულ მინდა რო დავჯდე და დავისვენო.
რა იწვევს ასეთ მოვლენას ?? არ ვიცი რის ექიმთნ მივიდე არადა ვგრძნობ რომ რაგაც სერიოზულად მენძრევა, თანაც რაც დრო გადის უფრო მემატება. ეს დამეწყო რამდენიმე წლის წინ.მაგრამ მემართებოდა ძალიან იშვიათად.ერტი ორი დღით. ნელ ნელა უმატა,უმატა და ეხლა სულ ასე ვარ.
მირჩიეთ რომელ ექიმთან მივიდე ,რა ორგანოს გამოკვლევები გავაკეთებინო?

დიდი მადლობა წინასწარ. იმედია გულთან მიიტანთ ჩემს გასაჭირს და მიპასუხებთ. ალბათ კითხვებიც გაგიჩნდებატ დიაგნოზის დასმამდე.მზად ვარ გიპასუხოთ კიდეც.

Posted by: audrey-girl 11 May 2011, 23:52
GIO BIO
QUOTE
სულ მინდა რომ მეძინოს. თან ეს ძილი რაგაცნაერად არაჯანსაღი ძილია

ეს დღის განმავლობაში რომ გძინასვ მაშინ, თუ დილით რომ გეღვიძება მასინაც მასეა?
საერთოდ როგორი დღის რეჟიმი გაქვს, როგორი სამუშაო გრაფიკი? ან საერთოდ რამდენად ასრულებ ფიზიკურ სამუშაოს?
QUOTE
ეს დამეწყო რამდენიმე წლის წინ.მაგრამ მემართებოდა ძალიან იშვიათად.ერტი ორი დღით. ნელ ნელა უმატა,უმატა და ეხლა სულ ასე ვარ.

ამ ორი წლის განმავლობაში სხვა რაიემ ჩივილი ხომ არ გქონია?
წონაში ხომ არ მოგიმატია?

Posted by: GIO BIO 12 May 2011, 12:45
ეს სულ ასეა დღეც და დილასაც.ფიზიკურად არ ვმუშაობ თუ რაგაცის გაკეტება მიწევს ზალიან მსუბუქი და იშვიათად.წონაში მოვიმატე ცოტა 70 ვიყავი და 83 ვარ, ნუ + -. თუ მომატება ქვია ამას.მსუქანი არ ვარ.უფრო ასაკისგან მოვიმატე მგონი.
თქვენ ექიმი ხართ??
ღვიძლმა ტუ იცის ასეთი სიმპტომები? ან ნაღველმა, ან ელენთამ. მე სადGაც აქ მგონია ჩემი პრობლემა. ეხოსკოპია გამიკეთეს ღვიძლზე და მითხრეს ქონი გაქვსო გარშემოო ტუ რაგაც, ნორმაზე ბევრად მეტიო.
ც Gეპატიტი არ მაქვს არც ბ და საერტოდ არ მქონია არანაერი გეპატიტი.ნაღვისბუშტი ძAლიან სუსტი მაქვს პატარა ცხიმიან ან ნიგვზიან საჭმელზე მკლავს მძმარავს და რავიცი.გასტრიტიც მაქვს საჭმელს ზალიან ცუდად ვინელებ.ოდნავ ცხიმიანი საჩმელი და ღამე არ მაძინებს და გული მერევა. მერე დამიბარეს ზონდი უნდა გადაგაყლაპოტო და ვეგარ მივედი. ანესთეზიით გავაკეტებინებ ტორე დამახრჩობენ ეგენი.
ეს ყველაფერი დიდი ხანია მაწუხებს და მიჩვეული ვარ.ეს ძილის პრობლემა არ მქონდა ადრე ეს კუჩნაწლავის პრობლემა სულ მქონდა. მოკლედ თუ თქვენ ექიმი ბრძანდებით იქნებ მითხრათ რა დაავადებას შეიძლება ქონდეს ასეთი სიმპტომები. ვგულისხმობ ამ უაზრო ძილის და უენერგიობის ამბავს.ესაა ეხლა ჩემი პრობლემა.

დიდად გმადლობთ წინასწარ.

Posted by: audrey-girl 12 May 2011, 16:47
GIO BIO
ჯერ იქიდან დავიწყებ რომ ექიმი არ ვარ ჯერ, სტუდენტი ვარ მე4 კურსის.

მე რაც ვიფიქრე თავიდან ეს იყო ფარისებრი ჯირკვლის ჰორმონების დისბალანსი, მაგრამ რაც შენ დაწერე ეს უკვე გასტრიტის სიმპტომებია. ამიტომ გირჩევ რაც უფრო მალე გაიკეთებ ენდოსკოპიურ გამოკვლევას მით უკეთესი. ის რაც შენ გაწუხებს ანუ ზოგადი სისუტე, ძილიანობა, უენერგიობა გასტრიტის დროს არის გამოხატული ხოლმე.
გადა ამისა დაგჭირდება სისხლის საერთო ანალიზის გაკეთება, რათა შემოწმდეს ჰემოგლობინის და ერითროციტების დონე.
ბოლოს როცა იყავი ექიმთან ალბათ გაიკეთე სისხლის საერთო და მაშინ როგორი ანალიზი იყო თუ გახსოვს?
რაც შეეხება ზონდის გადაყლაპვას ჯერ არავინ დახრჩობილა და შენც არაფერი მოგივა, ამიტომ ნუ ნერვიულობ. smile.gif

Posted by: GIO BIO 13 May 2011, 01:53
სისხლი ნორმაში მქონდა მემგონი, მაგრამ ეს იყო ალბატ 3 წლის წინ.ეხლა გადაწყვეტილი მაქვს ბოლომდე გამოვიკვლიო. მე უფრო ღვიძლი მგონია. საჭმელის ჭამა და ეგრევე გეგონება ერტი 10 საძილე ტაბლეტი დავლიე,ისე მეხუჭება ტვალები ზალას ვატან რომ გავახილო მაგრამ ერტ წუტში ისევ მეხუჭება. ნუ ჭამა აძლიერებს ჩემს ძილად მივარდნის პროცესს. ერთმა მითხრა რომ ღვიძლმა იცისო ,რაღაც ჯანგბადის იაღლიში ხდებაო და ტვინი ვერ იკვებება ჯანგბადითო.ამიტო გეძინებაო. ნუ ეს მოყვარულის დონეზეა გათვითცნობიერებული მედიცინაში ,ასე რომ ექიმის რჩევა კარგი იქნებოდა მაგრამ რაღაც იგვიანებს ქალბატონი ექიმი ტუ ბატონი ექიმი. მოკლედ მაინც მადლობა laudrey-girl
ara adre gadamaylapes eg zondi da damazines ise. ეგრე უფრო ადვილია.
ეს გასტრიტი ბავშობიდან მაქვს და ესეთი რაღაცა არ მქონია არასდროს.არამგონია მთლად გასტრიტი იყოს დამნაშავე.

Posted by: amelia 13 May 2011, 09:51
ალბათ აქ უნდა ვიკითხო. მოკლედ, თიტქმსი ორი კვირაა ფეხის (ტერფის) ტკივილი მაწუხებს საშინლად. თავიდან ვაბრალებდი მაღალ ქულსებს, მაგრამ უკვე ერთი კვირაა ჩამოვედი მაღალი ქუსლიდან და ტკივილი კი არ შემცირდა, გამაგიჟა. მტკივა ისე, რომ ღამე მაღვიძებს და ხელს რომ დავიდებ ტერფზე (ან ზემოდან, აქ ქვემოდან) თითქოს საშინელი დაბეჟილი მაქვს და გუშინ ძლივს დავიბანე, რადგან საპნის წასმის დროს ფეხის თითები ისე მეტკინა (ხელი რომ მოვკიდე), რომ ჩავიკეცე, ვიფიქრე გული არ წამივიდესთქო. სიარულით ძლივს დავდივარ. რომ დავდივარ, ტერფი (თუ რაც ქვია თითები როგორც კი მთავრდება იქ, ქვედა მხრიდან, სირბილე სადაც არის) თითო ნაბიჯის გადადგმაზე საშინლად მტკივა. არ ვიცი რა მჭირს, ინტერნეტში სადაც უცხოურ საიტებზე მსგავსი სახის ჩივილები წავიკითხე, ყველა ვარაუდს გამოთქვამს, რომ ალბათ ეს არისო Плантарный фасциит (აზრზე არ ვარ ეს რა არის, ეს სურათი ახლავს თან).
http://www.radikal.ru
ასეთი ტკივილი ფეხს თუ შეეძლო არ ვიცოდი, ვჩერდები ხოლმე სიარულისას, რადგან ტვინში მირტყამს უკვე ტკივილი და მოკლედ, დამღალა. თავიდან მართლა ფეხსაცმელებს ვაბრალებდი.
ერთი სიტყვით, ორთოპედი მინდა ხომ სავარაუდოდ?
ძალიან გთხოვთ თუ იცით რომელიმე ორთოპედის ნომერი, დამიწეროთ და ასევე თუ იცით ხვალ რომელ კლინიკაში შეიძლება წავიდე, რომელიც მუშაობს და სადაც ორთოპედი იქნება?
წინასწარ მადლობა
თუ გნებავთ აქ დაწერეთ, თუ გინდათ პიემი მომწერეთ აქ თუ არ გინდათ
P.შ. ხო, ფეხის თითი თითოეული მტკივა, ოდნავ რომ მოვკიდებ ხელს და ოდნავ თუ მოვუჭერ.
მირჩიეთ რა გთხოვთ რამე

Posted by: Neo777 13 May 2011, 13:12
amelia

ღუდუშაურში ტრავმატოლოგთან smile.gif


Posted by: qeraiuristi 17 May 2011, 20:00
ვინმეს თუ შეუძლია მითხრას რისი წამალია დიმედროლი და მისი გადაჭარბებული დოზა რა შედეგს იწვევს ?

Posted by: K_rati 17 May 2011, 20:59
qeraiuristi
დიმედროლი არის ანტიალერგიული წამალი, რომელსაც ასევე ახასიატებს დამამშვიდებელი და დამაძინებელი მოქმედება და ამ მიზნითაც გამოიყენება.
გადაჭარბებულმა დოზამ შეიძლება გამოიწვიოს ძლიერი ძილიანობა, დაბინდული მხედველობა და ცნობიერება, პირის სიმშრალე, ჰალუცინაციები. ასეთ დროს აუცილებლად უნდა მიმართოტ ექიმს

Posted by: Hyperzet 17 May 2011, 21:07
გამარჯობა smile.gif

არსებობს პრეპარატი, რომელიც უმატებს აგრესიას?
კონკრეტულად, აგრესიის მოსამატებელი პრეპარატზეა საუბარი
აფთიაქებში თუ იყიდება , რეცეპტით ან ურეცეპტოდ?
წინასწარ მადლობა smile.gif


Posted by: audrey-girl 17 May 2011, 22:37
GIO BIO
რა ქენი იყავი ექიმთან? თუ იყავი გვითხარი ჩვენც რა ამბებია. რა დიაგნოზი დასვა? რა ანალიზები გაიკეთე. თუ არ დაგეზარება დაგვიწერე. smile.gif
რაც შეეხება ექიმს, აქ ექიმი არ შეოდის ხოლმე, ამიტომ ტყუილად გაქვს იმედი smile.gif
amelia
თუ არ გეზარება შენც გაგვაგებინე რა ამბებია, რა გითხრა, რა დიაგნოზია?

Posted by: dito_Metalhead 17 May 2011, 22:54
რამე ძილის წამალი მირჩიეთ რა, 10-12 საათი რო მიმასიკვდილოს, ფასიც მიაწერეთ ბარემ. მადლობა წინასწარ smile.gif

Posted by: audrey-girl 18 May 2011, 21:35
dito_Metalhead
მწვანე ვალერიანი რომ სცადო? ორი აბი დალიე ძილის წინ. მე ძალიან მშველის ხოლმე. აქამდე დაგილევია რაიმე დამამშვიდებელი და საძილე საშვალება?
ან რად გინდა საძილე საშვალება? არ გძინავს კარგად?

Posted by: dito_Metalhead 18 May 2011, 22:24
audrey-girl
ადრე ვსვავდი ცოტა ხანს რაღაც დამამშვიდებელს სახელი არ მახსოვს, 5-6 საათამდე ვერ ვიძინებ სადღაც 2 წელია უკვე.

Posted by: audrey-girl 19 May 2011, 18:57
dito_Metalhead
უძილობა არც ისე მარტივია როგორც ერტი შეხედვით ჩანს. არ გირჩევ თვითნებურად წამლების მიღებას, ეს პრობლემას ვერ მოაგვარებს. ხომ არ ჯობია ფსიქიატრის კონსულტაციას?!

Posted by: GABRIELA 20 May 2011, 11:50
გავიკეთე ექოსკოპია და მითხრეს ღვიძლში არის ჰემანგიომაო, რას ნიშნავს და რამდენად საშიშია?ძალიან შემეშინდა

Posted by: nakanaka 22 May 2011, 19:57
გამარჯობა!ვარ 20 წლის და მაწუხებს ოფლიანობა იღლიაში მაშინაც კი როცა მცივა. რისი ბრალი შეიდლება იყოს და რა უნდა გავაკეთო რომ ასე ძალიან აღარ შემაწუხოს? მადლობა

Posted by: gvelis_wiwila 22 May 2011, 20:50
ხალხნო რევმატიზმის დიაგნოზის დასადგენად მარტო აწეული სტრეპტოკოკი საკმარისია? თუ სხვა მაჩვენებკებიც უნდა იყოს ნორმიდან ამოვარდნილი/?

Posted by: leopardibuba 23 May 2011, 11:36
მდა
--------------------------------------------

Posted by: ქეთუ 23 May 2011, 16:03
სახსრების ტკივილი აი რევმატიზმება რო იცის ისეთი გლანდების გარდა რამ იცის?

რევმაზიტმები არ მაქვს, ჩიყვი არ მაქვს, გლანდები არაა ამოსაჭრელი და რატო მტკივა ვერ ვიგებ sad.gif

Posted by: K_rati 23 May 2011, 16:44
ქეთუ
კერძოდ რომელი სახსრები გტკივა? ორივე მხარეს თუ ცალ მხარეს?

Posted by: cimi 24 May 2011, 14:37
თუ ვინმემ იცით ვისთან მივიდე ნაწლავების სამკურნალოდ?ბრმა ნაწლავისა და ნაწლავების ანთება მითხრა ეხომ.მინდა ნორმალური სპეციალისტი,ქირურგთან არ მივდივარ ,მაშინვე ბრმანაწლავის ოპერაციას მეტყვის.ეხოზე კი მითხრეს რომ საოპერაციო არა ხარ მკურნალობა გჭირდებაო.

Posted by: ქეთუ 26 May 2011, 10:45
QUOTE (K_rati @ 23 May 2011, 16:44 )
ქეთუ
კერძოდ რომელი სახსრები გტკივა? ორივე მხარეს თუ ცალ მხარეს?

აი ხელები მიმძმიდება თიტქოს და ფეხებიც მასე.
ზამტარ-ზაფხულ ნასკები მაცვია, თუ არადა მასე მომდის ფეხებზე.
ხელებზე არ ვიცი რისი ბრალია sad.gif
და მითხრეს გლანდებმა იცის მასეო +ყელი 1წელი ისე მტკიოდა ძლივს მომირჩინეს მარა ეხლაც არ არის კარგ დრეში.

Posted by: rock in rose 26 May 2011, 16:40

cimi

ძმები მუხაშავრიები რომ არიან, რესპუბლიკურის უკან კლინიკა აქვთ, იმათთან მიდი. ძალიან ძლიერი ექიმები არიან

ექიმები და არა ექიმბაშები!!!

Posted by: Coffee_and_Cigarettes 30 May 2011, 19:16
10 დღის წინ გაკეთებული ნემსები დღემდე მტკივა ხელს რო ვიდებ , რა ვქნა? დავიღალე იოდის სმით, მგონი არ მშველის, თან ნანემსარის ადგილები ცოტა გამაგრებულია weep.gif

Posted by: kato_stropa 1 Jun 2011, 13:26
1კვირაა დღეში 4ჯერ საკმაოდ ძლიერმოქმედ პრეპარატებს ვსვამ.ნუ,პრინციპში ანტიბიოტიკია 2,მაგრამ მაინც ძლიერმოქმედია.
+ 3ჯერ ამიღეს ვენიდან სისხლი 3 დღეში,დაახლ.200გრ.
შეიძლება ამ ყველაფერმა გამოიწვიოს ნერვული აღგზნებადობა,გაღიზიანება,ძილისკენ მიდრეკილება,სულ ცუდ ხასიათზე ვარ ბოლო დღეებია და გემოს შეგრძნება დავკარგე,მაწონსაც კი მწარე გემო აქვს და მარწყვსაცsad.gif
შეიძლება ეს ბევრი წამლის ბრალია??

Posted by: supinatori 1 Jun 2011, 16:25
ფეხის ტერფის და თითების ჩასაფენები http://www.supinatori.com
* * *

Posted by: shtori 14 Jun 2011, 20:00
რამდენიმე დღეა თავს უცნაურად ვგრძნობ. თითქოს აფორიაქებული ვარ, გული აჩქარებული, ხან სუნთქვა მიჭირს, გულის რევის შეგრძნება მაქვს. ავადმყოფობის შემდგეი პერიოდისავით მაქვს. ავად კი არ ვყოფილვარ.
ეს დღეები შოკოლადს აღარ ვჭამ,მანამდე ძალიან ბევრს ვჭამდი და შეიძლება მაგის გამო ვიყო ასე? თუმცა ეს უფრო ადრე დამეწყო

Posted by: likaterine 14 Jun 2011, 23:28
კბისქვეშა ჯირკვალი (მგონი ლიმფური კვანძი ქვია, არ ვიცი ზუსტად) მაქვს გადიდებული და მტკივა. რომელ ექიმს მივმართო თუ შეგიძლიათ დამაკვალიანეთsmile.gif მადლობა წინასწარ..

Posted by: dito_Metalhead 15 Jun 2011, 23:25
კუჭის წყლულის სიმპტომები მითხარით რა ვინმემ

და რისგან არის ეს დაავადება გამოწვეული smile.gif

Posted by: BloodElf 16 Jun 2011, 08:19
kato_stropa
ექიმი არ ვარ მაგრამ აქ არავინ პასუხობს მგონი კითხვებს და .. გაციებული ხომ არ ხარ? მეც დამემართა მსგავსი რაღაც გაციებული ვიყავი ..
* * *
ეხლა რაც შეეხება ჩემს კითხვას .. დედაჩემი 55 წლისაა და დაახლოებით 1-2 თვის წინ გაუჩნდა ფეხზე , მუხლს ქვემოთ მოყავისფრო ლაქები.. რაღაცით სისხლჩაქცევას გავდა აქაიქ ქონდა ფეხზე ასეთი ლაქები... ნელ ნელა იმატებდა თან ახლა თითქმის მთელს ფეხზე აქვს თან ამბობს რომ სტკივა.. ვერაფრით ვერ დავითანხმე ექიმთან წასვლაზე.. და იქნებ ვინმემ მითხრათ რა შეიძლება იყოს ან რომელ ექიმს მივმართო რჩევისთვის

Posted by: likaterine 17 Jun 2011, 02:46
BloodElf

ვენების გაგანიერება ხომ არ აქვს? დედაჩემს აწუხებს და მსგავსი ლაქები მასაც აქვს ხოლმე, გამოუვა ისევ გაუქრება და ა.შ. თუ დედათქვენსაც იგივე პრობლემა აქვს, მაშინ ანგიოლოგს უნდა მიმართოთ.

Posted by: dito_Metalhead 17 Jun 2011, 17:26
QUOTE (dito_Metalhead @ 15 Jun 2011, 23:25 )
კუჭის წყლულის სიმპტომები მითხარით რა ვინმემ

და რისგან არის ეს დაავადება გამოწვეული smile.gif


Posted by: monchita 17 Jun 2011, 22:34
QUOTE
QUOTE (dito_Metalhead @ 15 Jun 2011, 23:25 )
კუჭის წყლულის სიმპტომები მითხარით რა ვინმემ

და რისგან არის ეს დაავადება გამოწვეული

დისპეპსიური მოვლენები- გულძმარვა, ბოყინი, ტკივილი კუჭის მიდამოში განსაკუთრებით უზმოზზე და საკვების მიღების შემდეგ დისკომფორტის შეგრძნება. ეს ყველა ჩამოთვლილი ჩივილი მიუთითებს კუჭ-ნაწლავის ტრაქტის რომელიმე პათოლოგიაზე და აუცილებელია გასტროენტეროლოგთან ვიზიტიsmile.gif მარტო ანამნეზის საფუძველზე არ ხდება კუჭის წყლულის დიაგნოზის დასმაsmile.gif აუცილებელია ზონდით გამოკვლევა და უშუალოდ კუჭის დათვალიერებაsmile.gif რაც შეეხება დაავადების ეტიოლოგიას- დიდი მნიშვნელობა ენიჭება მოუწესრიგებელ კვების რეჟიმს, თვითნებურ, არარაციონალურ დიეტებს ასევე მჟავე, ცხარე, მწვავე საკვების მიღებას დიდი ხნის განმავლობაში, რაც თავდაპირველად სხვადასხვა სახის გასტრიტებს იწვევს და შემდეგ თუ არ ჩატარდება სათანადო მკურნალობა შესაძლოა, კუჭის ლორწოვანიც დაზიანდესsmile.gif გარკვეული მნიშვნელობა ენიჭება გენეტიკურ ფაქტორსაცsmile.gif
* * *
QUOTE
1კვირაა დღეში 4ჯერ საკმაოდ ძლიერმოქმედ პრეპარატებს ვსვამ.ნუ,პრინციპში ანტიბიოტიკია 2,მაგრამ მაინც ძლიერმოქმედია.
+ 3ჯერ ამიღეს ვენიდან სისხლი 3 დღეში,დაახლ.200გრ.
შეიძლება ამ ყველაფერმა გამოიწვიოს ნერვული აღგზნებადობა,გაღიზიანება,ძილისკენ მიდრეკილება,სულ ცუდ ხასიათზე ვარ ბოლო დღეებია და გემოს შეგრძნება დავკარგე,მაწონსაც კი მწარე გემო აქვს და მარწყვსაც
შეიძლება ეს ბევრი წამლის ბრალია??

რა თქმა უნდა შესაძლებელიაsmile.gif წაიკითხეთ წამლის ანოტაცია და გაეცანით გვერდით ეფექტებს და ჩივილების შესახებ აცნობეთ მკურნალ ექიმსsmile.gif

Posted by: MegMar1 21 Jun 2011, 00:41
ძალიან გთხოვთ, მიპასუხეთ. შეიძლება, რომ ქალს ორსულობის დროს დაემართოს ჩუტყვავილა და ბავშვი თანდაყოლილი დეფექტით დაიბადოს?მითუმეტეს თუ 5 კვირის ორსულს დაემართა. იქნებ ამიხსნათ, რამდენად ვნებს ნაყოფს ჩუტყვავილა?

Posted by: Natuka NGN 21 Jun 2011, 00:55
MegMar1
ჩუტყვავილა არ ვიცი და ძალიან საშიშია წითურა. ხვალ შევეცდები გავიგო.

Posted by: Nanuuu 21 Jun 2011, 10:40
გამარჯობა, თქვენი აზრი მაინტერესებს, კარდიოლოგს რო ვკითხე, არამგონია გულის ბრალი იყოსო, ჯერ თერაპევტთან მიდიო და....
ამ ბოლო დროს, დილას რო ვიღვიძებ, მკერდის არეში მაქვს ჭვალივით.... ძაან მაწუხებს, თითქოს ის ადგილი სადაც გულია გადაბმული, თუ როგორ ვთქვა არ ვიცი
ქაჩავს რაღაცნაირად, მარცხენა ხელს რო გავამოძრავებ მაგაზეც კი მქაჩავს, ამოსუნთქვაზე, მოძრაობეაზე...მერე დღის განმავლობაში თანდათან მივლის თავისით...
ყოველდღე არ მეორდება მაგრამ ნუ კვირაში 3-4ჯერ, დილას მაქვს ხოლმე... 24 წლის ვარ...არ ვეწევი, ვვარჯიშობ და ვცდილობ ჯანსაღად ვიკვებო, მეტი რაღა ვქნა
რისი ბრალი შეილება იყოს ხო არ იცით?

Posted by: ninca 21 Jun 2011, 13:17
გამარჯობა

ჩემს ქმარს რამოდენიმე დღეა ფეხის თითი აქვს დაბუჟებული. რისი ბრალი შეიძლება იყოს, ან რომელ ექიმს მივმართოთ? მადლობთ წინასწარ

Posted by: MegMar1 21 Jun 2011, 13:56
მეც ასე მითხრეს, წითურა არის საშიში, ჩუტყვავილა ნაკლებადო, მითუმეტეს პირველ ტრიმესტრშიო, მაგრამ მაინც ვნერვიულობ.

Posted by: audrey-girl 21 Jun 2011, 17:11
BloodElf
თავიდანვე მტკივნეული იყო ეგ ლაქები? როგორი ლაქებია შეგიძლია აღწერო? კანიდან ოდნავ ამობურცულია თუ არა?
* * *
ninca
QUOTE
ჩემს ქმარს რამოდენიმე დღეა ფეხის თითი აქვს დაბუჟებული. რისი ბრალი შეიძლება იყოს, ან რომელ ექიმს მივმართოთ? მადლობთ წინასწარ

რის მერე გაუჩნდა დაბუჟება?
სავარუდოდ ნევროლოგს უნდა მიმართოთ.
Nanuuu
კარდიოლოგთან იყავი და გაგსინჯა თუ როგორ?

Posted by: ninca 21 Jun 2011, 17:44
audrey-girl
ამბობს, დიდი ხანი ვიყავი ჩაკუზულიო.. მაგრამ მესამე დღე გავიდა უკვე და რავიცი აბა..
მადლობთ გამოხმაურებისთვის

Posted by: audrey-girl 21 Jun 2011, 17:49
ninca
QUOTE
ამბობს, დიდი ხანი ვიყავი ჩაკუზულიო.. მაგრამ მესამე დღე გავიდა უკვე და რავიცი აბა..
მადლობთ გამოხმაურებისთვის

რამდენიმე დღე დაელოდეთ როგორი იქნება საქმე და მერე მიდით.

Posted by: Hana Arendtiii 21 Jun 2011, 19:09
ნევროლოგიის განყოფილება არის? მაინტერესებს რა არის ყველაზე კარგი და უსაფრთხო დამამშვიდებელი პრეპარატი?( ზალიან დამოკიდებულიც რომ არ გახდე მერე ისეთი მინდა დაახლოებით 50 წლამდე ქალისთვის, ჩემთვის არა smile.gif

Posted by: gvants 21 Jun 2011, 19:13
MegMar1
ამასწინათ ვკითხულობდი, რაც შორსაა მშობიარობის თარიღთან, მით უკეთესიაო.

--------------------

Posted by: audrey-girl 21 Jun 2011, 19:54
Hana Arendtiii
რისთვის გინდა? ვისთვის? ან რატომ?

Posted by: rock in rose 21 Jun 2011, 20:03
Hana Arendtiii

წამლის რჩევა და მითუმეტეს დამამშვიდებლის, ფორუმის წესებით აკრძალულია. ისედაც არ მიმაჩნია სწორად ურჩიო ადამიანს ვირტუალური სივრციდან მედიკამენტი smile.gif
ამიტომ, სამწუხაროდ ამ კითხვაზე პასუხს აქ ვერ გაგცემენ

იმედია სწორად გაიგებთ smile.gif

Posted by: Hana Arendtiii 22 Jun 2011, 01:02
rock in rose
კი, მესმის აბსოლიტურად smile.gif წამალში რაიმე ხალხური უფრო ვიგულისხმე, ბალახებისგან დამზადებული ნაყენი და ა.შ smile.gif

Posted by: rock in rose 22 Jun 2011, 19:05
Hana Arendtiii

QUOTE
წამალში რაიმე ხალხური უფრო ვიგულისხმე, ბალახებისგან დამზადებული ნაყენი და ა.შ


ასე გეთქვა მერე biggrin.gif

შავბალახა არის ასეთი მცენარე, დამამშვიდებელი და სედაციური მოქმედება ახასიათებს. მაგისგან მომზადებულ ნაყენს გაურიე 20 წვეთი პიტნის წვეთები(აფთიაქში იყიდება, პირდაპირ პიტნის წვეთები იკითხე) და ის დალიოს. თუ დაბალ წნევიანია, მაშინ არ გინდა პიტნის შერევა, სხვა შემთხვევაში კი ძალიან კარგია smile.gif

Posted by: Natuka NGN 22 Jun 2011, 20:33
Hana Arendtiii
მართალი გითხრა როკიმ. შავბალახა საუკეთესო დამაშვიდებელია. გამზადებლიც იყიდება აფთიაქებში. პინტის კოქტეილთანაც კარგია...

Posted by: laliko 23 Jun 2011, 13:16
არის აქ ვინმეე? 1 კვირაა მტკივა ხელების კუნთები ძვლები სახსრები, ლამისაა გავგიჟდე. რის ბრალი იქნება. ბავშვი მყავს 2 წლამდე მარა ეხლა ხო არ ავიყვანე პირველად.

Posted by: pawking 23 Jun 2011, 13:57
გამარჯობა, მაქვეს ერთი კითხვა, სავარაუდოდ რისგან შეიძლება იყოს გამოწვეული შარდში სისხლის გარევა (მამაკაცი) ?

Posted by: audrey-girl 23 Jun 2011, 20:10
pawking
რა ფერის სისხლი არის? ალისფერი და მუქი ფერის?

Posted by: m-009 25 Jun 2011, 02:00
კანზე სისხლ ჩაქცევები რისი ბრალი უნდა იყოს?[პროთრომბინი ნორმაშია,ვის მივახითხო?

Posted by: buub 29 Jun 2011, 16:48
CD სიმპტომები რას ნიშნავს ? (მგონი მასე ქვია ხო CD სიმპტომებიო ) ხოდა ვინც იციტ განმიმარტეთ რა

Posted by: METEORA 30 Jun 2011, 11:52

გამარჯობათ. დილას ფეხების და სახის შეშუპება რისი ბრალი შეიძლება იყოს?

Posted by: kato_stropa 30 Jun 2011, 12:04
METEORA
აქ მემგონი არავინ პასუხებს კითხვებს მოკვდავი ჰუზერების გარდა biggrin.gif

QUOTE
დილას ფეხების და სახის შეშუპება

ან ცილები გაქვს მომატებული და მაგის ბრალია(პირადი გამოცდილება)
ან თირკმელებმა შეიძლება გამოიწვიოს.

შარდზე ნორმალურად გადიხარ?
თუ გიჭირს,აუცილებლად მიმართე ექიმს,მაგრამ არამიანცში ნუ წახვალ,გაგწეწავენ როგორც შეუძლიათ და მერე კიბოს დიაგნოზს დაგისვამენ user.gif

და ისედაც,თუ გაიშუქებ თირკმელებს და გაესინჯები,კარგს იზამ smile.gif

Posted by: METEORA 30 Jun 2011, 13:48
kato_stropa
თირკმელების ექო გაკეთებული მაქვს. არაფერი ანახა. შარდის გზებზეც ვიმკურნალე. ეხლა ყველაფერი ნორმაშია. სხვა არაფერი შეიძლება იყოს მაგის გარდა?

Posted by: K_rati 30 Jun 2011, 17:15
METEORA
შარდის ანალიზი გაიკეტეთ, ექოზე წინ ცილა უნდა განისაზღვროს შარდში.

Posted by: REGVENI 30 Jun 2011, 19:02
ყელი (კისერი) მაქვს დაჭიმული თუ ყელში მიჭერს ვეღარ გავიგე თან კაი ხანია უკვე 2 თვეა დაახლოებით. უფრო სამსახურში მიშვება, კომპტან ვზივარ მთელი დღე. საღამოს სახში ამიშვებს ხოლმე. კიდე თუ დავლიე ლუდი ან ღვინო მაშჳნაც მიშვებს. ჭამა და ყლაპვა არ მიჭირს არც ტკივილი მაქვს. რისი ბრალი შეიძლება იყოს? ექიმთან ვიყავი და უფრო ნერვებიაო. სომნა რიცი დამინიშნა. თუ ვინმეს გეცნოთ სიმპტომი მიპასუხეთ. ანუ მხრებისმკენ მქაჩავს თითქოს, ხანდახან ყელშიც წამიჭერს. და კიდე გავცივდი ადრე და იმან დამიტოვა ხველება და ვახველებ ხოლმე

Posted by: F_R_E_Y_A 5 Jul 2011, 12:06
რა შემთხვევაში შეიძლება გჭირდებოდეს ყოველ თვე სისხლის გაწმენდა და გადასხმები, გქონდეს გამონაყარი და ძლიერი ოფლიანობა? ნარკომანია? თუ რამე ვირუსი?
უცნაურ თემაში რომ ვკითხულობ ვიცი მაგრამ სხვა ვერ ვნახე biggrin.gif

Posted by: jgnakvi 14 Jul 2011, 00:21
არ დამცინოთ და წელს შევამჩნიე საკუთარ თავს,ნესვის ჭამის მერე ყელი მტკივა,მეწვის მიხურს sad.gif რისი ბრალი შეიძლება იყოს?

Posted by: F4R2 16 Jul 2011, 23:21
თუ გაგიგიათ ან გქონიათ შემთხვევა როდესაც ნეიროსენსორული სმენის დაქვეითების მქონე პაციენტს სმენა აღდგენია ან ნაწილობრივ მაინც გაუმჯობესებულა აპარატის და იმპლანტანტის გარეშე?

Posted by: Registration 18 Jul 2011, 19:48
საორდინატორო რა კაბინეტია, შიგნით რა ხდება?

Posted by: Jeanne Hebutern 19 Jul 2011, 10:49
არ ვიცი აქ უნდა დავწერო თუ არა ეს კითხვა...

მოკლედ დამეცყო ვენეების ტკივილი. რო ვიღვიძებ დილით ძარღვები მტკივა და ისეთი დაჭიმული მაქვს თითქოს წინა ღამეს ბევრი ვიცეკვე ან ვივარჯიშე.. და შეიძლება მამენტ რაღაც ადგილზე დამილურჯდეს კიდეეც..
ვარ 22 წლის
და ეს შეიძლება წონის ბრალი იყოოსს???
ან ჯდომიის უმოძრაობიისს???
და რა ვუშველოო???

Posted by: MAGIC_WOMAN 25 Jul 2011, 10:39
გამარჯობა, გთხოვთ დამაკვალიანოთ ვის მივმართო ან რისი ბრალი შეიძლება იყოს 2 დღეა დამეწყო ხელის გულებზე და ფეხის გულებზე დაბუჟება-ღიტინს შეგრძნება, რომლის გამო ღამეც კი ცუდად მძინავს. ნერვული სისტემის ბრალი შეიძლება იყოს ???

Posted by: gurabe 26 Jul 2011, 20:57
არ ვიცი სწორად ვსვამ თუ არ კითხვას,მაგრამ...............


დაახლოებით ერთი კვირაა ცალი ფეხი (მუხლს ზემოთ,პატარა ადგილას) მიბუჟდება,რამეც მაღვიზებს ხოლმე,მხოლოდ ერთი ფეხი,როდესაც ვზივარ ცოტა დიდხანს მაშინაც იგივეა და დაახლოებიტ თუ შეგიძლიათ,რომ მითხრათ რისი ბრალი შეიძლება იყოს?.............................................

Posted by: Laverne_de_Montmorency 27 Jul 2011, 15:50
ეს ვინც იცით (არ ვიცი სად უნდა ვიკითხო), ცოფის აცრას თუ ვიკეთებ შესაძლებელია რომ მოვწიო? თუ როგორც ალკოჰოლი ეგეც მეკრძალება?

ერთი ღერი უკვე მოვწიე და მერე დავპანიკდი sad.gif იქნებ მიპასუხოთ რა tongue.gif

Posted by: Kaifistka 27 Jul 2011, 19:33
არვიცი აქ უდნა მეკიტხა თუ არა მაგრამ მაინც ვიკითხავ.

რაღაცა მიზეზების გამო ხშირად ვკარგავ დიდი რაოდენობის სისხლს,მაგალითად მენსტრუაცია და სხვა მიზეზებიც.
და ადრე დამიდგინეს რკინა დეფიციტური ანემია და ვსამდი წამლებს.
ახლა მგონი ისევ დამეწყო. ძალიან დაღლილი ვარ სულ, ახლად გამღოძიებულზეც ისევ მეძინება, მთელი დღე მაქვს დაღლილობის შეგრძნება, სპორტს ვერ მივსდევ ისე თუ ასე, მატყდება ფრჩხილები და ზოგჯერ დაღუნულად მეზრდება, წამოდგომისას თუ იმის წინ ვიწექი ხშირად მიბნელდება თვალებში და ცოტა ხანი ვერაფერს ვერ ვხედავ.
შეიძლება ვივიარაუდო რომ ისევ რკინადეფიციტური ანემია მაქვს?
ექიმთან ვერ წავალ ახლა, რკინის ტაბლეტკები მაქვს. დოზა აწერია მკრნალობისთვის დღეში 3-ჯერ შეიძლება და პრევენციისთვის დღეში ერთხელ და აქამდე რამდენჯერმე დღეში ერთხელ დავლიე დღეს დავიწყე ისევ სამჯერ დალევა.
ჩემი შეკითხვა მდოგომარეობს იამში რომ თუ შეიძლება ვივარაუდო ანალიზის გარეშე რომ მაქვს ისევ რკინადეფიციტური ანემია?! ან სხვა რისი ბრალი შეიძლება იყოს ჩემი სიმპტომები?

ჰო კიდე დამავიწყდა თმები მცვივა საშინლად.

Posted by: Mr-Banks 29 Jul 2011, 15:50
სანამ ახლ თემას გავხსნი, დამაკვალიანეთ...

ამ ბოლო დროს (ხშირდება დღითი-დღე), მირტყამს დენი რკინასთან შეხებისას. არის რაიმე გამოსავალი?

Posted by: Registration 1 Aug 2011, 16:28
დიდხანს სიარულისას ფერდის ტკივილის მიზეზი რაა? როგორც წესი, მარცხენა ფერდის.

Posted by: lurdes 1 Aug 2011, 20:43
გამარჯობა
გთხოვთ, მიპასუხოთ:
70 წლის ქალბატონს ძილის დროს აღვიძებს ფეხებში ძარღვების დაჭიმვისა და განასკვის შეგრძნება, რაც ძალიან აწუხებს. მხოლოდ წამოდგომის შემდეგ გრძნობს შვებას. ეს მხოლოდ ღამით, ძილის დროს ემართება. რისი ბრალია, როგორ მოიქცეს?

Posted by: All Shall Perish 1 Aug 2011, 21:47
სახლში კოდიციონერით 20 გრადუსი რომაა და გარეთ ხშირი გასვლა შემოსვლა ცუდია ჯანმრთელობისთვის?

Posted by: cortex-amygdala 3 Aug 2011, 14:24
QUOTE
სახლში კოდიციონერით 20 გრადუსი რომაა და გარეთ ხშირი გასვლა შემოსვლა ცუდია ჯანმრთელობისთვის?


ეს მეც მაინტერესებს, სამსახურში ვიყინები, გარეთ დღეში რამდენჯერმე მიწევს გამოსვლა და ვიხარშები, მერე ისევ გამყინვარებაში . რამდენად საზიანოა და რატომ??
* * *
აუჰ, აქ კითხვაზე პასუხის გაცემის იმედის ნაპერწკალიც არ ჩანს აშკარად, პასუხგაუცემელი კითხვების კორიანტელია

Posted by: matusala_5 3 Aug 2011, 14:51
QUOTE
ეს მეც მაინტერესებს, სამსახურში ვიყინები, გარეთ დღეში რამდენჯერმე მიწევს გამოსვლა და ვიხარშები, მერე ისევ გამყინვარებაში . რამდენად საზიანოა და რატომ??

არაა კარგი ტემპერატურის კონტრასტულად სწრაფად ცვლილება ,განსაკუთრებით გულ-სისხლძარღვთა სისტემისთვის.
ამიტომ 5-6 გრადუსს (წესით) არ უნდა აღემატებოდეს ტემპერატურული სხვაობა.
* * *
QUOTE
70 წლის ქალბატონს ძილის დროს აღვიძებს ფეხებში ძარღვების დაჭიმვისა და განასკვის შეგრძნება, რაც ძალიან აწუხებს. მხოლოდ წამოდგომის შემდეგ გრძნობს შვებას. ეს მხოლოდ ღამით, ძილის დროს ემართება. რისი ბრალია, როგორ მოიქცეს?

კოაგუროგრამაზე გაიკეთოს ანალიზი.

Posted by: Natuka NGN 3 Aug 2011, 15:44
აუ, ყველა სახსარი მტკივა sad.gif გამაცივა კონდიციონერმა sad.gif

Posted by: matusala_5 3 Aug 2011, 15:59
Kaifistka
QUOTE
არვიცი აქ უდნა მეკიტხა თუ არა მაგრამ მაინც ვიკითხავ.

რაღაცა მიზეზების გამო ხშირად ვკარგავ დიდი რაოდენობის სისხლს,მაგალითად მენსტრუაცია და სხვა მიზეზებიც.
და ადრე დამიდგინეს რკინა დეფიციტური ანემია და ვსამდი წამლებს.
ახლა მგონი ისევ დამეწყო. ძალიან დაღლილი ვარ სულ, ახლად გამღოძიებულზეც ისევ მეძინება, მთელი დღე მაქვს დაღლილობის შეგრძნება, სპორტს ვერ მივსდევ ისე თუ ასე, მატყდება ფრჩხილები და ზოგჯერ დაღუნულად მეზრდება, წამოდგომისას თუ იმის წინ ვიწექი ხშირად მიბნელდება თვალებში და ცოტა ხანი ვერაფერს ვერ ვხედავ.
შეიძლება ვივიარაუდო რომ ისევ რკინადეფიციტური ანემია მაქვს?
ექიმთან ვერ წავალ ახლა, რკინის ტაბლეტკები მაქვს. დოზა აწერია მკრნალობისთვის დღეში 3-ჯერ შეიძლება და პრევენციისთვის დღეში ერთხელ და აქამდე რამდენჯერმე დღეში ერთხელ დავლიე დღეს დავიწყე ისევ სამჯერ დალევა.
ჩემი შეკითხვა მდოგომარეობს იამში რომ თუ შეიძლება ვივარაუდო ანალიზის გარეშე რომ მაქვს ისევ რკინადეფიციტური ანემია?! ან სხვა რისი ბრალი შეიძლება იყოს ჩემი სიმპტომები?

ჰო კიდე დამავიწყდა თმები მცვივა საშინლად.

რა სიმტომებსაც შენ აღწერ ეგ არის ანემიების დროს,მაგრამ ჯობია მაინც გაიკეთო რკინის ცვლაზე სრულყოფილი გამოკვლევა,ხანგრძლივად თვითნებურად რკინის პრეპარატები არ მიიღო,რკინის დოზის გადაჭარბება იწვევს სხვა დარღვევებს ორგანიზმში,მაგალითად ღვიძლში ცვლილებებს...



QUOTE
დიდხანს სიარულისას ფერდის ტკივილის მიზეზი რაა? როგორც წესი, მარცხენა ფერდის.

ამ დროს ხდება ელენთი მოცულობით გადატვირთვა(სისხლით)ელენთის კაფსულის დაჭიმვა რაც დროებით ტკივილს იძლევა.




Natuka NGN
QUOTE
აუ, ყველა სახსარი მტკივა  გამაცივა კონდიციონერმა

კონდიციონერის გარეშეც მოვახერხე გაციებაsmile.gif gigi.gif

Posted by: lurdes 3 Aug 2011, 20:09
matusala_5
QUOTE
კოაგუროგრამაზე გაიკეთოს ანალიზი.

გმადლობთ smile.gif

ერთი კითხვაც მაქვს – რა სახის ანალიზია და სად შეიძლება გაკეთება, ვის მივმართოთ?

Posted by: matusala_5 3 Aug 2011, 21:52
lurdes
კოაგულოგრამა-სისხლის შედედების გამოკვლევაა.თითქმის ყველა ლაბორატორიში კეთდება,მე თვითონ "მრჩეველში "გავიკეთე(მაგრამ სხვაგან მგონი უფრო იაფია)

Posted by: lurdes 3 Aug 2011, 22:41
matusala_5
დიდი მადლობა! smile.gif

Posted by: matusala_5 3 Aug 2011, 23:13
lurdes
არაფრისsmile.gif
..................

Posted by: Solveig 4 Aug 2011, 00:52
საყლაპავის ეროზიის ეფექტური მკურნალობა არსებობს?
მჟავიანობის მარეგულირებლების გარდა, რა თქმა უნდა...

Posted by: matusala_5 4 Aug 2011, 10:54
QUOTE
საყლაპავის ეროზიის ეფექტური მკურნალობა არსებობს?
მჟავიანობის მარეგულირებლების გარდა, რა თქმა უნდა...

პირველ რიგში უნდა მოიხსნას ისეთი ფაქტორები (თუ აღინიშნება):კარდიის ახალაზია,რეფლუქს-ეზოფაგიტი+მედიკამეტოზურად მჟვანიანობის რეგულირება+მექანიკურად და ქიმიურად დამზოგველი დიეტა.
მოკლედ კონსერვატიული მკურნალობა საყლაპავის ეროზიისას ეფექტურია (მე სხვა მეთოდი არაფერი მახსენდება ,)

Posted by: buskunia 9 Aug 2011, 01:26
გამარჯობათ

იქნებ მალე მიპასუხოთ sad.gif

ესე იგი წუხელ საღამოდან მოყოლებული მტკივა სახის მარჯვენა ნახევარი მთლიანად,ძვლები,წარბი და წამწამებიც კი ))) სურვილი მაქვს ამოვიგლიჯო


მაქვს ერთი კბილი საშინელ მდგომარეობაში ზუსტად მარჯვენა მხარეს,ფაქტობრივად ჩაშლილია და მხოლოდ ძირია ამოსაღები,ვერა და ვერ მივედი და შეიძლება რომ კბილის ბრალი იყოს? ანუ ნერვი რო გადასცემდეს,რავიცი... ტკივილით არ მტკივა.... ან რა უნდა მტკიოდეს,ძირის გარდა აღარაფერია ....

Posted by: dzirkoka 9 Aug 2011, 11:57
შევამჩნიე, რომ შინდის კომპოტს და ტყემლის წვენს, როცა ვსვამ სუნთქვის შეკავება მემართება. ძლიერად არა, მაგრამ რაღაც უსიამოვნო შეგრძნებაა. თითქოს ბოლომდე ვერ ვისუნთქავო. სხვაზემ ასეთი არაფერი არ მემართება. მხოლოდ, ამ ორ რამეზე. რამე ალერგიულია????

Posted by: fat house cat 10 Aug 2011, 02:02
სულ თავი მტკივა, ხან თითქოს მიჭერს, ხან გვერდით ფეთქავს თუ რაღაც ჯანდაბას შვრება .. ხო და კიდევ თავს რომ ვიბან, ცხელი წყალი რომ მესხმება, მარჯვენა ნახევარსფეროში , შიგნით მტკივა
ადრე დენის პროცედურებს ვიტარებდი თმისთვის და დარსონვალის გადატარებისას ისევ მარჯვნივ მტკიოდა..

ვის მივაკითხო და რა ვქნა?

Posted by: Natuka NGN 10 Aug 2011, 03:17
fat house cat
ერთმა ძალიან ჭკვიანმა ექიმმა მასწავლა - თავის ტკივილი მსგავსი გაუწყლოვნების ბრალიაო. დღეში რამდენ წყალს იღებ? შენი პრობლემა შესაძლოა სრულიად მარტივად გადაწყდეს. მით უმეტეს ამ სიცხეებში 2 ლიტრაზე მეტი წყალი უნდა მიიღო.

Posted by: fat house cat 10 Aug 2011, 12:32
Natuka NGN
წყალს პირიქით, ძალიან ბევრს ვსვამ. სიცხის დროს საერთოდ sad.gif

Posted by: BloodElf 11 Aug 2011, 14:45
წარმოდგენა არ მქონდა სად უნდა მეკითხა და იქნებ აქ მიპასუხოთ smile.gif) ხომ ვერ მეტყვით სად იკვლევენ რადიციით დასხივებულ ადამიანებს ? ცენტრი კლინიკა რავი smile.gif

Posted by: Kate789 12 Aug 2011, 23:48
სალამი.

ყური მტკივა რამდენიმე დღეა, არ ვიცი რატომ.
შეიძლება ყურის ჩხირი ცუდად მომიხვდა და მაგიტომ. ან შეიძლება წყალი ჩამივიდა. ნამდვილად არ მახსოვს.
ანუ მტკივა არა ნიჟარა, არამედ აი თითს რომ ვიყოფ და ვაჭერ, ანუ პალპაციით.
იქნებ რამე მირჩიოთ. sad.gif

Posted by: პერწკლი 17 Aug 2011, 20:59
matusala_5

პიემში მინდოდა მომეწერა და სავსე გაქვთ და იქნებ გაათავისუფლოთ sad.gif

Posted by: Kaifistka 19 Aug 2011, 20:40
აქ ვიკითხო თუ არა არვიცი. საყლაპავის დიაფრაგმისეული ხვრელის თიაქრი მაქვს და მძიმეების აწევა მომიწია ეს დღეები და ძალიან ცუდად ვარ, ამტკივდა მუცელი, აღარ ვიცი რა ვქნა, რამე მირჩიეთ რაა cry.gif cry.gif cry.gif

Posted by: lucia 21 Aug 2011, 12:07
---------------------------------------------

Posted by: shtori 23 Aug 2011, 02:42
მუხლის სიდა მხარეს, ძარღვთან ლობიოს მარცვლის ოდენა გამაგრება მაქვს, ამობურცულია, გაწითლებული და თან მექავება. ჩხვლეტა ვიგრძენი და მის მერე მესამე დღეა ასე მაქვს. ნაკბენი იქნება? დღეს კიდე როგორც ნასიცხი ისე გამომაყარა. კი ძალიან ცხელოდა და გავოფლიანდი საშინლად, მარა მაინც ფეხზეც რომ მჭირს რაღაც მაგის გამოწვეული ხომ არ იქნება რამე ალერგიასავით? რა წავისვა?

Posted by: buub 23 Aug 2011, 16:32
ეს ერთი კვირაა გაღვიძებისთანავე მთელი დღის განმავლობასი სისუსტის და თავბრუსხვევის შეგძნება მაქ,თვალებში მიჭრელდება დედაჩემი მეუბნება ტუჩები გაქვს გალურჯებულიო და რისი ბრალი შეიძლება იყოს?

Posted by: rock in rose 23 Aug 2011, 17:03
buub

სისხლის საერთო ანალიზი გაიკთე პირველ რიგში, ჰემოგლობინი ხომ არ გაქვს დაბალი

Posted by: anuka2009 25 Aug 2011, 15:19
იქნებ იცოდეთ - თბილისში სად კეთდება ბუასილზე უსისხლო ოპერაცია?

Posted by: Gora Mborgali 27 Aug 2011, 21:00
შიდსზე და ცე-ჰეპატიტზე სად შეიძლება გაკეთდეს თბილისში ანალიზი და თუ შეიძება ფასიც რომ დაწეროთ? ისე უფასო არ უნდა იყოს?

Posted by: gvelis_wiwila 29 Aug 2011, 10:32
მოკლედ გუშინ ვიჯექი ჩემთვის მეგობართან. გამოვედი გარეთ და მარჯვენა მხარეს თIთქმის მთელ ზურგზე ამტკივდა შიგნიდან. თან ძლიერი ტკივილი იყო, და მთავარი ისაა რომ ჩასუნთქვისას მტკიოდა, ანუ ისე მტკიოდა რომ ვერ ვსუნთქავდი. ბოლოს პატარა შესუნთქვაზეც მტკიოდა.

მივედი სახლში, წამისვეს რაღაც მაზი, დავლიე ცხელი ჩაი, ცოტა ჩავთბი და გამიარა.

მაგრად კი მტკიოდა და რომ ვერ ვსუნთქავდი დავიზმენდი ცოტა gigi.gif

ხველა არ მქონია/მაქვს.

ჰოდა დღეს როგორ მოვიქცე? ჩვეულებრივ რიტმში გავაგრძელო ცხოვრება თუ რამე საშიშია? ანუ ფილტვების ანთება და ა.შ.

Posted by: Natuka NGN 30 Aug 2011, 13:00
Gora Mborgali
მაგაზე ყველაზე კარგად ინფო C ჰეპატიტის თემაში იციან.


gvelis_wiwila
ნერვის გაგიცივდებოდა. ფილტვებმა ტკივილი არ იცის. მოერიდე ორპირს და გაისინჯე მაინც სიცხე.

-----
მე, კი 4 დღეს თავის ტკივილს ვერაფერს ვუშვები sad.gif რაიმე ვირუსი ხომ არ დადის ნეტა დაბალი სიცხით და საშინელი ტავის ტკივილებით? გასკდა თავი sad.gif

Posted by: Megi-meghan 30 Aug 2011, 22:42
გამარჯობა!თუ შეგიძლიათ მითხრათ რისი მიმანიშნებელია ნახეთქები პირის კუთხეებში?როგორც ვიცი რომელიღაც ვიტამინის ნაკლებობას ნიშნავს და რომლის? წინასწარ დიდი მადლობა

Posted by: ToyotaMR2 31 Aug 2011, 10:48
რა მაინტერესებს გულის მხარეს ანუ მუცლის მარცხენა მხარეს რა ორგანოა როცა ვზივარ თითქოს რაღაც მაწვება და ყრუდ მტკივა მუცლის მარცხენა ქვედა ნაწილში ან როც ავწევარ და მარჯვენა მხარეს გადმოვტრიალდები ასე მგონია რაგაც მკიდიაო შიგნით და დაჭიმულიაო .რა შეიძლება იყოს

მადლობა

Posted by: Megi-meghan 31 Aug 2011, 12:24
გამარჯობა!თუ შეგიძლიათ მითხრათ რისი მიმანიშნებელია ნახეთქები პირის კუთხეებში?როგორც ვიცი რომელიღაც ვიტამინის ნაკლებობას ნიშნავს და რომლის? წინასწარ დიდი მადლობა

Posted by: pataputina 31 Aug 2011, 23:26
გამარჯობა
მოკლედ არ ვიცი სად უნდა ვიკიტხო და ამიტომ აქ ვიკითხავ smile.gif

ვის შეუძლია დამაკვალიანოს სად და რა სახის აცრებია საჭირო აზიაში კერძოდ ინდოეთი, ჩინეთი ნეპალი და ა.შ. მოგზაურობისას?

როგორც ვიცი, ბევრი აცრაა საჭირო :შ


Posted by: Natuka NGN 1 Sep 2011, 12:32
Megi-meghan
შნეი პრობელმა შეიძლება გამოწვეული იყოს B ჯგუფის ვიტამინების ნაკლებობით. აგრეთვე, შეხედე ენას, თუ ენა გათეთრებულია, სავარაუდოდ სოკო გაქვს და მაგას მკურნალობა უნდა. ენაზე სოკომაც იცის მსგავსი ნახეთქები ტუჩის კუთხეებში,

pataputina
სადღაც წერდნენ მაგაზე. ვეღარ ვიპოვნე თემა. ისე, ინფექტიურში არ იციან ნეტა ან სამინისტროში? მგონი გარკვეული ჩამონათვალია.

Posted by: gramatikikina 1 Sep 2011, 19:55
Natuka NGN
ხომ არ იცი, კუჭის გამოკვლევა ენდოსკოპიის გარეშე შეიძლება?
ან ვინმემ იქნებ იცოდეს
მე მითხრეს არაო
ცოტა ხნის წინ გავიგე, რომ თითქოს არის რაღაც სხვა მეთოდიც, მაგრამ კონკრეტულად არაფერი ვიცი

Posted by: Natuka NGN 1 Sep 2011, 19:59
gramatikikina
ერთადერთი ბარიუმის დალევა ვიცი, ოღონდ მაგი აჩვენებს არის თუ არა წყლული მგონი. თორემ ზედაპირის მდგომარეობას ვერ გაიგებ. შევეცდები ხვალ დაგიდგინო.

Posted by: DarkPrince 1 Sep 2011, 21:12
მეგობრებო მაინტერესებს ტაფლი ტუ არის დიეტური ??? მადლობთ წინასწარ...

Posted by: F4R2 1 Sep 2011, 22:10
DarkPrince
QUOTE
მეგობრებო მაინტერესებს ტაფლი ტუ არის დიეტური ??? მადლობთ წინასწარ...

თაფლზე ალერგიული თუ არა ხარ ჭამე, ოღონდ ზომიერად, მაგაზე კარგს მაინც ვერაფერს მისცემ საკუთარ ორგანიზმს smile.gif

Posted by: აჯღაბუჯღა 1 Sep 2011, 22:28
ჩემ შვილს გადავუღე კომპიუტერული ტომოგრაფია და ქალას შიდა წნევა აქვს. რა საშველი აქვს ამ მდგომარეობას? 10 წლისაა და გაიზრდება და მოურჩებაო, მაგრამ სანამ გაიზრდება მუდმივად ტკივა თავი. დაუნიშნეს შარდმდენები რომელმაც არ უშველა, რა შეიძლება გაკეთდეს?

Posted by: Natuka NGN 2 Sep 2011, 00:12
აჯღაბუჯღა
გოგოა თუ ბიჭი? ერტ უცნაური რამეს ვიტყვი. მეც მაწიხებდა ბავშვობაში და ზუსტად მსგავსი დიაგნოზი მქონდა. 12 წლისას დამისვეს კიდევ დამატებიტ რაღAც უბედურებები. ჰოდა, იყო ასეთი ანგიოლოგი გიორგი ნაცვლიშვილი. ჩემმა მშობლებმა მიმიყვანეს სულ სხვა "აღმოჩენილი" პრობლემის გამო. უყურა ჩემეს უყურა და ტავში ჭკუა არ გაქვთ არცერთსო smile.gif რას აწვალებთ ამ ბაღანეს ამ თმებით, შეჭერითო, მეტად თამამი დიაგნოზია ბავშვისთისო. მართლაც ძლიან გრძელი თმა მქონდა. თმის შეჭრის შემდეგ თავის ტკივილი აღარ მქონია.

Posted by: აჯღაბუჯღა 2 Sep 2011, 01:23
Natuka NGN
რა უცნაურია. smile.gif გოგოა და გრძელი თმა აქვს, რომელსაც არ მაჭრევინებს ცოცხალი თავით, არც მავარცხნინებს. ახტაჯანა მაიმუნია. ვცდი ერთი, ვნახოთ რა იქნება. გმადლობ smile.gif
საფეთქლები გტკიოდა შენც?

Posted by: Natuka NGN 2 Sep 2011, 01:37
აჯღაბუჯღა
საფეთქლები ძირითადად. მთელი თავიც.
აუ, კლავიატურას გადასცილდა წარწერები და რას ვბეჭდავ...
თუ სულ არ შეგაჭრევინებს, შეუმოკლე მაინც და არ დაუქაჩო. ანუ ზედმეტი რეზინა და დაკაჩვა არ უნდა. თმა ძლაინ მძიმეა ისე. თუ ხშირია კიდევ უფრო.

Posted by: monchita 2 Sep 2011, 01:44
gramatikikina
ენდოსკოპიის გარდა გამოიყენება ბარიუმის ფაფით კონტრასტირება, მაგრამ ენდოსკოპიას ცხადია ვერ შეცვლისsmile.gif
* * *
QUOTE
ერთადერთი ბარიუმის დალევა ვიცი, ოღონდ მაგი აჩვენებს არის თუ არა წყლული მგონი. თორემ ზედაპირის მდგომარეობას ვერ გაიგებ

ვერ აჩვენებს რა თქმა უნდა, ბარიუმით მარტო ფუნქციური მდგომარეობის შეფასებაა შესაძლებელი, ზოგადად წყლულის დიაგნოსტიკისათვის ენდისკოპია ოქროს სტანდარტია და შეუცვლელია

Posted by: gramatikikina 2 Sep 2011, 15:51
monchita
Natuka NGN
კაი, მივალ და ალბათ ორივე ექნებათ. ესე მგონია ძალით მტენიან ამ ენდოსკოპიას

Posted by: აჯღაბუჯღა 2 Sep 2011, 20:42
Natuka NGN
ვეცდები რამენაირად დავიყოლიო. რეზინს კი არ იკარებს თავზე, მაგრამ მაინც.
დიდი მადლობა smile.gif

Posted by: DarkPrince 2 Sep 2011, 20:59
QUOTE (აჯღაბუჯღა @ 1 Sep 2011, 22:28 )
ჩემ შვილს გადავუღე კომპიუტერული ტომოგრაფია და ქალას შიდა წნევა აქვს. რა საშველი აქვს ამ მდგომარეობას? 10 წლისაა და გაიზრდება და მოურჩებაო, მაგრამ სანამ გაიზრდება მუდმივად ტკივა თავი. დაუნიშნეს შარდმდენები რომელმაც არ უშველა, რა შეიძლება გაკეთდეს?

გმერტმა დაგეხმაროს !!! ისე საკაიფო ავატარი გაქვს !!! Depeche mode-ს მსმენელი ახრ ???

Posted by: აჯღაბუჯღა 2 Sep 2011, 21:34
DarkPrince
კი, მაგის ხათრით დავაყენე. ოღონდ ხანდახან გოგა ხაინდრავა გონიათ ხოლმე და მეჩხუბებიან gigi.gif

:ოფტოპიკ:

Posted by: Megi-meghan 3 Sep 2011, 13:41
თუ შეგიძლიათ ვინმემ მითხრათ ცხვირის ნიჟარების ჰიპერტროფიის მორჩენა რამდენად ადვილია, დაახლოებით რამდენ ხანში შეიძლება მისი მორჩენა?წინასწარ დიდი მადლობა

Posted by: REZiKA87 4 Sep 2011, 00:10
ხაალხ.. მოკლედ მგონი თირკმელები მტკივა და წნევაც მომატებული მაქვს (130-140 - 80-ზე) რისი ბრალი შეიძლება იყოს?

Posted by: CrazyForFeeling 7 Sep 2011, 17:58
მაინტერესებს რა დაავადებები გადადის ნერწყვით?

Posted by: DarkPrince 7 Sep 2011, 18:58
QUOTE (CrazyForFeeling @ 7 Sep 2011, 17:58 )
მაინტერესებს რა დაავადებები გადადის ნერწყვით?

აი ამ კითხვაზე იტყვიან "ვაპროს ნა მილიონ" smile.gif ძალიან ბევრი დაავადება გადადის ნერწყვით...და თუ შენ გაინტერესებს სერიოზული დაავადებები...ჰეპატიტები (ა ჰეპატიტის ე.წ. "ბოტკინი" გარდა) არ გადადის ნერწყვით თუ პირის ღრუში არ გაქვს რია ჭრილობა, შიდს-იც მასეა...არ გადადის !!!

Posted by: CrazyForFeeling 7 Sep 2011, 20:00
QUOTE
ჰეპატიტები (ა ჰეპატიტის ე.წ. "ბოტკინი" გარდა) არ გადადის ნერწყვით


ზუსტად ეს მაინტერესებდა

გმადლობ smile.gif

Posted by: Tommy Guerrero 10 Sep 2011, 01:33
http://forum.ge/?f=15&showtopic=34299257

Posted by: gramatikikina 11 Sep 2011, 13:32
QUOTE
2 კვირის წინ მაწუხებდა თავის ძლიერი ტკივილი, უხასიათობა , ჭამის მადა მქონდა დაკარგული წონაშიც დავიკელი


ახლა ისევ დამეწყო მახველებს და ღამე ისე მცხელა საბანს ვერ ვიფარებ
ექიმთან მივიდე თუ ისევ გამივლის?
საშიშია რამე?

ამას უპასუხეთ რა

Posted by: buskunia 15 Sep 2011, 23:52
ალბათ აქ უნდა დავსვა ეს კითხვა...

მოკლედ რამდენჯერმე დამემართა ასეთუ რაღაც: თავიდან ყურები დამიგუბდა,მერე თითქოს თვალებში მომაწვა სისხლი და ჯერ ყველაფერი გათეთრდა, შემდეგ გაშავდა,ფაქტობრივად ვერაფერს ვხედავდი ( ეს ხდება სულ რამდენიმე წამის განმავლობაში,დაახლოებით 10-20 წამი) თავი ისე გამიხურდა ასე მეგონა ცეცხლი მეკიდა... 3ჯერ დამემართა ცეკვაზე როცა დავდიოდი,(მაგის გამო გამოვედი კიდეც sad.gif 3-4ჯერ ხან ეკლესიაში,ხან კიბეებზე ამოსვლისას... დავაკონკრეტებ იმასაც,რომ ერთ პერიოდში არ მომხდარა,პირველად სადღაც 12-13 წლის ასაკში მომივიდა ასეთი რაღაც,მეორედ 5-6 წლის შემდეგ ზემოთ აღნიშნულ სიტუაციებში

დღეს დილითაც განმეორდა,ახალი ამდგარი ვიყავი,მაღაზიაში ჩავედი და ისე მოხდა რომ კიბეზე ფეხი დამისრიალდა და დავეცი )))) თავი არ დამირტყავს და სერიოზულად არაფერი მტკენია..ჩემითვე წამოვდექი და გამოვიარე,მაგრამ სადღაც 10 წამში ვიგრძენი ყურების დაგუბება,თვალებში დამიბნელდა და ძლივს ამოვაღწიე.. სიარული რომ აღარ შემეძლო,ჩავიკუზე,თავი დავხარე ძირს და რამდენიმე წამში ვიგრძენი,რომ დამიბრუნდა მხედველობაც და შემდგომ სმენაც


რა შეიძლება იწვევდეს???

sad.gif

Posted by: Megi-meghan 25 Sep 2011, 13:28
თუ შეგიძლიათ ვინმემ მითხრათ ფეხის ქუსლებში დაგროვებული მარილების მოშორება თუ არის შესაძლებელი და როგორ?? წინასწარ დიდი მადლობა!

Posted by: ravi_shen 26 Sep 2011, 21:38
არ ვიცი სად უნდა ვიკითხო და ამიტომ აქ ვიკითხავ,
მოკლედ ჩემს სოფელში ამ ბოლო დროს ძალიან ბევრი ადამინი კვდება სიმსიმვნით,
ბაბუაჩემი ფილტვების სიმსვნით გარდაიცვალამ,
ბებიაჩემს ახლა კუჭის სიმსიმვნე დაუდგინეს..
ასევე სამეზობლოშიც ძალიან ბევრი ვიცი რომ სიმსიმვნით გარდაიცვალა..
მაქვს ეჭვი რომ ან წყლის ბრალი შეიძელბა იყოს ან მიწის ან რა ვიცი რისი, შეიძლება ჩერნობილის ტრაგედიის ბრალიც არის ნუ მოკლედ აშკრააა რომ ის მიდამო გამოსაკვლევია..
ვის შეიძელბა რომ მივმართო და ვინ ეხება ეს?

Posted by: qeto555 27 Sep 2011, 00:12
ზურგი მაწუხებს უკვე თვეა :| აი მხარს რომ უკან გავწევ ან გვერდულად შევბრუნდები მტკივა. რისი ბრალი უნდა იყოს ან ვის მივმართო? :|

Posted by: pleita 28 Sep 2011, 18:16
გამარჯობა, ე.ი დამინიშნეს მედიკამენტები: "თიორელი" და "არკოქსია" . მაინტერესებს ალკოჰოლის მიღება სასტიკად აკრძალულია მათი მიღების დროს?? თუ დიდად არ აქვს გავლენა??
გმადლობთ

Posted by: Megi-meghan 8 Oct 2011, 18:21
ქრონიკულ ცისტიტს უშედეგოდ ვმკურნალობ 2წელია.ავადმყოფობა მიმწვავდება მკურნალობის კურსის დამთავრებისთანავე. შეიძლება თუ არა ცისტიტი გამოწვეული იყოს სისხლში რაიმე მიკრობის არსებობით?წინასწარ დიდი მადლობა!
* * *
კიდევ რაგაც მაინტერესებს!შეიძლება თუ არა ფიელონეფრიტი გამოწვეული იყოს სეფსისისაგან??

Posted by: Megi-meghan 10 Oct 2011, 19:15
თუ შეგიძლიათ ვინმემ მითხრათ ფეხის ქუსლებში დაგროვებული მარილების მოშორება თუ არის შესაძლებელი და როგორ?? წინასწარ დიდი მადლობა!

Posted by: ქეთუ 10 Oct 2011, 20:01
გაციებული ვარ რამოდენიმე დღეა და ყურები მაქვს დაგუბებული და გავქანდე ექიმთან თუ გაციების ბრალია? sad.gif

დავიტყანჯე 2დღეა sad.gif

Posted by: rock in rose 10 Oct 2011, 20:54
ქეთუ

იცის ეგ გაციებამ. მაგას უმკურნალე და ყურების დაგუბებამაც უნდა გაგიაროს, თუ არ გაგივლის მერე მიდი ექიმთან.

Posted by: Megi-meghan 11 Oct 2011, 18:37
თუ შეგიძლიათ ვინმემ მითხრათ ფეხის ქუსლებში დაგროვებული მარილების მოშორება თუ არის შესაძლებელი და როგორ?? წინასწარ დიდი მადლობა!

Posted by: Kate789 14 Oct 2011, 04:43
ბოლო ხანებია ლავიწის (უფრო მკერდისკენ, ვიდრე კისრისკენ) ძვალი მტკივა და რისი ბრალი შეიძლება იყოს???

დღეს რამდენიმეჯერ ამტკივდა. sad.gif

Posted by: m-009 15 Oct 2011, 20:18
კეფაზე,მხრებზე და ხელებზე თითქოს გირები დამადესო ისეთი შეგძნება მაქვს,წნევა რომ გავიზომე დაბალი მაქვს ,მაგრამ ისეთი დაბალი არა რომ ესეთი სიმპტომები მქონდეს, ხომ ვერ მეტყვით დაახლოებით რისი ბრალი უნდა იყოს?

Posted by: taamo 17 Oct 2011, 18:30
გამარჯობა. უკვე სამი დღეა გამუდმებით მაწუხებს გულისრევის შეგრძნება, მაგრამ ამ დროის განმავლობაში არ მქონია პირღებინება. იქნებ მითხრათ რომელ ექიმს მივმართო ან რისი ბრალი შეიძლება იყოს??( ღვიძლის და კუჭშეკრულობის ბრალი არაა, უკვე ვცდე ამისთვის გარკვეული საშუალებები, მაგრამ შედეგი არ გამოიღო)

Posted by: Megi-meghan 17 Oct 2011, 19:50
თუ შეგიძლიათ ვინმემ მითხრათ ფეხის ქუსლებში დაგროვებული მარილების მოშორება თუ არის შესაძლებელი და როგორ?? წინასწარ დიდი მადლობა!

Posted by: lurdes 17 Oct 2011, 22:55
გთხოვთ, მირჩიოთ პოლივიტამინები ხანშიშესული ადამიანისთვის
სასურველია არ იყოს ამერიკული წარმოების

Posted by: tete- 18 Oct 2011, 00:44
კანზე ნამზეურის შემდეგ გამიცნდა კოპლები( ჩემი ნამდვილი კანის ფერი) ეს რა უბედურებაა? რა წავისვა ხომ ვერ მირჩევთ თუ რამე მჭირს ვაFშე

Posted by: rock in rose 18 Oct 2011, 09:13
Te0

ლაქებია ეგ, ამიტომ არ ვარგა ნამზეური. დერმატოლოგთან უნდა მიხვიდეთ

Posted by: matilda123 18 Oct 2011, 21:05
QUOTE
http://forum.ge/?showtopic=34203346&f=&st=330&#entry27893546
ამ თემას იქნებ მიხედოთ?

Natuka NGN აქტიურად დაგეხმარები მაგრამ ახალი წლის მერე, ეხლა გამოცდისთვის ვემზადები , თან ვმუშაობ და სისტემატურად ვერ შევძლებ ფორუმზე პოსტვას. smile.gif))
გამოცდის მერე პრობლემაა არაა 2kiss.gif
PM-ში ვერ მოგწერე პმ-ბოქსი გადავსებული გაქვს.

Posted by: Megi-meghan 19 Oct 2011, 19:25
თუ შეგიძლიათ ვინმემ მითხრათ ფეხის ქუსლებში დაგროვებული მარილების მოშორება თუ არის შესაძლებელი და როგორ?? წინასწარ დიდი მადლობა!

Posted by: simebi 19 Oct 2011, 22:19
ომეგა 3 _ის მიღება მინდა, და ტაბლეტების ან რამე სახით არის რამე(ხარისხიანი)?

Posted by: Natuka NGN 19 Oct 2011, 22:58
matilda123
QUOTE
Natuka NGN აქტიურად დაგეხმარები მაგრამ ახალი წლის მერე, ეხლა გამოცდისთვის ვემზადები , თან ვმუშაობ და სისტემატურად ვერ შევძლებ ფორუმზე პოსტვას. ))
გამოცდის მერე პრობლემაა არაა 

დიდი მადლობაა 2kiss.gif
ისე მევსება ეს PM რომ მეც ვერ ავსწრებ გასუფთავებას biggrin.gif მოკლა ამ რეორტებმა განათლებიდან.

Posted by: matilda123 20 Oct 2011, 00:08
Megi-meghan
QUOTE
თუ შეგიძლიათ ვინმემ მითხრათ ფეხის ქუსლებში დაგროვებული მარილების მოშორება თუ არის შესაძლებელი და როგორ?? წინასწარ დიდი მადლობა!   შ

კი... თბილისში კამოს ქუჩაზე(დასაწყისში) იყო რევმატოლოგიის ცენტრი. იქ მუშაობდა სახელი ზუსტად აღარ მახსოვს მგონი ბაადურ რაჭველიშვილი (სახელი მართლა ზუსტად აღარ მახსოვს sad.gif(() ეგ ნიშნავდა მედიკამენტოზურ მკურნალობას რომელიც ამცირებდა მნიშვნელოვნად მაგ წანაზარდებს ( ე.წ. "დეზებს") .
გარდა ამისა არსებობს ლითოტრიპსია( ანუ მაგ კალციფიკატების დაშლა) სპეც. აპარატით, რაც საკმაოდ მტკივნეული პროცესია, მაგრამ შედეგიანი. მოშორება ოპერაციით, სპეციალური ორთოპედიული ფეხსაცმლების ტარება(ამცირებს ტკივილს)ან სუპინატორების ხმარება, (ოღონდ პროფესიონალურის).
* * *
m-009
QUOTE
ეფაზე,მხრებზე და ხელებზე თითქოს გირები დამადესო ისეთი შეგძნება მაქვს,წნევა რომ გავიზომე დაბალი მაქვს ,მაგრამ ისეთი დაბალი არა რომ ესეთი სიმპტომები მქონდეს, ხომ ვერ მეტყვით დაახლოებით რისი ბრალი უნდა იყოს?


პირველ რიგში ხერხემლის პათოლოგია(კისრის მალებისარეში) და სისხლის მიმოქცევის პათოლოგიაა გამოსარიცხი , პლუს ამას ფარისებრი ჯირკვლის ჰორმონებიც გამოსაკვლევია. ამისათვის გინდა ორთოპედი ან რევმატოლოგი , რომელიც გაგიშვებს რენტგენზე ( კისრი არის რენტგენი), სისხლის აღება,( ელექტროლიტები, შედედება, ფარისებრ.ჯირკვ.ჰორმონები ).
მოკლედ არ არის იოლი კითხვა, გააჩნია როგორი ცხოვრების წესი გაქვს, რამდენ სითხეს ღებულობ, სპორტს თუ ეწევი, სწორად თუ იკვებები და ა.შ. მიდი კარგ ინტერნისტთან და ის დაგაკვალიანებს smile.gif))

taamo
QUOTE
გამარჯობა. უკვე სამი დღეა გამუდმებით მაწუხებს გულისრევის შეგრძნება, მაგრამ ამ დროის განმავლობაში არ მქონია პირღებინება. იქნებ მითხრათ რომელ ექიმს მივმართო ან რისი ბრალი შეიძლება იყოს??( ღვიძლის და კუჭშეკრულობის ბრალი არაა, უკვე ვცდე ამისთვის გარკვეული საშუალებები, მაგრამ შედეგი არ გამოიღო)

თუ მარტო გულისრევაა გასტროენტეროლოგს. თუ გულისრევას თავბრუსხვევაც ახლავს მაშინ ყელ-ყურ-ცხვირის ექიმს.

Posted by: *tamtam* 20 Oct 2011, 03:05
სალამი, მოკლედ გწერთ მთელი ბუკეტით ჩივილების და იქნებ დამაკვალიანოთ საიდან დავიწყო smile.gif

1) პირველ რიგში მაწუხებს გულის "ჭვალვა" და პერიოდული ტკივილები, რამდენიმე დღის წინ 13 საათი მქონდა "ჭვალი" გულზე, ძლივს გამიარა, პერიოდულად მიმეორებს.

2) მაქვს სტრესი, უხასიათობა, დეპრესია და ა.შ.

3) მტკივა იღლიის ფოსო, ჯირკვალი თუ რაც არის.. პერიოდულად მაქვს ხოლმე და მერე გადამდის.


Posted by: kkoxo 22 Oct 2011, 05:03
ჰიპოკრატეს ფიცის დამცველნო :_P


მაქვს ძალიან ბევრი თმა მთელ სხეულზე ,განსაკUტრებით ბევრი მაქვს ზურგში ,ფიზიკურად არ ვარ გამხდარი მაგრამ წონაში ზოგადად ვერ ვიმატებ და ხომ არ დვაბრალო ჭარბთმიანობას ,ვფიქრობ რომ ვიტამინებს მაცლის ეს თმა ამსთან არ ვიპარსავ ამ თმებს და უკეთესი ხომ არ არის რომ დავიპარსო ?ამსთან მეშინია იმის რომ უფრო მეტი შეიძლება ამოვიდეს და უხეში ამასთან მიუხედავდ იმისა რომ ყოველDღე ვივლებ წყალს ზოგჯერ 2-ჯერაც მაინც მცვივა და დილას ლოგინში ვხედავ თმებს ჰა დამეხმარეთ ანუ რატომ მცვივა ეს თმა სხეულიდან და უკეთესი ხმ არ არის გავიპარსო ეს თმები ამასთან რამდენად სწორია ჩემი სეხედულება ,რომ ამდენი თმა ბევრ ენერგიას მოიხმარს


გმადლობთ .
ველოდები ტქვენს პასუხს

Posted by: matilda123 22 Oct 2011, 13:23
kkoxo
QUOTE
ჰიპოკრატეს ფიცის დამცველნო :_P

მაქვს ძალიან ბევრი თმა მთელ სხეულზე ,განსაკUტრებით ბევრი მაქვს ზურგში ,ფიზიკურად არ ვარ გამხდარი მაგრამ წონაში ზოგადად ვერ ვიმატებ და ხომ არ დვაბრალო ჭარბთმიანობას ,ვფიქრობ რომ ვიტამინებს მაცლის ეს თმა ამსთან არ ვიპარსავ ამ თმებს და უკეთესი ხომ არ არის რომ დავიპარსო ?ამსთან მეშინია იმის რომ უფრო მეტი შეიძლება ამოვიდეს და უხეში ამასთან მიუხედავდ იმისა რომ ყოველDღე ვივლებ წყალს ზოგჯერ 2-ჯერაც მაინც მცვივა და დილას ლოგინში ვხედავ თმებს ჰა დამეხმარეთ ანუ რატომ მცვივა ეს თმა სხეულიდან და უკეთესი ხმ არ არის გავიპარსო ეს თმები ამასთან რამდენად სწორია ჩემი სეხედულება ,რომ ამდენი თმა ბევრ ენერგიას მოიხმარს

გმადლობთ . ველოდები ტქვენს პასუხს

ჩაგეთვალა ... მაცინე გულიანად smile.gif))


Posted by: nica1990 25 Oct 2011, 13:12
Summer_Rain
მე მგონი ფსიქოლოგიც და აუცილებლა ნორმალური გასტროლოგი

Posted by: m-009 25 Oct 2011, 18:41
ადამიანის ნაკბენი საშიშია თუ არა? რაიმე აცრა საQიროა?

Posted by: tako44 25 Oct 2011, 23:16
დღეს დავაკვირდი, რომ B12-ის ამპულას აწერია 1 მგ ანუ 1 გრამია ნემსში, მაგრამ დანიშნული მაქვს ექიმისაგან 200იანი, ხოდა აფთიაქში რომ ვიკითხე დღეს , ასე მითხრეს, რომ ესო (ანუ ეს ერთგრამიანი ამბულა) არისო 500იანიო, რომელიც კეთდება 5 დღეში ერთხელო, 200იანიო, რაც შენმა ექიმმა დანიშნა, დეფიციტშიაო, ხოდა დავიბენი და ვინც ერკვევით, გამარკვიეთ რაა smile.gif)

Posted by: matilda123 26 Oct 2011, 00:06
tako44
QUOTE
დღეს დავაკვირდი, რომ B12-ის ამპულას აწერია 1 მგ ანუ 1 გრამია ნემსში, მაგრამ დანიშნული მაქვს ექიმისაგან 200იანი, ხოდა აფთიაქში რომ ვიკითხე დღეს , ასე მითხრეს, რომ ესო (ანუ ეს ერთგრამიანი ამბულა) არისო 500იანიო, რომელიც კეთდება 5 დღეში ერთხელო, 200იანიო, რაც შენმა ექიმმა დანიშნა, დეფიციტშიაო, ხოდა დავიბენი და ვინც ერკვევით, გამარკვიეთ რაა smile.gif)

1 მგ 1 გრამი არ არის, 1 მგ 0,001 გრამია, რადგან 1 გრამში 1000 მილიგრამია. ანუ 1 მილიგრამი = 1000გრამი და ექიმისგან რაც გაქვთ დანიშნული (200 გრამი თუ მილიგრამი) ამის მიხედვით გამოთვალეთ.
smile.gif))

Posted by: SPC 28 Oct 2011, 11:35
ექიმმა ანტიბიოტიკი დამინიშნა, დღეს პირველი დავლიე, სახელი არ მახსოვს ზ-ზე იწყება, ეხლა სამსახურში ვარ . ჭამამდე 1 საათით ადრეო, მოკლედ დვალიე და სადღაც 1 საათის და 20 წუთის მერე გამომივიდა ჭამა. ხოდა მუცელი ამტკივდა საშინლად, რაღაც მოვლითი ტკივლები მაქვს . ვაბშე შესაძლებელია რომ ასე ჭამამდე დალიო ანტიბიოტიკი? sad.gif( კუჭი გამიღიზიანა მგონი საშინლად.....მერე ცოტა გამიარა და ყავა და შოკოლადი ვჭამე (ტრადიციაა, როგორ დავარღვევდი biggrin.gif ) ხოდა ეხლა ისვე ამტკივდა.... ანუ ზოგადად მაინტერესებს რამდენად ნორმალურია ანტიბიოტიკის დალევა ჭამამდე? ან ეხლა ეს გამივლის თავისით? ყავა აღარ გავაგრძელო?

kkoxo
QUOTE
,ვფიქრობ რომ ვიტამინებს მაცლის ეს თმა
,რომ ამდენი თმა ბევრ ენერგიას მოიხმარს
ველოდები ტქვენს პასუხს


დალი ხარ ფაქტიურად biggrin.gif

Posted by: Natuka NGN 28 Oct 2011, 11:46
SPC
ანტიბიოტიკი ჭამამდე უნდა დაილიოს ან წამიდან 2-3 საათის შემდეგ. ყავა და შოკოლადი საკვები არაა. უნდა მიიღო მინიმუმ ხაჭო და პური. ანტიბიოტიკები იწვევენ დისბაქტერიოზს და ამ დროს ალქტობაქტერიების მიშველებაა საწირო, თუ ფაღარათო დაგეწყო, აუცილებელია, რომელიმე ჯგუფის ლაქტობაქტერიის მიღება.

Posted by: SPC 28 Oct 2011, 11:54
Natuka NGN
დიდი მადლობა,
ყავა მერე დავლიე, მანამდე ყიყლიყო ვჭამე , მეთქი უბრალოდ ყავამ კიდევ თავისი ხომ არ დაამატა მეთქი.
კიდევ ექვს დღეში უნდა დავლიო და 12ში, და ვფიქრობ 1 სათი არ დაველოდო და 40 წუთში ვჭამო

Posted by: france 1 Nov 2011, 15:56
არ ვიცი სად დამესვა შეკითხვა.

რა მაინტერესებს იცით ვერცხლის წყლის თერმომეტრისთვის ნორმალური ტემპერატურა (იღლიაში) 36.6 და ელექტრო თერმომეტრის შემთხვევაში 36.1?
ელექტრო თერმომეტრი ვიყიდე პირველად და აწერია 36.1 არის ნორმალური იღლიაშიო და დამაინტერესდა ვერცხლის წყლიანი თერმომეტრით რომ ვიზომავდი 36.6 ვთვლიდი ნორმალურ ტემპერატურად.

Posted by: kato_stropa 1 Nov 2011, 16:39
რა მინდა გკითხოთ:
დაახლ.12-13 წელია უშაქროდ ვსვამ ყავასაც,ჩაისაც და ზოგადად,ნაკლებშაქრიან წვენებს ვეტანები,უბრალოდ არ მიყვარს შაქარი.მაგრამ ბოლო რამდენიმე დღეა საშინელი მიდრეკილება გამიჩნდა-ჩაიში 2კოვზ შაქარს ვიყრი და ისე სიამოვნებით ვსვამ user.gif ყავასაც,რაღაც მომეწონასავით..რატო? გადავრჩები? sad.gif

Posted by: matilda123 1 Nov 2011, 19:06
france
გაზომვა შეიძლება რექტალურად, ენის ქვეშ, იღლიაში, ყურშიც..
36,1- 37,4 ნორმად თვლიან. 38 მდე თU აიწია სუბფებრილურია. 38,4 ზომიერი. 38,5-40,4 მაღალი. 40,5-დან ზემოთ ექსტრემალური ტემპერატურაა(ჰიპერპირექსია).
მიუხედავად იმისა თუ როგორია თერმომეტრი, ნორმა ყველასთვის ერთია. თუმცა საკმაოდ ვარირებს ზემოთაც ჩანს.

Posted by: france 1 Nov 2011, 19:33
matilda123

დიდი მადლობა. კარგად ამიხსენით მართლა : )

ანუ მე ახლა მაქვს 36.8 და ყელი მტკივა და ეს მაინც ნორმად ითვლება? გუშინ 38.1მდე ავიდა საღამოს და პარაცეტამოლი დავლიე და დღეს უკვე 37.3ს არ ასცილებია, მაგრამ პარაცეტამოლი ისევ დავლიე. იმედია სწორად ვიქცევი.

Posted by: matilda123 1 Nov 2011, 20:50
QUOTE
ანუ მე ახლა მაქვს 36.8 და ყელი მტკივა და ეს მაინც ნორმად ითვლება? გუშინ 38.1მდე ავიდა საღამოს და პარაცეტამოლი დავლიე და დღეს უკვე 37.3ს არ ასცილებია, მაგრამ პარაცეტამოლი ისევ დავლიე. იმედია სწორად ვიქცევი.

france
37,4 ნორმაა . პარაცეტამოლს გარდა სიცხის დამწევისა აქვს ტკივილგამაყუჩებელი და სუსტი ანთების საწინააღმდეგო მოქმედებაც. თუ ყელის ტკივილმა 3 დღეზე მეტხანს გასტანა ექიმს მიმართე რომ ანტიბიოტიკი დაგინიშნოს.

Posted by: konkretula 3 Nov 2011, 18:26
შეგიძლიათ ვინმემ ამიხსნათ , რატომ მმატებს მადას ასკორუტინი?? დღეში ერთ ტაბლეტს ვსვავ და ისეთ მადაზე მოვყავარ ბოლო 20 დღის მანძილზე 3 კილო მოვიმატე... როდესაც აქამდე ბევრჯერ მიცდია წონაში მომატება მაგრამ არანაირი ვიტამინები და მსგავსი ძვირადღირებული წამლები არ მოქმედებდნენ ჩემს მადაზე.

არადა ფარმაცევტმა მითხრა ასკორუტინს მადასთან მომატებაში არანაირი კავშირი არ აქვს და სრულიად სხვა დანიშნულებით სვავენო... :|

Posted by: matilda123 4 Nov 2011, 21:41
QUOTE
შეგიძლიათ ვინმემ ამიხსნათ , რატომ მმატებს მადას ასკორუტინი?? დღეში ერთ ტაბლეტს ვსვავ და ისეთ მადაზე მოვყავარ ბოლო 20 დღის მანძილზე 3 კილო მოვიმატე... როდესაც აქამდე ბევრჯერ მიცდია წონაში მომატება მაგრამ არანაირი ვიტამინები და მსგავსი ძვირადღირებული წამლები არ მოქმედებდნენ ჩემს მადაზე.

არადა ფარმაცევტმა მითხრა ასკორუტინს მადასთან მომატებაში არანაირი კავშირი არ აქვს და სრულიად სხვა დანიშნულებით სვავენო... :|


ეს ფარმაცევტებს უნდა ჰკითხო smile.gif) შენ უცოდინარი ფარმაცევტი შეგხვედრია. ასკორუტინის შემადგენლობაში შედის ასკორბინის მჟავა იგივე ვიტამინი "ცე", რომელიც ამაღლებს კუჭის წვენის მჟავიანობას.

Posted by: MOGZAVNILI-2 7 Nov 2011, 15:04
შარდვის შემდეგ ისეთი შეგრძნება მიჩნდება თითქოს რაღაც მძიმე ჩამოეკიდა შარდის ბუშტსო და რისი სიმპტომია ეგ? თან შრდს ცუდი სუნი უდის ძაან

Posted by: Leoner 10 Nov 2011, 18:38
პურნოლოგი მირჩიეთ რა ვინმე კარგი თბილისში.

Posted by: dito_Metalhead 10 Nov 2011, 19:41
ხალხნო ნეკნის მოტეხილობა სერიოზულია? ოპერაცია ან რამე ეგეთი ხო არ ჭირდება? sad.gif

Posted by: dito_Metalhead 11 Nov 2011, 20:01
მიპასუხეთ რა ვინმემ :/

303030

Posted by: matilda123 12 Nov 2011, 20:00
dito_Metalhead
QUOTE
ხალხნო ნეკნის მოტეხილობა სერიოზულია? ოპერაცია ან რამე ეგეთი ხო არ ჭირდება?

თუ ნეკნი მოტეხილობისას რაიმე შინაგან ორგანოს აზიანებს მაშინ კი, თუ არადა არა . იმასაც გააჩნია რამდენი ნეკნი გაქვს მოტეხილი.

Posted by: Leoner 13 Nov 2011, 19:42
ბრონქიტის ექიმი ვინმემ იცით კარგი ?

Posted by: MOGZAVNILI-2 14 Nov 2011, 16:58
ამ თემას პატრონი არ ჰყავს გაგვეცით ვინმემ პასუხი რა

Posted by: buub 15 Nov 2011, 14:44
ერთი თვეა მახველებს სიცხე არ მაქვს სისხლის ანალიზი გავიკეთე ლეიკოციტები მაქვს მომატებული რენდგენზე ფილტვები სუფთაა ნუ ფილტვების ანთება არ არის ექიმმა სუმამედი დამინიშნა ხოდა აქ რომელიღაც თემაში წავიკითხე "საშინელი ანტიბიოტიკია მთლინად ანგრევს ორგანიზმსო" ხოდა რას ნიშნავს ეს შემეშინდა რაგაც მაგის დალევა

Posted by: matilda123 15 Nov 2011, 20:43
MOGZAVNILI-2
QUOTE
შარდვის შემდეგ ისეთი შეგრძნება მიჩნდება თითქოს რაღაც მძიმე ჩამოეკიდა შარდის ბუშტსო და რისი სიმპტომია ეგ? თან შრდს ცუდი სუნი უდის ძაან


ცისტიტის სიმპტომებს ჰგავს. შარდის ზოგადი ანალიზი უნდა გაკეთდეს პლუს უნდა ინახოს პათოგენური ფლორა შარდში. მოკლედ გამომწვევი უნდა დადგინდეს. მიმართე უროლოგს, სამწუხაროდ მეტს ვერაფერს გეტყვი, მე უროლოგი არ ვარ.
* * *
buub
QUOTE
ერთი თვეა მახველებს სიცხე არ მაქვს სისხლის ანალიზი გავიკეთე ლეიკოციტები მაქვს მომატებული რენდგენზე ფილტვები სუფთაა ნუ ფილტვების ანთება არ არის ექიმმა სუმამედი დამინიშნა ხოდა აქ რომელიღაც თემაში წავიკითხე "საშინელი ანტიბიოტიკია მთლინად ანგრევს ორგანიზმსო" ხოდა რას ნიშნავს ეს შემეშინდა რაგაც მაგის დალევა

როცა ხანგრძლივად გახველებს, ფაქტია, რომ ფილტვებში ან ბრონქებში მთლად რარაც არ არის წესრიგში. ლეიკოციტების მომატება ანთების მიმანიშნებელია. 2 კვირაზე მეტად გახანგრძლივებული ხველა პროცესის ქრონიკულობაზე მიგვანიშნებს.მანდ ვინც გყავს ექიმი, მას აქვს საშუალება, გაგსინჯოს, სისხლის ანალიზს გაეცნოს, რენტგენის სურათი ნახოს, მოგისმინოს და ჩვენ მხოლოდ შენს მონაყოლს ვეყრდნობით და ასე ძნელია დიაგნოზის დასმა და მკურნალობა. გააკეთე ის რაც ექიმმა გითხრა.

Posted by: an apple 15 Nov 2011, 22:26
ლეიკოციტები 4,97 ნორმა 5,2_12,4 ერითროციტები 3,96 ნორმა4,2-5,4 ჰემოგლობინი 12,3 ნორმ 12_16, ჰემატოკრიტი 32,1 ნორმა37-47 ,ერითროციტების საშ მოცულობა 81,1 ნორმა81-99, ჰემოგლობინის საშ მოცულობა 30,9 ნორმა 27 -31, ჰემოგლობინის საშ კონცენტრაცია 38,1 ნორმა 27_31 ,კონცენტრაცია უჯრედში 32 ნორმა 33-37, ჰემოგლ კორპუსული შემადგენლობა 25,9 ნორმა 28,624-35, ერითროციტების გავრცელების ფართი 12,4 ნორმა 11,5-14,5 ,ჰემოგლ გავრცელების ფართი 2,46 ნორმა 2,2-3,2 . ანემია მაქვს? სამკურნალო ვარ? თუ კი ვის უნდა მივმარტო

Posted by: m-009 15 Nov 2011, 23:50
4 დღის წინ კონსერვის გახსნის დროს ხელი გავიჭერი,პატარაზე,უმნიშველოდ,ტეტანუსი მჭირდება?

Posted by: buub 16 Nov 2011, 21:19
დღეს დავლიე ანტიბიოტიკი სუმამედი ხოდა ამტკივდა კუჭი მერე წავიკითხე და გვერდით მოვლენებში ნახსენები იყო კუჭის ტკივილი ჩემ ექიმს ვერ დავუკავშირდი ხოდა ჯობია შევწყვითო თუ გავაგრძელო მაგის დალევა?

Posted by: matilda123 16 Nov 2011, 22:03
buub
QUOTE
დღეს დავლიე ანტიბიოტიკი სუმამედი ხოდა ამტკივდა კუჭი მერე წავიკითხე და გვერდით მოვლენებში ნახსენები იყო კუჭის ტკივილი ჩემ ექიმს ვერ დავუკავშირდი ხოდა ჯობია შევწყვითო თუ გავაგრძელო მაგის დალევა?

ანტიბიოტიკი კუჭს იშვიათად რომ ატკიებდეს, ღვიძლი ან ნაღველი შესაძლოა რომ გააღიზიანოს, მაგრამ შეწყვეტა არ შეიძლება, ან უნდა გააგრძელო ან სხვა ანტიბიოტიკით ჩანაცვლდეს. თუ მაინც თვლი რომ კუჭია, ომეპრაზოლი დალიე 20მგ დღეში ერთხელ წამლის მიღებამდე ნახევარი საათით ადრე.

Posted by: buub 17 Nov 2011, 00:08
matilda123
კუჭია ნამდვილად და მადლობაა

Posted by: m-009 17 Nov 2011, 00:39
4 დღის წინ კონსერვის გახსნის დროს ხელი გავიჭერი,პატარაზე,უმნიშველოდ,ტეტანუსი მჭირდება?

Posted by: ane mekeshi 17 Nov 2011, 10:50
ყველაზე კარგი ოტოლარინგოლოგი მირჩიეთ, რა.... სერიოზული ქირურგიული ჩარევაა საჭირო და მაინტერესებს, თქვენი აზრით, ვისი ნდობა შეიძლება.

Posted by: palindromi-x 18 Nov 2011, 11:33
თავის დაბანის შემდეგ გამიცივდა თავი, რამაც გამოიწვია თავის ტკივილი, თვალის ხამხამი,ჩხვლეტა ზურგის არეში, თვალების აცრემლება, ტემპერატურა 37-37,3. თავის ტკივილმა და თვალის ხამხამმა გამიარა, ტემპერატურა რჩება 37, თვალები ოდნავ მაინც მეცრემლება. ხომ ვერ მირჩევთ რა მივიღო?

Posted by: Natuka NGN 18 Nov 2011, 11:40
palindromi-x
დოქტორ ტაისის ეხინაციის წვეთები.

Posted by: palindromi-x 18 Nov 2011, 12:20
Natuka NGN
გმადლობთ ყურადღებისთვის

Posted by: natnatnati 20 Nov 2011, 23:13
გამარჯობათ,იქნებ დამეხმაროთ. დედაჩემს აქვს ბარძაყზე სიმსივნე-ნეიროფიბრომა,ამჟამად უკვე სისხლმდენია,უკვე 4ჯერ ჰქონდა სისხლდენა ამ სიმსივნიდან. საერთოდ 15 წელია ამავე ფეხზე აქვს ღრმა ვენის შევიწროება და ყოველდღიურად იღებდა ვარფარინს,რაც ეს სისხლდენები დაეწყო (1 თვეა) ვარფარინი აღარ მიუღია.კოაგულოგრამა ნორმალური აქვს,პროტრომბინი 75 აქვს,ინერი 1.2 . სისხლდენის პერიოდში ონკოლოგმა დაუნიშნა დიცინონი დღეში სამჯერ,და გვაინტერესებს თუ შეიძლება რომ გააგრძელოს ვარფარინის მიღება? უფრო სწორად დიცინონთან ერთად მიიღოს თუ არა? სისხლდენას ხო არ გამოიწვევს? იქნებ მირჩიოთ,წინასწარ გიხდით მადლობას

Posted by: simebi 21 Nov 2011, 13:45
6 თვის განმავლობაში ვეწეოდი სიგარეტს, დღეში საშუალოდ 2-3 ღერს , და ეხლა სულ დავანებე თავი, და ყველაფერი რაც დაგროვდა თუ დაგროვდა საერთოდ რამე, გამოვა ორგანიზმიდან, ან რამეზე მოახდენდა გავლენას?

Posted by: tos-ka 21 Nov 2011, 14:26
იქნებ ურჩიოთ რამე?
http://forum.ge/?f=82&showtopic=34325070

Posted by: teoni 24 Nov 2011, 19:53
შესაბამისი თემა ვერ ვიპოვე და აქ ვიკითხავ თუ შეიძლება,იქნებ ვინმემ იცოდეს პასუხი.ჩემს მეგობარს უბუჟდებოდა სახე,მარჯვენა ნაწილი,ხან გრძნობაც არ ქონდა,ცოტა თავის ტკივილიც.მიზეზი რომ დაედგინათ ზურგის ტვინიდან აუღეს სითხე,იქიდან მოყოლებული აქვს საშინელი თავის ტკივილები,სულ ეძინება.ვითომ სხვა გზა არ არსებობდა ხერხემალს რომ არ სცემოდნენ?

Posted by: natnatnati 24 Nov 2011, 20:47
ვარ 17 წლის, ამ ბოლო დროს პერიოდულად მტკივა ჭიპთან მარჯვენა მხარეს ზოლისებურად, ტკივილი მიძლიერდება ამოსუნთქვის დროს. თუ მოვიხრები მიყუჩდება და რომ ვსწორდები მტკივა და თითქოს მეჭიმება, დაახლობით 1 საათში გამივლის. რისი ბრალი შეიძლება იყოს, იქნებ მირჩიოთ რამე?

Posted by: matilda123 25 Nov 2011, 03:31
teoni
QUOTE
შესაბამისი თემა ვერ ვიპოვე და აქ ვიკითხავ თუ შეიძლება,იქნებ ვინმემ იცოდეს პასუხი.ჩემს მეგობარს უბუჟდებოდა სახე,მარჯვენა ნაწილი,ხან გრძნობაც არ ქონდა,ცოტა თავის ტკივილიც.მიზეზი რომ დაედგინათ ზურგის ტვინიდან აუღეს სითხე,იქიდან მოყოლებული აქვს საშინელი თავის ტკივილები,სულ ეძინება.ვითომ სხვა გზა არ არსებობდა ხერხემალს რომ არ სცემოდნენ?

გაბუჟება ნერვების დაზიანებასთანაა კავშირში როგორც წესი. ზურგის ტვინის სითხით ბევრი რამის გარკვევა შეიძლება და ალბათ ამიტომ აუღეს. თუმცა ჩემი აზრით იმდენ დაავადებას შეუძლოა მაგის გამოწვევა, რომ ჯერ სისხლის გამოკვლევით უნდა დაეწყოთ.
natnatnati
QUOTE
ვარ 17 წლის, ამ ბოლო დროს პერიოდულად მტკივა ჭიპთან მარჯვენა მხარეს ზოლისებურად, ტკივილი მიძლიერდება ამოსუნთქვის დროს. თუ მოვიხრები მიყუჩდება და რომ ვსწორდები მტკივა და თითქოს მეჭიმება, დაახლობით 1 საათში გამივლის. რისი ბრალი შეიძლება იყოს, იქნებ მირჩიოთ რამე?

რჩევა ის იქნება, რომ ექიმთან მიდი. ის გაგსინჯავს, გაგიკეთებს სისხლის ანალიზს, მუცლის ღრუს ექოსკოპიას და ა.შ. თანმიმ დევრულად და დაგისვამს დიაგნოზს.

Posted by: teoni 26 Nov 2011, 16:21
matilda123
მადლობა პასუხისთვის.რავიცი იმედია არაფერი დაუშავდება .თან გერმანიაში ხდება ეს,საქართველოში კი არა vis.gif vis.gif

Posted by: matilda123 26 Nov 2011, 19:23
teoni

QUOTE
მადლობა პასუხისთვის.რავიცი იმედია არაფერი დაუშავდება .თან გერმანიაში ხდება ეს,საქართველოში კი არა

უშუალოდ ლუმბალური პუნქციის მერე თავის ტკივილები ხშირია, რადგან თავის ქალის წნევა იზრდება. დამოკიდებულია იმაზეც რამდენი სითხე აიღეს, ბევრი არ შეიძლება. ეხლა მაგას სჭირდება წოლითი რეჟიმი, ბევრი სითხე და ანალგეტიკები და მოცდა, ტკივილები შეიძლება დიდანს გაგრძელდეს. თუ ძალიან აუტანელია უეჭველი მიმართეთ ექიმს, სტაციონალური მკურნალობა შეიძლება დასჭირდეს რამოდენიმე დღე.
რომ ამბობ გერმანიაში ხდება, შენ რა გგონია გერმანიაში ყველა ექიმი მაგარია?! აქაც არიან გამოუცდელი ექიმები , თან ძაან ბევრი, მაგრამ არ ინერვიულოთ ყველაფერი გაუვლის დროსთან ერთად.

Posted by: Pinturico 27 Nov 2011, 18:32
ბოდიში თუ არასწორ ადგილას ვპოსტავ, უბრალოდ შესაბამისი თემა ვერ ვიპოვე:

შეგიძლიათ მირჩიოთ სად შემიძლია გავიკეთო სქესობრივი გზით გადამდებ დაავადებებზე სრული გამოკვლევა. რომელია ამ მხრივ წამყვანი კლინიკა თბილისში? ან თუ ერთის მითითება რეკლამად ითვლება, რამდენიმე მირჩიეთ. ვიმეორებ, მჭირდება სრული გამოკვლევის ჩატარება ყველა შესაძლო დაავადებაზე.

Posted by: simebi 29 Nov 2011, 11:39
შეიძლება აქ არ უნდა მეკითხა : ადრე ვვარჯიშობდი ინტენსიურად და მერე გავანებე თავი ერთი 2-3 თვე, ეხლა ისევ დავიწყე ვარჯიში დღეში 2-3 საათი დაახლოებით 2 კვირაა, და შევამჩნიე რომ ორგანიზმი მთხოვს ბევრ ძილს(ალბათ ვარჯიშის გამო) 12-13 საათი და არ მაქვს ძილისთვის ამდენი დრო, და ვერ მეტყვით რისი ბრალია ეს და როგორ შევამცირო ორგანიზმის მოთხოვნილება ძილზე, კიკბოქსინგში ვვარჯიშობ და ფიტნესში კიდევ ცოტას (თუ აქვს ამას მნიშვნელობა)

Posted by: matilda123 1 Dec 2011, 23:16
Pinturico
ნინოშვილის ქუჩAზე არის ვენეროლოგიის და დერმატოლოგიის ცენტრი.
simebi
smile.gif დრო არავის არ გვაქვს, მაგრამ ჩვენი ორგანიზმი ძალებს იღდგენს ძილშI. შენ თუ ბუნებას გინდა შეეწინააღმდეგო, მაშინ სხვა გზებს მიმართე... ნორმალურია, რომ ძილიდ მოთხოვნილება გაქვს ვარჯიშის მერე, ორგანიზმი იღლება და ამიტომ გეძინება.

Posted by: iFFii 2 Dec 2011, 22:25
QUOTE
ბოდიში თუ არასწორ ადგილას ვპოსტა

QUOTE
შეიძლება აქ არ უნდა მეკითხა


ზონდის გადაყლაპვის გარეშე არ შეიძლება კუჭის გამოკვლევა?
sad.gif

Posted by: PANTALONE 5 Dec 2011, 16:44
არ ვიცი სად უნა დავპოსტო/ ალბათ აქ/



ართრიტის, რევმატიზმის ან პოდაგრის (რომელიღაცა მაქვს მგონი) ტკივილები რითი უნდა გავიყუჩო უცბად?

სანამ ექიმთან მისვლას მოვაბი თავი

Posted by: inesa1936 6 Dec 2011, 12:49
ამიხსენით - რას ნიშნავს:" მტკიცებულებებზე დამყარებული მედიცინა"? სხვა შემთხვევებში დიაგნოზი აპრიორი ისმება?

Posted by: letodiniusi 7 Dec 2011, 23:55
არ ვიცი აქ უნდა ვიკითხო თუ არა... გულისრევის შეგრძნება თითქმის მუდმივად რამდნეიმე დღის განმავლობაში,ჭვალისებური წამოვლითი ტკივილები მუცლის და ნაღვლის ბუშტის მიდამოებში ნაღვლის ბუშტის პრობლემებზე მიუთითებს თუ რამე სხვა შეიძლება იყოს?? :შ

Posted by: source 10 Dec 2011, 14:01
37 გრადუს სიცხეზე კაცმა უნდა დალიოს რამე? თუ გაივლის , გაციებისას რა .

Posted by: matilda123 11 Dec 2011, 03:15
iFFii
QUOTE
ზონდის გადაყლაპვის გარეშე არ შეიძლება კუჭის გამოკვლევა?

ზონდს არ ყლაპავ.
პაციენტი წვება გვერდზე და ექიმს შეყავს გასტროსკოპი პირის ღრუდან. მანამდე პაციენტი ასხურებენ ადგილობრივ ანესთეტიკს პირის ღრუში და გამოკვლევის პროცესი ყოველგვარი უსიამოვნო შეგრძნებების გარეშე მიდის.
PANTALONE
QUOTE
ართრიტის, რევმატიზმის ან პოდაგრის (რომელიღაცა მაქვს მგონი) ტკივილები რითი უნდა გავიყუჩო უცბად?

სანამ ექიმთან მისვლას მოვაბი თავი

ტკივილგამაყუჩებლად არასტეროიდული ანტიფლოგისტიკა გამოგადაგება, მაგ. იბუპროფენი 600მგ X 3/დღეში, აუცილებლად ჯერ ომეპრაზოლი დალიე 20მგ X 1/დღეში. თუ პოდაგრაა დიკლოფენაკით იმკურნალება.
კი დავწერე, მაგრამ ასე პაციენტის უნახავად წამლის დანიშვნა დანაშაულია . ამიტომ ჩემი თხოვნა იქნება- მიდი ექიმთან.
letodiniusi
QUOTE
არ ვიცი აქ უნდა ვიკითხო თუ არა... გულისრევის შეგრძნება თითქმის მუდმივად რამდნეიმე დღის განმავლობაში,ჭვალისებური წამოვლითი ტკივილები მუცლის და ნაღვლის ბუშტის მიდამოებში ნაღვლის ბუშტის პრობლემებზე მიუთითებს თუ რამე სხვა შეიძლება იყოს?? :შ


--------------------

მიმართე გასტროენეტეროლოგს. შესაძლოა ნაღვლის ბუშტია, შესაძლოა კუჭი ან 12-გოჯა, ასევე პანკრეასი და ზოგჯერ საკვერცხეების პრობლ. და აპენდიციტმაც იცის ზედა მუცლის ტკივილი. გულისრევა კიდევ ძალიან ბევრ ისეთ დაავადებასაც ახასიათებს, რომლებიც მუცელთან კავშირში არც არიან.
მოკლედ, მიმართე ექიმს.
source
QUOTE
37 გრადუს სიცხეზე კაცმა უნდა დალიოს რამე? თუ გაივლის , გაციებისას რა .

37,4 -ს ზემოთ ტემპერატურა ითვლება სიცხედ. გაციებული თუ ხარ გაციების ტაბლ. ან ჩაი მიიღე.

Posted by: letodiniusi 11 Dec 2011, 21:22
matilda123
მადლობააა smile.gif

Posted by: PANTALONE 12 Dec 2011, 22:54
matilda123
მადლობთ /

Posted by: ნინა 14 Dec 2011, 18:31
ქრონიკულ სურდოზე რას მირჩევთ? ანუ გაჭედილი ცხვირი... ანუ ცხვირში კლანჭები მამიჭირეს... smile.gif)))
წინასწარ მადლობა

Posted by: Golden Fish 16 Dec 2011, 03:42
გამარჯობათ.
მაინტერესებს რა განსხვავებაა ფონოფორეზსა და ელექტროფორეზს შორის და თუ ახასიათებთ რაიმე გვერდითი მოვლენები მათ? აქვს თუ არა მნიშვნელობა რა მოდელის აპარატურით იქნება გაკეთებული? ალბათ უკანასკნელი მოდელის აპარატურით აჯობებს არა გაკეთება?

Posted by: chuvak_lexo 17 Dec 2011, 17:23
matilda123
გამარჯობათ ერთი შეკითხვა მაქვს:
ზოგადად ღორის ხორცს რომ ვჭამ ხოლმე მეორე დღეს ან გარკვეული დროის მერე მეშლება კუჭი (ყოველთვის არა) და არის ისეთI წამალი რომ ეს ამაცილოს თავიდან?
ანუ ჭამის დროს დავლიო და ჩვეულებრივ გავიდწე კუჭში მუცლის წვის და ტკივილის გარეშე?

Posted by: Golden Fish 17 Dec 2011, 21:12
სად აკეთებენ მასაჟებს დისპლაზიური სახსრებისთვის? (დიდებისთვის)

Posted by: m-009 17 Dec 2011, 22:53
ისეთი შეგძნება მაქვს ფილტვების მიდამოში თითქოს მეკვანძება ,რისი ბრალი უნდა ოყოს? გაციებული ხომ არ ვარ?

და იქნებ დაწეროთ ტეტანუს რა სიმპტომები აქვს? დღეს სახლის კონსერვზე გავიჭერი ხელი ,სისხლი არ წამომსვლია და მაინც მჭირდება აცრა?

Posted by: matilda123 17 Dec 2011, 23:10
b]chuvak_lexo[/b]
QUOTE
გამარჯობათ ერთი შეკითხვა მაქვს:
ზოგადად ღორის ხორცს რომ ვჭამ ხოლმე მეორე დღეს ან გარკვეული დროის მერე მეშლება კუჭი (ყოველთვის არა) და არის ისეთI წამალი რომ ეს ამაცილოს თავიდან?
ანუ ჭამის დროს დავლიო და ჩვეულებრივ გავიდწე კუჭში მუცლის წვის და ტკივილის გარეშე?


ღორის ხორცის აუტანლობა გაქვს ხომ ხვდები თავადაც რა წამალი გინდა? შენი წამალი შენი თავის შეკავებაა...
m-009
QUOTE
ისეთი შეგძნება მაქვს ფილტვების მიდამოში თითქოს მეკვანძება ,რისი ბრალი უნდა ოყოს? გაციებული ხომ არ ვარ?

და იქნებ დაწეროთ ტეტანუს რა სიმპტომები აქვს? დღეს სახლის კონსერვზე გავიჭერი ხელი ,სისხლი არ წამომსვლია და მაინც მჭირდება აცრა?

ჩემი აზრით არა. ტეტანუსის ჩხირები არის ნიადაგში, მტვერში , ცხოველის და ადამიანების გამონაყოფებში.

Posted by: chuvak_lexo 18 Dec 2011, 15:42
matilda123
QUOTE
ღორის ხორცის აუტანლობა გაქვს ხომ ხვდები თავადაც რა წამალი გინდა? შენი წამალი შენი თავის შეკავებაა...

ყოველთვის არ მაქვს ხოლმე ხანდახან მაქვს და user.gif

Posted by: m-009 19 Dec 2011, 03:29
მარცხენა ხელი მტკივა+მარჩხენა ყელის მხარე (თითქოს დანას მიყრიან ისეთი შეგრძნება მაქვს) და მარცხენა მკერდის მხარეს ტკივილი მაქვს ,(გული არ არის გამოვიკვლიე) რისი ბრალი უნდა იყოს ეს სიმპტომები?

Posted by: მამა-ჯეMალი 19 Dec 2011, 15:21
2 დღით ვიყავი უკრაინაში, და ისეთი რეჟიმი მქონდა რომ მაგრად გადავიღალე და ღამეებიც ძალიან ცოტა მეძინა...

ღამე გამომაღვიძა არაბუნებრივმა კანკალმა, ყველაფერი მიკანკალებდა და თან მაგრად მციოდა.. არა და სახლში მაგარი სითბო იყო sad.gif კანკალი გავდა ნერვულს და არა ისეთს სიცივისგან რომ გაკანკალებს, ვერ გავიგე რა მჭირდა და კორსიზი დავლიე 2 ტაბლეტი და დამამშვიდა...ჩამეძინა მაგრამ მერე გამომაღვიძა სიმხურვალემ, სიცხე მქონდა მაღალი, ეხლაც მაქვს....

გულის რევის შეგრძნებაც მქონდა და კუჭშიც ვერ ვარ კარგად, და ეს ყველაფერი რამე ვირუსის ბრალია, გადაღლილობის თუ მოწამლვის?

Posted by: matilda123 19 Dec 2011, 20:44
მამა-ჯეMალი
QUOTE
2 დღით ვიყავი უკრაინაში, და ისეთი რეჟიმი მქონდა რომ მაგრად გადავიღალე და ღამეებიც ძალიან ცოტა მეძინა...

ღამე გამომაღვიძა არაბუნებრივმა კანკალმა, ყველაფერი მიკანკალებდა და თან მაგრად მციოდა.. არა და სახლში მაგარი სითბო იყო sad.gif კანკალი გავდა ნერვულს და არა ისეთს სიცივისგან რომ გაკანკალებს, ვერ გავიგე რა მჭირდა და კორსიზი დავლიე 2 ტაბლეტი და დამამშვიდა...ჩამეძინა მაგრამ მერე გამომაღვიძა სიმხურვალემ, სიცხე მქონდა მაღალი, ეხლაც მაქვს....

გულის რევის შეგრძნებაც მქონდა და კუჭშიც ვერ ვარ კარგად, და ეს ყველაფერი რამე ვირუსის ბრალია, გადაღლილობის თუ მოწამლვის?

სავარაუდოდ ენტეროვირუსული გრიპი გაქვს. ეხლა ეპიდემიასავითაა ევროპაში. მაღალი ტემპერატურაც მაგას მოწმობს.
m-009
QUOTE
მარცხენა ხელი მტკივა+მარჩხენა ყელის მხარე (თითქოს დანას მიყრიან ისეთი შეგრძნება მაქვს) და მარცხენა მკერდის მხარეს ტკივილი მაქვს ,(გული არ არის გამოვიკვლიე) რისი ბრალი უნდა იყოს ეს სიმპტომები?

ხელის( მხრის) ტკივილი( ტკივილის ყელ-კისრის მიდამოში გადაცემით) ახასიათებს მხრის სახსრის დაზიანებას, ტკივილი ძლიერდება ხელის მოძრაობისას და უფრო ღამით გვხვდება( ხშირად ტკივილი გაღვიძებს). ტკივილი დღისით გამოწვეულია მოძრაობისგან, როდესაც მუდმივად ერთიდაიგივე მოძრაობას ვაკეთებთ. ამ ტიპის ტკივილი შესაძლოა ხერხემლის დაზიანებამაც გამოიწვიოს, რევმატულმა დაავადებებმაც და სხავადსხვა სპონდილიტებმა და ართროზებმა. გული გამოვრიცხე, რადგან თვითონ თქვი რომ გულის დაავადებები გამოირიცხაო. მოკლედ ორთოპედთან ვიზიტია საჭირო და ის დასვამს დიაგნოზს.

Posted by: მამა-ჯეMალი 19 Dec 2011, 23:15
matilda123
QUOTE
სავარაუდოდ ენტეროვირუსული გრიპი გაქვს


საავადმყოფოში დავწვე თუ როგორ ვიმკურნალო? და სად მივიდე და გავესინჯო, ზუსტად რომ დავრწმუნდე რა მჭირს

Posted by: matilda123 20 Dec 2011, 01:03
მამა-ჯეMალი
ინტერნისტთან მიდი ჩვეულებრივთან ან შენი ოჯახის ექიმთან.

Posted by: Natuka NGN 20 Dec 2011, 01:03
მამა-ჯეMალი
მიდი ინფექციურში მაინც, თუ სიცხეები გაქვს, მით უმეტეს.

Posted by: buub 20 Dec 2011, 14:16
გახშირებული დაბალი წნევები რამ შეიძლება გამოიწვიოს?

Posted by: m-009 21 Dec 2011, 18:51
buub
მე მაქვს დაბალი წნევები,და ჩემს შემთხვევაში ნევროზის ბრალია,შეიძლება ჩიყვი გქონდეს, გაიკეთე სისხლის საერთო ანალიზი და ეხსკობია ,რა სიმპტომები გაქვს დაბალი წნევის გარდა?
matilda123
დამინიშნეს პოლიჟენი და ვაგოსტაბილი და ხომ არ იცი რომელიმეს შეუძლია რეაქციის მიღება? ( შიშები დამეწყო წამლებზე )

Posted by: matilda123 21 Dec 2011, 23:23
buub
დაბალი წნევა ხშირია ახალგაზრდა გამხდარ ქალებში. ამას კონსტიტუციური ჰიპიტონია ჰქვია და არ ითვლება დაავადებად.[
b]m-009[/b]

QUOTE
დამინიშნეს პოლიჟენი და ვაგოსტაბილი და ხომ არ იცი რომელიმეს შეუძლია რეაქციის მიღება? ( შიშები დამეწყო წამლებზე )

ნებისმიერ მედიკამენტს შეუძლია ალერგია ან აუტანლობა გამოიწვიოს.

Posted by: buub 22 Dec 2011, 01:36
m-009
საშინელი თავის ტკივილი, გულისრევის შეგრძნება და წყურვილი რავი ნევროზი მაინცდამაინც არ მგონია

matilda123
გამხდარიც არ ვარ 170 მდე ვარ სიამღლეში და წონა 55კგმდე მგონი ნორმალურია .?
კიდე ერთი კითხვა მაქ ერთი თვის წინ სისხლის საერთო გავიკეთე და ლიმფოციტების ნაკლებობა და ლეიკოციტების მომატება მქონდა ხოდა რამდენიმე დღის წინ კისრის ლიმფური კვანძი გამისივდა და მტკიოდა მეორე დღეს კი დაცხრა და რამ შეიძლება გამოიწვიოს?

Posted by: Woman No Cry 23 Dec 2011, 03:20
რა განსხვავებაა დისპლაზიურ სახსრებსა და ცრუ სახსრებს( ფსევდოართროზი) შორის და როგორი მკურნალობა სჭირდებათ მათ?

Posted by: matilda123 23 Dec 2011, 13:22
buub
QUOTE
გამხდარიც არ ვარ 170 მდე ვარ სიამღლეში და წონა 55კგმდე მგონი ნორმალურია .?
კიდე ერთი კითხვა მაქ ერთი თვის წინ სისხლის საერთო გავიკეთე და ლიმფოციტების ნაკლებობა და ლეიკოციტების მომატება მქონდა ხოდა რამდენიმე დღის წინ კისრის ლიმფური კვანძი გამისივდა და მტკიოდა მეორე დღეს კი დაცხრა და რამ შეიძლება გამოიწვიოს?

აბა რა ხარ, მსუქანი? არსებობს ბოდი მასი ინდექს , რომლებითაც ექიმები ვხელმძღვანელობთ სიმსუქნის გამოსათვლელად. შეგიძლია გუგლში დასერჩო და გამოითვალო. ლეიკოციტოზი უკეთეს შემთხვევაში ინფექციის არსებობაზე მიგვანიშნებს ორგანიზმში. ლიმფოპენია იმუნიტეტის დაქვეითებაზე. ლიმფ.ჯირკვლ. გადიდება იცის ასევე ინფექციამ და კიდევ სხვა რაღაცეებმა რაზეც ეხლა არ ვისაუბრებ. მიდი ექიმთან აუცილებლად თან გადაუდებლად.
Woman No Cry
QUOTE
რა განსხვავებაა დისპლაზიურ სახსრებსა და ცრუ სახსრებს( ფსევდოართროზი) შორის და როგორი მკურნალობა სჭირდებათ მათ?

ართროზი არის სახსრების დეგენერაციულ-დისტროფიული გადავგარება, რისი მიზეზიც სახსრის ხრტილის გადაგვარებაა. დისპლაზია ამოვარდნილობაა სახსრის.
მკურნალობა ორთოპედებმა უკეთესად იციან. დისპლაზის ვარჯისი, მასაჟები და რაღაც სპეც. შეხვევას აკეთებენ კიდევ , დამავიწყდა რა ჰქვია მაგ.ახალშობილ ბავშვებს რომ აქვთ ხოლმე, როდესაც მენ-ბარძაყის ამოვარდნილობაზეა საუბარი, ისეთი.
ართროზს ასევე ზ.აღ. მკურნ. და პლუს მედიკამენტები , ოპერაცია და ხელოვნ. სახსრის იმპლანტაცია , როდესაც საჭიროა.

Posted by: nina1992 26 Dec 2011, 16:21
იქნებ აქ მიპასუხოთ,
მსუბუქი დამამშვიდებელი მინდა რამე, ზედმეტად რო არ იწვევდეს მიჩვევას...
აი რო დავლიო საღამოს 9-10 სთვის და გამთიშოს და დავიძინო.
არაა ესეთ ი რამე?

Posted by: taamo 6 Jan 2012, 16:38
გამარჯობა. ედს მაქვს მაღალი (27), არაფერს ვუჩივი და როგორ გავიგო რის ბრალი შეიძლება იყოს???

Posted by: saturngirl 7 Jan 2012, 01:57
სადღაც 1 თვეა რაც აღმოვაჩინე, რომ ზოგიერთი რამის სუნი და გემო ძალიან უსიამოვნო გახდა. მაგალითად ჩემი სუნამოები, ორივეს ერთნაირად გამაღიზიანებელი მძაფრი მწარე სუნი აქვს ახლა ჩემთვის. ყავასაც იგივე სუნი აქვს, მძაფრი და მწარე და არც გემო მსიამოვნებს, ასევე კოკა-კოლასაც და კბილის პასტას.... სხვა საკვებზე ჯერ არ შემინიშნავს.... კიდევ ლიმონათებს.... ხელზე მქონდა გამონაყარი, წვრილი ბუშტუკებივით, პერიოდულად მიჩნდება ხოლმე ხელზე, მერე თავისით ქრება, ზაფხულში ფეხზეც იყო... ექიმთან რომ ვიყავი ეს ყველაფერი ალერგიას დაუკავშირა. დამინიშნა წამალი რაღაც ანტიგესტამინური. მაგრამ ყნოსვის და გემოს პრობლემა ისევ მაქვს..... რისი ბრალი შეიძლება იყოს?

Posted by: nadira64 7 Jan 2012, 10:24
-გამარჯობათ. გილოცავთ შობა-ახალ წელს.ასეთი შეკითხვა მაქვს: ჩემს 20 წლის ბიჭს დაეწყო ფეხისგულების წვა, უკვე სამი კვირაა, წვა ძლიერდება ღამის საათებში. გარეთ რომ გადის, კლებულობს. ფეხებზე ვენები არ ეტყობა, სისხლის საერთო და შარდის ანალიზი ნორმაშია. პოლიკლინიკის თერაპევტმა ყოველი შემთხვევისათვის დაუნიშნა ვენოდიოლი, მაგრამ შედეგი არ აქვს.რისი ბრალი შეიძლება იყოს და რომელ ექიმს მივმართოთ?

Posted by: tornikke 8 Jan 2012, 10:44
გამარჯობათ. გილოცავთ შობა-ახალ წელს. ერთი კვირაა მაქვს გრიპი,(მეგობარსაც იგივე ჭირს) სიმპტომები: მაქვს ყელი აწითლებული ძალიან. ნახველის შეგრძნება რის გამოც ვცდილობ ამოვახველო . მაქვს შავი ნახველი. ხველებით არ ვახველებ. არ მაქვს სიცხე. მაგრამ სუსტად ვგრძნობ თავს. უკვე 1 კვირაა ასე ვარ. ამ დროის განმავლობაში ვსავდი (ტაიქოლდს, ეფექტი ფლუს ჩაის) ასევე დიდი რაოდენობით ლიმნიან ჩაის. ამოსახველებლად- აცც-ს, ლაზოლვანს. მაგრამ არც გაუარესება მაქ და არც გამოჯანრმთელება. ისევ ისე ვარ. იქნებ მირჩიოთ როგორ მოვიქცე

Posted by: matilda123 8 Jan 2012, 22:53
saturngirl
მიზეზები შეიძლება იყოს 1) წამლისმიერი ალერგია, თუ უკანასკნელ ხანს მიღებული გაქვს რომელიმე ანტიბიოტიკი. 2) ცინკის დეფიციტი ორგანიზმში და კიდევ სხვა მიზეზები. შენს შემთხვევაში ალბათ უფრო ცინკის დეფიციტია, რადგან ბუშტუკებიანი გამონაყარი მას ახასიათებს და პლუს ამას გემოს და ყნოსვის ცვლილება მაფიქრებინებს ცინკის დეფიციტზე. გაიკეთე სისხლის ანალიზი.
nadira64
პოლინევროპათია იწვევს ფეხისგულებს წვას. მიმართეთ ნევროპათოლოგს. დიაგნოზის დადგენის შემდეგ დაგინიშნავენ B ჯგუფის ვიტამინებს და ალბათ ლიპონისმჟავასაც. ჯერ როგორც ინფუზიას და მერე ტაბლეტებს. მოკლედ, მიმართეტ ნევროპათოლოგს.
tornikke
შენ ალბათ მწეველი ხარ და ამიტომ გაქვს მოშავო ნახველი. კარგია რასაც სვამ თუმცა ეგ გაციების წამლები 5 დღეზე მეტხანს ნაჩვენები არაა. აცც არაა საჭირო თუ ნახველი არ გაქვს არც ლაზოლვანი. მიიღე ბევრი სითხე და ვიტამინები. ყოველი შემთხვევისთვის მიმართე ექიმს რომ ბრონქებზე და ფილტვებზე მოგისმინოს და თუ საჭიროდ ჩათვლის დაგინიშნავს ანტიბიოტიკს.

Posted by: saturngirl 9 Jan 2012, 02:00
matilda123
ძალიან დიდი მადლობა. სისხლის საერთო ანალიზი თუ რომელი? ანუ ექიმთან რომ მივალ რა ვუთხრა? ან ცინკის დეფიციტი საიდან გამიჩნდებოდა? ეგ გამონაყარი პერიოდულად ჩნდებოდა ხოლმე ამ ბოლო 2 წლის მანძილზე, მაგრამ ეს ყნოსვა და გემო ახალია...

Posted by: Polymorph 9 Jan 2012, 02:21
ვაიმე ხალხნო ძალიან შეშინებული ვარ და დამეხმარეთ რააა..

დაბალწნევიანი ვარ. 110- 80-ზე.... და უკვე 1 კვირაზე მეტია 130-85-ზე მაქვს..ჩემთვის მაღალია..გული მიცემს სასწაულად.. 24 საათი მიფრიალებს. ღამეც არ მაძინებს.ძალიან მაწუხებს..ეხლა კიდე ამ შუა ღამეს 140-90-ზე ამივარდა..სახლშიც მარტო ვარ..რა გავაკეთო ან რისი ბრალია sad.gif((( წამლებიც არ მაქვს ნამეტანი.. კლოფელინი მარტო.

არც გადავღლილვარ, არც უძილობა მაწუხებს, არც არაფერს ვუჩივი ..ჩიყვზეც გასინჯული ვარ ერთი 3 წლის წინ არაფერი მჭირს. 25 წლის ვარ.გოგო. ასე უცბად რა მეტაკა არ ვიცი sad.gif((( 1-იანვრიდან ასე ვარ.. არც არაფერი მიჭამია ისეთი.. ისედაც კვარტალში ერთხელ ვჭამ.


Posted by: shtori 9 Jan 2012, 06:26
matilda123

გამარჯობა. იქნებ მეც დამეხმაროთ...
მარჯვენა ფეხი მაწუხებს. მთელს გაყოლებაზე, გვერდით მღრნის და მიღუტუნებს თითქოს და ხან ფეხის გულზე გეგონება დენმა გაიარა მაჟრიალებს. ანუ მენჯიდან ფეხის გულამდე, განსაკუთრებით წვივის მონაკვეთში ვგრძნობ მომეტებულად.
რამდენიმე დღის წინ მუცელი + სახსართან (მენჯთან) მტკიოდა . ახლა კი ფეხზე გადაინაცვლა.
რამე მალამოს ხომ ვერ მირჩევდით ან აბებს?
გმადლობთ.

Posted by: tornikke 9 Jan 2012, 10:26
matilda123
მწეველი არ ვარ ! ცოტათი შავი არი ხოლმე. ეხლა დაივწყე ნიმესილის დალევა. ცოტათი უკეთესად ვარ. სითხეს ისედაც ძალიან ბევრს ვსვავ ყოველ დღე. ეხლა ხო საერთოდ. მადლობა გამოხმაურებისთვის.

Posted by: ToyotaMR2 9 Jan 2012, 12:20
QUOTE
გავაკეთო ან რისი ბრალია ((( წამლებიც არ მაქვს ნამეტანი.. კლოფელინი მარტო.

არც გადავღლილვარ, არც უძილობა მაწუხებს, არც არაფერს ვუჩივი ..ჩიყვზეც გასინჯული ვარ ერთი 3 წლის წინ არაფერი მჭირს. 25 წლის ვარ.გოგო. ასე უცბად რა მეტაკა არ ვიცი ((( 1-იანვრიდან ასე ვარ.. არც არაფერი მიჭამია ისეთი.. ისედაც კვარტალში ერთხელ ვჭამ.


ზუსთად მეც იგივე ასაკის ვარ და იგივენაირად ვარ როგორც შენ ..რა გვემართება ?იქნება გვითხრან .არასდროს არ ვყოფილვარ ასე

Posted by: matilda123 9 Jan 2012, 20:03
saturngirl
სისხლში ნახონ ცინკის რაოდენობა.
Polymorph
ფარისებრი ჯირკვლის ჰორმონები გამოიკვლიე კიდევ ერთხელ. 3 წელი საკმაოდ დიდი დროა. შEსაძლოა აგრეთვე პრემენსტრუალურ სინდრომთან იყოს დაკავშირებული ანუ თუ მაღალი წნევა ოვულაციის შემდგომ პერიოდს ემთხვევა.
shtori
მე ვფიქრობ ნერვუს იშიადიკუს გაქვს გაღიზიანებული აი ეს http://derekfajas.hu/images/lesugarzas_2.jpg . წამლების დანიშვნა ციტა საფრთხილოა გაუსინჯავად. ნევროპათოლოგს ან ქირურგს მიმართე ან კარგ ინტერნისტს.

Posted by: nadira64 9 Jan 2012, 20:26
QUOTE
matilda123

დიდი მადლობა

Posted by: Polymorph 9 Jan 2012, 23:00
matilda123
დიდი მადლობა მატილდ.. ისე შეიძლება კბილის ბრალი იყოს? სიბრძნის კბილები ამომდის.. ორივე ერთდროულად მიტევს biggrin.gif მაგრამ ერთერთი ამოდის კბილის ზემოთ..ზემოდან აწვება და ძალიან მაწუხებს.. გამისივდა ყბა,მიჭირს პირის გაღება,ღეჭვა და ხანდახან სიცხესაც მაძლევს.. იქნებ კბილის ბრალიცაა? ჰორმონზე მართალია მივალ.. რადგან დედაჩემსაც სჭირს ჩიყვი და ბებიაჩემსაც..მიდრეკილი ვიქნები მეც. ექიმმა მითხრა კაი 10 წელი არ დაგინახო აქ არაფერი გჭირსო..უბრალოდ როცა გათხოვდები და ორსულად იქნები მაშინ მოდი პროფილაქტიკისთვისო რადგან გენებში გაქო ი ვსო..user.gif

ToyotaMR2
ვბერდებით biggrin.gif

Posted by: PrtSc SysRq 9 Jan 2012, 23:41
გამარჯობათ.შეიძლება შეუფერებელ ქვეგანყოფილებაში ვსვამ კითხვას და წინასწარ გიხდით ბოდიშს იქნებ მიმითითოთ სად შეიძლება გამცენ პასუხი.მაინტერესებს რომელიმე ძლიერი კუჭში გამხსნელის სახელი იქნებ მითხრათ ძალიან მჭირდება.დიდი მადლობა წინასწარ

Posted by: shtori 10 Jan 2012, 00:04
matilda123
დიდი მადლობა პასუხისთვის. მივმართავ ექიმს.

მანამდე კი კიდე 2 კითხვით შეგაწუხებთ
QUOTE
მე ვფიქრობ ნერვუს იშიადიკუს გაქვს გაღიზიანებული

რა იწვევს ყველაზე ხშირად გაღიზიანებას?


და კიდევ, ჰორმონებზე ერთი ანალიზი კეთდება?

Posted by: matilda123 10 Jan 2012, 00:40
shtori
QUOTE
რა იწვევს ყველაზე ხშირად გაღიზიანებას?

გაციება, ტრავმა, ინფექცია და ა.შ.
QUOTE
და კიდევ, ჰორმონებზე ერთი ანალიზი კეთდება?

კითხვა ვერ გავიგე. არსებობს ბევრი ჰორმონი ორგანიზმში და ყველა მათგანი შესაძლოა სისხლში ინახოს.

Posted by: shtori 10 Jan 2012, 01:25
matilda123
სისხლის საერთო ანალიზი უნდა გავიკეთო ანუ ჰორმონების შესამოწმებლად?
ვიფიქრე ყველა ჰორმონზე სხვადასხვა ანალიზი ხომ არ სჭირდება თქო : ))
მაინტერესებს წესრიგში, ნორმაში თუ არიან ჩემი ჰორმონები.

Posted by: mr_gold 10 Jan 2012, 21:49
თუ სშგიძლიათ მიპასუხეთ. ჰემოგლობინი მაქ 60, ესე დაბალი არასდროს მქონიაა... საშიშია ?

Posted by: rock in rose 10 Jan 2012, 22:12
shtori
QUOTE
სისხლის საერთო ანალიზი უნდა გავიკეთო ანუ ჰორმონების შესამოწმებლად?


არა, სისხლის საერთო ანალიზით ვერ გაიგებ ამა თუ იმ ჰორმონის კონცენტრაციას სისხლში.

QUOTE
ვიფიქრე ყველა ჰორმონზე სხვადასხვა ანალიზი ხომ არ სჭირდება თქო : ))


სისხლის რაოდენობას აიღებენ და იქიდან გააკეთებენ ანალიზს - სხვადასხვა ჰორმონზე სხვადასხვას. და ფასიც განსხვავებული იქნება ბუნებრივია

Posted by: mr_gold 10 Jan 2012, 22:29
თუ შეგიძლიათ მიპასუხეთ წინა შეკიტხვაზე.

Posted by: m-009 10 Jan 2012, 22:47
ოქტომბრის დასაწყისში დავამთავრე ცოფის აცრები,ერთი თვის მერე დავიწყე ვალოკარდინის დალევა (თურმე ალკოგოლს შეიცავს) ,ეხლა მაინტერესებს ალკოგოლი რომ დავლიო რამე რეაქციას ხომ არ მომცემს?

Posted by: shtori 10 Jan 2012, 23:16
rock in rose
მე უნდა ვუთხრა რომელ ჰორმონზე მინდა ანალიზი? რომ ვერ ვერკვევი მასე?..
თმიანობა მომემატა ცოტა და ამიტომ მინდა sad.gif

Posted by: rock in rose 11 Jan 2012, 14:52
shtori

თავისუფალ ტესტოსტერონზე დაგჭირდება ანალიზის გაკეთება( და არა მხოლოდ) .

კი შენ უნდა უთხრა, მაგრამ კარგი იქნება თუ ენდოკრინოლოგთან გაივლი კონსულტაციას, თმიანობის მომატება მხოლოდ ტესტოსტერონის ამბავი არ იქნება.



Posted by: shtori 11 Jan 2012, 16:10
rock in rose
მადლობა smile.gif

Posted by: elee 11 Jan 2012, 20:12
ბოლო 3 კვირის მანძილზე ხშირად მაქვს 36-37 გრადუსი ტემპერატურა საღამოობით, მაქვს ღამით გაძლიერებული ოფლიანობა, კიდევ თავის აუტანელი ტკივილი, ხშირად კანკალი და მთელ სხეულში მტვრევის შეგრძნება, ვის მივაკითხო ან რა ვქნა საერთოდ?

Posted by: tasikotasiko 12 Jan 2012, 02:45
დღეს ჩემმა შვილმა კანფეტი ჭამა და მგონი პატარა ნაგლეჯი პარკიც შეყვა,საშიშია?ვერ იხსენებს გადაყლაპა თუ არა და გავსკდი ნერვიულობით

Posted by: EJDER 12 Jan 2012, 09:27
რა მაინტერესებს..

ნივთიერების ცვლის დარღვევა რამ შეიძლება გამოიწვიოს და რა შედეგები მოჰყვება?

Posted by: source 12 Jan 2012, 15:00
მაინტერესებს სავაჭრო ცენტრებში ტანსაცმლის მოზმოება გასინჯვა და შეძენა არის თუ არა ისეთ შემთხვევაში საშიში
როგორიცაა ვთქვათ ჯერ ჩაიცვა სოკოვანით დაავადებულმა დამიანმა ან ვენერიულად დაავადებულმა ადამიანმა ვთქვათ
შარვალი, მაიკა . არის შანსი გადადების ამ დაავადებების? შემდეგ ჯამნმრთელმა რომ ჩაიცვას.

Posted by: matilda123 12 Jan 2012, 20:51
elee
ზოგადი ინტოქსიკაციის სიმპტომები გაქვს. მიმართე ინტერნისტს. სისხლის საერთო ანალიზი +ფარისებრი ჯირკვლის ჰორმონების გამოკვლევა და ამის მიხედვით დანარჩენი პროცედურები და ანალიზები.
tasikotasiko
თუ კუჭის მოქმედება ნორმალურია და მუცლის ტკ-ბი არ აქვს, საშიში არაა, მაგრამ ყოველი შემთხვევისთვის ჰკითხე პედიატრს.
source
თეორიულად არის მაგის შანსი, მაგრამ პრაქტიკულად ნაკლებად.
EJDER
ძალიან ბევრმა დაავადებამ შეიძლება გამოიწვიოს. მათ შორის ენდოკრ.დაავადებებმა, ელექტროლიტური წონასწორობის მოშლამ, რომელიმე ენზიმის დეფიციტმა ან გენურმა დეფექტმა და ა.შ.

Posted by: gurabe 13 Jan 2012, 20:07
არ ვიცი სად ვიკითხო( sad.gif ) და აქ დავწერ(თუ არასწორია,შემისწორეთ):
თითი მაქვს გასიებული,მარჯვენა შუა თითი,რამდენი მე დღეა(არ მიმქცევია ყურადღება,რადგან საგანგაშოდ არ მაქვს),დღსე მთელი მკლავი მეტკინა და ზურგის მარჯვენა მხარეც,პატარა ადგილზე...

ბევრი წერა მიწევს და შეიძლება ამნ გამოიწვიოს?...

პ.ს. როდესაც ცოტა ბევრს ვწერდი,ხელზე ვენები მებერებოდა და მტკიოდა(ადრე ესე ვიყავი)

Posted by: matilda123 13 Jan 2012, 20:21
gurabe
QUOTE
თითი მაქვს გასიებული,მარჯვენა შუა თითი,რამდენი მე დღეა(არ მიმქცევია ყურადღება,რადგან საგანგაშოდ არ მაქვს),დღსე მთელი მკლავი მეტკინა და ზურგის მარჯვენა მხარეც,პატარა ადგილზე...

ბევრი წერა მიწევს და შეიძლება ამნ გამოიწვიოს?...

კი.

Posted by: gurabe 13 Jan 2012, 20:58
matilda123
გმადლობ............ smile.gif

Posted by: MeAndYou 16 Jan 2012, 12:10
ესე იგი, წყალთან რომ მაქვს შეხებაა 1-2 წუთი, კანი მირბილდება ხელებზე. იჭმუხნება. რომ მოხარშავ, ისე biggrin.gif
აი, ზოგადად დიდხანს წყალში ყოფნის დროს რომ ურბილდებათ ხოლმე კანი, ეგრე, ორონდ მე ძალიან მოკლე დროში.

რა შეიძლება იყოს ამის მიზეზი?

Posted by: tasikotasiko 16 Jan 2012, 15:23
დაბალ წნევებზე ცხელი წყლით ბანაობა შეიძლება?ისევ მითამაშებს წნევა 90/60 ზე და მეშინია ბანაობა sad.gif

Posted by: teo25 16 Jan 2012, 16:26
გამარჯობათ,
ჩემს დედამთილს ხელისგულზე გაუჩნდა წვრილი, წყლულოვანი გამონაყარი,
რომელიც სკდება და კანი აქერცლილი და გაუხეშებული რჩება.
დღეს ჩევამჩნიეთ რომ ფეხისგულზეც იგივე გამოუვიდა და მეორე ხელზეც გადავიდა.
ხომ ვერ მეტყვით რა არის?


პ.ს. თუ არასწორ თემაში ვიკითხე მაპატიეთ.

Posted by: matilda123 16 Jan 2012, 21:58
teo25
QUOTE
გამარჯობათ,
ჩემს დედამთილს ხელისგულზე გაუჩნდა წვრილი, წყლულოვანი გამონაყარი,
რომელიც სკდება და კანი აქერცლილი და გაუხეშებული რჩება.
დღეს ჩევამჩნიეთ რომ ფეხისგულზეც იგივე გამოუვიდა და მეორე ხელზეც გადავიდა.
ხომ ვერ მეტყვით რა არის?

წყლულოვანი თუ ბუშტუკოვანი? კანის წყლული არის ეს მაგალითად http://www.radikal.ru
რადგან ამბობ სკდებაო, მე ბუშტუკებად წარმომიდგა შენი მონაყოლი და თუ ასეა, მაშინ სავარაუდოდ სოკოა (პაციენტის უნახავად, დიაგნოზის დასმა რთულია) და ეს გადამდებია. მიმართეთ დერმატოლოგს.
MeAndYou
QUOTE
ესე იგი, წყალთან რომ მაქვს შეხებაა 1-2 წუთი, კანი მირბილდება ხელებზე. იჭმუხნება. რომ მოხარშავ, ისე biggrin.gif

არის ასეთი ფენომენი "მრეცხავის ხელები " ჰქვია .შენი კანი ალBატ ზედმეტად მგრძნობიარეა.
tasikotasiko
QUOTE
დაბალ წნევებზე ცხელი წყლით ბანაობა შეიძლება?ისევ მითამაშებს წნევა 90/60 ზე და მეშინია ბანაობა sad.gif

შენც მაინცდამაინც აბაზანას ნუ მიიღებ, "იდუშავე" და ეგაა smile.gif)

Posted by: teo25 16 Jan 2012, 22:51
matilda123
ბუშტუკებივითაა , მართალი ხართ.
დიდი მადლობა პასუხისთვის.

Posted by: MeAndYou 17 Jan 2012, 10:32
matilda123
QUOTE
არის ასეთი ფენომენი "მრეცხავის ხელები " ჰქვია .შენი კანი ალBატ ზედმეტად მგრძნობიარეა.


ადრე არ იყო, ეხლა გახდა, ბოლო რამდენიმე თვეა.
ამიტომ დავვინტერესდი, რა შეიძლება იყოს მიზეზითქო.

Posted by: tasikotasiko 17 Jan 2012, 18:15
ოქტომბრის დასაწყისში დავამთავრე ცოფის აცრები,დღEს რომ დავრეკე ვაქცინაციის ცენტრში მითXრეს შეგიძლია დალიოო,ასე ზეპირად ვერ ვენდობი და რა მაინტერესებს,ბოლო ორი თვეა ვალოიკარდინს ვსვამ (რომელიც თრუმე სპირტს შეიცავს) და ეხლა ვთქვათ არაყი რომ დავლიო რამე რეაქციას მომცემს?ძალიან დაპანიკენული ვარ

Posted by: shilaida 17 Jan 2012, 22:14
Kარგი ართრიტოლოგი ვინ არის თბილისში,იქნებ მირჩიოთ?
მადლობა წინასწარ!

Posted by: nina1992 18 Jan 2012, 17:30
მოკლედ ძალიან მადარდებს ეს საკითხი ბოლო დროს sad.gif
შუბლზე წარბთან ახლოს გამომივიდა მეჭეჭის მაგვარი, მანამაც მქონდა მაგრამ ძალიან პატარა და ამობურცულიც არ იყო..,შუბლზე ხელს თუ გადავისმევდი მაშინ ვგრძნობდი,
ბოლო დროს ზრდა დაიწყო, მე კიდევ ვიტყავებ sad.gif მოკლედ ნერვები მეშლება, მოვიგლიჯავ მერე ისევ იზრდება
ვის უნდა მივმართო რო სამუდამოდ მომცილდე?

sad.gif

Posted by: matilda123 18 Jan 2012, 19:37

niana
დერმატოლოგს მიმართე.

Posted by: niseti 20 Jan 2012, 14:14
დღის განმავლობაში კარგად ვგრძნობ თავს და საღამოს სიცხეს მაძლევს, დილით ისევ კარგად ვარ.უკვე ასეთ მდგომარეობაში ვარ ორი კვირაა.ხანდახან ძვლებიც მტკივა. ექიმთან ვიყავი და ანალიზებზე გამიშვა. მსგავს მდგომარეობაში როგორ უნდა მოვიქცე?

Posted by: matilda123 20 Jan 2012, 14:23
niseti
სიცხე და ტანში მტვრევა ზოგადი ინტოქსიკაციაა , ახასიათებს ძალიან ბევრ დაავადებას და გასარკვევია საიდან მოდის ეს. ექიმი მართალია, გაიკეთე ჯერ სისხლის საერთო ანალიზი, შარდის ანალიზი. მერე სისხლის და შარდის სურათის მიხედვით სხვა გამოკვლევები დაგენიშნება.

Posted by: niseti 20 Jan 2012, 17:28
matilda123
კარგია რომ თქვენი და იმ ექიმის აზრი დაემთხვა. მადლობა

Posted by: miu-miu 21 Jan 2012, 13:52
ჩუტყვავილა მქონდა 3 კვირის წინ და მენსტრუაცია არ მომივიდა დანიშნულ დღეს. შეიძლება ეს ამან გამოიწვიოს? რა შეიძლება ხდებოდეს? და როგორ მოვიქცე?

Posted by: matilda123 21 Jan 2012, 14:10
miu-miu
QUOTE
ჩუტყვავილა მქონდა 3 კვირის წინ და მენსტრუაცია არ მომივიდა დანიშნულ დღეს. შეიძლება ეს ამან გამოიწვიოს? რა შეიძლება ხდებოდეს? და როგორ მოვიქცე?

თეორიულად შესაძლოა , ნებისმიერი მწვავე დაავადება( განსაკუთრებით ინფექციური ან ვირუსული)არის სტრესი და ახდენს გავლენას ქალის ორგანიზმზე, რა თქმა უნდა შესაძლოა ციკლი დაირღვეს. მოიცადე 1 მენსტრუაციული ციკლი და თუ არ მოწესრიგდა მერე მიმართე გინეკოლოგს.

Posted by: tasikotasiko 23 Jan 2012, 19:52
2 დღეა სიმშრალე მაქბს და სულ მწყურია,რისი ბრალი უნდა იყოს?
* * *
100-80 ზე მაქვს ორი დღეა წნევა,როგორ მოვაწესრიგო?

Posted by: Summer_Rain 26 Jan 2012, 18:16
თუ შეგიძლიათ მიპასუხეთ,
ჰელიკობაქტერია პილორის კოპროტესტი სად კეთდება საიმედოდ და როგორ
მადლობა

Posted by: tasikotasiko 28 Jan 2012, 15:50
ერთI საათია წამალი დავლიე და რეაქცია არ მოუცია და ესეიგი აღარ მომცემს ხო?

Posted by: lurdes 29 Jan 2012, 19:19
დედას ჰქონდა მაზოლი ფეხის თითზე, სპეციალურმა პლასტირმა არაფერი უშველა,
ნეკა თითია, გაუსივდა ძალიან, ფეხსაცმლის ჩაცმა უჭირს. პლასტირის შემდეგ არყის კომპრესი გაიკეთა, ასევე ალოეს საფენი

გთხოვთ, გვირჩიოთ რამე

Posted by: mia-maria 29 Jan 2012, 21:04
აშკარად, ენერგიის ნაკლებობას განვიცდი და სულ დაღლილი ვარ. რისი ბრალი უნდა იყოს? ჰო კიდევ კონცენტრაცია მიჭირს და რაღაცეები მავიწყდება თითქოს sad.gif

Posted by: DarkPrince 29 Jan 2012, 22:23
მეგობრებო რას იტყვით, ამ პლანშეტზე რას იტყვით, იქნებ იცოდეს ვინმემ როგორი ხარისხისაა

http://www.amazon.com/Coby-7-Inch-Android-Internet-Touchscreen-Tablet/dp/B005HUH88K/ref=sr_1_1?ie=UTF8&qid=1327860288&sr=8-1

Posted by: lady t 30 Jan 2012, 13:35
გამარჯობათ
არ ვიცი აქ უნდა ვიკითხო თუ არა.
საზარდულის თიაქარზე მითხრეს რომ ოპერაცია არ შველის და ისევ განმეორდება აუცილებლად.რამდენად სწორად ვარ ინფორმირებული?
და

მაინტერესებს ამ თიაქარის შემთხვევაში შესაძლებელია თუ არა ფეხმძიმობა და მშობიარობა,რამდენად ხელს შეუშლის ეს ამ პროცესებს?

წინასწარ დიდი მადლობა.

Posted by: gio1993gio 1 Feb 2012, 21:41
გამარჯობა დგეს მუცელი ამტკივდა კუჭის არეში და ახლა ამ საღამოს მომცა 38 სიცხე და რა არის ჩვეულებრივი გრიპია თუ რამე სხვაა წინასწარ მადლობა დახმარებისთვის

Posted by: matilda123 2 Feb 2012, 02:46
lady t
QUOTE
საზარდულის თიაქარზე მითხრეს რომ ოპერაცია არ შველის და ისევ განმეორდება აუცილებლად.რამდენად სწორად ვარ ინფორმირებული?
და

მცდარია აზრი რომ აუცილებლად იქნება რეციდივი. რეციდივის ასაცილებლად დგამენ სპეც. "ბადეს".
QUOTE
მაინტერესებს ამ თიაქარის შემთხვევაში შესაძლებელია თუ არა ფეხმძიმობა და მშობიარობა,რამდენად ხელს შეუშლის ეს ამ პროცესებს?

ჩემი აზრით გართულდება თიაქარი, მაგრამ უფრო ზუსტად ქირურგი გიპასუხებს. იყო თემა ქვემოთ "თქვენს კითხვებს უპასუხებს პროფეს. ქირურგი"-ო თუ რაღაც ამდაგვარი... მოკლედ იქ იკითხე.
gio1993gio
QUOTE
გამარჯობა დგეს მუცელი ამტკივდა კუჭის არეში და ახლა ამ საღამოს მომცა 38 სიცხე და რა არის ჩვეულებრივი გრიპია თუ რამე სხვაა წინასწარ მადლობა დახმარებისთვის

თუ ენტეროვირუსია, ფაღარათიც უნდა გქონდეს. მაშინ ჯდება სიმპტომატიკაში. მაგრამ განავლის ანალიზის და სისხლის ანალიზის გარეშე დიაგნოზის დასმა რთულია, რადგან თითქმის ყველა მწვავე(ან გამწვავებულ) გასტოინტესტინალურ დაავადებას ახასიათებს ეს სიმპტომები.
მიმართე ექიმს.

Posted by: lady t 4 Feb 2012, 19:07
matilda123
გმადობთ
უკვე აღარ მქონდა პასუხის იმედი.

Posted by: irinka-ira 4 Feb 2012, 22:20
ამ ბოლო დროს ენერგიის ნასახი არ მაქვს, წნევაც დამივარდა.... ალბად გადავიღალეე, ნეტა რამე საშიში ხომ არააააა?

Posted by: matilda123 5 Feb 2012, 02:56
irinka-ira
QUOTE
ამ ბოლო დროს ენერგიის ნასახი არ მაქვს, წნევაც დამივარდა.... ალბად გადავიღალეე, ნეტა რამე საშიში ხომ არააააა?

თებერვალ-მარტში ქვეითდება იმუნიტეტი, ეს შეიძლება ვთქვათ "ნორმალურია" მიიღე B ჯგუფის ვიტამინები ან ნებისმიერი ვიტ. კომპლექსი. მეტი ყურადღება მიაქციე კვებას, ძილს და თუ ისევ სისუსტე გექნება, მიმართე ექიმს.

Posted by: zuria71 8 Feb 2012, 10:58
სიცხის დამწევი რა საშუალება არსებობს ისეთი რომ ორსულობას არ ავნოს?

Posted by: matilda123 8 Feb 2012, 14:04
zuria71
QUOTE
სიცხის დამწევი რა საშუალება არსებობს ისეთი რომ ორსულობას არ ავნოს?

ადრე პარაცეტამოლს უნიშნავდნენ. ბოლო გამოკვლევებმა აჩვენა რომ , ეს ბავშვებში დაბადების შემდგომ ასთმას და სათესლეების(თუ ბიჭია) მდებარეობის ანომალიებს იწვევს. ამიტომ სჯობია ყველაფრის მიღებისგან თავი შეიკავოს. და თუ 38,5 ზემოთ ავიდა მაშინ გინეკოლოგს მიმართეთ და ის გირჩევთ რა ქნათ.

Posted by: Einsam 12 Feb 2012, 13:44
რაც ამინდები გაურესდა სულ მეძინება. აი რამდენიც არ უნდა მეძინოს ღამე, სულ მეძინება. ყავა, ენერგეტიკული სასმელები რას აღარ ვსამ. ცოტა ხანი მიშველის და მერე ისევ ასე ვარ. რა ვქნა არვიცი.
შეიძლება უბრალოდ ისევ რკინის ნაკლებობა მაქვს. ადრე მქონდა რკინადეფიციტური ანემია და როგორ მივხვდე რის გამო მაქვს ეს მუდმივი დაღლილობის და მოთენთილობის შეგრძნება?!
ისე იმ პროდუქტების ჭამით დიდად არ ვიკლავ თავს რაც შეიცავს რკინას. პლიუს ქალური დღეებიც დიდხანს მაქვს. ახლა შემიძლიადავლიო რკინის შემცველი ტაბლეტები, მაგრამ საერთოდ რა და რამდენი უნდა ვჭამო რკინის დღური დოზა რომ შევივსო?
მადლობა წინასწარ mo.gif

Posted by: matilda123 13 Feb 2012, 00:37
Einsam
რკინის პრეპარატების მიღება სისხლის ანალიზის გარეშე არ შეიძლება.
შესაძლოა ჩვეულებრივი სეროტონინის დეფიციტი გქონდეს. არის ასეთი ჰორმონი სეროტონინი, რომელსაც "ბედნიერების" ჰორმონს ეძახიან და მისი რაოდენობა ორგანიზმში მზიანი დღეების მატებასთან ერთად მატულობს. როდესაც მზე ნაკლებად ჩანს ჩვენს ორგანიზმში მატულობს მელატონინი- სეროტონინის ანტაგონისტი.
თუ რას ემსახურება ეს 2 ჰორმონი, ქვემოთ ლინკებია და იქ წერია (შევეცადე გასაგები ენით დაწერილი ტექსტი შემერჩია)
http://www.onmeda.de/lexika/anatomie/serotonin.html
http://www.onmeda.de/lexika/anatomie/melatonin.html

Posted by: keen 15 Feb 2012, 20:58
ეჰ, vano_t აღარ შემოდის ფორუმზე?

....................................................

Posted by: Natuka NGN 16 Feb 2012, 00:14
Einsam
QUOTE
შეიძლება უბრალოდ ისევ რკინის ნაკლებობა მაქვს. ადრე მქონდა რკინადეფიციტური ანემია და როგორ მივხვდე რის გამო მაქვს ეს მუდმივი დაღლილობის და მოთენთილობის შეგრძნება?!

თუ რკინადეფიციტური ანემია გქონდა, ეს დაამხასიატებელია ჰიპოთირეოზისთვის. რადა ჰიპოთირეოზის ყევლა ნიშნანს ამბობ, მათ შორის რკიანდეიფციტიც მაგია. ამიტომ ენდოკრინოლოგთან ვიზიტს გირჩევდი.

Posted by: mishka---777 16 Feb 2012, 00:35
8 წლის ბავშვს ეს რამოდენიმე (3-4) დღეა აღენიშნება თვალის ქუთუთოების შესიება(შეშუპება) და რას მირჩევთ? რის ბრალი შეიძლება იყოს?
ვიცი რომ თირკმელებმა და ღვიძლმაც იცის ხოლმე,მაგრამ 8 წლის ასაკში??? თანაც ჩივილების გარეშე???
ან რომელ ექიმთან ჯობია მიყვანა?
დაახლოებით 1 თვის წინ საერთო ანალიზი გავუკეთეთ,სახის ფერი არ მოგვწონდა უბრალოდ და მაშინ ანალიზებმა ყველაფერი ნორმაში აჩვენა,თუმცა მაშინ არ ჰქონდა ქუთუთოებზე შეშუპება.
ტკივილით არაფერს არ იტკიებს,არც არავითარი ჩივილები არ აქვს.სხვა არანაირი პათოლოგია არ აღენიშნება ერთის გარდა:
დეკემბრის შუა რიცხვებში,თვალის ქვედა ნაწილში სახის ძვალზე მიიღო მცირე ტრავმა,რაღაც რკინას მიარტყა და მის შემდეგ ცალი თვალის გუგა უფრო მცირე ზომის აქვს,ვიდრე მეორე.თუმცა ესეც არანაირად არ მოქმედებს არც მხედველობაზე და ასევე არც არანაირი ჩივილები (თავბრუსხვევა,თავის ტკივილები,ღებინების შეგრძნება) არ აქვს.
რას მირჩევთ კეთილო ხალხო?

Posted by: tasikotasiko 16 Feb 2012, 02:04
გულის მხარეს მაქვს დისკონფორტი,თითქოს გაბუჟEბული მაქვსო,თითქოს მაქვს სიცარიელე და ეს სიცარიელე დისკონფორტს მიქმნის,აგრეთვე მგონია რომ გული არ ფუნქციონირებს და ეს მაგიჟEბს, ვითOმ ეს ყოველივე ნევროზის ბრალია?

Posted by: matilda123 16 Feb 2012, 22:59
Natuka NGN
QUOTE
თუ რკინადეფიციტური ანემია გქონდა, ეს დაამხასიატებელია ჰიპოთირეოზისთვის. რადა ჰიპოთირეოზის ყევლა ნიშნანს ამბობ, მათ შორის რკიანდეიფციტიც მაგია. ამიტომ ენდოკრინოლოგთან ვიზიტს გირჩევდი.

არაა მასე smile.gif
ეგ რასაც აღწერს ზოგადი სიმპტომატიკაა და მილიონ დაავადებას ახასიათებს. მაინცდამაინც რკინადეფიციტურ ანემიასთან კავშირში ნუ განიხილავ და კიდევ, მე პირადად არ ვიცოდი თუ რკინადეფიციტური ანემია ჰიპოთირეოზის სიმპტომატიკაში შედიოდა, ეგ სად წაიკითხე, მართლა მაინტერესებს
? mishka---777
QUOTE
8 წლის ბავშვს ეს რამოდენიმე (3-4) დღეა აღენიშნება თვალის ქუთუთოების შესიება(შეშუპება) და რას მირჩევთ? რის ბრალი შეიძლება იყოს?
ვიცი რომ თირკმელებმა და ღვიძლმაც იცის ხოლმე,მაგრამ 8 წლის ასაკში??? თანაც ჩივილების გარეშე???
ან რომელ ექიმთან ჯობია მიყვანა?
დაახლოებით 1 თვის წინ საერთო ანალიზი გავუკეთეთ,სახის ფერი არ მოგვწონდა უბრალოდ და მაშინ ანალიზებმა ყველაფერი ნორმაში აჩვენა,თუმცა მაშინ არ ჰქონდა ქუთუთოებზე შეშუპება.
ტკივილით არაფერს არ იტკიებს,არც არავითარი ჩივილები არ აქვს.სხვა არანაირი პათოლოგია არ აღენიშნება ერთის გარდა:
დეკემბრის შუა რიცხვებში,თვალის ქვედა ნაწილში სახის ძვალზე მიიღო მცირე ტრავმა,რაღაც რკინას მიარტყა და მის შემდეგ ცალი თვალის გუგა უფრო მცირე ზომის აქვს,ვიდრე მეორე.თუმცა ესეც არანაირად არ მოქმედებს არც მხედველობაზე და ასევე არც არანაირი ჩივილები (თავბრუსხვევა,თავის ტკივილები,ღებინების შეგრძნება) არ აქვს.
რას მირჩევთ კეთილო ხალხო?

პედიატრთან ვიზიტია აუციელებლი, რომელიც შარდის სტატუსს გაგაკეთებინებთ, თირკმლების დაავადება რომ გამოირიცხოს. ნაწერიდან გამომდინარე ვიფიქრე, რომ შეშუპება ორმხრივია, თუ არაა მასე?
ტრავმას რაც შეეხება, გუგების ზომაში სხვაობა სერიოზული პათოლოგიაა და თავისი მიზეზები აქვს. ნამდვილად გუგებზე ამბობ, თუ თვალის ჭრილზე? და როცა რაღაცას მიარტყა, მაშინვე უნდა მისულიყავით გადაუდებელი დახმარების ქირურგთან, ეხლა მაინც მიდით.

Posted by: mishka---777 16 Feb 2012, 23:54
matilda123
QUOTE
ნაწერიდან გამომდინარე ვიფიქრე, რომ შეშუპება ორმხრივია,

ორმხრივი,ანუ ორივე თვალზე ? დიახ 2-ვე ქუთუთო აქვს შესიებული.
დილით ახალ გაღვიძებულზე განსაკუთრებით.
QUOTE
ნამდვილად გუგებზე ამბობ, თუ თვალის ჭრილზე?

ნამდვილად გუგებზე.პოლიკლინიკის ოფთალმოლოგთან გვყავდა და ნევროპათოლოგთანაც. ორივე ექიმისაგან მივიღეთ ასეთი პასუხი: "ბავშვს საჩემო არაფერი სჭირს"-ო.
თუმცა რამე რომ არ გამოგვეპაროს,კარგი იქნება თუ ენცეფალოგრამას გაუკეთებთო.
QUOTE
როცა რაღაცას მიარტყა, მაშინვე უნდა მისულიყავით გადაუდებელი დახმარების ქირურგთან

ტრავმა არც ისე საგანგაშო იყო.ოდნავ ნაკაწრი ჰქონდა კანზე და შემდეგ დაუსივდა ის ადგილი,მეტი არაფერი.
ამიტომაც არ ჩავთვალეთ საჭიროდ ექიმთან ვიზიტი.გუგების ხვაობაც ამის შემდგომ შევამჩნიედ 2-3 კვირის შემდეგ.

თვალზე კი მინდა ექიმთან მივიყვანო აუცილებლად.როგორც ინტერნეტ სივრცეში ინფო მოვიძიე,თურმე ამას "ანიზოკორია" ეწოდება (გუგებს ვგულისხმობ).მაგრამ ზუსტად არ ვიცი სად მივიყვანო,ორთაჭალაში თვალის კლინიკაში თუ ახალ მზერაში.

Posted by: Natuka NGN 17 Feb 2012, 00:26
matilda123
შეიხედე ხოლმე ენდოკრინოლოგის თემაში და მასკედს დაეკითხე. პირველი მაგას გვეკითხება და B12 ვიტამინი და რკინა გაისინჯეთო. მეტსაც გტყვი, ხანდახან, ჰიპოთირეოზი შეიძლება რკინა-დეფიციტურმა ანემიამაც გამოიწვიოსო. ერთმაენთთან კავშირშია. მე მისგან ვიცი smile.gif

* * *
http://ru.wikipedia.org/wiki/%C3%E8%EF%EE%F2%E8%F0%E5%EE%E7
Анемический синдром: анемия — нормохромная нормоцитарная, гипохромная железодефицитная, макроцитарная, В12-дефицитная.
http://sanatorij.narod.ru/gipotireoz.htm
Гипотиреоз может сопровождаться малокровием - анемией. Наряду с нарушением превращения, в печени бета-каротина - оранжевого пигмента - это может придать коже бледный и слегка желтоватый оттенок.
http://www.medlinks.ru/article.php?sid=11
2. Гематологические анемии
- железодефицитная гипохромная
- нормохромная
- пернициозная
- фолиеводефицитная

3. Хирургические
- желчнокаменная болезнь
ერთი სიტყვით, 16 წელი ფარისებრმა გამტანჯა. იმდენი მაქვს მასზე ნაკითხი და იმდენი საკუთარ თავზე გადატანილი რომ... ასე რომ, ერთ-ერთი სიმპტომია
* * *
ჰოდა, მუმივი დაღლილობა და მოთენთილობა ჰიპოთირეოზის უმთავრესი ნიშანია, დღეს ჩვენში ეს სიე გავრცელებულია, ლამის ყოველ მესამეს სჭირს, არათუ მეორეს. ამიტომ ენდოკრინოლოგთან ვიზიტი არ აწყენდა მაგ იუზერს.

Posted by: Pathetikos 17 Feb 2012, 14:37
დილით გავიღვიძე ზურგში, მარცხენა მხარეს, "დაჭიმული" ნერვით თუ რაც ქვია და ძლივს ვინძრეოდი. გუშინ მთელი დღე ვიჯექი და ერთი კი ვითომ "გავივარჯიშე" ჩემი ჭკუით და დამეჭიმასავით ზურგში კუნთები, მაგრამ ასეთ დღეში თუ ჩავვარდებოდი, რას ვიფიქრებდი? biggrin.gif
მოკლედ, რა ვქნა? ახლაც ლამის გაუნძრევლად ვარ, იმიტომ რომ ხანდახან ტკივილი ფეთქავს ხოლმე, ზოგჯერ მთელ ზურგზე და ტანზე გადადის დაძაბულობა.
რამე ვარჯიში უნდა გავაკეთო?
თბილი შემოვიხვიე დაჭიმული ნერვის ადგილას.

Posted by: matilda123 17 Feb 2012, 22:22
Natuka NGN
QUOTE
შეიხედე ხოლმე ენდოკრინოლოგის თემაში და მასკედს დაეკითხე. პირველი მაგას გვეკითხება და B12 ვიტამინი და რკინა გაისინჯეთო. მეტსაც გტყვი, ხანდახან, ჰიპოთირეოზი შეიძლება რკინა-დეფიციტურმა ანემიამაც გამოიწვიოსო. ერთმაენთთან კავშირშია. მე მისგან ვიცი smile.gif

მასკედი ჩვეულებრივი ექიმია( თან ჰომეოპათი მგონი და ეგენი დაავადებას სხვა ჭრილში განიხილავენ, თუმცა მე მის პროფესიონალიზმში ეჭვი არ შემაქვს და ბევრს სასარგებლო რჩევა მისცა ამ განყოფილებაში) , ისეთივე როგორც ათასი სხვა და მასკედის დაწერილი ჭეშმარიტებად არ უნდა მივიღოთ.
QUOTE
может сопровождаться
შეიძლება ახლდეს არ ნიშნავს, რომ ჰიპოთირეოზის მასახიათებელი სიმპტომია. საერთოდ დაავადებებს აქვთ თავისი სპეციფიკური სიმპტომატიკა და პაციენტს თუ ანემია სჭირს, ალბათ 0,1% თუ იფიქრებს ჰიპოთირეოზზე. ესეც მაშინ როდესაც სხვა სპეციფიკური სიმპტომატიკაა გამოხატული.
ჰიპოთირეოზზე გეტანხმები, რომ ბევრს სჭირს, მაგრამ მე ჰიპოთირეოზით გამოწვეული ანემია არ მინახავს.
არსებობს ჰეროლდის "შინაგანი მედიცინა", რომელიც დღეს ინტერნისტების "ბიბლიაა" შეიძლება ითქვას smile.gif. ქართულადაც არის ნათარგმნი ნიკა ქურიძის მიერ, შეგიძლია იქ წაიკითხო რაც გაინტერესებს. ინტერნეტის ინფოს მე პირადად არ ვენდობი.
QUOTE
მუმივი დაღლილობა და მოთენთილობა ჰიპოთირეოზის უმთავრესი ნიშანია,

შენ რაც დაწერე მაგას წონაში მომატებაც ემატება smile.gif, მაგრამ მაგას ზოგადი სიმპტომატიკა ჰქვია და ათას დაავადებას ახასიათEბს, მათ შორის ანემიასაც smile.gif ჰიპოთირეოზის უმთავრესი ნიშნებია: მშრალი კანი, სიცივის აუტან;ლობა, თმის ცვენა, ბრადიკარდია, მიქსედემა, შეკრულობა, ჩახლეჩილი ხმა . ბევრ პაციენტს უვითარდება არტერიოსკლეროზი. სხვა გართულებებზე აღარ ვსაუბრობ, რომელიც შესაძლოა სიკვდილით დამთავრდეს.
mishka---777
QUOTE
ორმხრივი,ანუ ორივე თვალზე ? დიახ 2-ვე ქუთუთო აქვს შესიებული.
დილით ახალ გაღვიძებულზე განსაკუთრებით.

შარდის ანალიზია გასაკეთებელი.
კი, ანიზოკორიაა რასაც თქვენ ამბობთ და სხვა მიზეზებთა შორის ტრავმამაც ა(ნუ აქ სახის ტრავმას ვგულისხმო)ბ შეიძლება გამოიწვიოს. თვალის ექიმს მიმართეთ და მოუყევით რის შემდეგაც განუვითარდა.
Pathetikos
სითბო, მასაჟი. რაიმე გამახურებელი მაზი შეიზილე, თუ ტკივილი გაუსაძლისია , ორთოპედს მიმართე და ზურგზე ნერვების ბლოკადას გაგიკეთებს და ტკივილი გაგივლის.

Posted by: mishka---777 17 Feb 2012, 23:44
matilda123
მადლობა ყურადღებისათვის.
კიდევ ერთი შეკითხვა:
QUOTE
შარდის ანალიზია გასაკეთებელი.

ეს მხოლოდ ჩვეულებრივი შარდის ანალიზია გასაკეთებელი,თუ შარდში კონკრეტულად რაიმეს შემცველობაზე?
ჩიყვის დროს როგორც ვიცი მგონი "შარდში იოდის შემცველობა"-ზე კეთდება ანალიზი და ამ შემთხვევაში როგორ იქნება ხომ ვერ მეტყვით?

წინასწარ კიდევ ერთხელ გიხდით მადლობას.

Posted by: tasikotasiko 18 Feb 2012, 01:53
ფილტვებზე გაკვანძის შეგრძნებას რა უნდა იწვევდეს?დავიტანჯე ეს ორი დღEა

Posted by: matilda123 18 Feb 2012, 17:33
mishka---777
QUOTE
ეს მხოლოდ ჩვეულებრივი შარდის ანალიზია გასაკეთებელი,თუ შარდში კონკრეტულად რაიმეს შემცველობაზე?
ჩიყვის დროს როგორც ვიცი მგონი "შარდში იოდის შემცველობა"-ზე კეთდება ანალიზი და ამ შემთხვევაში როგორ იქნება ხომ ვერ მეტყვით?

შარდის სტატუსი ჰქვია, (საუკეთესოა დილის შარდი.) როდესაც იზომება შარდის ხვედრითი წონა, ცილა, გლუკოზა, კეტონები, ბილირუბინი, ერითროციტები, ლეიკოციტები.
tasikotasiko
QUOTE
ფილტვებზე გაკვანძის შეგრძნებას რა უნდა იწვევდეს?დავიტანჯე ეს ორი დღEა

ფილტვები არ იკვანძება. ვერ ვხვდები რას გულისხმობ, გტკივა, გაბუჟებულია თუ რას გულისხმობ?

Posted by: mishka---777 18 Feb 2012, 21:31
matilda123

ორშაბათს მივაკითხავ ალბათ ენმედიც-ს.
მანდ გავაკეთებინებ ანალიზს.
კიდევ ერთხელ გიხდით მადლობას.

Posted by: matilda123 19 Feb 2012, 00:08
mishka---777
QUOTE
კიდევ ერთხელ გიხდით მადლობას.

არაფერს smile.gif მალე გამოჯანმრთელებას გისურვებთ.

Posted by: Natuka NGN 19 Feb 2012, 02:38
matilda123
ახლა არ ვაპირებ კამათს ამ საკითხზე შენთან, ვიკის ბმულზე ჩამოთვლილია როგორც სინდრომი ჰიპოთირეოზის და კიდევ მეორეზე. მხოლოდ ერთი ნახე - მოჯეტ სოპროვოჯდატსიაო. შენ არ გინახავს არ ნიშნავს, რომ არ ახასიათებს, ხოლო რაც ჩამოთვალე, თმის ცვენა, ყაბზობა, კანის სიმშრალე და ა.შ. არცერთი არ მქონდა, ეს კლინიკური ისმპტომატიკაა, კი ბატონო, მაგრამ არის სუბკლინიკური ჰიპოთირეოზიც. ამ ყველაფერთან ერთად ანემია მქონდა. თუ არ გინახავს პაციენტი მსგავსი სინდრომით, აგერ ვარ smile.gif თუმცა საინტერესო ისაა, რომ ეს ანემია ყოვეთვის არ მქონდა და მიზეზს ვერ პოუობდნენ სისხლის გადახმაში რა იყო. თან კანის სიმშრალე არასოდეს მქონია, არც ჩახლეჩილი ხმადა არც ბრადიკარგია.
http://rustamsamedov.narod.ru/physlib/diaghypo.html
ესევ საინტერესო ბმულია:
Система кроветворения
У больных, как правило, развивается гипохромная анемия (что обусловлено снижением всасывания железа в тонком кишечнике и отсутствием стимулирующего влияния тиреоидных гормонов на эритропоэз), реже - В12-дефицитная анемия (вследствие нарушения всасывания витамина B12 в кишечнике).
http://www.medlinks.ru/article.php?sid=11
• Анемический синдром: нормохромная нормоцитарная, гипохромная железодефицитная, макроцитарная, В12-дефицитная анемия.
Характерные для гипотиреоза нарушения тромбоцитарного ростка ведут к снижению агрегации тромбоцитов, что в сочетании со снижением уровня в плазме факторов VIII и IX, а также повышенной ломкостью капилляров усугубляет кровоточивость.
რაზე ვკამათობთ არ ვიცი.
Г. А. Мельниченко
Кафедра эндокринологии Московской Медицинской Академии им. И. М. Сеченова
მერე რა რომ რუსული სკოლაა? ანემია ერთ-ერთი დამახასიატებელია ჰიპოთირეოზისთვის და ეს მე არ მომიგონებია.
მსგავსი პაციენტი მე ვიყავი. smile.gif
http://www.ithyroid.com/anemia.htm
ესეც საინტერესო ბმულია, სულ ბოლომდე მსგავსი კორელაცია არ არისო, მაგრამ რკინა-დეფიციტი შეიძლება გახდეს ჰიპოთირეოზის მიზეზიო, ხოლო სპილენძის დეფიციტი ჰიპერთირეოზისო.
ან კიდევ ამ ფორუმზეც:
INTRODUCTION: Anemias are diagnosed in 20-60% patients with hypothyroidism. Real values of the degree of anemia are estimated by radioisotopic analysis due to the lower volume of plasma in hypothyroidism causing false high levels of hemoglobin in blood. Anemia is often the first sign of hypothyroidism. Diagnosis of hypothyroidism should be considered in every case of anemia with uncertain etiology because sometimes signs of overt hypothyroidism needn't necessarily be evident. Microcytic, macrocytic and normocytic are regularly described anemias. CLASSIFICATION: Microcytic anemia is usually ascribed to malabsorption of iron and loss of iron by menorrhagia. Macrocytic anemia is caused by malabsorption of vitamin B12, folic acid, pernicious anemia and inadequate nutrition. Pernicious anemia occurs 20 times more frequently in patients with hypothyroidism than generally. Macrocytosis is found in up to 55% patients with hypothyroidism and may result from the insufficiency of the thyroid hormones themselves without nutritive deficit. Normocytic anemia, so-called uncomplicated anemia, arises due to thyroid hormones deficit itself not followed by nutritive deficit. This type of anemia is considered to be an adaptation to a decreased basal metabolism. Thyroid hormones directly or indirectly, through erythropoietin, stimulate growth of erythroid colonies (BFU-E, CFU-E). Normocytic anemia is characterized by reticulopenia, hypoplasia of erythroid lineage, decreased level of erythropoietin, mainly regular erythrocyte survival. Acanthocytosis findings in cytologic blood smear suggest hypothyroidism in about 90% of
http://www.prep4usmle.com/forum/thread/52973/
ერთი სიტყვით, ძლაინ უაზრო კამათი გამოგვდის. Anemia is often the first sign of hypothyroidism.
ამიტომ, სანამ თმსი ცვენა, ბრადიკარდია და ა.შ. აქტიურად დაიწყებოდეს, თუ ანემიაა, აუცილებელია ფარსიებრის პრობლემის გამორიცხვა. ვინაიდან, თავად რკინადეფიციტმა იქით შეიძლება გამოიწვიოს ეს ჰიპოთირეოზი.

Posted by: matilda123 19 Feb 2012, 13:52
Natuka NGN
QUOTE
ახლა არ ვაპირებ კამათს ამ საკითხზე შენთან

smile.gif))
კარგი გოგო ხარ შEნ. ეს რაც შენ დაწერე ძალიან საინტერესოა. მაგრამ ეგ ყველაფერი ჯერ სამედიცინო წიგნებში არ შესულა, რადგან კვლევის დონეზეა.მეც წავიკითხე ეხლა, რომ კვლევების დროს ჰიპოთირეოზით დაავადებულ პაცინეტენთა უმეტესობას ანემიაც აღმოუჩინეს. ზუსტი მიზეზი ჯერ დაუდგენელიაიმედია. ბევრი თეორიაა, მაგრამ ჯერ დაუზუსტებელი.
ჩემთვის მაგალითად ჰაშიმოტოს ჰიპოთირეოზის დროს უფრო თეორიულად ადვილად წარმოსადგენია ანემია. რადგან ჰაშიმოტოს ჩიყვი აუტოიმუნური დაავადებაა და ამ დროს წარმოქმნილი ანტისხეულები შესაძლოა კუჭის და ნაწლავების ლორწოვანსსაც აზიანებენ. თუ კუჭის ლორწოვანი დაზიანდა დაიკლებს პარიეტალური უჯრედების რიცხვი, რომლებიც ინტრინზიკ ფაქტორს წარმოქმნიან და თუ ეს ფაქტორი აღარ არის არ მოხდება ვიტ. B12 შეტვისება ნაწლავებში , რადგან მხოლოდ ინტრინზიკ-ფაქტორთან კომბინაჩიაში ხდება ნაწლავების კედლების მიერ ვიტ.B 12 შეწოვა. და თუ ეს ასეა მაშინ გამოდი პერნიციოზული ანუ B12 დეფიციტური ანემია
მაგრამ ფერიტინის და ფარისებრის ჰორმონების კორელაციის მექანიზმი რაღაც კარგად ვერ წარმოვიდგინე. და მე თუ მოქმედების მექანიზმი კარგად ვერ გავიგე, ის რაღაც ჩემთვის ბუნდოვანი რჩება.

პ.ს. ისე ეხლა შენ რომ დაგეწერა ანემიაოო, მე ამდენი ტვინის განძრევა და ჰიპოთირეოზის დროს ანემიის გამომწვევ მექანიზმებზე ფიქრი არ მომიწევდა. კარგია ყოველდღე რაღაცას რომ იგებს ადამიანი.

Posted by: tasikotasiko 19 Feb 2012, 21:00
matilda123
ფეხი რომ გაგეკვანძება აი მასეთი შეგრძნება მაქვს.როგორ ავხსნა სხვანაირად არ ვიცი,ფაკტია რომ მაწუხებს

Posted by: misha-htc 20 Feb 2012, 12:32
Mainteresebs el. Sokeri SeiZleba adamianze ar moqmedebdes? Cemi megobari amtkicebda - arsebobs iSviaTi daavadeba roca adamians tkivilis SegrZneba ar aqvs da Sokeric ar moqmedebso smile.gif magram ravici Tu tvinidan el signalebi miewodeba kidurebs maSin el Sokerma unda gamoiwvios am signalebis CaxSoba yvela variantSi da ar unda qondes mniSvneloba aranair daavadebas da tkivilis aRqmas - Cemi diletanturi azriT asea da ravici smile.gif

mapatieT am kiTxvisTvis - Tqven aq karg saqmes akeTebT da me suleluri kiTxviT SemogeWeriT magram Tu mipasuxebT madlobeli dagrCebiT smile.gif

Posted by: Masked 20 Feb 2012, 14:08
matilda123
მედ. წიგნები არ არის ძალიან ბევრი დაწერილი. ენდოკრინოლოგიაში განსაკუტრებიტ, ამიტომ არის კოველი წელი პროფ. საკუტარი გამოკვლევები იწერება.
პერნიციöზე ანემიი კი მორბუს ჰაშიმოტო( ეს არის ძველი ბეგრიფფ ეხლა აქვს სახელი ტჰირეოპატჰიები) დაკავშირებული არის მეორე აუტოიმმ. დაავადებასტან მით ინტრინსიკფაკტოა AK და პარიეტალცელლ AK სთან. ჰიპოტჰ. ნაკლებობა ტჰირონინის და ტჰიროქსინის დომინანც იწვევს ანემიას , და ასევე პირიქიტ, ანემიი შეიძლება გამოიწსიოს "ჰიპოტჰრიეზე" ტეესჰა ამაღლება.


Posted by: Riddle 20 Feb 2012, 19:18
იმედია, შესაფერის ადგილას ვკითხულობ.

მომატებული აზოტის დროს რამე სახის მკურნალობა არ ინიშნება? წამლები, რამე.

Posted by: mishka---777 20 Feb 2012, 22:07
მიმასწავლეთ თუ შეიძლება არამიანცის ტერიტორიაზე კონკრეტულად სად კეთდება ბავშვის თავის ტვინის ექოსკოპია?
მითხრეს რაღაც ახალი შენობა არისო და იქ აკეთებენო და ზუსტად ვერ მივხდი,ვერ მივაგენი სადაა კონკრეტულად. user.gif

Posted by: matilda123 20 Feb 2012, 23:11
tasikotasiko
QUOTE
matilda123
ფეხი რომ გაგეკვანძება აი მასეთი შეგრძნება მაქვს.როგორ ავხსნა სხვანაირად არ ვიცი,ფაკტია რომ მაწუხებს

არსებობს ასეთი რამ კუნთის ანთება და ტკივილი, იმას აქვს ზურგში მასეთი შეგრძნება. ორთოპედს მიმართე. Masked
Masked
QUOTE
მედ. წიგნები არ არის ძალიან ბევრი დაწერილი. ენდოკრინოლოგიაში განსაკუტრებიტ, ამიტომ არის კოველი წელი პროფ. საკუტარი გამოკვლევები იწერება.
პერნიციöზე ანემიი კი მორბუს ჰაშიმოტო( ეს არის ძველი ბეგრიფფ ეხლა აქვს სახელი ტჰირეოპატჰიები) დაკავშირებული არის მეორე აუტოიმმ. დაავადებასტან მით ინტრინსიკფაკტოა AK და პარიეტალცელლ AK სთან. ჰიპოტჰ. ნაკლებობა ტჰირონინის და ტჰიროქსინის დომინანც იწვევს ანემიას , და ასევე პირიქიტ, ანემიი შეიძლება გამოიწსიოს "ჰიპოტჰრიეზე" ტეესჰა ამაღლება.

წავიკითხე რაღაცეები , ვიღაცეების კვლევები, რომლებიც ვირთხებზე ცდის შედეგად იყო დაწერილი. ნეტში ბევრი ინფო არაა, თან ენდოკრინოლოგია არ არის ჩემი სფერო მასე დეტალებში ვერ ვერკვევი.
QUOTE
მომატებული აზოტის დროს რამე სახის მკურნალობა არ ინიშნება? წამლები, რამე.


კი, პირველ რიგში კუჭის მოქმედება უნდა მოწესრიგდეს. საჭმელი კუჭში დიდხანს არ უნდა რჩებოდეს ამიტომ კარგია ლაქტულოზის მიღება. და მეტრონიდაზოლის მიღება, რათა შემცირდეს ამონიაკის მწარმოებელი ნაწლავური ფლორა. და თავისთავად ძირითად დაავდებას უნდა მიხედვა (აქ, ღვიძლის პათოლოგიას) .

მოკლედ გეტყვის ნორმალურ ფიზიოლოგიას , პათოლოგია რომ გაიგო. ამონიაკს ( აზოტს რასაც ეძახიან) ორგანიზმში გამოყოფენ ნაწლავური ბაქტერიები და რაც დიდხანს ყოვნდება საკვები ნაწლავებში( მაგ. შეკრულობის დროს) მით მეტი ბაქტერიაა და შესაბამისად მეტი ამონიაკი წარმოიქმნება. ეს ამონიაკი მერე მიყვება სისხლს და მიდის ღვიძლში , რათა იქ გაიფილტროს და Harnstoff-ად გარდაიქმნას, ქართულად დამავიწყდა რა ჰქვია, მგონი შარდოვანა( ზუსტად არ მახსოვს , თავს ვერ დავდებ) და როდესაც რაღაც მიზეზის გამო ( ჰეპატიტი, ან ღვიძლის შეგუბება, ციროზი,კიბო და ა.შ.) ღვიძლის ფუნქცია დარღვეულია და ღვიძლი ვერ იღებს ამონიაკს, გაუფილტრავი და გადაუმუშავებელი ამონიაკი მიყვება ისევ სისხლს და ყველაზე ცუდ შემთხვევაში ტვინს აზიანებს. ამას უკვე ღვიძლისმიერი ენცეფალოპათია ჰქვია და ცალკე პათოლოგიაა.

Posted by: Riddle 21 Feb 2012, 13:54
matilda123
დიდი მადლობა დეტალური განმარტებისთვის.

აზოტი/ამონიაკი ერთი და იგივეა, კრეატინინიც? ღვიძლზე მგონი პრობლემა არ არის (მგონი), პროსტატის ჰიპერპლაზიაა დიაგნოზი ამ ეტაპზე და შარდის ბუშტი+თირკმელები არის ცუდ მდგომარეობაში. მხოლოდ ბევრი სითხე (ჩაი, მინერალური წყალი, წვენები) დაუნიშნა მკურნალმა ექიმმა კრეატინინის დონის დასაწევად და კუჭის მოქმედებისთვის წამალი. ანუ, ეს სრულიად საკმარისია ე.წ. აზოტის დასარეგულირებლად?

Posted by: fikria 21 Feb 2012, 15:42
37,2–37,4 ტემპერატურა მაკვს ხშირად და რისი ბრალია თუ შეგიძლიათ მითხრატთ? sad.gif ფილტვები სუფთაა, სისხლის ანალიზი ნორმალურიsad.gif

Posted by: Natuka NGN 21 Feb 2012, 16:11
fikria
დააკვირდი, თუ ციკლის მეორე ანხევარშია, ანუ მენსტრუაციის მოსვლამდე 7-10დღით ადრე, ნორმალურია. ნუ ეძებ ნურაფერს. smile.gif

Posted by: fikria 21 Feb 2012, 23:13
Natuka NGN
ხოოო, ზუსტად მაგ დროს მაქვს ხოლმე, მაგრამ ადრე არ მქონდა, რამდენიმე თვეა დამეწყო , ჰორმონები მეთამაშებიან ნეტა??? smile.gif
დიდი მადლობაააააsmile.gif

Posted by: geoeka 22 Feb 2012, 01:04
QUOTE
37,2–37,4 ტემპერატურა მაკვს ხშირად და რისი ბრალია თუ შეგიძლიათ მითხრატთ?ფილტვები სუფთაა, სისხლის ანალიზი ნორმალური

არავითარ შემთხვევაში არ უნდა მიუშვათ თავის ნებაზე - სუბფებრილიტეტი ანუ დაბალი სიცხეები ძირითადად ახასიათებს ქრონიკულ ანთებით პროცესს. საჭიროა დამატებითი გამოკვლევები მიზეზის დასადგენად. რაც შეეხება ანალიზების სისუფთავეს, ესეც შესაძლებელია, ყოფილა შემთხვევები, როდესაც ანალიზი რაღაც ეტაპზე არ ასახავს არსებულ მდგომარეობას, თანაც უნდა გაითვალისწინოთ შესაძლო შეცდომა ანალიზში.
* * *
QUOTE
აზოტი/ამონიაკი ერთი და იგივეა, კრეატინინიც? ღვიძლზე მგონი პრობლემა არ არის (მგონი), პროსტატის ჰიპერპლაზიაა დიაგნოზი ამ ეტაპზე და შარდის ბუშტი+თირკმელები არის ცუდ მდგომარეობაში. მხოლოდ ბევრი სითხე (ჩაი, მინერალური წყალი, წვენები) დაუნიშნა მკურნალმა ექიმმა კრეატინინის დონის დასაწევად და კუჭის მოქმედებისთვის წამალი. ანუ, ეს სრულიად საკმარისია ე.წ. აზოტის დასარეგულირებლად?

კრეატინინის დონის მომატება თირკმლის ფუნქციას ახასიათებს, როგორც ჩანს პროსტატის ჰიპერპლაზიამ გამოიწვია შარდის არარეგულარული გამოყოფა, რამაც თავის მხრივ გამოიწვია ზედა საშარდე გზების და თირკმელების დაზიანება, რაზეც მეტყველებს კრეატინინის მატება. ამ შემთხვევაში ექიმი ცდილობს სითხეების გაზრდით, "გამორეცხოს" თირკმელი, მაგრამ თუ შარდვა არ არის აღდგენილი, სითხით დატვირთვა რას მოგცემთ? ამიტომ ჯერ უნდა იქნას აღდგენილი რეგულარული შარდვის პროცესი და ამის შემდეგ შესაძლოა სითხეებმაც მოგცეთ შედეგი, თუ დაზიანება მსუბუქია. თუ არა, მაშინ საერთოდ არ არის საკმარისი გატარებული ზომები. საჭიროა მონიტორინგის ჩატარება.

Posted by: matilda123 22 Feb 2012, 01:25
Riddle
QUOTE
ზოტი/ამონიაკი ერთი და იგივეა, კრეატინინიც?

არა, კრეატინინი სხვა რამეა. კრეატინინით თირკმლების ფუნქცია მოწმდება. რაც უფრო მაღალია კრეატინინი სისხლში მით უფრო ცუდად მუშაობს თირკმელი.

Posted by: geoeka 22 Feb 2012, 01:27
QUOTE
სიცხის დამწევი რა საშუალება არსებობს ისეთი რომ ორსულობას არ ავნოს?
ნუ მიიღებთ ფარმაკოპრეპარატებს, არსებობს მცენარეული ალტერნატივა, რომელიც არ გამოიწვევს არავითარ გართულებებს. პარაცეტამოლი ზოგ ქვეყანაში არის აკრძალული პრეპარატი. თქვენ შეგიძლიათ გამოიყენოთ ჩაი ჟოლოს მურაბით, ჟოლოს ფოთლებით, ცაცხვის, თავშავას ჩაი, შეგიძლიათ გამოიყენოთ ოყნები ძმრით ან სოდით, ოყნის ტემპერატურა არ უნდა იყოს მკვეთრად განსხვავებული ანუ ცივი, ოთახის ტემპერატურაზე დაბალი არ უნდა იყოს, სასურველია უფრო თბილი, არყის ან სპირტის შეზელვები (უნდა წაისვათ მთელს სხეულზე და აორთქლებისას ტემპერატურა მოიკლებს, შესაძლებელია ძმრიანი წინდების ჩაცმა, ასევე კვერცხის გულის წასმა ფეხისგულებზე. არსებობს კიდევ სხვა მცენარეული პრეპარატებიც, თუ ეს ჩამონათვალი საკმარისი არ აღმოჩნდება.

* * *
QUOTE

ფეხი რომ გაგეკვანძება აი მასეთი შეგრძნება მაქვს.როგორ ავხსნა სხვანაირად არ ვიცი,ფაკტია რომ მაწუხებს
ეს სიმპტომი ახასიათებს კუნთის სისხლისმიმოქცევის დარღვევას, ხშირად კალცის ნაკლებობას სისხლში, ასევე შესაძლებელია სხვა მიკროელემენტის ნაკლებობაც, შესაძლებელია ვენური უკმარისობისძთვისაც და სპორტსმენებში, გაზრდილი დატვირთვების შემთხვევებში. პირველ რიგში გირჩევთ კალცის პრეპარატების მიღებას, თუ ეს არ მოგიხსნით ამ სიმპტომს, შემდეგ უნდა გამოირიცხოს ზემოთჩამოთვლილი მიზეზები.

Posted by: Natuka NGN 22 Feb 2012, 03:10
geoeka
თუ მენსტრუაციის დაწყებისთანავე სიცხე ნორმას უბრუნდება, ეს არის ბუნებრივი რეაქცია პროგესტერონზე. რატომაა,რომ ორსულებს დაბალი სიცხეები აქვთ და ეს ხომ ნორმაა? სწორედ იგივე რეაქციაა ციკლის მეორე ნახევარში. პროგესტერონი იმატებს და ორგანიზმი რეაგიებს.

* * *
fikria
დავწერე რაზეა რეაქცია. პროგესტერონზე. კარგად დააკვირდი, თვიურისმოსვლაზე თუ სიცხე არ იკლებს, მაშინ სხვა რამ ეძებე, მაგრამ თუ ნორმაში დგება და თავს კარგად გრძნობ - შენი ორგანიზმი ბავშვს ითხოვს smile.gif ნუ ეს ბოლო ხუმრობა იყო smile.gif
* * *
QUOTE
შეგიძლიათ გამოიყენოთ ოყნები ძმრით ან სოდით, ოყნის ტემპერატურა არ უნდა იყოს მკვეთრად განსხვავებული ანუ ცივი, ოთახის ტემპერატურაზე დაბალი არ უნდა იყოს, სასურველია უფრო თბილი, არყის ან სპირტის შეზელვები (უნდა წაისვათ მთელს სხეულზე და აორთქლებისას ტემპერატურა მოიკლებს, შესაძლებელია ძმრიანი წინდების ჩაცმა, ასევე კვერცხის გულის წასმა ფეხისგულებზე. არსებობს კიდევ სხვა მცენარეული პრეპარატებიც, თუ ეს ჩამონათვალი საკმარისი არ აღმოჩნდება.

რა ძმარი, რა სპირტი და რა სოდის ოყნები ორსულზე? და საერთოდ რითი გინდათ დამისაბუთით, რომ სოდიანი წყლის ნაწლავებში შეშვება რაიმენაირად მარგებელია?
და საერთოდ, ძმარმა შიეძლება შოკი გამოიწვიოს.
მაღალი ტემპერატიროს დასაგდებად საუკეთესოა ჩვეულებრივი წყალი და არა ალკოჰოლი ან სოდა.

Posted by: Solveig 22 Feb 2012, 03:18
geoeka
QUOTE
, შეგიძლიათ გამოიყენოთ ოყნები ძმრით ან სოდით,

facepalm.gif
ბროწეულის ოყნებზე რა აზრისა ხართ?

QUOTE
კვერცხის გულის წასმა ფეხისგულებზე

biggrin.gif ვახ, რას არ წაიკითხავ, რა....

Posted by: Riddle 22 Feb 2012, 13:19
geoeka
matilda123
დიდი მადლობა პასუხისთვის. ამ ეტაპზე კათეტერი არის ჩადგმული. გამოდის, სითხეების მიღება ყოფილა მართლა გამოსავალი. :/ მეგონა, კიდევ შეიძლებოდა რამე.

Posted by: fikria 22 Feb 2012, 14:33
QUOTE
არავითარ შემთხვევაში არ უნდა მიუშვათ თავის ნებაზე - სუბფებრილიტეტი ანუ დაბალი სიცხეები ძირითადად ახასიათებს ქრონიკულ ანთებით პროცესს. საჭიროა დამატებითი გამოკვლევები მიზეზის დასადგენად. რაც შეეხება ანალიზების სისუფთავეს, ესეც შესაძლებელია, ყოფილა შემთხვევები, როდესაც ანალიზი რაღაც ეტაპზე არ ასახავს არსებულ მდგომარეობას, თანაც უნდა გაითვალისწინოთ შესაძლო შეცდომა ანალიზში.
* * *

და რამდენჯერ უნდა გავიკეთო ანალიზი, რომ საბოლოოდ ჩამოვყალიბდე?

Posted by: gio_9999 22 Feb 2012, 22:20
მოგესალმებით ყველას

თერაპევტებთან მაქვს შეკითხვა:

ბოლო ორი თვეა მაქვს უაზროთ მერყევი ტემპერატურა, მიზეზი ვერ დავადგინე

ესეიგი თუ მთელი დღე სახლში ვარ ან სამსახურში ოფისში ვზივარ ტემპერატურა მერყეობს 36.4 დან 36.8 –მდე
საკმარისია ქუჩაში გავიდე, მანქანით ან ფეხით ვიარო ცოტა, ეგრევე მიწევს 37.0 – 37.2–მდე, სახლში შემოვალ და ნახევარ საათში ან უფრო მალე ეგრევე ჩამოდის 36.4 – 36.6 მდე

ემოციურად დატვირთული ვარ ეს ბოლო ხანია, რაღაცეებზე ვნერვიულობ, იქნებ მაგის ბრალია? ერთი სიტვით იქნებ დამაკვალიანოთ ვის მივმართო, რისი ბრალი შეიძლება იყოს, ან იქნებ ესეთია ჩემი ორგანიზმის თავისებურება, უბრალოდ ადრე არ შემიმჩნევია, ორი თვის წინ გავიზომე და იმის მერე სულ ვიზომავ და მასე მაქვს, სუბიექტური ცუდი შეგრძნება პრინციპში არ მაქვს
ყველას უმორჩილესად გთხოვთ, იქნებ დამეხმაროთ.

წინასწარ ყველას მადლობა

Posted by: an apple 23 Feb 2012, 14:28
გამარჯობათ,ჩემს მეგობარს დაეწყო ქავილი ხელებსა და ფეხის ტერფებზე. თავიდან ქავილი აწუხებდა მხოლოდ საღამოს ,უჩნდებოდა პატარა ,წითელი ლაქები, და ის ადგილები ებერებოდა ,დილით ჩვეულებრივად უცხრებოდა და არანაირი ნიშანი აღარ ქონდა. დაიწყო დიეტა და ალერგიული წამლები და მალამო ,ალერგოლოგის რჩევით,მაგრამ დღეს საშინელ მდგომარეობაში ჩავარდა ,ხელის მტევნები დაუსივდა და ქავილმა კინაღამ გააგიჟა. გარეგანი შემოწმებით გულის რიტმი აქვს დარღვეული და წინა თვეში მენსტრუალური ციკლი ჩაუვარდა ,თუ ამას რაიმე კავშირი აქვს ზემოთაღნიშნულთან.

Posted by: matilda123 23 Feb 2012, 22:42
gio_9999
QUOTE
  გამოგზავნილია: 22 Feb 2012, 22:20  #29854838     ბარათი · პროფილი · პირადი მიმოწერა · ელფოსტა · ვებგვერდი · im · yim
მოგესალმებით ყველას

თერაპევტებთან მაქვს შეკითხვა:

ბოლო ორი თვეა მაქვს უაზროთ მერყევი ტემპერატურა, მიზეზი ვერ დავადგინე

ესეიგი თუ მთელი დღე სახლში ვარ ან სამსახურში ოფისში ვზივარ ტემპერატურა მერყეობს 36.4 დან 36.8 –მდე
საკმარისია ქუჩაში გავიდე, მანქანით ან ფეხით ვიარო ცოტა, ეგრევე მიწევს 37.0 – 37.2–მდე, სახლში შემოვალ და ნახევარ საათში ან უფრო მალე ეგრევე ჩამოდის 36.4 – 36.6 მდე

ემოციურად დატვირთული ვარ ეს ბოლო ხანია, რაღაცეებზე ვნერვიულობ, იქნებ მაგის ბრალია? ერთი სიტვით იქნებ დამაკვალიანოთ ვის მივმართო, რისი ბრალი შეიძლება იყოს, ან იქნებ ესეთია ჩემი ორგანიზმის თავისებურება, უბრალოდ ადრე არ შემიმჩნევია, ორი თვის წინ გავიზომე და იმის მერე სულ ვიზომავ და მასე მაქვს, სუბიექტური ცუდი შეგრძნება პრინციპში არ მაქვს
ყველას უმორჩილესად გთხოვთ, იქნებ დამეხმაროთ.

არაფერი განგაშო არ გაქვს . 37,4 მდე ტემპერატურა ნორმად ითვლება.
an apple
QUOTE
გამარჯობათ,ჩემს მეგობარს დაეწყო ქავილი ხელებსა და ფეხის ტერფებზე. თავიდან ქავილი აწუხებდა მხოლოდ საღამოს ,უჩნდებოდა პატარა ,წითელი ლაქები, და ის ადგილები ებერებოდა ,დილით ჩვეულებრივად უცხრებოდა და არანაირი ნიშანი აღარ ქონდა. დაიწყო დიეტა და ალერგიული წამლები და მალამო ,ალერგოლოგის რჩევით,მაგრამ დღეს საშინელ მდგომარეობაში ჩავარდა ,ხელის მტევნები დაუსივდა და ქავილმა კინაღამ გააგიჟა. გარეგანი შემოწმებით გულის რიტმი აქვს დარღვეული და წინა თვეში მენსტრუალური ციკლი ჩაუვარდა ,თუ ამას რაიმე კავშირი აქვს ზემოთაღნიშნულთან.

ხელების ქავილი, წითელი ლაქები, შესიება , მენსტრუალური ციკლის დარღვევა ჩემთვის პირადად ღვიძლის პათოლოგიასთან ასოცირდება. მაგრამ, უფრო რომ გავერკვეთ. გაიკეთეთ სისხლის ანალიზი- ტრანსამინაზებზე(GPT, GOT, Gama-GT, AP) , ბილირუბინზე, ფარისებრი ჯირKვლის (TSH) ჰორმონზე და აუცილებლად მუცლის ღრუს ექოსკოპია. კინაღამ დამავიწყდა,- თქვენს მეგობარს ჰეპატიტი ხომ არ აქვს რომელიმე?

Posted by: an apple 23 Feb 2012, 23:18
matilda123
ახლა ვკითხე და არ აქვს ჰეპატიტი გადატანილი და არც რაიმე სერიოზული დაავადება არ ახსენდება,ეს პირველი სერიოზული ვიზიტია ექიმთან. არის 21 წლის. ბატონები გაიხსენა და მართლა , ერთი წლის წინაც ჰქონდა ციკლთან დაკავშირებით პრობლემა ,მაგრამ ყურადღება არ მიუქცევია ( თვეში 2-ჯერ დაემართა დენა) .
QUOTE
ტრანსამინაზებზე(GPT, GOT, Gama-GT, AP) , ბილირუბინზე
იქნებ მოკლედ მითხრათ რას წარმოადგენს ეს ანალიზები . ფარისებრზე გამოკვლევას აპირებს ,მაგრამ ქავილისგან დილით ისეთი სტრესი აქვს ,ანალიზებზე არ წავიყვანე ,როგორც ვიცი გავლენა აქვს. დიდი მადლობა პასუხისთვის.

Posted by: Natuka NGN 23 Feb 2012, 23:36
an apple
მიდი შეამოწმე ფაროსებრზე აუცილებლად. იმ სიმპტომებს მეუბენები, რაც ჩემს თანამშრომელს ქონდა და ავტოიმუნური ტოქსიური ჩიყვი აღმაოჩნდა და პრედნიზოლონი დასჭირდა. სტრესი არ ახდენს დიდ გავლენას ამ ანალიზებზე. ანტი-ტპო აუღეთ აუცილებლად და ტსჰ.

Posted by: an apple 23 Feb 2012, 23:41
Natuka NGN
მადლობა ნატუკა. მასაც ასეთი ქავილი ქონდა? ბავშვი არის საშინლად გატანჯული , სახე აქვს დასიებული ყოველ დილით. გარეგნულად არ აღენიშნება არც წონის ცვლილება და არც თმის ცვენა-კანის გამოშრობა.

Posted by: Natuka NGN 23 Feb 2012, 23:52
an apple
არა რატომ თმის ცვენა ან კანის გამოშრობა? პირიქით, იმას რასაც გეუბნები ეს სიმპტომები არა ხასიატებს. პირიქით, კანი უფრო ნაზი და ტენიანია და არა მშრალი.
მაგრამ ღვიძლის პათოლოგიაც შესამოწმებელია როგორც მატილდამ დაგიწერა, უმიზეზოდ არაფერი არ ხდება.

Posted by: an apple 24 Feb 2012, 00:06
Natuka NGN
მსგავსი შეკითხვები დააყარეს და იმიტომ ვთქვი. ანუ ჩვენ გვჭირდება ენდოკრინოლოგი . ღვიძლის შემთხვევაში რაიმე სხვა კლინიკური ნიშანი ხომ არ უნდა იყოს კიდევ, ან სისხლის ანალიზზე თუ აისახება .

Posted by: buburatino 24 Feb 2012, 10:36
იქნებ იცოდეთ? ახლობელმა შემომჩივლა რომ ძილში გაღვიძებისას აფიქსირებს რომ ენა აქვს აი როგორ გითხრათ დუნის დაავადები ს დროს რომ არის ისეთ მდგომრეობაში და პირი აქვს ღია..რომ გაიღვიძებს მერე ჩვეულებრივ..სრულიად ჯანსაღი და გონიერი ადამიანია..და რა შეიძლება იყოს ამის მიზეზი? რაიმე ნერვიული ან ფსიქ.ან ორგანული დავადების დააწყისის ნიშანი ხომ არ არის? თუ სუნთქვა უჭირს და პირღია სუნთქვის დროს ხდება ენის მასეთი მდგომრეობა ძილისას...?? რას ფიქრობთ?

Posted by: Ssalome 24 Feb 2012, 10:45
გამარჯობა, ჩემს მეგობარს (32 წლის) ქონდა წნევის მოულედნელი ვარდნა 70 - 30 ზე, მისი ნორმალური წნევაა 90-60 ზე. იყო მისული დაზღვევის ოჯახის ექიმთან, გაუკეთეს სისხლის ანალიზი, გემოგლობინი აქვს ცოტა დაწეული (66). დაუნიშნეს B ჯგუფის ვიტამინები და ფერუმ ლეკის ტაბლეტები. წნევის ვარდნის მიზეზად დაასახელეს მუშაობის წესი (10 დან 5 მდე კომპიუტერთან ზის).

დრო და დრო ისევ უვარდება წნევა, ორსულად არ არის, სხვა ჩივილებიც არა აქვს. სურს სრული გამოკვლევის ჩატარება, მაგრამ არ იცის საიდან დაიწყოს და ვის მიმართოს, თუ დაგვაკვალიანებთ მადლობელი დაგრჩებით. smile.gif

Posted by: gio_9999 24 Feb 2012, 11:54
matilda123

QUOTE
არაფერი განგაშო არ გაქვს . 37,4 მდე ტემპერატურა ნორმად ითვლება.


კი მაგრამ რაიმე ინფექციის სიგნალი ხომ არ არის მეთქი მაგას ვშიშობ, მაგალითად აივ ინფექციის ან ვირუსული ჰეპატიტის, ან სხვა უბედურების, ზემოთხსენებულ ინფექციებს ახასიათებს ხანგრძლივი სუბფებრილიტეტი, თუმცა ჩემი შემთხვევა შეიძლება თუ არა სუბფებრილიტეტს მივაკუთნვნოთ ეგ არ ვიცი, (ტემპერატურა მოსვენებულ მდგომარეობაში და სახლის პირობებში ნორმალურია, გარეთ გასვლისას იწევს 37.2-მდე მაქსიმუმ, ისიც დღის მეორე ნახევარში) ?


Posted by: Natuka NGN 24 Feb 2012, 15:36
Ssalome
შეიძლება არასწორი დღსი რეჟიმცი იყოს მიზეზი, მაგრამ აუცილებელია ფარისებრის შემოწმება. სუ ამას ვაწვები, მარა ახლა ვხვდები თურმე ოპეარციამდე რა ცუდად ვიყავი sad.gif და ჩემი უამრავინპრობლემის მიზეზი სწორედ ეს ყოფილა. ახლა სრულაიდ სხვა თვალებიტ შვეხედე სამყაროს.

Posted by: Nanuka444 26 Feb 2012, 20:46
არ ვიცი, ვინმე მიპასუხებს თუ არა, მაგრამ მაინც ვიკითხავ.

1 თვის წინ დაახლოებით გავცივდი (ვირუსი იყო სავარაუდოდ), ლაქტაციის გამო არ მიმკურნალია სერიოზულად, 2-3 დღე ვსვი რაღაც სიროფი (კოვონია) და ცოტა ცე ვიტამინის ტაბლეტები დავლიე. სხვა არაფერი.
ძლიერი ხველება მქონდა კარგა ხანს, მერე გამიარა თითქოს, მერე ისევ თავიდან შემომიბრუნდა და ეხლა ისევ გამიარა.
მაგრამ მარჯვენა მხარეს, ფილტვებში რაღაც უსიამოვნო შეგრძნება დამრჩა, თითქოს ტკივილი არ არის, მაგრამ აი ძალიან დისკომფორტს ვგრძნობ. სუნთქვას მიშლის, ღრმად ჩასუნთქვა მიჭირს, მცირე დატვირთვაზეც კი სუნთქვა მიძნელდება.

ეს შეიძლება რამე სერიოზული იყოს, თუ უბრალოდ გაციების დროს ხდება ხოლმე?

არასდროს მქონია ფილტვებზე პრობლემა, ზოგადად ჯანმრთელ ადამიანად ვითვლები, უბრალოდ ეხლა შეიძლება დაქვეითებული მქონდეს იმუნიტეტი ლაქტაციის გამო.

Posted by: matilda123 27 Feb 2012, 00:18
Nanuka444
QUOTE
მაგრამ მარჯვენა მხარეს, ფილტვებში რაღაც უსიამოვნო შეგრძნება დამრჩა, თითქოს ტკივილი არ ალიან დისკომფორტს ვგრძნობ. სუნთქვას მიშლის, ღრმად ჩასუნთქვა მიჭირს, მცირე დატვირთვაზეც კი სუნთქვა მიძნელდება.

ეს შეიძლება რამე სერიოზული იყოს, თუ უბრალოდ გაციების დროს ხდება ხოლმე?

არასდროს მქონია ფილტვებზე პრობლემა, ზოგადად ჯანმრთელ ადამიანად ვითვლები, უბრალოდ ეხლა შეიძლება დაქვეითებული მქონდეს იმუნიტეტი ლაქტაციის გამო.

ექიმმა უნდა მოგისმინოს ფილტვებზე. გამოსარიცხია პნევმონია, პლევრიტი და ნევრიტი ან მიოზიტი. ეს 2 უკანასკნელი ხველების და სჰეციებისგან ჩნდება. მე ვვარაუდობ, რომ უფრო ეს 2 ბოლო გაქვს, მაგრამ აუცილებლად მიმართე ექიმს.

Posted by: Megi-meghan 27 Feb 2012, 12:46
თუ შეგიძლიათ მირჩიოთ კარგი ნევროპათოლოგი.წინასწარ მადლობა

Posted by: Ssalome 27 Feb 2012, 16:52
Natuka NGN
დიდი მადლობა გამოხმაურებისათვის, აუცილებლად გადაამოწმებს smile.gif

Posted by: saliha82 28 Feb 2012, 19:42
გამარჯობათ დაახლოებით ერტთი თვეა მტკივა ხან კისერი ხან თავი ,რისი ბრალი შეიძლება იყოს რა გამოკვლევები ჩავიტარო და საერთოდ ვის მივმართო?მადლობა წინასწარ

Posted by: Nanuka444 28 Feb 2012, 22:24
matilda123
დიდი მადლობა გამოხმაურებისთვის.
ვიყავი დღეს ექიმთან, ფილტვების ანთება გადამიტანია ფეხზე მდგომს.
ბოლომდე არ არის განკურნებული და დამინიშნეს ნემსები.

Posted by: monica8 28 Feb 2012, 23:41
გთხოვთ მიპასუხოთ ტემპერატურა მაქვს 35 3 ხაზი. გაკეთებული მაქვს ოპერაცია 7 წლის წინ. ამოღებული მაქვს საშვილოსნო და ყველაფერი. მქონდა მიომა გთხოვთ მთხრათ ვის მივმართო ან რა ანალიზები გავიკეტო.
* * *
ძალიან შემაწუხა დაბალმა ტემპერატურამ.

Posted by: matilda123 29 Feb 2012, 01:17
saliha82
QUOTE
გამარჯობათ დაახლოებით ერტთი თვეა მტკივა ხან კისერი ხან თავი ,რისი ბრალი შეიძლება იყოს რა გამოკვლევები ჩავიტარო და საერთოდ ვის მივმართო?მადლობა წინასწარ

ზუსტად სად გტკივა?
Nanuka444
QUOTE
ვიყავი დღეს ექიმთან, ფილტვების ანთება გადამიტანია ფეხზე მდგომს.
ბოლომდე არ არის განკურნებული და დამინიშნეს ნემსები.

გამოჯანმრთელება smile.gif
monica8
QUOTE
გთხოვთ მიპასუხოთ ტემპერატურა მაქვს 35 3 ხაზი. გაკეთებული მაქვს ოპერაცია 7 წლის წინ. ამოღებული მაქვს საშვილოსნო და ყველაფერი. მქონდა მიომა გთხოვთ მთხრათ ვის მივმართო ან რა ანალიზები გავიკეტო.
* * *
ძალიან შემაწუხა დაბალმა ტემპერატურამ.

რაში გამოიხატება თქვენი შეწუხება, რა ჩივილები გაქვთ? ზოგისთვის ეგ ნორმალური ტემპერატურაა , მე ვიცი ადამიანი ვისაც 35,5- 35,6 აქვს და არაფერი არ სჭირს.

Posted by: fikria 29 Feb 2012, 09:57
საშინელი უძილობა მაქვს, საძილე წამალიც კი არ მაძინებს და რა უნდა ვქნა? ვინმემ რამე მირჩიეთ, თორემ გამოვიფიტე უკვე ძაან!!!!!!:(

Posted by: saliha82 29 Feb 2012, 10:58
matilda123
თავი მტკივა ასე ვთქვათ კისრისა და თავის გამყოფ ადგილას(ზუსტად როგორ გითხრათ არ ვიცი) ხანდახან ვერ ვგებულობ თავი მტკივა თუ უფრო კისერი.მეხსიერებაც და ყურადღების კონცენტრაციის უნარიც საგრძნობლად გაუარესებული მაქვს შეიძლება რომ ამის ბრალი იყოს?

Posted by: irinka-ira 29 Feb 2012, 11:14
saliha82
მაგას მიხედვა უნდა. არ მინდა შეგაშინო მაგრამ აუცილებლად მიმარტე ექიმს, სისხლძარღვების ბრალი შეიძლება იყოს....

Posted by: tatuuu 29 Feb 2012, 19:55
matilda123
გამარჯობა
ვარ 15 კვირის ორსული, მესამე დღეა ვარ გაციებული, ყელი მტკიოდა პირველ დღეს, ეხლა ისე აღარ. სიცხე დღის განმავლობაში არ მაქვს, საღამოს მაქვს 37.1.
ვიყავი ტერაპევტთან და ფილტვები სუფთა მაქვს, ყელი მაქვს ცოტა შეწითლებული
ბევრ სითხეებს ვსვავ, ჩაი+ლიმონი+მალინა+თაფლი და ა.შ
ყელის ტკივილის არაფერი დამინიშნა და მაინტერესებს საშიში ხომ არაა? ანუ წამალს არაფერს არ ვიღებ რეალურად.
და კიდე ერთი რამ, რისიც ყველაზე მეტად მეშინია.
გუშინ რომ მითხრა ექიმმა ფილტვები სუფთააო, შეიძლება ზეგ რამე იყოს ფილტვებში? ყოველ დღე ხომ არ ვივლი ტერაპევტთან? user.gif

Posted by: monica8 29 Feb 2012, 23:46
შეწუხება იმაში გამოიხატება რომ სულ მცივა. ადრე არ მქონდა ასეთი დაბალი ტემპერატურა.

Posted by: matilda123 1 Mar 2012, 00:55
saliha82
ორთოპედს მიმართე. გამოსარიცხია კისრის მალების პათოლოგია, ცთომილი ნერვის და სამწვერა ნერვის გაღიზიანება.
monica8
ჰორმონების გამოკვლევა გაიკეთე სისხლში , ყურადღება მიაქციე ფარისებრ. ჯირკვლ. ჰორმონებს. სიცივის შეგრძნება შესაძლოა მაქედან მომდინარეობს.
tatuuu
თუ კლინიკურად ჩივილები არ გაგიუარესდება, საშიში არაფერი არაა. თორემ ისე თეორიულად კი არის შესაძლებელი დაავდება პროგრესირდეს და ფილტვ. ანთებაში გადაიზარდოს. მაგრამ შენ როგორც წერ გააგრძელე მასე, სვი ბევრი სითხე, თაფლიანი ჩაი მათ შორის, ყელი გამოივლე გვირილის ნაყენით და შეხედე როგორ იგრძნობ თავს მომდევნო Dღეებში.
მალე გამოჯანმრთელდი smile.gif

Posted by: bui89 1 Mar 2012, 10:59
ეხლა ძალაინ ნუბურ კითხვას დავსვამ, მარა მშრალი ხველის საწინააღმდეგოდ რომელ წამალს მირჩევდით სასურველია ტაბლეტებში biggrin.gif

Posted by: miranda14 1 Mar 2012, 22:09
მოგესალმებით smile.gif ე.ი ჩემს მეგობარს რამოდენიმე კვირაა უკვე რაც აწუხებდა თვალის ტკივილები, ოფტამოლოგმა გაუშვა მაგნიტო რეზონანსულ ტომოგრაფიაზე და იქ უტხრეს რომ კეფის ნაწილში აქვს სისხლძარღვების კონგლომეტარები, სასწრაფო ოპერაცია არ არისო მაგრამ რომ გასკდეს ჰემატომა განვითარდება ამიტომ ქირურგიული ჩარევა აუცილებელიაო sad.gif შეგიძლიათ თურქეთში გადაიყვანოთ და იქ ლაზერით დაუშალოთო. მაინტერესებს რამდენად საშიში ოპერაციაა? აქ ვერ კეთდება ლაზერით დაშლა?? (ჯერ ნევროლოგთან არ მისულა) რამე წამლები ვერ გაიწოვს? ან რამდენი დაუჯდება ოპერაცია? ან რომელ საავადმყოფოს მირჩევდით ოპერაცია რომ გაიკეთოს აქ? რა ინფორმაციაც გაქვთ მომაწოდეთ sad.gif დიდი მადობა წინასწარ

Posted by: AbyssWalker 2 Mar 2012, 12:52
სურდო მაქვს ერთი-ორი კვირაა. ინჰალაცია რო გავიკეთო მეშველება? ინჰალატორი სახლში მაქვს

Posted by: tatuuu 2 Mar 2012, 13:48
matilda123
შედარებით უკეთესად ვარ, სიცხე აღარ მაქვს, ნუ ეს სურდო ვერ მოვიშორე მაინც smile.gif
დიდი მადლობა

ისე რა მაინტერესებს, ეს დღეები სულ სახლში ვიყავი და 2 დღეში მიწევს გარეთ გასვლა და საშიში ხომ არ იქნება? ვგულისხმობ ანუ ხომ არ შემიბრუნდება ისევ გაციება? sad.gif უბრალოდ აუცილებლად ვარ წასასვლელი თორე ნამდვილად არ გავიდოდი ისე მეშინია ეხლა მითუმეტეს გაციების და ვირუსების

Posted by: Megi-meghan 2 Mar 2012, 16:24
თუ შეგიძლიათ მირჩიოთ ნამდვილად კარგი ნევროპათოლოგი.წინასწარ მადლობა

Posted by: AbyssWalker 2 Mar 2012, 16:43
matilda123
QUOTE
სურდო მაქვს ერთი-ორი კვირაა. ინჰალაცია რო გავიკეთო მეშველება? ინჰალატორი სახლში მაქვს

jump.gif jump.gif jump.gif jump.gif

Posted by: Janiashvili 2 Mar 2012, 17:31
ერთი შეკითხვა მაქვს, ადრე რეკლამა იყო რაღაცა ყოვლისშემძლე წამლის...

მოხუცი კაცი კითხულობდა რისგან შველოდა ეგ წამალი, მაგის სახელი ხომ არი იცით?
კიდე მაგ რეკლამაზე ბალახს ესხმებოდა წყალი როგორც მახსოვს(ვითომ წვიმა იყო)

Posted by: AbyssWalker 2 Mar 2012, 17:52
Janiashvili
სალხინოო?

Posted by: Janiashvili 2 Mar 2012, 18:23
AbyssWalker
შენ გაიხარე ებისვოლკერო

Posted by: Nanuka444 2 Mar 2012, 21:07
Janiashvili
QUOTE
მოხუცი კაცი

სოკოლოვი იყო ეგ "მოხუცი კაცი" biggrin.gif

სალხინოს რეკლამა იყო ეგ, კი.

Posted by: matilda123 2 Mar 2012, 22:38
bui89
QUOTE
ხლა ძალაინ ნუბურ კითხვას დავსვამ, მარა მშრალი ხველის საწინააღმდეგოდ რომელ წამალს მირჩევდით სასურველია ტაბლეტებში biggrin.gif

პარაკოდინი.
QUOTE
სურდო მაქვს ერთი-ორი კვირაა. ინჰალაცია რო გავიკეთო მეშველება? ინჰალატორი სახლში მაქვს

არ გაწყენს, მაგრამ სურდოს ცხვირის წვეთები და სპრეი უნდა.
QUOTE
სე რა მაინტერესებს, ეს დღეები სულ სახლში ვიყავი და 2 დღეში მიწევს გარეთ გასვლა და საშიში ხომ არ იქნება? ვგულისხმობ ანუ ხომ არ შემიბრუნდება ისევ გაციება? sad.gif უბრალოდ აუცილებლად ვარ წასასვლელი თორე ნამდვილად არ გავიდოდი ისე მეშინია ეხლა მითუმეტეს გაციების და ვირუსების

თუ თბილად იქნები არაფერი არ შეგიბრუნდება.
QUOTE
მოგესალმებით smile.gif ე.ი ჩემს მეგობარს რამოდენიმე კვირაა უკვე რაც აწუხებდა თვალის ტკივილები, ოფტამოლოგმა გაუშვა მაგნიტო რეზონანსულ ტომოგრაფიაზე და იქ უტხრეს რომ კეფის ნაწილში აქვს სისხლძარღვების კონგლომეტარები, სასწრაფო ოპერაცია არ არისო მაგრამ რომ გასკდეს ჰემატომა განვითარდება ამიტომ ქირურგიული ჩარევა აუცილებელიაო sad.gif შეგიძლიათ თურქეთში გადაიყვანოთ და იქ ლაზერით დაუშალოთო. მაინტერესებს რამდენად საშიში ოპერაციაა? აქ ვერ კეთდება ლაზერით დაშლა?? (ჯერ ნევროლოგთან არ მისულა) რამე წამლები ვერ გაიწოვს? ან რამდენი დაუჯდება ოპერაცია? ან რომელ საავადმყოფოს მირჩევდით ოპერაცია რომ გაიკეთოს აქ? რა ინფორმაციაც გაქვთ მომაწოდეთ sad.gif დიდი მადობა წინასწარ

მე ქირურგი არ ვარ, მაგრამ როცა საოპერაციოა, მითუმეტეს თავის სიხლძარღვები უნდა გაკეთდეს ოპერაცია, რათა შემდგომი გართულებები ავიცილოთ თავიდან.

Posted by: AbyssWalker 2 Mar 2012, 22:48
2kiss.gif

Posted by: taamo 8 Mar 2012, 21:45
გამარჯობა! 4-5 თვეა ხელის ერთმა თითმა ფორმა შეიცვალა, აშკარად დეფორმირებულია, და სხვა თითებთან შედარებით ცოტა უფრო მსხვილია, ხანდახან მოხრაც მიჭირს. ხომე ვერ მეტყოდით რისი ბრალია? ან რომელ ექიმს მივმართო?

Posted by: taamo 11 Mar 2012, 11:43
იქნებ ვინმემ გამცეთ პასუხი...............

Posted by: matilda123 12 Mar 2012, 00:27
taamo
შესაძლოა ართრიტი გაქვს რომელიმე ან ტოფუსია. ორთოპედს მიმართე.

Posted by: rusudani7 16 Mar 2012, 11:51
თითქმის ორი წელია მაქვს ალერგიული ხველა, პერიოდულად ხან გამიძლიერდება, ხან შემიმცირდება მივიღე ალერგიის, ანტიბიოტიკები და რა ვიცი მრავალი სხვადასხვა წამლები, ახლობელმა მირჩია ყოველ დილით მივიღო ორი კბილი ნიორი , 10 დღის შემდეგ ხველება შემიჩერდა , როგორც კი შევწყვეტ ნიორის მიღებას ( დაგაყლაპვას) რამდენიმე დღეში ხველება ისევ დამეწყება, მაინტერესებს რაიმე უკუჩვენება ხომ არ ააქვს ნიორს, ზოგმა მითხრა პროთრომბინს წევსო თქვენ ხომ არ მირჩევთ რამეს?

Posted by: tamuna_mpt 16 Mar 2012, 19:02
დილაობით მაქვს გაციების სიმპტომები . სურდო . ხველა, ცოტა ყელში დისკომფორტი. ამასთან ერთად თვალების ქავილი. ეს ყველაფერი მსუბუქ ფორმებშია. მაგრამ დღის მეორე ნახევრისკენ თავისით გადამდის. შეიძლება ეს ალერგია იყოს?

Posted by: matilda123 17 Mar 2012, 00:55
rusudani7
ხველას ბევრი რამ იწვევს. ნიორზე ვერაფერს გეტყვით, მარტო ის ვიცი რომ ფიტონციდებს შეიცავს, ანტისეპტიკია , აქვს ფუნგიციდური, ანთების საწინააღმდეგო. ანტიპროტოზოული და ანტივირუსული მოქმედება. მაგრამ ყველა ეს მოქმედება სუსტია და სამკურნალო თვისებები ბოლომდე გამოკვლეული არაა.
მიმართეთ ექიმს. გამოიკვლიეთ ფილტვები, ბრონქების მგრძნობელობა, გაიკეთეთ ასთმის ტესტირება, ალერგიის ტესტი, გამორიცხეთ რეფლუქს-ეზოფაგიტი და თუ 40-ს გადაცილებული ხართ , ავთვისებიანი პროცესიც გამოსარიცხია.
tamuna_mpt
QUOTE
დილაობით მაქვს გაციების სიმპტომები . სურდო . ხველა, ცოტა ყელში დისკომფორტი. ამასთან ერთად თვალების ქავილი. ეს ყველაფერი მსუბუქ ფორმებშია. მაგრამ დღის მეორე ნახევრისკენ თავისით გადამდის. შეიძლება ეს ალერგია იყოს?

არაა გამორიცხული. მიმართე ალერგოლოგს.

Posted by: unknown lady 17 Mar 2012, 15:57
საშინელი სურდო მაქვს
მუდმივად მაცემინებს, მეწვის ცხვირი. ამდენი მოხოცვისგან კანიც გამიწითლდა

მესმის გრიპია, სიცხეც მქონდა 37,3 ორი დღის წინ, მაგრამ რამე არ იცით, რომ დააჩქაროს სურდოს გაქრობა?


დავიღალე, ცხვირსახოცით მძინავს sad.gif((

Posted by: ia_Baggins 17 Mar 2012, 22:55
8 თვეა გაჭიმული 37 მაქვს სიცხე, კიდე რაღაც წვრილმანი ჩივილები მაქვს და ვინმე კარგი ექიმი მიმასწავლეთ რა... თან რო კარგ სპეციელისტებთანაც გამაგზავოს თუ საჭირო იქნება

დაზღვევაც მაქვს მაგრამ ისეთი ცუდი გამოცდილება მაქვს ბრმად უცხო ექიმთან მისვლის რო აღარ მინდა რაა sad.gif(


Posted by: Salome_L 24 Mar 2012, 18:27
გამარჯობა,

ჩემი მეუღლე ეს ბოლო პერიოდია იძახის ქუსლები მტკივაო, რისი ბრალი შეიძლება იყოს. ხერხემალი თუ სხვა რამ? ან შესაძლო მიზეზების ჩამონათვალი რომ მითხრათ ვინმემ??

მე ცოტა სხვაგვარი ტკივილი მქონდა, შეგრძნება რომ თითქოს ქუსლის ძვლები დაიფშვნებოდა. ნუ კალცის ნაკლებობას დავაბრალე და კალცი ვსვი თან ლაქტაციის პერიოდი მქონდა. მაგრამ აი მეუღლის შემთხვევაში მიზეზი სად ვეძებოთ?

Posted by: matilda123 25 Mar 2012, 00:02
ia_Baggins
QUOTE
8 თვეა გაჭიმული 37 მაქვს სიცხე, კიდე რაღაც წვრილმანი ჩივილები მაქვს და ვინმე კარგი ექიმი მიმასწავლეთ რა... თან რო კარგ სპეციელისტებთანაც გამაგზავოს თუ საჭირო იქნება

37,4 მდე ტემპერატურა სიცხედ არ ითვლება.
Salome_L
ორთოპედს მიმართეთ.

Posted by: Limes 28 Mar 2012, 14:26
ბევრმა ხველამ (არამშრალმა) შეიძლება პერიოდულად ნახველში სისიხლის არსებობა გამოიწვიოს? ოღონდ ძალიან ცოტა დოზით????

Posted by: matilda123 28 Mar 2012, 22:52
Limes
QUOTE
ბევრმა ხველამ (არამშრალმა) შეიძლება პერიოდულად ნახველში სისიხლის არსებობა გამოიწვიოს? ოღონდ ძალიან ცოტა დოზით????

ექიმს მიმართე, სისხლი ნახველში ყოველთვის პათოლოგიაა.

Posted by: January Kid 30 Mar 2012, 01:48
რა არის სათესლე ჯირკვლის ვარიკოზი? რა სიმპტომები აქვს? რა იწვევს? რას უქმნის საფრთხეს? რამდენად საშიშია?

Posted by: ___ATINATI___ 1 Apr 2012, 21:45
ვინმემიქნებ მირჩიოს რომელექიმსუნდამივმართო.უკვე რამდენჯერმე გამიმეორდა- გულისცემამიჩქარდება ძალიან (ისე რომ დათვლას ვერ ვასწრებ),მიჭირს სუნთქვა, სახე მიწითლდება( თითქოს ლაქები მაქვს მთელ სახეზე) და ისეთი განცდა მაქვს რომ სახეც და ყელიც მისივდება.რისი ბრალი შეიძლება იყოს.

Posted by: faverliki 1 Apr 2012, 21:53
QUOTE
ვინმემიქნებ მირჩიოს რომელექიმსუნდამივმართო.უკვე რამდენჯერმე გამიმეორდა- გულისცემამიჩქარდება ძალიან (ისე რომ დათვლას ვერ ვასწრებ),მიჭირს სუნთქვა, სახე მიწითლდება( თითქოს ლაქები მაქვს მთელ სახეზე) და ისეთი განცდა მაქვს რომ სახეც და ყელიც მისივდება.რისი ბრალი შეიძლება იყოს.

შსიებას დ ალაქებს ალერგიაც იწვევს მაგრამ რადგან გულიც აჩქარებულია მიმართეთ ანგიოლოგს შეიძლება რაიმე პრობლემა იყოს სისხლძარღვებზე

Posted by: ___ATINATI___ 2 Apr 2012, 16:40
QUOTE
შსიებას დ ალაქებს ალერგიაც იწვევს მაგრამ რადგან გულიც აჩქარებულია მიმართეთ ანგიოლოგს შეიძლება რაიმე პრობლემა იყოს სისხლძარღვებზე


დიდი მადლობაsmile.gif

Posted by: Glekhi 3 Apr 2012, 12:01
ბატონებო იქნებ დამეხმაროთ. ერთი თვეა მარჯვენა ხელი მაქვს დაჭიმული აწევა მიჭირს რაგაცეების . მარა იმდენად მცირე დისკომფორტი იყო რო არ ვაქცევდი ყურადღებას. ეხლა ერთი კვირაა დამეწყო დაბუჟება. ანუ დღეში ერთი ხუთჯერ მაინც მიბუჟდება. გავიკეთე ანალიზები სისხლის საერთო+კოალგრამა და ყველაფერი ნორმაშია. რას ვქნა იქნებ დამაკვალიანოთ.

Posted by: fall 7 Apr 2012, 16:55
ძალიან გთხოვთ იქნებ დამეხმაროთ. 4 წლის წინ მყავდა ჭიები, ვიმკურნალე გოგორის კურკით და თითქოს მიშველა. ბოლო 3 წელია დამეწყო ჭინჭრის ციება, ვიყავი ჯერ დერმატოლოგთან, მერე ვმკურნალობდი ალერგოლოგთან, მაგრამ მიზეზს ვერ იგებდნენ რა იწვევდა. ეხლა უკვე ქრონიკული ჭინჭრის ციება მაქვს. აღმოჩნდა, რომ ქრონიკული ურტიკარია გამოწვეულია ნაღვლის ბუშტში ან წვრილ ნაწლავებში ლამბლიების და ჭიების არსებობის გამო. მირჩიეთ ვის მივმართო, სად გავიკეთო ანალიზები და ასე შემდეგ. თქვენი ნებისმიერი რჩევა ჩემთვის ძალიან მნიშვნელოვანია.

Posted by: matilda123 8 Apr 2012, 01:25
Glekhi
ნერვი გექნებათ გაღიზიანებული ან დაჭიმული ან რაღაცით მოჭყლეტილი. ორთოპედს მიმართეთ .
fall
ნებისმიერ ოჯახის ექიმს შეუძლია გიმკურნალოს. მაგისი მკურნალობაა მეტრონიდაზოლი 500 მგ 5-7 დღის გან-ში.
ლამბლიაზის არსებობა დგინდება განავლის ანალიზით

Posted by: fall 8 Apr 2012, 12:34
matilda123
QUOTE
ნებისმიერ ოჯახის ექიმს შეუძლია გიმკურნალოს. მაგისი მკურნალობაა მეტრონიდაზოლი 500 მგ 5-7 დღის გან-ში.
ლამბლიაზის არსებობა დგინდება განავლის ანალიზით

დიდი მადლობა, იცოცხლეთ...
ანუ თქვენ გაგიგიათ მსგავსი რამ? ქრონიკული ჭინჭირს ციება შეიძლება ჭიებისგან იყოს გამოწვეული?
* * *
თუ შეგიძლიათ მირჩიეთ საქართველოში ყველაზე ძლიერი თერაპევტი, ძალიან მჭირდება... მადლობა წინასწარ

Posted by: Lollipop 8 Apr 2012, 18:40
თირკმლების პოლიკისტოზზე მაინტერესებს რამდენად საშიშია. ეხო გადავიღე უკვე. კისტები არის 1-2 სანტიმეტრი. ნუ დიდად არ დავუპანიკებივარ ეხოსკოპისტს მარა მაინც მაინტერესებს.

Posted by: matilda123 8 Apr 2012, 20:19
fall
QUOTE
ანუ თქვენ გაგიგიათ მსგავსი რამ? ქრონიკული ჭინჭირს ციება შეიძლება ჭიებისგან იყოს გამოწვეული?

გამიგია არა, სიმპტომია ერთერთი.
QUOTE
თუ შეგიძლიათ მირჩიეთ საქართველოში ყველაზე ძლიერი თერაპევტი, ძალიან მჭირდება... მადლობა წინასწარ

ვერაფერს გირჩევთ, არ ვიცი. მანდ იკითხეთ.
Lollipop
QUOTE
თირკმლების პოლიკისტოზზე მაინტერესებს რამდენად საშიშია. ეხო გადავიღე უკვე. კისტები არის 1-2 სანტიმეტრი. ნუ დიდად არ დავუპანიკებივარ ეხოსკოპისტს მარა მაინც მაინტერესებს.


პატარა ზომის კისტები ჩივილებს როგორც წესი არ იწვევს .

Posted by: Kate789 8 Apr 2012, 21:13
პოლიპებთან დაკავშირებით მაქვს შეკითხვა და ვის შეიძლება მივმართო???
ანუ რომელ თემაში დავსვა შეკითხვა.

Posted by: fall 9 Apr 2012, 10:28
matilda123
QUOTE
გამიგია არა, სიმპტომია ერთერთი.

QUOTE
ვერაფერს გირჩევთ, არ ვიცი. მანდ იკითხეთ

დიდი მადლობა, გაიხარეთ

Posted by: ქართლოსი 11 Apr 2012, 12:53
Summer_Rain
ვაკეში ტატიშვილის კლინკაშია ზონდის გადაყლაპვის გარეშე მკურნალობს კუჩზე გვარი თუ გავიხსენე დავწერ

Posted by: Solveig 15 Apr 2012, 22:41
ერთი კითხვა მაქვს. მოკლედ, უკვე მეორედ შევამჩნიე ასეთი რამ...

ხელის ცერი თითის სახსარში უცებ ძალიან მწვავე ტკივილი ვიგრძენი. მერე ამ მწვავე ტკივილმა გაიარა, მაგრამ ახლა მტკივა მოძრაობის და თითის დაჭერის დროს, გალურჯებულიცაა. შარშან მქონდა ერთხელ, სხვა სახსარში, ზუსტად ისეთივე სილურჯით. 2-3 დღეში მთლიანაგ გაქრა. რა შეიძლება, რომ იყოს? თითქოს მკვეთრი მოძრაობა არ გამიკეთებია და ვერ მივხვდი, რატომ შეიძლებოდა, მომხდარიყო,...

Posted by: matilda123 16 Apr 2012, 22:48
Solveig
QUOTE
ხელის ცერი თითის სახსარში უცებ ძალიან მწვავე ტკივილი ვიგრძენი. მერე ამ მწვავე ტკივილმა გაიარა, მაგრამ ახლა მტკივა მოძრაობის და თითის დაჭერის დროს, გალურჯებულიცაა. შარშან მქონდა ერთხელ, სხვა სახსარში, ზუსტად ისეთივე სილურჯით. 2-3 დღეში მთლიანაგ გაქრა. რა შეიძლება, რომ იყოს? თითქოს მკვეთრი მოძრაობა არ გამიკეთებია და ვერ მივხვდი, რატომ შეიძლებოდა, მომხდარიყო,...

მე ართრიტზე ვიფიქრებდი. მით უმეტეს რომ ქალებში პროცენტულად ხშირია. მიმართეთ ორთოპედს.

Posted by: Solveig 17 Apr 2012, 02:15
matilda123
გმადლობთ.

და, სილურჯე რისგან შეიძლება იყოს გმაოწვეული? მე უფრო სისხლძარღვზე ვიფიქრე, რაკი ისედაც მაქვს ეგ პრობლემა.

Posted by: SPC 17 Apr 2012, 18:30
გამარჯობათ,
ჩემი მეგობარი ამჟამად არის საზღვარგარეთ და ასეთი პრობლემა აქვს.
1 კვირაა ზუსტად დაეწყო კუჭის აშლილობა და ამასთანავე შენიშნა რომ კუჭში რომ გადიოდა თითქოს სტაფილოსფერი-ფორთოხლისფერი იყო , შემდეგ დღეებში კი თითოს უფრო მოყვითალო.

ასევე ამბობს რომ თითოს თვალები მაქვს ოდნავ ყვითელი.

გასაგებია რომ ჰეპატიტს აქვს მსგავსი სიმპტომები მაგრამ კიდევ რა შეიძლება იყოს? რადგან ეს აშლილობა ზუსტად მაშინ დაეწყო რაც საზღვარგარეთ წავიდა იქნებ კლიმატის და საჭმლის ცვლილებების გამოა? იქ 6 თვიანი კონტრაქტი აქვს და ჯერჯერობით ვერ მოდის მაგრამ ძალიან ნერვიულობს. თუ რამეა ჩამოვა ეხლავე ეს თუ ასე საშიშია. ანუ რა ქნას ეხლა? როდემდე დაელოდოს ან არ დაელოსო ამ სიმპტომების გავლას?

Posted by: matilda123 17 Apr 2012, 22:09
Solveig
QUOTE
და, სილურჯე რისგან შეიძლება იყოს გმაოწვეული? მე უფრო სისხლძარღვზე ვიფიქრე, რაკი ისედაც მაქვს ეგ პრობლემა.

მოკლედ, ეხლა გადავიკითხე რაღაცეები. არსებობს სხვადასხვა ვასკულიტები, რომლებსაც ახასიათებს მსგავსი სიპტომები.
და თუ ვასკულიტია CRP და BSG უნდა იყოს აწეული. ANA და ANCA შეიძლება აწეული იყოს, შეიძლება არც.
* * *
SPC
QUOTE
ჩემი მეგობარი ამჟამად არის საზღვარგარეთ და ასეთი პრობლემა აქვს.
1 კვირაა ზუსტად დაეწყო კუჭის აშლილობა და ამასთანავე შენიშნა რომ კუჭში რომ გადიოდა თითქოს სტაფილოსფერი-ფორთოხლისფერი იყო , შემდეგ დღეებში კი თითოს უფრო მოყვითალო.

ექიმს მიმართოს და განავალი მიკრობიოლოგიურად გამოიკვლიოს.

Posted by: SPC 17 Apr 2012, 23:10
matilda123
საქმე იმაშია რომ ისეთ ადგილასაა იქ ვერ მიმართავს, მაშინ აქ უნდა ჩამოვიდეს სასწრაფოდ. რომ დაიცადოს რამოდენიმე დღე და თუ არ გაუარა მერე ჩამოვიდეს? თუ სასწრაფოა?

Posted by: matilda123 18 Apr 2012, 00:37
SPC
QUOTE
საქმე იმაშია რომ ისეთ ადგილასაა იქ ვერ მიმართავს, მაშინ აქ უნდა ჩამოვიდეს სასწრაფოდ. რომ დაიცადოს რამოდენიმე დღე და თუ არ გაუარა მერე ჩამოვიდეს? თუ სასწრაფოა?


თვითონ გადაწყვიტოს, რა უფრო მნიშვნელოვანია.
ფაღარათი ათასი დაავადების სიმპტომი შეიძლება იყოს- დაწყებული უბრალო საკვების აუტანლობით და დამთავრებული მძიმე ინფექც. დაავადებით. თუ სუბიექტურად თავს ძალიან ცუდად არ გრძნობს, თუ განავალში სისხლი არ აქვს, თU წყალივით განავალი არ აქვს , თუ სიცხე , გულისრევა, მუცლის ტკივილებიდა ა.შ. არ აქვს, დაიცადოს ერთი-ორი დღე.
არ დალიოს იქაური წყალი, მაღაზიის იყიდოს. არ ჭამოს იქაური საჭმელი, თვითონ მოამზადოს. თუ ჩივილები იგივე დარჩა. ექიმს უნდა მიმართოს.

Posted by: SPC 18 Apr 2012, 01:48
matilda123
უფრო ფერი რომ იყო მოსტაფლიოსფრო როცა ტუალეტსი მიდიოდა მაგიტომ შეშინდა


დიდი მადლობა

Posted by: Solveig 18 Apr 2012, 02:47
matilda123
დიდი მადლობა.

ან ტისხეულებზე რა შემიძლია, რომ ვთქვა: სისხლძარღვების პრობლემა დიდი ხანია, რაც მაქვს (ლამის 15 წლის ასაკიდან, ახლა 32 წლისა ვარ), მაგრამ რამდენიმეჯერ სხვადასხვა დროს საქართველოსა და გერმანიაში ჩატარებულმა გამოკვლევებმა რევმატული დაავადება არ აჩვენა, მხოლოდ ქვედა კიდურების ვენების ვარიკოზის დიაგნოზი დამისვეს (ეს უკვე გერმანიაში).

თითს რაც შეეხება, დღეს უკვე საერთოდ აღარ მაწუხებს.

Posted by: xatia2525 20 Apr 2012, 11:27
გამარჯობა, მტკივა მენჯის ძვალი,მუხლშიც ჩადის ტკივილი. იქნებ მითხრათ რის ექიმთან უნდა მივიდე? რენდგენი უნდა გადავიღო ? ცხადია ისე გარედან შეხედვით ვერ მიხვდება. ვარ 30 წლის
* * *
სად არის ყველაზე კარგი ექიმი, რომელ საავადმყოფოში?

Posted by: matilda123 20 Apr 2012, 23:57
xatia2525
QUOTE
გამარჯობა, მტკივა მენჯის ძვალი,მუხლშიც ჩადის ტკივილი. იქნებ მითხრათ რის ექიმთან უნდა მივიდე? რენდგენი უნდა გადავიღო ? ცხადია ისე გარედან შეხედვით ვერ მიხვდება. ვარ 30 წლის

ორთოპედთან მიდი , თუნდაც ყოფილი ტრავმატოლოგიის ინსტიტუტში.

Posted by: xatia2525 21 Apr 2012, 10:24
გმადლობთ და ამაზე რენდეგნი კეტდება თუ ანალიზი თუ რა?

Posted by: lazy 21 Apr 2012, 12:56
გამარჯობათ,მაინტერესებს ნეიროცირკულატორული დისტონია რამდენად საშიში დაავადებაა და მისი თვითგანკურნება თუ არის შესაძლებელი,თავისით რომ გაიაროს ამ დაავადებამ, თუ რაიმე საერიოზულ მედიკამენტურ მკურნალობას საჭიოებს? და კიდევ, ამ დაავადებამ ხომ არ შეიძლება რამე ორგანო მაგალითად გული ან კუჭი დააზიანოს???

Posted by: SPC 21 Apr 2012, 22:57
ერთი კითხვა,,,
ათასში ერთხელ (2 თვეში ერთხელ ალბათ), 2 წუთის განმავლოვობაში მტკივდება ორივე მხარეს რაღაც, აი დაახლოებით თირკმელები სადაცაა ოღონდ წინა მხრიდან. ვიცი დბეილურად ვხსნი მაგრამ ასეა დაახლოებით, საკმაოდ ძლიერად მტკივა ხოლმე ეს 2 წუთი, გულის რევის შეგრძნებამდე. მაგრამ როგორც ვთქვი 2 თვეში ერტXელ თუ ხდება, წელიწადში სულ 4-5 ჯერ ალბათ მეტი არა. მაქვს ძალიან დიდი ხანია და რა შეიძLება იყოს? ღირს ყურადღების მიქცევა?

Posted by: lucia 25 Apr 2012, 22:38
რაღაც კითხვა მაქვს და აქ ხომ არ პოსტავს ვინმე ნევროლოგი/ნევროპათოლოგი, რომ მივწერო? მადლობა წინასწარ

ან იქნებ სპეციალისტი მირჩიოთ კარგი?

Posted by: Damageplan 26 Apr 2012, 00:07
დაახლოებით 1 თვის წინ მქონდა ძლიერი ვირუსი და ბრონხიტი. ფილტვების ანთება რო არ წასულიყო ანტიბიოტიკებსაც ვიღებდი.
ეხლა ისევ გავცივდი და ფილტვების მოსმენაზე (არც თუ ისე სანდო) ექიმმა მითხრა, რო ძლიერი ბრონხიტი მაქვს.

ხან ძალიან მახველებს, ხან ძალიან მიმშვიდდება და ხან საერთოდ არ მახველებს.
გარეთ რო გავდივარ იმ დროს ამიტყდება ხოლმე ხველა...
ფილტვების ანთების საშიშროება კიდევ მაქვს? და მომენტალურად ხველა როგორ შევიჩერო ხოლმე sad.gif არა სასურველ მომენტებში
ან რა წამალი დავლიო რო ბრონხიტიც მომირჩინოს, ფილტვების ანთებაც არ დამემართოს და ხველაც გამიქროს

Posted by: Nikitavar 26 Apr 2012, 14:49
გამარჯობა,

მაინტერესებს რა დაავადებაა, როდესაც გტკივა მუხლები დიდხანს ფეხგაუშლელად ჯდომის დროს? ამ ბოლო დროს საერთოდ გამიუარესდა მდგომარეობა და დიდხანსაც რომ არ ვიჯდე მაინც მტკივა. ასევე მაინტერესებს, თუ რის ექიმს მივმართო, ვინ კურნავს აღნიშნულ პრობლემას? გთხოვთ დამაკვალიანოთ, გმადლობთ წინასწარ.

Posted by: ბარბაროსი 26 Apr 2012, 17:26
გამარჯობათ
3 დღეა მაწუხებს დაბალი სიცხეები 37-37.5 მაქვს მცირედი სურდო რისი ბრალი შეიძლება იყოს
მაქვს ალერგიული რინიტი და ჰაიმორიტი და შეიძლება თუ არა ამისგან იყოს გამოწვეული სიცხე და რა ვქნა?

Posted by: LAZIKA-GEO 27 Apr 2012, 23:26
http://forum.ge/?f=43&showtopic=33764102
არადა აქვე ყოფილა მე კიდე სად აღარ ვეძებე .. რას ფიქრობთ ჩემს დაავადებაზე? chest.gif

Posted by: matilda123 27 Apr 2012, 23:34
Damageplan
QUOTE
დაახლოებით 1 თვის წინ მქონდა ძლიერი ვირუსი და ბრონხიტი. ფილტვების ანთება რო არ წასულიყო ანტიბიოტიკებსაც ვიღებდი.
ეხლა ისევ გავცივდი და ფილტვების მოსმენაზე (არც თუ ისე სანდო) ექიმმა მითხრა, რო ძლიერი ბრონხიტი მაქვს.

ვინ გითხრა, რომ ბრონხიტი გაქვს, მკურნალობაც დაანიშნინე. აქაური რჩევებით ანტიბიოტიკის მიღება არ შეიძლება.Nikitavar
QUOTE
მაინტერესებს რა დაავადებაა, როდესაც გტკივა მუხლები დიდხანს ფეხგაუშლელად ჯდომის დროს? ამ ბოლო დროს საერთოდ გამიუარესდა მდგომარეობა და დიდხანსაც რომ არ ვიჯდე მაინც მტკივა. ასევე მაინტერესებს, თუ რის ექიმს მივმართო, ვინ კურნავს აღნიშნულ პრობლემას? გთხოვთ დამაკვალიანოთ, გმადლობთ წინასწარ.

ორთოპედს მიმართე. შეიძლება მენისკი გაქვს დაზიანებული, შეიძლება ბეკერის ცისტა გაქვს და შეიძლება რამე სხვა. barbarian
QUOTE
მაქვს ალერგიული რინიტი და ჰაიმორიტი და შეიძლება თუ არა ამისგან იყოს გამოწვეული სიცხე და რა ვქნა?

კი, შეიძლება.

Posted by: COCOA 30 Apr 2012, 14:04
matilda123

გამარჯობა, 8 თვეა მაწუხებს ასეთი რამ: საჭმელს ვერ ვყლაპავ, თითქმის ყველაფერს ვიღებ უკან.როგორც მითხრეს ნევროზულია
ვიყავი ნევროპათოლოგთან, გამომიწერა უამრავი ფსიქოტროპული წამალი, მაგრამ არ მიშველა. როგორც ერთ-ერთმა ფორუმელმა მითხრა,მასაც ჭირდა მსგავსი რამ და მოურჩინეს წერტილოვანი თერაპიით,
თქვენ ხომ არ იცით რამე ამ დაავადების შესახებ ან ხომ არ იცით ვინმე ვისაც შეიძლება მივმართო
მადლობა

Posted by: Toxic:) 30 Apr 2012, 14:34
უკაცრავად, ერთი კითხვა მაქვს იქნებ დამეხმაროთ:
მამაჩემმს გაიკეთა მუცლის ღრუს კტ და ასეთი რამე დაუწერეს:

ასწვრივ კოლინჯზე ისახება მრავლობითი დივერტიკულები - ეს რას ნიშნავს?????

წინასწარ დიდი მადლობა!
* * *
უკაცრავად, ერთი კითხვა მაქვს იქნებ დამეხმაროთ:
მამაჩემმა გაიკეთა მუცლის ღრუს კტ და ასეთი რამე დაუწერეს:

ასწვრივ კოლინჯზე ისახება მრავლობითი დივერტიკულები - ეს რას ნიშნავს?????

წინასწარ დიდი მადლობა!

Posted by: matilda123 30 Apr 2012, 20:50
COCOA
QUOTE
გამარჯობა, 8 თვეა მაწუხებს ასეთი რამ: საჭმელს ვერ ვყლაპავ, თითქმის ყველაფერს ვიღებ უკან.როგორც მითხრეს ნევროზულია
ვიყავი ნევროპათოლოგთან, გამომიწერა უამრავი ფსიქოტროპული წამალი, მაგრამ არ მიშველა. როგორც ერთ-ერთმა ფორუმელმა მითხრა,მასაც ჭირდა მსგავსი რამ და მოურჩინეს წერტილოვანი თერაპიით,
თქვენ ხომ არ იცით რამე ამ დაავადების შესახებ ან ხომ არ იცით ვინმე ვისაც შეიძლება მივმართო
მადლობა

რადგან გითხრეს, ე.ი. ექიმტან იყავით და გამოკვლევები ჩატარებული გაქვთ. აკუპუნქტურით შესაძლებელია მკურნალობა რა თქმა უნდა, ვის მიმართო არ ვიცი. გაიკითხეთ მანდ. Toxic:)
QUOTE
ასწვრივ კოლინჯზე ისახება მრავლობითი დივერტიკულები - ეს რას ნიშნავს?????

დივერტიკულები/დივერტიკულოზი ნაწლავების კეთილთვისებიანი დაავადებაა( ძირითადად). თუ საჭიროდ ჩათვლის ქირურგი ოპერაციას გაუკეთებენ.

Posted by: lurdes 30 Apr 2012, 23:12
გამარჯობა
ძალიან გთხოვთ დამეხმაროთ
დედას აქვს მწვავე ანგინა, ჩირქოვანი
თვალით შესამჩნევია საკმაოდ ყურის ქვემოთ
ექიმმა დაუნიშნა ბაკტოქსი, იღებდა 6 დღე
ამასთან ერთად სავლებებს
ცოტა დაუცხრა მაგრამ არ განკურნებულა
განმეორებით ვიზიტზე დღეს დაუნიშნა აზიმაკი

ახლა მაინტერესებს, საშიში ხომ არა რის ეს პრეპარატი
რადგან ანოტაციაში წავიკითხეთ გვერდითი მოვლენების შესახებ
რას გვირჩევთ ამ წამალთან დაკავშირებით და საერთოდ, იქნებ დაგვაკვალიანოთ
წინასწარ დიდი მადლობა

Posted by: COCOA 1 May 2012, 11:56
matilda123
კი გამოკვლევები ჩატარებული მაქვს. და რას ნიშნავს აკუპუნქტურა??

Posted by: matilda123 1 May 2012, 21:49
COCOA
QUOTE
კი გამოკვლევები ჩატარებული მაქვს. და რას ნიშნავს აკუპუნქტურა??

http://en.wikipedia.org/wiki/Acupuncture

Posted by: ToyotaMR2 2 May 2012, 14:34
სამსახურში 9 საათს ვატარებ კომპთან და ეს ორი დღეა მარცხენა თეძოში ყრუდ მტკივა და მთელ ფეხზე გადის მერე და რისი ბრალია .მხოლოდ ჯდომისას ვგრძნობ რომ მტკივა და როცა ვწვები დასაძინებლად

Posted by: მანია 5 May 2012, 16:41
გამარჯობათ,სადღაც წავიკითხე ნევროზის სიმპტომია ესეთი წამოზიდვა და ხომ ვერ მეტრყვით რა არის ეს წამოზიდვა ??

Posted by: მანია 7 May 2012, 17:38
და კიდევ ერთი კითხვა..ვარ 20 წლის ბიჭი..მოკლედ მაქვს ასეთი შეგრძნება თითქოს კისერი და თავი მაქვს დამძიმებული,ხან კიდევ კისრიდან თავში რაღაც მაწვება და თითქოს თავი უნდა მომძვრეს..ამ დროს წნევას რომ ვიზომავ მაქვს 100/70-60 ზე ხან 110/70-60-ზე,პულსი 60-65 დარტყმა წუთში ხან 70.. იქნებ მითხრათ რისი ბრალი შეიძლება იყოს???

Posted by: nukri1959 10 May 2012, 06:42
გამარჯობათ. წინასწარ მაბლობას გიხდით ვინც შემეხმიანებით. ვარ მეზღვაური ამჯამად ვიმოფები გემზე 2 თვეში
კონტრაქტი მითავდება, სახლში დავბრუნდები და მერე მივხედავ ჯანმრთელობას მანამდე კი მინდა თქვენი დახმარება და რჩევა როგორ მოვიქცე.....
მარჯვენა მხარეს, ნეკნების ქვეშ მაქვს ყრუ ტკივილი, თითქოს რაღაცნაირად დამძიმებული, ხელის დაჭერით ცოტაოდენ
ტკივილს ვგრძნობ, ძალიან უმნიშლენოს, (ტკივილი არ მემატება). ცხელი სასმელის მიღებისას, (მაგალითად ჩაი)
მაქვს წვის შეგრძნება, თითქოს რაღაცა ჩაიღვარა შიგნით. ერთგვარი დისკომფორტის შეგრძნება მაქვს. ამ მდგომარეობაში
ვარ თითქმის ერთი თვე. ჩემი აზრით ნაღვლის ბუშტი უნდა იყოს.
6 თვის უკან, თოდუას ცენტრში გავიკეთე სამედიცინო გამოკვლევა, საყლაპავ მილში და კუჭში შეინიშნებოდა
მცირეოდენი ეროზია, როგორც მითხრეს ძალია უმნიშვნელო, ეხოსკოპიამ რაიმე დარღვევები არ აჩვენა, ასე მითხრეს
ყველაფერი წესრიგშიაო, (კენჭები არგაქვს... და ასე შემდეგ)
ველოდები თქვენგან რჩევა დარიგებებს. წინასწარ მადლობა ყურადღებისთვის

Posted by: snow white:))) 10 May 2012, 14:06
matilda123
QUOTE
დივერტიკულები/დივერტიკულოზი ნაწლავების კეთილთვისებიანი დაავადებაა( ძირითადად). თუ საჭიროდ ჩათვლის ქირურგი ოპერაციას გაუკეთებენ.

უკაცრავად და ამხელა სისულელეს რატომ წერთ????რა კეთილთვისებიანი სიმსივნე?რის სიმსივნე?პოლიპოზში თუ გერევა და კარგად გქონია მაშინ საქმე?აი რა არის დივერტილი და საერთოდ დივერტიკულოზი.ეს არის გამობერილი ნაწილი ნაწლავი კედელში!!!!!!!!!!!
Дивертикулез - заболевание, при котором образуются мешковидные выпячивания стенки толстой кишки - дивертикулы.
Дивертикулезом толстой кишки называют заболевание, при котором происходит образование единичных или множественных грыжеподобных выпячиваний слизистой оболочки (дивертикулов) наружу за пределы кишечной стенки. Дивертикулез толстой кишки встречается у 25% взрослого населения. У женщин заболевание встречается чаще, чем у мужчин: 55% и 45% соответственно.
თუ არ იცი საერთოდ ნუ პოსტავ და ნუ სეგყავს ადამიანი შეცდომაში.თან რჩევებს იძლევა. facepalm.gif :facepalm


COCOA
QUOTE
გამარჯობა, 8 თვეა მაწუხებს ასეთი რამ: საჭმელს ვერ ვყლაპავ, თითქმის ყველაფერს ვიღებ უკან.როგორც მითხრეს ნევროზულია
ვიყავი ნევროპათოლოგთან, გამომიწერა უამრავი ფსიქოტროპული წამალი, მაგრამ არ მიშველა. როგორც ერთ-ერთმა ფორუმელმა მითხრა,მასაც ჭირდა მსგავსი რამ და მოურჩინეს წერტილოვანი თერაპიით,
თქვენ ხომ არ იცით რამე ამ დაავადების შესახებ ან ხომ არ იცით ვინმე ვისაც შეიძლება მივმართო
მადლობა

შენ კიდე მატილდას კი ნუ უსმენ .ლარინგოსკოპია გჭირდება.რაიმე მექანიკური არ იყოს.რაც ხელს გიშლის ყლაპვაში.თუ ესეც გამოირიცხა ,მაშინ ფარისებრის ექოსკოპია გაიკეთე!

Posted by: Solveig 13 May 2012, 21:07
lurdes
QUOTE
თვალით შესამჩნევია საკმაოდ ყურის ქვემოთ

რა არის შესამჩნევი? ლიმფური კვანძია გადიდებული, თუ რა?

nukri1959
QUOTE
მარჯვენა მხარეს, ნეკნების ქვეშ მაქვს ყრუ ტკივილი, თითქოს რაღაცნაირად დამძიმებული, ხელის დაჭერით ცოტაოდენ
ტკივილს ვგრძნობ, ძალიან უმნიშლენოს, (ტკივილი არ მემატება). ცხელი სასმელის მიღებისას, (მაგალითად ჩაი)
მაქვს წვის შეგრძნება, თითქოს რაღაცა ჩაიღვარა შიგნით. ერთგვარი დისკომფორტის შეგრძნება მაქვს. ამ მდგომარეობაში
ვარ თითქმის ერთი თვე. ჩემი აზრით ნაღვლის ბუშტი უნდა იყოს.

ჩაის გარდა რაიმე მიზეზთან ვერ აკავშირებ? მაგალითად, როცა გშია, ან იღებ ცხიმიან, შემწვარ საკვებს, მდოგვს, ნიორს და ა. შ.? ლოკალიზაციის მიხედვით ნაღვლის ბუშტს გავს და ალბათ აჯობებს, რომ მუცლის ღრუს ექოსკოპია გაიკეთო, როცა ჩამოხვალ.

Posted by: matilda123 13 May 2012, 21:09
snow white:)))
QUOTE
უკაცრავად და ამხელა სისულელეს რატომ წერთ????რა კეთილთვისებიანი სიმსივნე?რის სიმსივნე?პოლიპოზში თუ გერევა და კარგად გქონია მაშინ საქმე?აი რა არის დივერტილი და საერთოდ დივერტიკულოზი.ეს არის გამობერილი ნაწილი ნაწლავი კედელში!!!!!!!!!!!
Дивертикулез - заболевание, при котором образуются мешковидные выпячивания стенки толстой кишки - дивертикулы.
Дивертикулезом толстой кишки называют заболевание, при котором происходит образование единичных или множественных грыжеподобных выпячиваний слизистой оболочки (дивертикулов) наружу за пределы кишечной стенки. Дивертикулез толстой кишки встречается у 25% взрослого населения. У женщин заболевание встречается чаще, чем у мужчин: 55% и 45% соответственно.
თუ არ იცი საერთოდ ნუ პოსტავ და ნუ სეგყავს ადამიანი შეცდომაში.თან რჩევებს იძლევა. facepalm.gif :facepalm

სისულელე თავად დაპოსტე , კითხვა იცი? სიმსივნე სადმე წერია ჩემს დაპოსტილში?
დაგუგლე , რომ გაიგო სხვაობა დაავადებასა და სიმსივნეს შორის და დააკოპირე მერე აქ. გეხერხება copy-paste ვხედავ უკვე.


QUOTE
შენ კიდე მატილდას კი ნუ უსმენ .ლარინგოსკოპია გჭირდება.რაიმე მექანიკური არ იყოს.რაც ხელს გიშლის ყლაპვაში.თუ ესეც გამოირიცხა ,მაშინ ფარისებრის ექოსკოპია გაიკეთე!

და სანამ Kომენტარს გააკეთებ ჩემს ნაწერზე, პოსტები დაკვირვებით წაიკითხე ქალბატონო ფიფქია, მათ შორის მაგ ადამიანისაც, სადაც წერს რომ გამოკვლევები უკვე ჩატარებული აქვს . წარმატებები weiter ბოღმის ფრქვევაში.


* * *
მანია
QUOTE
ა კიდევ ერთი კითხვა..ვარ 20 წლის ბიჭი..მოკლედ მაქვს ასეთი შეგრძნება თითქოს კისერი და თავი მაქვს დამძიმებული,ხან კიდევ კისრიდან თავში რაღაც მაწვება და თითქოს თავი უნდა მომძვრეს..ამ დროს წნევას რომ ვიზომავ მაქვს 100/70-60 ზე ხან 110/70-60-ზე,პულსი 60-65 დარტყმა წუთში ხან 70.. იქნებ მითხრათ რისი ბრალი შეიძლება იყოს???

წნევა ნორმალური გაქვს, პულსიც. წამოზიდვა რა არის არ ვიცი . თითქოს რომ გაბოყინებს ის ?
QUOTE
გამარჯობათ. წინასწარ მაბლობას გიხდით ვინც შემეხმიანებით. ვარ მეზღვაური ამჯამად ვიმოფები გემზე 2 თვეში
კონტრაქტი მითავდება, სახლში დავბრუნდები და მერე მივხედავ ჯანმრთელობას მანამდე კი მინდა თქვენი დახმარება და რჩევა როგორ მოვიქცე.....
მარჯვენა მხარეს, ნეკნების ქვეშ მაქვს ყრუ ტკივილი, თითქოს რაღაცნაირად დამძიმებული, ხელის დაჭერით ცოტაოდენ
ტკივილს ვგრძნობ, ძალიან უმნიშლენოს, (ტკივილი არ მემატება). ცხელი სასმელის მიღებისას, (მაგალითად ჩაი)
მაქვს წვის შეგრძნება, თითქოს რაღაცა ჩაიღვარა შიგნით. ერთგვარი დისკომფორტის შეგრძნება მაქვს. ამ მდგომარეობაში
ვარ თითქმის ერთი თვე. ჩემი აზრით ნაღვლის ბუშტი უნდა იყოს.
6 თვის უკან, თოდუას ცენტრში გავიკეთე სამედიცინო გამოკვლევა, საყლაპავ მილში და კუჭში შეინიშნებოდა
მცირეოდენი ეროზია, როგორც მითხრეს ძალია უმნიშვნელო, ეხოსკოპიამ რაიმე დარღვევები არ აჩვენა, ასე მითხრეს
ყველაფერი წესრიგშიაო, (კენჭები არგაქვს... და ასე შემდეგ)
ველოდები თქვენგან რჩევა დარიგებებს. წინასწარ მადლობა ყურადღებისთვის

მარჯვ. ნეკნქვეშა არეში იმყოფება ღვიძლი, ნაღვლის ბუშტი და მსხვ. ნაწლავის ნაწილი. მაგრამ ამასთანავე ტკივილები ი ამ არეში შესაძლოა პროეცირებული იყოს მარჯვ. თირკმლიდან, , კუჭიდან, 12-გოჯა ნაწლავიდან, პანკრეასის თავიდან.
რადგან საჭმლის მიღებისას დისკომფორტი გაქვს , მე უფრო კუჭს ვიფიქრებდი. გასტროსკოპია გაქვს ისევ გასაკეთებელი და გასტროენტეროლოგთან ვიზიტი გჭირდება. ექოსკოპია ნაკლებად ინფორმაციულია კუჭის, 12-გოჯას და საყლაპავის დაავადებებისას.

Posted by: kkatrin 14 May 2012, 15:48
გამარჯობათ , ყელი მტკივა და ცხვირში სისველე მაქ,რამე ისეთი წამალი მირჩიოეთ რომ აქ შეჩერდეს და არ გართულდეს , მეშინია ბავსვს არ გადაედოს.

Posted by: snow white:))) 15 May 2012, 12:24
matilda123
QUOTE
დივერტიკულები/დივერტიკულოზი ნაწლავების კეთილთვისებიანი დაავადებაა( ძირითადად). თუ საჭიროდ ჩათვლის ქირურგი ოპერაციას გაუკეთებენ.


ეს შენი პოსტი არ არის?კეთილთვისებიანი დივერტიკულთან როგორ შესაბამისობაში მოდის?ეს სიტყვა თავად რა მიზნით დაწერე? facepalm.gif კოპი პეისტი გინდა შენ რომ გუგლს მაინც დაუჯერო თუ მაგდენი ცოდნა ვერ მიიღე ,დივერტიკული რომ არ იცი რა არის?რა ცხოველია არ უნდა არიგო სიბრძნეები! facepalm.gif
QUOTE
დივერტიკულოზი ნაწლავების კეთილთვისებიანი დაავადებაა

QUOTE
დივერტიკულოზი ნაწლავების კეთილთვისებიანი დაავადებაა

QUOTE
დივერტიკულოზი ნაწლავების კეთილთვისებიანი დაავადებაა

biggrin.gif
* * *





QUOTE
რადგან გითხრეს, ე.ი. ექიმტან იყავით და გამოკვლევები ჩატარებული გაქვთ

ეს შენ დაასკვენი არა დედუქციის უზადო უნარით?პაციენტს არ უთქვამს რომ გამოკვლევა ჩაიტარა.
[quote]მათ შორის მაგ ადამიანისაც, სადაც წერს რომ გამოკვლევები უკვე ჩატარებული აქვს .

ვაა.მართლააა?თუ შენ დაასკვენი?
არსად არ აკონკრეტებს რომ ლარინგოსკოპია აქვს გაკეთებული,გამოკვლევას ეგება სისხლის საერთო ანალიზს ეძახის.

Posted by: Solveig 16 May 2012, 00:47
snow white:)))

შეეცადე, კორექტული იყო...

კამათი და შესწორება უფრო ზრდილი ფორმითაც შეიძლება. შენ კიდევ თავიდანვე შეურაცმყოფელი ფორმით ჩაერთე საუბარში.


Posted by: matilda123 19 May 2012, 00:27
snow white:)))
[quote]ს შენი პოსტი არ არის?კეთილთვისებიანი დივერტიკულთან როგორ შესაბამისობაში მოდის?ეს სიტყვა თავად რა მიზნით დაწერე? facepalm.gif კოპი პეისტი გინდა შენ რომ გუგლს მაინც დაუჯერო თუ მაგდენი ცოდნა ვერ მიიღე ,დივერტიკული რომ არ იცი რა არის?რა ცხოველია არ უნდა არიგო სიბრძნეები! facepalm.gif
QUOTE[/quote]
smile.gif))
[quote]ეს შენ დაასკვენი არა დედუქციის უზადო უნარით?პაციენტს არ უთქვამს რომ გამოკვლევა ჩაიტარა.
[quote]მათ შორის მაგ ადამიანისაც, სადაც წერს რომ გამოკვლევები უკვე ჩატარებული აქვს .

ვაა.მართლააა?თუ შენ დაასკვენი?
არსად არ აკონკრეტებს რომ ლარინგოსკოპია აქვს გაკეთებული,გამოკვლევას ეგება სისხლის საერთო ანალიზს ეძახის.
[/quote]
ფსიქიატრი არ ვარ, მაგრამ ზოგად დიაგნოზს დაგისვამ smile.gif - რთული ბავშვობა გქონდა გეტყობა და არც ეხლა განებივრებს ცხოვრება დიდად. არასრულფასოვნების კომპლექსი გაქვს, ვერ რეალიზდები როგორც პროფესიონალი და როგორც პიროვნება და გაბოროტდი. Eძებ შანსებს, რომ შენი ცოდნა და უპირატესობა დაამტკიცო smile.gif)) მაგრამ ჩემთან არ გინდა ფიფქია, მე და შენ სხვადასხვა ენაზე ვლაპარაკობთ smile.gif))
პ.ს. მთლად შენი ცოდნის და დედუქციის შესაბამისი პასუხი ვერ გაგეცი, უკაცრავად. მაგრამ შემდეგში გამოვასწორებ
yes.gif

Posted by: JPEG 19 May 2012, 20:09
კუწის ექიმი რომელია? ანუ დასახელება smile.gif
მტკივა ხოლმე 22 წლის ვარ და სერიოზულს არაფერს ვფიქრობ ცუდა კვებას ვაბრალებ
როგორ ხდება გამოკვლევა კუჭის კიდე შლანგი და მაგის ჯანია?

Posted by: Solveig 19 May 2012, 22:20
JPEG
გასტროენტეროლოგთან უნდა მიხვიდე.


Posted by: taamo 20 May 2012, 21:06
გამარჯობა, ბოლო ერთი თვის განმავლობაში უკვე მეორედ დამემართა ასე: თითქოს ცხვირში სასუნთქი გზები დამიხშეს და ჰაერი აღარ მყოფნიდა, მეგონა რო ვიგუდებოდი. მე ნერვებს დავაბრალე უფრო. რის ბრალი შეიძლება იყოს ან ვის მივმართო ხომ ვერ მეტყვით?

Posted by: lizi11 25 May 2012, 18:46
დედას ნერვოპათოლოგმა გაუკეთა მუხლის მყესებში 'ჩინური მეთოდით' 6 წერტილში თითო ნემსი. (ლიდაზა+ნოვიკაინი+ლიდოკაინი+დექსამეტაზონი). ნანემსრები გამაგრდა დაუწითლდა და ტკივა. ქირურგის რჩევით მიიღო ანტიბიოტიკი და ალერტეკი. გაიკეთა გადასხმა ვენაში "რინგერის ხსნარი+ც ვიტამინი+სუპრასტინი 1 ამპულა. ნანემსრებზე იდებდა სველ საფენებს+"ვიშნევსკი" და ახლა ისვამს "უზნაძე #2." მუხლის ქვევით შეშუპებული აქვს ფეხი და ნანემსრებიც ტკივა.
რა ვქნა??????

Posted by: pisopiso 31 May 2012, 19:45
ღვიძლის პრობლემებით რომელ ექიმთან უნდა მიხვიდე? დამაკვალიანეთ რა. კერძოდ ეხომ აჩვენა რაღაც ცვლილებები და სასწრაფოდ მინდა ექიმთან მისვლა

Posted by: cin_namon 31 May 2012, 23:25
ვაიმე ვკვდები sad.gif
ფეხის კუნთზე ძარღვი გადამეფსკვა ზედიზედ 6ჯერ მაინც..ლოდევ რო განმეორდეს მოვკვდებიsad.gif
ჰარი არ მყოფნის...

რა უნდა გავაკეთო ასეთ დროს, ფეხი როგორ დავდო, ავდგე ჯობია თუ?
ეხლა გაუნძრევლად ვწევარ და მეშინია, კიდევ ერთს ვერ გადავიტან


Posted by: Solveig 1 Jun 2012, 09:35
pisopiso
გასტროენტეროლოგ/ჰეპატოლოგთან უნდა მიხვიდე.

Posted by: Lacrimosa_ 3 Jun 2012, 01:21
კისერში ერთი კვირაა მრგვალი რაღაც გამიჩნდა და ცოტათი მაწუხებს. ალბათ ჯირკვალია გაღიზიანებული ხომ? ცუდად დაემთხვა იღლიაში ეპილაციას და უფრო იმიტომ მეშინია.
რომელ ექიმთან უნდა მივიდე? და სანამ ექიმთან მივალ, მანამ რა შემიძლია გავაკეთო? ჩემს ოჯახის ექიმთან კარგა ხნით ადრე უნდა ჩაეწერო და არ მაქვს იმედი, რომ უახლოეს დღეებში მოვხვდები. თანაც სამსახურიდანაც მიჭირს გამოსვლა
* * *
Solveig

კისერში ერთი კვირაა მრგვალი რაღაც გამიჩნდა და ცოტათი მაწუხებს. ალბათ ჯირკვალია გაღიზიანებული ხომ? ცუდად დაემთხვა იღლიაში ეპილაციას და უფრო იმიტომ მეშინია.
რომელ ექიმთან უნდა მივიდე? და სანამ ექიმთან მივალ, მანამ რა შემიძლია გავაკეთო? ჩემს ოჯახის ექიმთან კარგა ხნით ადრე უნდა ჩაეწერო და არ მაქვს იმედი, რომ უახლოეს დღეებში მოვხვდები. თანაც სამსახურიდანაც მიჭირს გამოსვლა

შევიფუთე ყელი და დღეს ცოტათი დაპატარავებულია ზომაში.

Posted by: Marcha 7 Jun 2012, 16:35
მე-5 დღეა საღამოობით 38 მაქვს, ერთ დღეს არ მქონდა... დილას 36.4

პირველ დღეს დილის 6-ზე 38 მქონდა, რვის ნახევარზე 36,4. მეგონა თერმომეტრი მოიშალა

გუშინწინწინ სიცხე საერთოდ არ მქონდა, გუშინწინ, ვა შემაქციაო... გუშინ დილას ისევ აღარ მქონდა მარა საღამოს რომ ისევ ამიწია გავგიჟდი... დღეს დილას აღარ დამიწია, 37,4 მქონდა... ვიკადრე და ერთი ნიმესილი დავლიე, აქამდე არაფერი დამილევია, თან თავიც მტკიოდა და ორ რამეს ერთად უშველის მეთქი

ეხლა რაღაც ვერ ვარ, გავიზომე და 35.5 მაქვს, მეორედ გავიზომე ალბატ კარგად არ მედო მეთქი... მართლა მაგდენი მაქვს!

სხვადღეებში შუადღეს არ გამიზომია

რას უნდა მივაწეროთ სიცხის ასეთი ხტუნვა/თამაში?

შაბათს მანქანით სადღაც მივდიოდით და თავში კაი ხანს ცუდად მიბერავდა, მეტი სხვა მიზეზი არაფერი ვიცი

თავი მტკივა ამ ავადმყოფობასთან ერთად მაგრამ გაბმით და უსაშველოდ არა, უფრო გაბრუება, სიცხიანს რომ აქვს ხოლმე

ხო, ბოლოს სიცხე 10-იოდე წლის წინ მქონდა

Posted by: Lollipop 7 Jun 2012, 19:30
არ ვიცი აქ უნდა ვიკითხო თუ არა, მაღალ წნევაზე აბასთუმანში წასვლა ცუდია?

Posted by: ravi_shen 9 Jun 2012, 21:37
კეფა მტკვია ძაალიან აი რომ ვახველებ ისე მტკვია რომ მეჭედება რაღაც რავქნაა? :შ

ხოდა თავს რომ წინ წამოვწევ კეფა მტკვია და კიდევ სუნთქვის დროს გულ მკერდი მტკვია ისე რომ მირჩევნია არ ვისუნთქო

Posted by: lurdes 13 Jun 2012, 23:32
გთხოვთ, მირჩიეთ კარგი თერაპევტი

Posted by: Archer 15 Jun 2012, 00:04
ისე მგონი საქართველოში ნომენკლატურული ერთეული თერაპევტი უნდა იყოს ფრჩხილებს გარეთ და არა პირიქით wink.gif


lizi11
QUOTE
რა ვქნა??????

ჩინური "მაგიის" აღარ დაიჯერო

ToyotaMR2
QUOTE
სამსახურში 9 საათს ვატარებ კომპთან და ეს ორი დღეა მარცხენა თეძოში ყრუდ მტკივა და მთელ ფეხზე გადის მერე და რისი ბრალია .მხოლოდ ჯდომისას ვგრძნობ რომ მტკივა და როცა ვწვები დასაძინებლად

სავარაუდოდ ხერხემლის. აღარ შეუძლია ხერხემალს ამდენი ჯდომა. ცოტა გაინძერი, გამოინძერი და ნევროლოგთან მიდი smile.gif

ravi_shen
ყოველი შემთხვევისთვის წნევა გაიზომე, მაგრამ ისე ეგ ნევროლოგის საქმეს გავს. მიდი ნევროლოგთან

Posted by: MaDD 15 Jun 2012, 03:08
კეთილთვისებიანი სინოვიომა.. .. ჯერ ბიოფსია მერე ოპერაცია..

იქნებ ვინმემ ამიხსნათ რა მომივა სავარაუდოდ? დედაჩემი პედიატრია და როცა რას ამბობენ ექიმები ვიცი.. არც ისე კარგადაა საქმე ვხვდები მაგრამ რა შეიძლება საბოლოოდ გამოიწვიოს ამ ყველაფერმა?

ჯერ ბიოფსიამ ან როგორ კეთდება საერთოდ ბიოფსია?

Posted by: Archer 15 Jun 2012, 14:52
MaDD
ჯობია ონკოლოგის თემაში დასვა ეგ შეკითხვა

Posted by: taamo 15 Jun 2012, 19:08
გამარჯობა. მესამე დღეა ფეხები კოჭებთან დამისივდა და სიარულის დროს მტკივა, ვერ ვიგებ რისი ბრალი შეიძლება იყოს. ზუსტად 2 წლის წინაც ასე დამემართა და მაშინ იმ პერიოდს დაემთხვა, როცა ჩუტყვავილა მქონდა და იმას დაბრალდა ფეხების დასიება. ეხლა ხომ ვერ მეტყვით რას შეიძლებოდა გამოეწვია?

Posted by: lucia 18 Jun 2012, 23:17
გამარჯობა, მეგაცისტერნამაგნას შესახებ შეგიძლიათ დაწეროთ? როდისმე რთულდება, თუ სულ ერთ მდგომარეობაშია? მადლობა

Posted by: NewsAvatar 19 Jun 2012, 12:17
გამარჯობათ.

user posted image
უკვე რამოდენიმე თვეა შევამჩნიე ჭორფლებივით ტერფის იმ მიდამოებში სადაც შემოხაზული მაქ.. სხვებზე მინახავს ოღონდ მოხუცებულების ტერფზე, მე ჯერ არ ვარ მოხუცებული და ვერ მეტყვით რა უნდა იყოს??

Posted by: NewsAvatar 20 Jun 2012, 16:13
აქ არავინ ხართთთთთთ?!!!!!!!!!!!!!!!!!!

Posted by: kala 22 Jun 2012, 16:00
გმარჯობა დახმარება მჭირდება არ ვიცი თემა სწორედ შევარჩიე?
თუ აქ არ უნდა დამეპოსტა მაპატიეთ (არ დამვორნოთ)

მაინტერესებს ბრმანაწლავს რა სიმპტომები აქვს? ვიცი რომ ფეხის გაჩერებაა ერთ ერთი სიმპტომი...
წელის ტკივილიც იცის? რამდენიმე დღეა მარჯვნივ მტკივა რაღაც ვერ გავიგე ბრმანაწლავია, ნაღველი (თუ ქალური)
ფეხს ხან მიჭEრს ხან არა... ამიტომ ვერ გავიგე რა ხდევბა sad.gif
(ჯერ-ჯერობით რაღაც მიზეზების გამო ვერ ვახერხებ ექიმთან ვიზიტს და ამიტომ გადავწყვიტე მეკითხა აქ)

(უკაცრავად მაგრამ ჩვეულებრივზე ხშირად ვშარდავ) user.gif

Posted by: rock in rose 22 Jun 2012, 16:20
kala
QUOTE
(უკაცრავად მაგრამ ჩვეულებრივზე ხშირად ვშარდავ)


ცოტას და ხშირად შარდავ თუ ხშირად და ბევრს?

Posted by: kala 22 Jun 2012, 17:07
rock in rose
QUOTE

ცოტას და ხშირად შარდავ თუ ხშირად და ბევრს?

ბევრს ძირითადად... ბრმანაცლავია? sad.gif

Posted by: rock in rose 22 Jun 2012, 17:27
kala

შეიძლება თირკმელიც იყო

თან წელის არეშიც გქონია ტკივილები. როგორც კი მოახერხებ ექიმთან მიდი

ნუ გეშინია smile.gif 2kiss.gif

Posted by: kala 22 Jun 2012, 17:41
QUOTE (rock in rose @ 22 Jun 2012, 17:27 )
kala

შეიძლება თირკმელიც იყო

თან წელის არეშიც გქონია ტკივილები. როგორც კი მოახერხებ ექიმთან მიდი

ნუ გეშინია smile.gif 2kiss.gif

ჰო წელი უფრო მტკივა ვიდრე მუცელი
თან ეხლა ნოშპა დავლიე და ცოტა მომეშვა
გუშინ საერთოდ არ მტკიოდა

დიდი მადლობაა... yes.gif

Posted by: rock in rose 22 Jun 2012, 18:11
kala

რისი მადლობა smile.gif

....................

Posted by: kala 22 Jun 2012, 18:36
QUOTE (rock in rose @ 22 Jun 2012, 18:11 )
kala

რისი მადლობა smile.gif

....................

ცოტა დავმშვიდდი... yes.gif smile.gif

Posted by: Freestyle 23 Jun 2012, 11:47
ყურის ბიბილოში შიგნიდან ჯირკვალი მაქვს რაგაც.პატარა ბურთულა ზის,ერთი თვის წინ მქონდაცოტა გვერდით,შემდეგ ნელნელა გაქრა და ეხლა ეს გაიჩითა,რამე საშიში ხო არაა? თან ეს უფრო დიდია

Posted by: infuzoria 23 Jun 2012, 12:15
ნაღვლის ბუშტის ანთების დროს რა წამლის მიღება შეიძლება,ექიმის გარეშე?ანუ სანამ ექიმი მოვა რითი შეიძლება ტკოვილის გაყუჩება? ნოშპის მიღება შეიძლება ამ დროს?

Posted by: ravi_shen 25 Jun 2012, 18:35
ხალხო
მუცლის არეში ვგრძნობ დიდი ხანია ტკივილებს საკვერცხეებშიც დაბლა
წელიც მტკივა

შეიძელბა თუ არა რომ ეს ბრმანაწლავის სიმპტომები იყოს
აი რაღაც ყრუ ტკივილები მაქვს

ფეხი არ მიჩერდება და არც სიცხე არ მაქვს
რომ ვწვები და ფეხს რომ ვხრი არ მტკივა

თუ შეიძეკლბა უბრალოდ საკვერცხეები გამიცივდა

rock in rose იქნებ შენ იცოდე

Posted by: rock in rose 25 Jun 2012, 19:11
ravi_shen

კი, დიდი შანსია საკვერცხე იყოს. თუმცა ექოსკოპიის გაკეთება მოგიწევს, აპენდიციტის(ხალხურად ბრამანწლავის ანთება) ქრონიკული ფორმა რომ გამოირიცხოს.

თუმცა მაინც მგონია რომ სკვერცხეა.

Posted by: ravi_shen 25 Jun 2012, 20:33
rock in rose
დედაჩემსაც ეგრე გონია და მეც

დღეს გული წამივიდა კინაღამ ჩემ მეგობარს ჩავაკვდი ხელში

Posted by: rock in rose 25 Jun 2012, 22:07
ravi_shen

QUOTE
დღეს გული წამივიდა კინაღამ ჩემ მეგობარს ჩავაკვდი ხელში


ნერვიულობისგან?

ნუ გეშინია, მიხვალ ექიმთან და თვითონაც გეტყვის როგორ მარტივად მოგვარდება ეგ პრობლემა 2kiss.gif


Posted by: ravi_shen 25 Jun 2012, 22:18
rock in rose
არა ასე უბრალოდ ვიჯექი და უცბათ ცუდათ გავხდი რა დამემართა ვერ მივხცდი
ძალიან ცხელოდა და მე მგონი მაგის ბრალია

ახლაც მტკივა მცუელი მაქვს შებერილი ჯერ 10 დღე ვერ მივალ ექიმთან არ მცალია და იმედია ამ 10 დღეში არ გადაირევე :<

მადლობა გამხნევებისთვის :*

Posted by: Marguerite Gautier 26 Jun 2012, 20:37
ვაა მეც მუცლის არეში ოღონდ მარცხენა მხარეს წელისკენ ვგრძნობ რაღაცას მაგრამ ტკივილსაც ვერ ვარქმევ, არადა მაწუხებს აშკარად. საკვერცე მეგონა იმიტომ რომ მანდაც მქონდა ხოლმე ზოგჯერ რაღაც შეტევისმაგვარი, ექოსკოპია გადავიღე და ყველაფერი ნორმაში გაქვსო ორივე საკვერცხე კარგად მუშაობს და არაფერი პათოლოგია არ გაქვსო. ხოდა ეხლა ვეღარ გავიგე რა ჯანდაბაა, ხან საერთოდ ვერ ვგრძნობ მარა ხან მონოტონურად თითქოს შიგნიდან რაღაც დაფუთფუთებს. დღეს ცხენზე ვიჯექი და ოდნავ როგორც კი ამახტუნა სუნთქვა შემეკვრა თითქოს რაღაც გაიკვანძაო, სირბილის დროსაც მომსვლია მასე... და რა შეილება იყოს? რავიცი თუ მიხვდით მაინც რამეს sad.gif ექიმთან მეშინია მისვლა არ მითხრას ბრმა ნაწლავიაო.

Posted by: weasel 26 Jun 2012, 21:38
სალამი.ერთი კვირაა მახველებს.ლიზლი რომ არის ეგ მივიღე ტაბლეტიც,კაფსულაც მაგრამ შედეგი არ არის.
სიცხე არამაქვს.
ფილტვების ანთება ხომ არ მექნება? user.gif
რამე მირჩიეთ. weep.gif
* * *
არავინ არ ხართ ამ განყოფილებაში?? sad.gif((

Posted by: martosuli90 1 Jul 2012, 20:05
უბრალოდ დავამახსოვრებ ამ გვერდს და მერე წავიკიტხავ

Posted by: infuzoria 2 Jul 2012, 13:52
შიმშილს ,რომ ვერ ვიტან რისი ბრალი უნდა იყოს? თავბრუს ხვევა და გულის რევის შეგრძნებები მეწყება ოდნავ თუ შემაგვიანდება ჭამა,ისე რეჯიმში ვარ 3 ჯერ ერთი და იგივე დროს ვიკვებები,კვების რაციონი ნორმალური,არ ვსვავ ,არ ვეწევი-რისი ბრალი უნდა იყოს? rolleyes.gif

Posted by: japarasi 8 Jul 2012, 10:57
გამარჯობათ! ვის შეუძლია მიპასუხოს, მაქვს ბოყინი, გულის შეჩერების შეგრძნება. ვერ გავიგე გულია თუ კუჭი. მაქვს კუჭში სიმძიმის შეგრძნება. გმადლობტ

Posted by: Limes 8 Jul 2012, 13:48
ესეიგი გავცივდი და ცხვირიდან გამონადენთან ერთად სისიხლი მომდის, ოღონდ გამონადენი ხანდახან მწვანე ფერისაა, ასეთი არაფერი მინახავს და სერიოზულია რამე?

Posted by: Limes 10 Jul 2012, 09:19
ამ თემაში არავინ შემოდის???


...................................................



Posted by: მაჰათმა განდი 10 Jul 2012, 10:48
თითების ტკაცუნი რა შესაძლო ნეგატიურ შედეგების მომტანია ხელის სახსრებისთვის?



მადლობა პასუხისთვის.




პატივისცემით,
განდი.
არწივი.

Posted by: Lacrimosa_ 11 Jul 2012, 21:26
rock in rose

თქვენს გარდა აქ პასუხს არავინ გვცემს smile.gif

მეორე დღეა ხელზე რაღაც ადგილი მომექავება და თითქოს უზარმაზარმა რწყილმა მიკბინა, ისე მიწითლდება და იბურცება. ხელის დადებისას სიმხურვალე იგრძნობა. პირველ დღეს ერთი გამომივიდა ხელზე და 3 ფეხზე.
ექიმთან მინდოდა მისვლა, მაგრამ სამუშაოდან ვერ ვთავისუფლდები, თანაც მეგონა რაღაც მწერმა მიკბინა თქო.
მეორე დღეს სხვა ადგილი გამიწითლდა, ძველები კი შედარებით გაუფერულდა და აღარც მექავება.
დავლიე ალერტეკი ორივე დღეს, მივიღე ბორჯომი, გაწითლებულ ადგილებზე შევიზილე ნიორი და ერთზე ღამეც დავიტოვე.
რა შეიძლება იყოს?
ვიცი, თვითმკურნალობა არ შეიძლება, მაგრამ ვერაფრით მოვახერხებ შაბათამდე ექიმთან მისვლას (მით უფრო, რომ დაზღვევას ვადა გაუვიდა, იგივესთან აღარ ვაგრძელებთ და ოჯახის ექიმიც ცოტათი ნაწყენია სავარაუდოდ smile.gif )

Posted by: rock in rose 11 Jul 2012, 21:58
Lacrimosa_

თავისუფლად შეიძლება ალერგიული ხასიათის იყოს. თან ალერტეკის მიღების ფონზე გაუფერულებულა და ქავილიც აღარაა.

რახან ვერ ახერხებთ ექიმთან მისვლას გააგრძელეთ ალერტეკის მიღება, დღეში 1 აბი. ხო და დააკვირდით მდგომარეობას. ასევე მოერიდეთ ალერგიული საკვების მიღებას ცოტა ხნით, სანამ უკეთ გახდებით.

ისე კი სისხლის საერთო ანალიზის გაკეთება კარგი იქნებოდა.

Posted by: sallli1 11 Jul 2012, 23:25
ნაღვლის ბუშტი მაქვს ამოჭრილი და შეიძლება თუ არა ვარჯიში და რა დოოზით თანაც მუცლის პრესის

Posted by: Lacrimosa_ 12 Jul 2012, 17:16
rock in rose
დიდი მადლობა ყურადღებისათვის.

ძველები კი გაუფერულდა, მაგრამ ახლები გამომდის. ხელზე და ფეხზე. სხვა ადგილებში არა. ნაკბენი თუ არის რაიმესი, რა შეიძლება გამოვიყენო?

ექიმთან ორშაბათს ჩავეწერე, მანამდე თუ არ მოვკვდი ან მოვრჩი smile.gif

Posted by: rock in rose 12 Jul 2012, 17:34
Lacrimosa_

QUOTE
ძველები კი გაუფერულდა, მაგრამ ახლები გამომდის. ხელზე და ფეხზე. სხვა ადგილებში არა. ნაკბენი თუ არის რაიმესი, რა შეიძლება გამოვიყენო?


კანზე წასასმელად შეგიძლიათ გამოიყენოთ პსილო-ბალზამი. ალერტეკის სმა განაგრძეთ და მემგონი მოგვარდება პრობლემა.

სისხლის ანალიზი კი აუცილებელიცაა. კარგია ექიმთან რომ მიდიხართ. smile.gif


QUOTE
დიდი მადლობა ყურადღებისათვის.


რისი მადლობა smile.gif

Posted by: SpyZooka 13 Jul 2012, 20:38
ყველაზე კარგი ყელის ტკივილი წამალი რაარი? სტრეპტოციდი და სტრეპსილსი არ მშველის.

Posted by: damomoda 17 Jul 2012, 12:32
მაქვს ქრონიკული ქოლეცისტიტი. მარჯვენა ფერდქვეშა არე გამობერილი მაქ. მაქ სიმძიმის შეგრძნება. და რას იტყვით ძალიან საშიში ხო არაა.
კენჭები ულტრაბგერამ ვერ აღმოაჩინა.
ეხლა მედიკამენტოზური მკურნალობის თავი არ მაქვს ნამდვილად

Posted by: rock in rose 17 Jul 2012, 13:20
damomoda

QUOTE
მაქვს ქრონიკული ქოლეცისტიტი. მარჯვენა ფერდქვეშა არე გამობერილი მაქ. მაქ სიმძიმის შეგრძნება. და რას იტყვით ძალიან საშიში ხო არაა.
კენჭები ულტრაბგერამ ვერ აღმოაჩინა.
ეხლა მედიკამენტოზური მკურნალობის თავი არ მაქვს ნამდვილად


ექიმთან მაგ მიზეზით ბოლოს როდის იყავით?

დიეტა და მედიკამენტოზური მკურნალობა აუცილებელია

Posted by: damomoda 17 Jul 2012, 14:35
rock in rose
დაახლ. 3 თვის წინ ვიყავი

Posted by: ravi_shen 17 Jul 2012, 15:06
ყურიდან ნაცხი სად უნდა ავიღო_

ონიაშვილზე იყო სადღაც მგონი ხო?

Posted by: rock in rose 17 Jul 2012, 15:45
damomoda

მაშინ მკურნალობა არ დაგინიშნეს?

ravi_shen

არ ვიცი სამწუხაროდ

Posted by: ravi_shen 17 Jul 2012, 16:20
rock in rose
ხვალ გადავწყვიტე როგორღაც მივაღწიო ექიმებამდე და ანალიზების გაკეთებით დავიწყებ biggrin.gif

Posted by: rock in rose 17 Jul 2012, 17:08
damomoda

QUOTE
ხვალ გადავწყვიტე როგორღაც მივაღწიო ექიმებამდე და ანალიზების გაკეთებით დავიწყებ


საკვერცხის მიდამოში რომ გტკიოდა იმის საქმე როგორაა? ხომ ხარ კარგად biggrin.gif

Posted by: ravi_shen 17 Jul 2012, 18:17
rock in rose
არ ვყოფილვარ ექიმთან და ხვალ მვიდივარ

რისი ბრალია უნდა გავიგო ძAლიან მაქვს ტკვილები შეიძლება ღვიძლის ბრალიც იყოს

იმიტომ რომ ანთება გამორიცხულია ჩემი მდგომარეობის გამო givi.gif

მადლობა მოკითხვისთვის 2kiss.gif

Posted by: rock in rose 17 Jul 2012, 18:28
ravi_shen

ექიმის რას გეტყვის გამაგებინე რა, არანაკლებ მაინტერესებს biggrin.gif

QUOTE
მადლობა მოკითხვისთვის


რისი მადლობა 2kiss.gif

Posted by: ravi_shen 17 Jul 2012, 19:58
rock in rose
ჰოოო, მეც მაინტერესებს ძალიან იმედი მაქვს ყველაფერი კარგად იქნება თუ არადა ერთი ამგისიც biggrin.gif

* * *
rock in rose
იშიაზი მაქვს გაქანებული და გამოქანებული :<
ნერვი მაქვს გაცივებული :>

Posted by: damomoda 19 Jul 2012, 11:01
rock in rose
QUOTE
მაშინ მკურნალობა არ დაგინიშნეს?

საერთოდ ეს დაავადება უკვე 5 წელია მაქვს და რამე სერიუზული გართულება მოყვება?

Posted by: rock in rose 19 Jul 2012, 11:09
ravi_shen

საჯდომი ნერვის ანთება გითხრეს? ოჰო, მაგას კარგად უნდა უმკურნალო, არ შეგეშინდეს ოღონდ biggrin.gif

damomoda
QUOTE
საერთოდ ეს დაავადება უკვე 5 წელია მაქვს და რამე სერიუზული გართულება მოყვება?


დიეტა აუცილებლად დაგჭირდებათ

Posted by: ravi_shen 19 Jul 2012, 11:17
rock in rose
ნემსები გამომიწერეს რაღაცები მარა ვინ იკეთEბს biggrin.gif
ისე მეშინია ნემსის სამ მეტრზე რო ვხედავ ღნავილს ვიწყებ biggrin.gif


Posted by: rock in rose 19 Jul 2012, 11:35
ravi_shen
QUOTE
ნემსები გამომიწერეს რაღაცები მარა ვინ იკეთEბს
ისე მეშინია ნემსის სამ მეტრზე რო ვხედავ ღნავილს ვიწყებ


აბა რა უნდა ქნა? biggrin.gif

დასალევით ვერ შეგიცვლის?

Posted by: damomoda 19 Jul 2012, 11:46
rock in rose
QUOTE
დიეტა აუცილებლად დაგჭირდებათ

რამე ლინკი დამიდე რა სადაც დიეტის სია იქნება

Posted by: ravi_shen 19 Jul 2012, 12:07
rock in rose
არაფერი ჩემი ფეხები რა უნდა ვქნა
რო ვდგები წელი მიჩერდება biggrin.gif გამივლის თავისით ალბათ
არაო ტაბლეტებით ვერ შევცვლიო biggrin.gif
ხოდა ნუ შეცვლის რა ვუყო ერთი მაგისიც biggrin.gif

Posted by: rock in rose 19 Jul 2012, 12:12
damomoda

აი ესაა http://www.dieta.ru/glossary/holicdiet/?lq=21

სამწუხაროდ ქართულად ვერ ვიპოვე და რუსულად ვნახე


ravi_shen
QUOTE
არაფერი ჩემი ფეხები რა უნდა ვქნა
რო ვდგები წელი მიჩერდებაგამივლის თავისით ალბათ
არაო ტაბლეტებით ვერ შევცვლიო
ხოდა ნუ შეცვლის რა ვუყო ერთი მაგისიც


ურჩი პაციენტი ხარ რა biggrin.gif

Posted by: ravi_shen 19 Jul 2012, 15:35
rock in rose
სასწაული
პროსტა შანი არაა ნემსს არ გავიკეთებ biggrin.gif
რაღაც მალამო გამომიწერა და იმას წავისვამ მარტო :დ
მადლობა შნე რომ დაინტერესდი ჩემი ჯანმრთელობით
ნამდივლი ექიმი ხარ biggrin.gif

Posted by: rock in rose 19 Jul 2012, 16:42
ravi_shen

QUOTE
რაღაც მალამო გამომიწერა და იმას წავისვამ მარტო :დ


ნუ, თუ ასეთ სასტიკ უარს აცხადებ მაშინ ხო biggrin.gif

QUOTE
მადლობა შნე რომ დაინტერესდი ჩემი ჯანმრთელობით
ნამდივლი ექიმი ხარ


ვაიმე, დიდი მადლობა 2kiss.gif

Posted by: Legatus 21 Jul 2012, 12:40
გამარჯობა, იროდანის შემცვლელი ხომ არ იცით რამე წამალი?

Posted by: maverick16 21 Jul 2012, 16:03
თუ შეგიძლიათ ცინკის შემცველი პრეპარატი რო მითხრათ რამდენიმე ვარიანტი სასურველია ძვირი არ ღირდეს smile.gif

Posted by: MALI 22 Jul 2012, 02:44
ასეათი შეკითხვა მაქვს

მოზარდს საჭმლის მირთმევის შემდეგ ხშირად ეგერევე ეწყება კუჭის მოქმედება და დაახლოებით ფაღარათის სახით, არანაირი ტკივილი არც მუცლის არეში და არც სწორ ნაწლავში არ არენიშნება

რისი ბრალი შეიძლება იყოს?
კი ვაპირებს გასტროენტეროლოგთან მიყვნას მაგარმ საშიში ხომ არ არის? მაინტერესებს კომპეტენტური პირის აზრი

მადლობა

Posted by: ananonano 30 Jul 2012, 16:12
მარცხენა მხარეს მენჯის ძვალი რომ მთავრდება,ვგრძნობ ყრუ ტკივილს,რომ ვმოძრაობ ტკივილიც გადაინაცვლებს ხოლმე...რისი ბრალი შეიძლება იყოს?

Posted by: sallli1 30 Jul 2012, 23:18
იქნებ დამეხმართ? მესამეწელია თმა მცვივა ვარ 15 ის გადავწყვიტე მივიდე ექიმთან. ვისთან მივიდე არ ვიცი დეა რა უნდა გავიკეთ ალბათ ნლიზები მინდა? არ ბვიციმკელდ დამეხმარეთ რა რამე მირჩიეთ მალე გავმელოტდები sad.gif

Posted by: GeoWarr1or 31 Jul 2012, 09:43

გამარჯობათ გაიმორიტის ყველა სიმპტომი მაქვს დღეს ნიმესილი დავლიე და თითქმის ყველა სიმპტომი გააქრო გარდა ტკივილისა წარბებსშორის და თვითო წარბისა და შესაძლებელია რო ეს არ იყოს გაიმორიტი ჯერ ექიმთან არ ვყოფილვარ.
ქიმთან ვიყავი თერაპევთთან მაგრამ იქ სხვა სიმპტომებით ვიყავი და ის ვირუსი მოვკალი ანტიბიოტიკებით 2 მა ნემსმა უშველა მაგრამ დღეს დილით აღმოვაჩინე რო გაიმორიტის ნიშნები მაქვს,პირველად დამემართა მემგონი ცხოვრებაში (ბევრს ვყვინთავდი,ვცურავდი,წყალი "ვყლაპე" ცხვირით ბევრი,სიგარეტს ვეწეოდი,კარიესი მაქვს კბილზე,ვირუსი მქონდა რომელმაც ალბათ გამირთულა მოკლედ ყველაზე ხელსაყრელი პრიბობები იყო გაიმორიტისთვის)

Posted by: tamunak 31 Jul 2012, 12:11
სისხლის საერთო ანალიზი გავიკეთე და თრომბოკრიტის რაოდენობა 0.36 მაქვს, ნორმაა- 0.1-0.3...
რას ნიშნავს მეტყვით?

Posted by: an_only 9 Aug 2012, 05:45
არ ვიცი სად უნდა დავწერო და ამიტომ აქ ვწერ. მაქვს ასეთი შეგრძნება, თითქოს კისრიდან დაწყებული და მერე მიყვება კეფას და თავის ზედა ნაწილამდე მცივა და ეს ესაა ჟრუანტელი დამივლის, რაც არ ხდება, ვერც გავიგე ეს სიცივეა თუ გაბუჟება, ეს 2-3 დღეა ასე ვარ და რას შეიძლება ეს გამოეწვია? ძალიან ცუდი შეგრძნებაა sad.gifsad.gifsad.gif

Posted by: nit123 10 Aug 2012, 11:12
გამარჯობათ
3დღეა მაქვს სიცხე საღამოობით 37,2
Gვერდით თემაში მითხრეს რომ ერთერთი ანალიზი იმუნოგლობულინ M და G ზე უნდა გავიკეთო
და გამარკვიეთ ეს იგივე ჰერპესის ვირუსია თუ მონონუკლეოზის დასადგენი ანალიზია

Posted by: SpyZooka 11 Aug 2012, 16:59
საჭმლის ჭამის დროს წვენის მიყოლება საჭმელზე ასუქებს?

Posted by: მეგრელა 11 Aug 2012, 23:51
nit123
QUOTE
3დღეა მაქვს სიცხე საღამოობით 37,2

ეგრე ვარ ზუსტად 5-6 დღეა, და ძააან მოთენთილი

Posted by: larakroft 12 Aug 2012, 10:59
გამარჯობათ.. იცით რა მაინტერესეიბს.. მარჯვენა ხელზე, არათითზე, საქორწინო ბეჭდის ქვეშ დღეს დილით ამექავა და ცოტა ხანში სითხით სავსე 2 ცალი ბუშტუკი გამიჩნდა, ასეთივე ბუშტუკი გამიჩნდა ფეხზე წვივთან..ისიც მექავებოდა..
არადა ჩუტყვავილა გადატანილი მაქვს.. რა შეიძლება იყოს?
.. sad.gif

Posted by: Lord Alfred Douglas 16 Aug 2012, 14:13
ჰაიმორიტის დროს მოწევა შეიძლება?

Posted by: ravi_shen 16 Aug 2012, 22:48
არ ვიცი რამე მჭირს თუ ჩემი სქელი და გრძელი თმის ბრალია

ხშირად მიოფლიანდება და მექავება გაშლილი რომ მაქვს და რაიმე კანის დაავადება ხო არ მჭირს biggrin.gif

პ.ს. პარაზიტები არ მყავს biggrin.gif

Posted by: Damageplan 19 Aug 2012, 10:55
მომენტალურად თაბრუს ხვევა მეწყება, კონტროლს ვკარგავ და ძირს დაცემას ვიწყებ sad.gif
რისი ბრალია და რითი ვუშველო?
გარეთ თუ ვარ მითუმეტეს მაწუხებს

Posted by: shtori 22 Aug 2012, 21:32
ერთი კითხვა მაქვს და თუ ვინმე მიპასუხებს დიდად მადლობელი დაგრჩებით.

ტანზე, უთმო ადგილას შევნიშნე თთქმის გამჭირვალე, ძალზედ სუსტი, მაგრამ გრძელი ასე 2სმ-ის ბეწვი.
ინტერნეტში ვერ ვიპოვე პასუხი, რატომ ამოვიდა ეს ბეწვი? სხვა ფორუმებზეც წერდნენ მსგავს შემთხვევაზე. პასუხი არავის ქონდა. ვიღაც კი წერდა სიბერის ბრალიაო : )) თუმცა მე ჯერ ახალგაზრდა ვარ და... ასაკით... იქნებ ორგანიზმი დამიბერდა?..

რისი მანიშნებელია? დარღვევა მაქვს რამე? რის ექიმს უნდა მივმართო?


თუ წაიკითხავ უპასუხოდ არ დამტოვო, გთხოვ.






* * *
Damageplan
მე ექიმი კი არ ვარ, მაგრამ როგორც ვიცი დაბალი წნევის გამოც ხშირად ეხვევათ თავბრუ.
და ისე საშიშია ხშრირი თავბრუსხვევა. ექიმს გაესინჯო უნდა.

Posted by: ckupee 30 Aug 2012, 18:19

WBC leikociti 15.9 m. 4.4-10.0 109/l Sedegi norma
q. 4.4-10.0 blasti
promielociti
RBC eriTrociti 4.4 m. 4.5-5.9 1012/l mielociti
q. 4.0-5.0 metamielociti
HGB hemoglobini 130 m. 140-175 g/l CxirbirTviani neitrofili 4
0.7 0-5%
0-0.5x109l
q. 123-153 segmentbirTviani neitrofili 73
11.6 50-70%
2.2-7x109l
HCT hematokriti 39.2 m. 40-52 % eozinofili 1
0.1 1-4%
0.044-0.4x109l
q. 35-47 bazofili 0-1%
0-0.1 x109l
MCV eriTrocitis saSualo moculoba 87.8 m. 80-96 mkm3 prolimfociti
q. 87-94 limfociti 15
2.3 20-40%
0.88-4x109l
MCH hemoglobinis saSualo Semcveloba eriTrocitSi 29.1 m. 26-34 pkg/l monociti 7
1.1 2-8%
0.08-0.8x109l
q. 27-34 plazmuri ujredi 0-0.5%
0-0.05x109l
MCHC Hb-is saSualo koncentracia eriTrocitSi 331 m. 336-360 g/l SeniSvna
q. 325-351 aRiniSneba leikocitozi
absoluturi neitrofiloziT,
SefardebiTi limfopenia,
neitrofilebSi sustad gamoxatuli toqsogenuri marcvlovaneba.
RDV eriTrocitis ganawilebis farTi 17.9 m. 12-16 %
q. 12-14
PLT Trombociti 272 m. 180-350 109/l
q. 180-320
MPV Trombocitis saSualo moculoba 7.5 m. 7,3-9,0 mkm3
q. 8-10
ESR eriTrocitebis daleqvis siCqare
(vestergrinis meTodiT) 23 q. <50 w. <25 mm/sT mm/sT
>50 w. <30 mm/sT
m. <50 w. <15 mm/sT
>50 w. <20 mm/sT
RET retikulociti   2-15 ‰
იქნებ მითხრათ რამდენად საშიშია სიცოცხლისთვის? 2თვეა რაც ვიმშობიარე, საკვერცხეები ნორმაშიაო ექო გადავიღე და საეღთოდ მთლიანი მუცლის ღრუ დავითვალიერე...
და საერთოდ რა ხდება იქნებ ამიხსნად და რამდენად საშიშია სიცოცხლისთვის
მადლობთ

Posted by: tasikotasiko 31 Aug 2012, 12:50
ბზიკის ნაკბენის დროს ,როდესაც არანაირ შესიებას ადგილი არ ქონია ,აუცილებელია თუ არა დიეტის დაცვა? (საკვებიდან რა უნდ ამოვიღო და რამდენი ხანი? )

Posted by: tamunadea 4 Sep 2012, 15:54
გამარჯობა!!ზოლოფტის შესახებ მიდნა ისევ მოგწეროთ,,,12 წლის ბავშვზე ანუ მე ვერანბაირად ვერ ვუკავშირდები ექიმს,,,და ვერც მივდივარ გარკვეული მიზეზების გამო,sad.gif ახლა მოგწერთ როგორი თანმიმდევრობით ქონდა დანიშნული..დიაგნოზი ასეთი,ქცევითი აშლილობა შემოფარგლული ოჯახური კონფლიქტით,, ზოლოფტი ასეა დანიშნული,1/4 აბი დილით 3 დღე 1/2 დილით 6 დღე 1/2 დილით და 1/2 შუდღეს 1 თვე.. მერე 1 აბი დლით და 1/2 აბი შუადღეს 10 დღე 1 აბი დილით და 1 აბი შუადღით არ შეწყვიტოთო,, მაგრამ ბავშვი უკვე მშვენივრად გრძნობს თავს და 4 თვეა უკვე სვამს და მინდა რომ შეწყვიტოს,,არ გავუფრთხილებივარ რომ ამდენი ხანი უნდა დაელია ეს წამალი თორემ რამე სხვას გამოვაწერინებდი..დაიღალა ბავშვიც ამდენი წამლების სმით,, ხან გვავიწყდება და არ შეიძლება დავიწყებაო და არც შეწყვეტა თვითნებურადო და ძალიან გთხოვთ დამეხმაროთ და მომწეროთ რა თანმიმდევროვით მივცე ეს წამალი რომ დასასრულამდე მივიდეს? წინასწარ დიდი მადლობა

Posted by: ninucanini 6 Sep 2012, 23:25
გამარჯობათ, ვინმეს ხომ ა შეუძლია მითხრას სად შეიძლება მიკა წვერავას მიგნება? რომელ საავადმყოფოშIა? მადლობა წინასწარ.

Posted by: kusakadareli 7 Sep 2012, 10:37
მოგესალმებით. არ ვიცი სწორად ვწრ ამ თემაში თუ არა მაგრამ შესაბამისი თემა ვერ ვიპოვე.

როგორც დავასკვენი იმუნიტეტი მაქვს ძალიან დაქვეითებული (ფაქტიურად ჩემი ორგანიზმი უბრალო გაცივებასაც კი 10 დღეზე უფრო ადრე ვერ უმკლავდება). თუ შეგიძლიათ მირჩიოთ რა ვიტყამინები შეიძლება მივიღო ამ დროს, მაგ: კალცის დალევა რომ დავიწყო რამდენად გამართლებულია? ან სხვა რამე საშუალება თუ არებობს იმუნიტეტის გასაძლიერებლად.

წინასწარ მადლობა.

Posted by: rock in rose 7 Sep 2012, 13:37
ninucanini
QUOTE
გამარჯობათ, ვინმეს ხომ ა შეუძლია მითხრას სად შეიძლება მიკა წვერავას მიგნება? რომელ საავადმყოფოშIა? მადლობა წინასწარ.


ვაკეში, მე-9 საავადმყოფოში მუშაობს

Posted by: an_only 8 Sep 2012, 19:30
გამარჯობათ, არ ვიცი სად უნდა დავსვა ეს კითხვა და ისევ აქ მოვედი, მოვიწამლე 3 დღის წინ, არც გული ამერია არც კუჭის მოქმედება მქონია, ანუ ვერ გამოვიდა მოწამვლა ორგანიზმიდან, რის შედეგადაც დამეწყო ღვიძლის ტკივილი და დღემდე მტკივა, კუჭის მოქმედება გუშინ მქონდა ჩვეულებრივად, ვსვამდი რეგიდრონს და კარსილს, მაგრამ ღვიძლი მტკივდება ისევ სიარულის მერე sad.gif და კიდევ რაც ძალიან მაწუხებს, ორი ღამეა მეღვიძება მთლად სველს ისე რომ მეც ვერ ვიგებ რატომ არც სიცხე არაფერი და ვერ ვიგებ ამ ოფლიანობის მიზეზს და იქნებ მითხრათ, ძალიან ვნერვიულობ და თან ღვიძლი ისე მტკივდება დღის ბოლოს რომ ვერ ვლაპარაკობ და ვერც დავდივარ sad.gif((

Posted by: mainc 14 Sep 2012, 12:20
ჩვენთან flu shot-ებს აკეთებენ თუ იცით? ვაქცინის სახით, რომ მერე გრიპმა და ვირუსებმა ვეღარ დაგვამარცხონ?

ანუ ამგვარ რაიმეს ვგულისხმობ http://www.babycenter.com/404_is-it-safe-t...regnant_2488.bc

Posted by: freeandhappy 18 Sep 2012, 18:12
დიაგნოზი: ვეგეტო დისტონია

სიმპტომი: ყველაზე დისკომფორტული. მოხრჩობის შეგრძნება მაქვს (არ ვაზვიადებ) sad.gif გამივლის? cry.gif

ანალიზები ,ასევე ეხო, ჩიყვის გამოსარიცხად ჩავიტარე
ეხოკარდიოგრამაც, არაფერია.

Posted by: kusakadareli 23 Sep 2012, 18:31
გამარჯობათ. შეგიძლიათ მირჩიოთ კარგი ოტორინოლარინგოლოგი, რომელიც დიაგნოზს სწორად დამისვამს და მკურნალობას ჩამიტარებს. ბაქტერიოფაგში ხახის ანალიზი ჩავიტარე და რაღაც ჩხირები აღმომაჩნდა რამდენადაც მივხვდი. და არ ვიცი ვის შEიძლება მივმართო მკურნალობისთვის.

წინასწარ დიდი მადლობა.

Posted by: MARIPUSKU 26 Sep 2012, 11:51
გამარჯობათ. გთხოვთ მირჩიეთ კარგი თერაპევტი, რომელიც ბინაზე შეძლებს მოსვლას მწოლიარე ავადმყოფის გასასინჯათ. მადლობთ

Posted by: DOLPHIN 27 Sep 2012, 09:42
თბილისში პლაზმოფერეზი სად კეთდება ხარისხიანად?

Posted by: ToyotaMR2 27 Sep 2012, 13:36
QUOTE
გამარჯობათ. გთხოვთ მირჩიეთ კარგი თერაპევტი, რომელიც ბინაზე შეძლებს მოსვლას მწოლიარე ავადმყოფის გასასინჯათ. მადლობთ


ავადმყოფს რა აწუხებს ..სასწრაფო რომ გამოიძახო არაა?

Posted by: Calvados 27 Sep 2012, 20:49
აუუუ თუ იცით ხუთი თვის ორსულისთვის თუ შეიძლება კბილის დაპლომბვა ან ამოღება?და საფრთხეს ხომ არ შეუქმნის ნაყოფს ეს?sad.gif(((

Posted by: chupa82 11 Oct 2012, 14:55
არ ვიცი სად ვიკითხო, იქნებ დამეხმაროთ, ინსულტ გადატანის მამაკაცისთვის მინდა კარგი სპეციალისტი და რომელ ექიმთან მივიყვანო ალბათ ნერვოპათოლოგთან? ვის მირჩევდით , სრული გამოკვლევა რომ ჩაუტაროს თავის ტომოგრაფიით და ზედმეტად არ გაგვაწვალოს, რუსეთიდან უნდა ჩამოვიდეს სპეციალურად მაგისთვის

Posted by: MAST_WEB 11 Oct 2012, 15:41
გამარჯობათ, ვარ 23 წლის, ხშირად მიბუჟდება კეფის არე და ზოგჯერ მტკივა კიდეც. რისი ბრალი შეიძლება იყოს?
ერთი უცნაური და ცოტა სახალისო მომენტი შევამჩნიე, სიცილის შემდეგაც მემართება ეს smile.gif
იქნებ მითხრათ როგორ მოვიქცე, ვის მივმართო

Posted by: taamo 13 Oct 2012, 14:52
გამარჯობა. ართრიტზე ვმკურნალობდი რევმატოლოგთან, ფეხზე რომ დამხედა მითხრა ბრტყელტერფიანი ხარო. ზუსტად ხომ ვერ მეტყვით რომელ ექიმს ან სად შეიძლება მივმართო ნორმალური კონსულტაციისათვის?

Posted by: martateka 18 Oct 2012, 12:03
მაინტერესებს, რა იწვევს სისხლში ანტისტრეპტოლიზინის მომატებას, გლანდები რომ გამოვრიცხოთ? (ბავშვს თითქმის ერთი წლის წინ გავუკეთე გლანდებისა და პოლიპების ოერაცია). ნორმა 200-ია და აქვს 226, ძალიან მომატებული შეიძლება არ არის, მაგრამ მაინც. ბავშვი 6 წლისაა და თვალებს ხშირ-ხშირად ჭუთავს, ამ ბოლოს ცოტა აბრიალებსავით. თვალის ექიმთან მყავდა, მხედველობა წესრიგში აქვს. ექიმმა, უფრო მშრალი თვალის სინდრომი აქვსო და წვეთები დამინიშნა, რომელსაც შედეგი არ მოუტანია. ნევროპათოლოგთან ჯერ არ მყოლია. იქნებ მირჩიოთ რამე. წინასწარ გიხდით მადლობას smile.gif

Posted by: MacBook 18 Oct 2012, 14:09
სალამი,
ხალხო თამბაქოს გამო ფილტვებზე სად უნდა შემოწმდეს ადამიანი???

ძალიან გთხოვთ ვინც იცით მალევე გამომეხმაურეთ...

Posted by: natuki89 18 Oct 2012, 14:09
QUOTE (chupa82 @ 11 Oct 2012, 14:55 )
არ ვიცი სად ვიკითხო, იქნებ დამეხმაროთ, ინსულტ გადატანის მამაკაცისთვის მინდა კარგი სპეციალისტი და რომელ ექიმთან მივიყვანო ალბათ ნერვოპათოლოგთან? ვის მირჩევდით , სრული გამოკვლევა რომ ჩაუტაროს თავის ტომოგრაფიით და ზედმეტად არ გაგვაწვალოს, რუსეთიდან უნდა ჩამოვიდეს სპეციალურად მაგისთვის

მე გირჩევ ინგოროყვას კლინიკას მიმართო , მაგალითად ედიშერ მაღლაშვილი, საკმაოდ ძლიერი სპეციალისტია, თუნდაც მამამისი გიორგი მაღალაშვილი.

Posted by: BLACK METAL 19 Oct 2012, 13:15
გამარჯობათ. მაინტერესებს დაძაბული მუშაობის გრაფიკი როცა გაქვს და ძილი არ გყოფნის. დასვენების გარდა რაიმე წამალი თუ არსებობს? ან ვიტამინი, ან რამე რომ ჯანმრთელობა არ შეგერყეს. სტრესის მომხსნელი რამე საშუალება.

Posted by: rock in rose 19 Oct 2012, 17:40
BLACK METAL

ასე კონკრეტულად წამალი არ არსებობს, უბრალოდ კარგად უნდა იკვებოთ - სფრულფასოვნად, ხოლო თუ ვერ ახერხებთ, მაშინ პოლივიტამინური კომპლექსი მიიღეთ.

Posted by: keep_walking 19 Oct 2012, 18:21
ამ ბოლო დროს ჰაერი არ მყოფნის
სახლშიც და მანქანაშიც სულ ვაფრიალებ ფანჯრებს და ამინც ხრჩობის შეგრძნება მაქვს
რა ვქნა?

Posted by: Mad Dog Gambino 22 Oct 2012, 15:21
ბიძიებო და დეიდებო გადავრჩები თუ მოსაჭრელი მაქ ფეხი? smile.gif

http://www.radikal.ru

Posted by: ninia84 22 Oct 2012, 15:40
ჰემოგლობინი როცა 63 აქვს ზრდასრულ ადამიანს, რა შეიძლება რომ მიიღოს? ექიმთან ვერ ახერხებს მისვლას

Posted by: newsgirl 24 Oct 2012, 10:02
რამე ეფექტური ხველებისთვის... SOS. წინა კვირას გავცივდი, სიცხითა და სურდოთი. 2 დღეში უკეთ გავხდი და დავუბრუნდი ცხოვრებას და მერე მალევე ყელმა მომიჭირა. უფრო სწორად ბრონხები გამიცივდა როგორც ჩანს. დღეს და ხვალ ექიმთან ვერ მივალ, სამწუხაროდ. ზეგამდე არ მინდა ასე სიკვდილი biggrin.gif ხმა არ მაქვს საერთოდ. ამოხველება მიჭირს. ანგინა არ მაქვს. ხმის დაბრუნება მინდა sad.gif

Posted by: Calvados 29 Oct 2012, 18:22
ძალიან გთხოვთ, დამეხმარეთ...ასეთი რამ მაინტერესებს, რეზუსუარყოფითიანებისთვის არსებობს ნემსი ანტიიმუნოგლობულინი რომელიც კეთდება ორსულობის დროს და საკმაოდ ძვირიც ღირს, თუ იცით, არის რაიმე სახის ანალიზი, რომელიც დაადასტურებს ეს ნემსი ნამდვილად გაკეთდა თუ არა?

Posted by: jgnakvi 31 Oct 2012, 11:58
ძალიან გთხოვთ მიპასუხოთ, გაზაფხულის და შემოდგომის პერიოდებში ყოველთვის ვსვავდი ვიტამინებს, ტერავიტ ანტისტრესს, ვინაიდან ყოველთვის ძალიან ვიტვირთები გონებრივი მუშაობით და ვიფიტები ხოლმე. ამასთან პრობლემა მაქვს ცოტათი ზდმეტ წონასთან,მაგრამ ტერავიტ ანტისტრესი არ მასუქებდა საერთოდ, ახლაც ეს დღეებია ვიგრძენი რომ კრიზისი მეწყენა, საშინლად გამოფიტული ვარ, მაგრამ აღმოჩნდა რომ ტერავიტ ანტისტრესი აღარ იშოვება სააფთიაქო ქსელებში, ექიმის რჩევა მჭირდება რომელი ვიტამინით შეიძლება შევცვალო ეს ჩემი ტერავიტ ანტისტრესი? ანუ შემადგენლობიტ რომ ჰგავდეს და შედეგიც რომ მქონდეს (თან რო არ გამასუქოს)

ძალიან გთხოვთ უპასუხოდ არ დამტოვოთ რააა smile.gif)

Posted by: GABRIELA 18 Nov 2012, 23:43
3დღეა Mაქვს თავისტკივილები კეფის არეში
Dა საფეტქელთან,თვალები მეწვის და თითქოს რაღაც
Mაწვება ,ყელტან სპაზმივით მაქვს Dა გული მიკანკალებს ისე ,თითქოს
Aმოვარგებაო ყელთან .ცოტა შანერვიულო შEმხვდა და
Dამეწყო მასშEმდეგ. მარცხენა თვალიც მითAმაშებს.რა
შეიძLება მჭირდეს ? ძალიან მეშინია

Posted by: Natuka NGN 19 Nov 2012, 00:23
GABRIELA
წნევა გაისინჯე. რამდენი აგქვს გინახავს?

Posted by: MONTY 19 Nov 2012, 02:10
რომელიმე ძილის წამალი თუ არის 100% თ უსაფრთხო ? უძილობა არ მაწუხებს განსაკუთრებულად მაგრამ მაწუხებს შეცვლილი რეჟიმი და მგონი საუკეთესო მეთოდი ამ პრობლემის მოგვარების იქნება თუ კი ისეთ ძილის წამალს გამოვიყენებ რომელიც მიღებიდან მალევე დამაძინებს smile.gif
სხვაგვარად დროებითია პრობლემის მოგვარება , კომპიუტერის დამსახურებით მუდმივად მაწუხებს ეს პრობლემა , შესაბამისად მინდა ისეთი ძილის წამალი რომელსაც ყოველ დღე გამოვიყენებ 6 თვის განმავლობაში და არანაირ პრობლემას არ გამოიწვევს smile.gif

Posted by: GABRIELA 19 Nov 2012, 09:56
წნევა 117 მქონდა 80 ზე ,ჩემი წნევა 90 60ზე არის ისე sad.gif.

Posted by: Natuka NGN 19 Nov 2012, 16:12
GABRIELA
ჰოდამ აგწევია და მაგის ბრალია. დალიე რაიმე მსუბუქი დამამშVიდებელი, მაგალიტად ვალერიანი-ბარამბოთი, ქართველეი უშვებენ და ძლაინ ეფექტურია.

Posted by: GABRIELA 19 Nov 2012, 16:31
დიდი მადლობა დახმარებისთვის Natuka NGN

Posted by: d_tsabo 19 Nov 2012, 21:14
ლორისტა,ნონბლონი,აგრეგანი,მაქსიბილი,რეიტოლი ეს წამლები დაუნიშნეს დღეს და პოლიკნინიკაში მყავდა და დამატებითი გამოკვლევები სად ჩავუტარო თუ არის საჭIრო?

Posted by: MAST_WEB 26 Nov 2012, 11:52
თავისტკივილები მაწუხებს , მინდა რომელიმე კლინიკაში მისვლა რომ გამოვიკვლიო რა მჭირს. სად მირჩევთ?
მთლად რომ გამწეწონ და უარესად ამატკიონ თავი ეგეთი არ მინდა smile.gif))))

თუ აქ რეკლამირება გამოდის გთხოვთ PM-ში მითხრათ

Posted by: Just_smile 26 Nov 2012, 17:44
ბარიუმი ცუდი დასალევია? user.gif გული მერევახოლმე ზოგადად სითხე წამლებზე cry.gif

Posted by: MAST_WEB 26 Nov 2012, 18:08
QUOTE (MAST_WEB @ 26 Nov 2012, 11:52 )
თავისტკივილები მაწუხებს , მინდა რომელიმე კლინიკაში მისვლა რომ გამოვიკვლიო რა მჭირს. სად მირჩევთ?
მთლად რომ გამწეწონ და უარესად ამატკიონ თავი ეგეთი არ მინდა smile.gif))))

თუ აქ რეკლამირება გამოდის გთხოვთ PM-ში მითხრათ

???..................................................................

Posted by: rock in rose 26 Nov 2012, 18:35
Just_smile
QUOTE
ბარიუმი ცუდი დასალევია?  გული მერევახოლმე ზოგადად სითხე წამლებზე


პირიქით, ძალიან გემრიელია smile.gif

მართლა biggrin.gif

Posted by: Just_smile 26 Nov 2012, 20:20
rock in rose
QUOTE
პირიქით, ძალიან გემრიელია

არ იუ ქიდინგ მი? spy.gif მე რარაც რეჰიდრონივით მგონია და წინასწარ ვიხეთქავ გულს weep.gif weep.gif

Posted by: rock in rose 28 Nov 2012, 13:55
Just_smile

არა რა რეჰიდრონივით, ტკბილია და გემრიელი

Posted by: natiko22 28 Nov 2012, 23:28
გამარჯობათ. კითხვა მაქვს ასეთი- 1 კვირის წინ როგორც ჩანს გავცივდი,დამეწყო ხველება,იმ დღესვე დავიწყე ინჰალაციის გაკეთება ნებულაიზერით და პულმოლანის დალევა. 3 დღეში ამილაგდა ფაქტიურად ხველება. შიგადაშიგ მახველებს,მაგრამ ტემპერატურა დამრჩა-ყოველ საღამოს მაძლევს 37.2,37.1-ს. გუშინ გადავიღე რენდგენი,ფილტვები სუფთა მაქვს,თერაპევტთანაც ვიყავი და მოსმენითაც არაფერია.ბრონქიტიც კი არ არის. დღეს უკვე 37.3 მაქვს მთელი საღამო. არაფერი არ მაწუხებს,არაფერი არ მტკივა. რისი ბრალი შეიძლება იყოს ეს ტემპერატურა? გარეთ არ გავდივარ,უკვე აღარ ვიცი რა ვქნა. რომელ ექიმს მივმართო ან რა ანალიზი ჩავიტარო? რას შეიძლება გამოეწვია ტემპერატურა?

Posted by: kethrin-eka 29 Nov 2012, 16:15
გამარჯობათ, ბოლო ხანია ტუჩის კუთხეებს და მის მიმდებარე ადგილებზე გამიჩნდა ნახეთქები. მაქვს წვა და ხან ქავილის შეგრძნება. ინტერნეტით მოვიძიე ინფორმაცია და რაღაც ამდაგვარია ( ჰქვია ქეილიტი)

http://www.radikal.ru

იქნება ვინმემ იცით რით შეიძლება ვიმკურნალო? მაზი ან რამე სხვა...

Posted by: Just_smile 2 Dec 2012, 21:10
rock in rose
დავლიე user posted image


რა მაინტერესებს ღვიძლის ფუნქციას თუ რაღაც ეგეთს როგორ აკეთებენ? ანუ რა პროცედურაა? rolleyes.gif

Posted by: >>levani9<< 3 Dec 2012, 13:01
გამარჯობათ . ადრე 18 წლის რომ ვიყავი მაჯა ამოვიგდე. ეხლა რაღაც ძაან შემაწუხა ტკივილმა. ადრეც მტკოდა ხოლმე ცუდ ამინდებში მაგრამ ეხლა რაღაც ხშირად მტკივა
რას მირჩევთ? რამე გამოსავალი არის თუ ნაღრძობს არაფერი ეშველება და ასე უნდა ვითმინო ?

Posted by: rock in rose 3 Dec 2012, 14:12
Just_smile

სისხლის ანალიზის საშუალებით დგინდება ღვიძლის ფუნქცია, სხვადასხვა ტესტს ატარებენ. ცალკე ვირუსებზე და ცალკე ფერმენტებზე.

>>levani9<<

გამარჯობა, ანთებითი პროცესი იქნება და შესაბამისი მკურნალობა დაგჭირდება

Posted by: >>levani9<< 3 Dec 2012, 14:17
rock in rose
ვისთან მირჩევ მისვლას ? ოპთამოლოგთან მივიდე ? ეხლა 25 ვარ. და შეიძლება ანთება არის ესეიგი ხომ.

Posted by: rock in rose 3 Dec 2012, 14:30
>>levani9<<
QUOTE
ვისთან მირჩევ მისვლას ? ოპთამოლოგთან მივიდე ? ეხლა 25 ვარ. და შეიძლება ანთება არის ესეიგი ხომ.


ორთოპედთან მიდი, მე ანთება მგონია, გამწვავდა პროცესი და ალბათ ამიტომ. ზუსტად ვერ გეტყვი დიაგნოზს, ჯერ ექიმი არ გავმხადარვარ biggrin.gif user.gif

Posted by: >>levani9<< 3 Dec 2012, 14:59
rock in rose
QUOTE
ოპთამოლოგთან მივიდე ?

ეს მაგარი დამიწერია biggrin.gif
გაიხარე დიდი მადლობა smile.gif

Posted by: Just_smile 3 Dec 2012, 18:09
rock in rose
QUOTE
სისხლის ანალიზის საშუალებით დგინდება ღვიძლის ფუნქცია, სხვადასხვა ტესტს ატარებენ. ცალკე ვირუსებზე და ცალკე ფერმენტებზე.

user.gif

Posted by: rock in rose 3 Dec 2012, 18:31
>>levani9<<

არაფრის smile.gif

Just_smile

რა იყო ხო არ გეშინია? user.gif

Posted by: Just_smile 3 Dec 2012, 18:35
rock in rose
რავი user.gif თან არავის უთქვამს, უბრალოდ სახეზე ვირუსოვანი მეჭეჭები მქონდა, მოვიწვი და სანამ მოვიწვავდი რომ გავესინჯე მკითხეს ღვიძლის ფუნქცია გაკეთებული გაქვსო და არამეთქი, მერე კარგად რომ გამსინჯეს biggrin.gif აღარაფერი უთქვამთ, რომ ან გჭირდება ან რამე, უბრალოდ მომიწვეს და რაღაც წასასმელები დამინიშნეს, მაგრამ მე ჩამრჩა გულში user.gif და მინდა გავიკეთო user.gif

Posted by: rock in rose 3 Dec 2012, 19:07
Just_smile

იმისთვის რომ დამშვიდდე გაიკეთე user.gif

Posted by: nino877 4 Dec 2012, 13:51
ლუმბალური პუნქცია
ყველაგნ კეთდება? და ფასების საქმის კურსში ხომ არ ხართ ?
და რა ანალიზები კეთდება ლეიკემიის გამოსარიცხად?

Posted by: kirtsxeli 6 Dec 2012, 15:21
ამდენ წერას არ ჯობია ექიმს მიმართოთ?

Posted by: Natuka NGN 6 Dec 2012, 16:48
kirtsxeli
შენ პოსტებს აგროვებ ჯანმედში?!!!

Posted by: tatuuu 10 Dec 2012, 22:15
გამარჯობა
ხამდახან მიბუჟდება ხოლმე ხელები და ტუჩები, მერე ნელ ნელა მთლიან სახეზე გადადის დაბუჟება და იმის მაგივრად რომ გამიაროს უფრო მემატება... თან ამ დროს ვერ ვამოძრავებ ხელებს ნორმალურად
ძალიან იშვითად ხდება ესე, ხანდახან ნერვიულობის დროსაც
რისი ბრალი შეიძლება იყოს? ან საშიშია?

Posted by: melano-melano 10 Dec 2012, 23:17
დაბალ სიცხეებთან დაკავშირებით სად არის თემა?

Posted by: Just_smile 10 Dec 2012, 23:40
შარდის ბუშტის ავსებისას გულის რევის შეგრძნება მეწყება ამ ბოლო დროს შევამჩნიე, ქრონიკული გასტრიტი დამიდგინეს და შეიძლება მაგის ბრალი იყოს? user.gif mo.gif

Posted by: iN broWn 25 Dec 2012, 15:30
გამარჯობათ...
თიტები მაქვს მარცხენა ხელზე დამწვარი, ერთ კვირაზე მეტი გავიდა და არ მირჩება
მესამე ხარისხია...
სიცივეში ხშირად სეროზული სითხე მომდის, ვერ ვიხვევ მეკრობა მერე...

ხოდა არსებობს რამე ლეიკო დამწვრობისთვის?
ძაან მაწუხებს, ვიციი სახსაში როა დიდი დრო დაჭირდება
ხოდა რამე მინდა რო დავიფიქსირო
არი რამე ??//
პანთენოლს ვხმარობდი ტავიდან ახლა რაღაც სახლსი დამზადებულ მალამოს მისმევენ

Posted by: ბურსა 5 Jan 2013, 16:52
რა სიმპტომები აქვს ფილტვებში წყალის ჩადგომას?

Posted by: blokator 8 Jan 2013, 06:06
nino877
QUOTE
ლუმბალური პუნქცია ყველაგნ კეთდება? და ფასების საქმის კურსში ხომ არ ხართ ?და რა ანალიზები კეთდება ლეიკემიის გამოსარიცხად? 

მართალია გვიან გპასუხობ მაგრამ შემიძლია ნომერი მოგცე კლინიკის პირადში, თუ ისევ გჭირდება,სადაც დარეკავ და გაგირკვევენ ყველაფერს

Posted by: nino877 8 Jan 2013, 13:29
blokator
QUOTE
მართალია გვიან გპასუხობ მაგრამ შემიძლია ნომერი მოგცე კლინიკის პირადში, თუ ისევ გჭირდება,სადაც დარეკავ და გაგირკვევენ ყველაფერს

არ არის გვიანი 1 თებერვლიდან მიაქტიურდება ახალი დაზღვევა და მერე ვაპირებ მისვლას
მომეცი კი ნომერი
მადლობა

Posted by: gio_xabaza 13 Jan 2013, 10:11
ვარ 16წლის, მქონდა ჩუტყვავილა, (ნუ მაქ მარა უკვე მოხმა და გამიარა თითქმის) უკვე მეათე დღეა ასეა, უკვე გამივლის წესით, და კითხვა რაშI მდგომარეობს, რამდენი ხანი არ შეიძლება ვარჯიში გავლის პერიოდიდან? ამბობენ ინფექცია ღვიძლთან რჩება და სისხლშიო და დატვირთვები არ შეილება რაღაც პერიოდიო

Posted by: paraNOik 16 Jan 2013, 03:20
სავარაუდოდ თირკმელებთან დაკავშირებით აქ ვიკითხო? facepalm.gif facepalm.gif facepalm.gif

Posted by: Kobalty 19 Jan 2013, 21:14
შემთხვევით ხომ არ იცით სად შეიძლება ესა თუ ის წამალი მოვიკითხო?
ცალკე თემა არის ამისთის?
სახელდობრ: ვაზელინის ზეთი მჭირდება ბავშვისთვის..
სად შეიძლება აქ შოვნა?
გმადლობთ წინასწარ.

Posted by: buub 23 Jan 2013, 03:36
ბურსა
ტკივილი გულ-მკერდის არეში,სუნთქვის უკმარისობა, ხველა,შესაძლოა ტემპერატურა,ქოშინი.


Kobalty
ავერსში იყო ადრე მახსოვს დარეკე დააზუსტე

Posted by: tacka 24 Jan 2013, 17:03
ჩუტყვავილას დროს არსებობს საკვები რისი ჭამაც არ შეიძლება? smile.gif

Posted by: lmlmlm 25 Jan 2013, 11:50
გამარჯობათ, ერთი სამი დღის წინ დამეწყო სურდო შეციება, საღამოს ამიწია სიცხემ 39 მდე, დამეწყო ხველა, ყელის ტკივილი, მაქვს ნახევლი მუქი ფერის, ეხლა მტკივა თვალის დაბლა ანუ ცხვირის გასწვრივ ცალ მხარეს და საფეთქელი, სიმძიმის შეგრძნება მაქ განსაკუთრებით თავს როცა დავხრი, ცხვირიდან ცალ ნესტოდან მაქ მუქი გამონადენი, და გაჭედილი ცხვირი, ციხე 38,2 მიგუბდბეა ყურები, როგორც ვხდები ჰაიმორიტის სიმპტომები მაქ და თუ იცით რომელი წამალი არის კარგი აფთიაქში რომ ვიყიდო? სიცხის დასაწევად და გაციებისთვის ნიმესილს ვსვამდი ეს დღეები და ნაფტიზინს ვიწვეთებდი, ყელის ტკივილისთვის კი სტრეპსილს ვსვამდი,

Posted by: Solveig 31 Jan 2013, 12:32
mari110

აუცილებლად უნდა მიმართოთ ჰეპატოლოგს. ღვიძლის და ნაღვლის ბუშტის გამოსაკვლევად.

კარგი იქნება, თუ მანამდე B და C ჰეპატიტებზე გაიკეთებს სისხლის ანალიზს.

Posted by: grumbler_girl 6 Feb 2013, 20:47
ცოტახნის წინ ფეხები მომეკვეთა და დავეცი მარცხენა ფეხს სააერთოდ რამოდენიმე წამს ვერ ვგრძობდი და ვერ ვდგავდი ისევ მეკეცებოდა... ვერაფერს ვერ დაუკავშირე რა შეიძლება იყოს, წნევა არ დამვარდნია
თუ შეიძლება სასწრაფოდ მიპასუხეთ
ძალიან შემეშინდაა...
ხო ისე ჰიპოტონიკი ვარ მაგრამ ამას ვერ დაუკავშირეეე

Posted by: _DelgadA_ 7 Feb 2013, 00:35
ბუსკოპანის სანთელი ვერსად ვერ ვიშოვე, დეფიციტშიაო და სად ვიყიდო ვერსად ვერ მიმასწავლით ? :ს

Posted by: infuzoria 7 Feb 2013, 11:01
პადაგრას პეობლემაზე რომელ ექიმს უნდა მივმართო?

Posted by: grumbler_girl 7 Feb 2013, 13:04
აქ პასუხის გაცემა არ იციან???

იყოს აღარ მინდა მშობლებს ვუთხარი უკვე.

Posted by: rock in rose 7 Feb 2013, 13:06
grumbler_girl

არ შემოდის ექიმი და ტყუილს ხომ არ მოგწერდა ვინმე დილეტანტი?

ადრე ყავდა ამ თემას თერაპევტი, მაგრამ რა ხანია ის ადამიანი არ შემოდის...

Posted by: Masked 7 Feb 2013, 13:41
QUOTE (grumbler_girl @ 7 Feb 2013, 13:04 )
აქ პასუხის გაცემა არ იციან???

იყოს აღარ მინდა მშობლებს ვუთხარი უკვე.

გოგონა, ან გაქვს ბანდშაიბენფოაფალლ - დისკუსპროლაპს (L4, L5) ან არის ტავის ტვინიდან. გაქვს ტკივილი ხერხემალის, არ არის აუცილებელი იკოს ტკივილი ხერხემალის. შენ დაწერე რომ არა გაქვს გრძნობა, ეს შემთხვევაში ტავის ტვინი უნდა გამოკვლევა.
ნოიროლოგმა უნდა გადაწკვიტა რა უნდა გამოკვლევა, ხერხემალის ტუ ტავის ტვინის.

Posted by: grumbler_girl 7 Feb 2013, 14:11
QUOTE (Masked @ 7 Feb 2013, 13:41 )
QUOTE (grumbler_girl @ 7 Feb 2013, 13:04 )
აქ პასუხის გაცემა არ იციან???

იყოს აღარ მინდა მშობლებს ვუთხარი უკვე.

გოგონა, ან გაქვს ბანდშაიბენფოაფალლ - დისკუსპროლაპს (L4, L5) ან არის ტავის ტვინიდან. გაქვს ტკივილი ხერხემალის, არ არის აუცილებელი იკოს ტკივილი ხერხემალის. შენ დაწერე რომ არა გაქვს გრძნობა, ეს შემთხვევაში ტავის ტვინი უნდა გამოკვლევა.
ნოიროლოგმა უნდა გადაწკვიტა რა უნდა გამოკვლევა, ხერხემალის ტუ ტავის ტვინის.

პირველად დამემართა ასე, არ მქონია არასდროს არანაირი თავის ტვინის პრობლემა, ცოტა ხნის წინ მსუბუქი ტვინის შერყევა მქონდა და არ დავწოლილვარ საერთოდ, ერთი ამას ვფიქრობ რო შეიძლება გამოეწვია... გრძნობა აღდგა რომოდენიმე წამში, ახლა სულ ოდნავ მტკივა მუხლის სახსარში ვადგავ ჩვეულებრივად
ნევროლოგს ვის მირჩევ გიგინეიშვილს???

Posted by: Masked 7 Feb 2013, 14:26
grumbler_girl
რას ნიშანვს არა გქონდა გრძნობა, უფრო კონკ. უნდა დაწერა, დაკარგე გრძნობა? ტუ დაკარგე კონროლლი ფეხების, ტუ ვერ გრძნობ ფეხებს ტუ როგორ. გაქვს სხვა ტკივილი გარდა მუხლი.

Posted by: grumbler_girl 7 Feb 2013, 14:33
QUOTE (Masked @ 7 Feb 2013, 14:26 )
grumbler_girl
რას ნიშანვს არა გქონდა გრძნობა, უფრო კონკ. უნდა დაწერა, დაკარგე გრძნობა?  ტუ დაკარგე კონროლლი ფეხების, ტუ ვერ გრძნობ ფეხებს ტუ როგორ. გაქვს სხვა ტკივილი გარდა მუხლი.

ჰოო კონტროლი და ვერც ვგრძნობდი საერტოდ მარცხენა ფეხს მთლიანად,ვერ ვადგავდი. რამოდენიმე წამის განმავლობაში, ხო დავწერე რო რამოდენიმე წამი მხოლოდ...
არა მხოლოდ მუხლის სახსარი მტკივა ესეც დაცემის შედეგად შეიძლება...
და კიდე თავი მტკივა

Posted by: Masked 7 Feb 2013, 18:49
grumbler_girl
უნდა კონს. ნოიროლოგთან გოგონა ჩქარა. აუცილებელად!

Posted by: ashley judd 13 Feb 2013, 03:25
დაღლილობის ან გამოუძინებლობის დროს მახველებს ხოლმე თითქოს ჰაერი არ მყოფნიდეს, რამე დაავადების სიმპტომი ხო არ შეიდძლება იყოს?

რომელ ექიმს მივმართო? მირჩიეთ

Posted by: rapu_zel 21 Feb 2013, 16:32
გამარჯობათ,იმდენი რამე მჭირს აღარ ვიცი ვის მივმართო ვარ 21 წლის,მაწუხებს მუდმივი თავის ტკივილები,სექტემბერში რო ავდექი სავარძლიდან თავბრუ დამეხვა ამანკალა ვეგარ შევიკავე თავი და დავეცი,მცვივა თმები საშინელ მდგომარეობაში მაქვს ვერაფერს ვუხერხებ უკვე და წონაში დავიკელი 15-კილო ეს ყველაფერი მოხდა 4-5 თვის განმავლობაში შეიძლება ოდნავ მეტი და საერთო სურათით რა შეიძლება მჭირდეს?

Posted by: rock in rose 21 Feb 2013, 17:08
rapu_zel

ენდოკრინოლოგთან მიდი აუცილებლად

Posted by: rapu_zel 22 Feb 2013, 19:41
QUOTE (rock in rose @ 21 Feb 2013, 17:08 )
rapu_zel

ენდოკრინოლოგთან მიდი აუცილებლად

მაინც რა მჭირს ?ისე ზოგადი მონაცემებით რამ იცი ეს?

Posted by: rock in rose 22 Feb 2013, 20:02
rapu_zel

მხოლოდ იმის თქმა შემიძლია აქედან, რომ ენდოკრინოლოგთან უნდა მიხვიდე, რა გჭირს უკვე ის გეტყვის ჰორმონალური სტატუსის შეფასების შემდეგ smile.gif

Posted by: cin_namon 23 Feb 2013, 02:03
ოდესმე მივიღებ ალბათ პასუხს..

ალცჰაიმერის დაავადების პირველი ნიშნები რა არის??

Posted by: rock in rose 23 Feb 2013, 09:33
play it fucking loud

მეხსიერების დაქვეითეიბა

Posted by: cin_namon 23 Feb 2013, 10:58
rock in rose

მარტივი სიტყვები რო გავიწყდება და ვერ იხსენებ ეგ არის მაგის სიმპტომი??

Posted by: rock in rose 23 Feb 2013, 11:02
play it fucking loud

კი სამწუხაროდ, აფაზია ახასიათებს ანუ სიტყვათა მარაგის გაღარიბება.

ასევე ცოტა ხნის წინ მომხდარი ფაქტების და ახლო წარსულის გახსენების უნარის დაქვეითება.

Posted by: cin_namon 23 Feb 2013, 12:47
rock in rose
აქ აკეთებენ მაგის გამოკვლევებს???

და ახალგაზრადა ადამიანს შეიძლება დაემართოს??
მე როგორც ვიცი ასაკში ჩნდება ეს user.gif

Posted by: rock in rose 23 Feb 2013, 13:44
play it fucking loud

50 წლის შემდეგ უფრო ხშირია, თუმცა იშვიათ შემთხვევებში შეიძლება ახალგაზრდასაც დაემართოს.

ზუსტი დიაგნოსტიკა ამ დაავადების ადამიანის სიცოცხლეში არ ხდება, ანუ იმისთვის რომ გადაჭრით დაისვას ეს დიაგნოზი თავის ტვინის მიკროსკოპული კვლევაა საჭირო, რაც ადამიანის სიკვდილის შემდეგ არის შესაძლებელი. თუმცა, იშვიათ შემთხვევაში შესაძლებელია თავის ტვინის ბიოფსიის გაკეთებაც.

პაციენტის სიცოცხლეში კი ექიმი გამორიცხავს ისეთ დაავადებებს, რომლებიც იწვევენ ისეთ სიმპტომებს, როგორიც აქვს ალცჰაიმერს, მაგალითად: სიმსივნე, ტრავმა, ინფექციები, ნივთიერებათა ცვლის მოშლა. როცა ეს ყველაფერი გამირიცხება მერე მიიტანენ ეჭვს ამ დაავადებაზე

Posted by: cin_namon 23 Feb 2013, 13:52
rock in rose
მადლობა

.............................

Posted by: giorgi --- 24 Feb 2013, 03:01
გამარჯობა,
თუ შეიძლება მითხარით ნებისმიერი კლინიკა, სადაც შემეძლება ხვალ (კვირას) მისვლა, ფილტვებზე მოსმენა და რენტგენის გადაღება.
3 დღეა ანთების სიმტომები მაქვს და დედააფეთქებულ სადაზღვევოს კლინიკაში მითხრეს, რომ ორშაბათამდე არ ეცალათ. დღეს სიცხეც დაემატა ამ სიმპტომებს და მინდა, რომ აუცილებლად გავესინჯო ხვალ.

Posted by: rock in rose 24 Feb 2013, 10:32
giorgi ---
გაგიმარჯოს
რესპუბლიკურში მაგალითად, ან არამიანცში

Posted by: damomoda 25 Feb 2013, 11:37
ხველების დროს ბრონქები მტკივა ცოტა, მკერდის არეში. სიცხე მაქვს 37,1. სურდო არ არის. ცოტა ნახველს ვიღებ ხოლმე. ცოტა სუსტად ვარ. რა მჭირს?
ბრონქიტი თუ ფილტვების ანთება?

Posted by: rock in rose 25 Feb 2013, 14:19
damomoda

მკერდის ძვალთან ტკივილის შეგრძნება და ხველა მწვავე ბრონქიტის ნიშნებია, ექიმთან უნდა მიხვიდეთ. რაც მალე მით უკეთესი

Posted by: damomoda 25 Feb 2013, 14:38
rock in rose
QUOTE
მკერდის ძვალთან ტკივილის შეგრძნება და ხველა მწვავე ბრონქიტის ნიშნებია, ექიმთან უნდა მიხვიდეთ. რაც მალე მით უკეთესი

ეხლა მე თვითონ ვიღებ ამოსახველებელს და ანთებისსაწინააღმდეგოს (პანადოლი და ლიზლი). და თუ დიდხანს გაგრძელდა მაშინ რენტგენს გადავიღებ რო პნევმონია გამოირიცხოს.

Posted by: rock in rose 25 Feb 2013, 20:06
damomoda

თვითმკურნალობას არ გირჩევ, რადგან მარტივად შეიძლება ანთებითი პროცესი გავრჩელდეს ფილტვებზე და მერე მართლა პნევმონია მიიღო. უბნის ექიმს გაესინჯე, მოგისმინოს ფილტვებში და ანტიბიოტიკიც დაგჭირდება.

QUOTE
ლიზლი


არაა მაინც და მაინც ეფექტური ეს წამალი და შენ როგორ ატყობ შვება გაქვს?

Posted by: damomoda 25 Feb 2013, 20:38
rock in rose
QUOTE
არაა მაინც და მაინც ეფექტური ეს წამალი და შენ როგორ ატყობ შვება გაქვს?

საერთოდ არ მეხმარება ნახველის ამოღებაში. თითქოს არც არაფერს არ ვსმადე. და თქვენ რას მირჩევთ?

Posted by: rock in rose 25 Feb 2013, 21:14
damomoda

ლიბექსინი უკეთესია, ამას გირჩევდი

Posted by: winipux 26 Feb 2013, 16:51
გამარჯობა!4 დგის წინ ვიყავი ექიმთან დამინიშნა ოტრივინი,ერესპალი,და კეტოტიფენი.2 დგეა დამეწყო გულის არეში ტკივილი სუნტქვის გაძნელება ,დილიტ საერტოდ ვერ ვსუნთავ ცხვირიდან,როგორც შევიშხურებ ოტრივინს მერე მეშვება.აი მომენტალურად მეკეტება სუნტქვა და გული მიჩქარდება მეწყება ქოშინი პირით სუნტქვისას მაინც ვერ ვპოულობ შვებად დიდად მაინც შფოთვარე ვარ....რა შეიდზლება იყოს?სანამ მივალ ექიმტან იკნებ რამე მირჩიოტ,

Posted by: _DelgadA_ 26 Feb 2013, 23:52
გამარჯობა, სახლის პირობებში ბარიუმის ფაფა რომ მივცე კუჭ ნაწლავის გასაწმენდად კარგი იქნება?

Posted by: damomoda 27 Feb 2013, 11:08
rock in rose
QUOTE
ლიბექსინი

კაი

Posted by: Pompiero 3 Mar 2013, 09:16
გამარჯობა smile.gif
რა მიზეზებით შეიძლება იყოს გამოწვეული გემოს შეგრძნების დაკარგვა?
გრძნობს სუსტად მხოლოდ მჟავეს და ცხარეს. სრულებით ვერ გრძნობს ტკბილს და მარილს.



Posted by: newsgirl 3 Mar 2013, 11:36
რაღაც ვირუსი რომ არის ქალაქში, ღორის გრიპი არა და მეორე, რომ ახველებს ხალხს, მგონი ეგ მჭირს. ან იქნებ ღორის გრიპიცაა. არ ვიცი. გუშინ დამეწყო ხველა, მშრალი. სიცხე მაქვს 37. ძვლები მტკივა, წელი ნეკნები ზურგი.... თავს სუსტად ვგრძნობ. acc-ს ვსვამ. ვინმემ გამოიარეთ რამე მსგავსი? მე გადმომდეს.

Posted by: gangsteri 3 Mar 2013, 17:39
გამარჯობათ, როგორ გავიგო რა მდგომარეობაში მაქვს ორგანოები ასპირინის დიდი დოზით მიღების შემდგომ, მედ პუნტში მისვლის გარდა სხვა მეთოდი არ არსებობს? მაგალითად რაიმე ტესტირება ან მსგავსი

გმადლობთ წინასწარ

http://forum.ge/?showtopic=34504387&hl=

Posted by: rapu_zel 5 Mar 2013, 18:45
გამარჯობათ თერაპევტებო სიცხე მაქვს ოცდაჩვიდმეტი და რისი ბრალი შეიძლება იყოოს?1 თვის წინ ვიყავი თერაპევტთთან და რაღაც ვირუსიააო დავლიე წამლები გამიარა მაშინ ცოტა სურდოც მქონა მაგრამ ახლააა არაფერი მჭირს მააგისმაგვარი და სულ შემთხვევით აღმოვაჩინე სიცხე სისუსტის ფონზე

Posted by: temart 6 Mar 2013, 13:45
წითელას საწინააღმდეგო აცრის ცენტრები სად არის ხონ არ იცით?





Posted by: rapu_zel 6 Mar 2013, 21:14
აქ მგონი მხლოდ შეკითხვები ისმევა და პსუხს არავინ გვცემს

Posted by: an_only 6 Mar 2013, 22:55
გამარჯობათ, მაინტერესებს თუ სეიძლება დადგინდეს ვენერიული დაავადებით დასნეულების სავარაუდო პერიოდი, კონკრეტულად გონორეის?

Posted by: rock in rose 7 Mar 2013, 19:35
an_only

ინკუბაციური პერიოდი ამ დაავადების 2-დან 30-დღემდეა და შემდეგ უკვე კლინიკური ნიშნები ვლინდება, სავარაუდოდ შეიძლება ითქვას როდის მოხდა დაინფიცირება.

Posted by: ravi_shen 7 Mar 2013, 19:52
კარგი ტერაპევტი სად არის ხომ არ იცით?
ö.....................................ö

Posted by: anri1992 9 Mar 2013, 10:55
ლანსოპრაზოლით ლანსოლით ხომ არ მკურნალობდით ვინმე?

Posted by: rock in rose 9 Mar 2013, 12:59
anri1992

კონკრეტულად რა გაინტერესებს? smile.gif

Posted by: alkagolik1 9 Mar 2013, 21:42
რამე კარგი დამამშვიდებელი მცენარეული საშვალება თუ იცით ვინმემ მირჩიეთ რა,ნევროზისთვის და უძილობისთვის. 50 წლის ასაკიან ადამიანზე "კარგად" რომ იმოქმედოს?

Posted by: rock in rose 10 Mar 2013, 09:00
alkagolik1

ვამელანი მაგალითად, ვალერიანს შეიცავს დიდი დოზით, პიტნასა და ბარამბოს.


Posted by: alkagolik1 10 Mar 2013, 12:24
QUOTE (rock in rose @ 10 Mar 2013, 09:00 )
rock in rose

მადლობთ...................................................................................

Posted by: pvm 12 Mar 2013, 18:50
იქნებ რამე მირჩიოთთთთთთთთთთთ

ძალიან მაღალი როე რის გამო შეიძლება იყოს? 69 წლის პაციენტს აქვს 75 როე. ვერავინ ვერაფერი დაადგინა.
პაციენტი არის დიაბეტიანი, აქვს გულის პრობლემები და სახსრების ტკივილები, განსაკუთრებით წელის მიდამოში. იქნებ მირჩიოთ რამეეეეეეეე თუ რის ექიმს შეიძლება მივამართოთ?
მადლობთ წინასწარრრრრრრრრრრ

Posted by: taso 12 Mar 2013, 18:55
დაბალი ტემპერატურა (35-36) მაქვს რამდენიმე დღეა. არადა ჩემი ნორმალური ტემპერატურა - 36,6-ია. :/
არ ჩამქოლოთ ექიმს რატომ არ მიმართეო. მეშინია რატომღაც. :/
რისი ბრალი შეიძლება იყოს?

Posted by: sakheli 12 Mar 2013, 20:49
მარტი პასუხებს რომ ითხოვთ თერაპევტებისგან გამოკვლევის პასუხებიც რომ დაწეროთ უფრო გარკვევით არ იქნება ხალხსაც დაეხმარებით შეოძლება ერთი და იგივე სიმპტომები ჩივილები აწუხებდეს ადამიანს უფრო გაუადვილებთ თუ რომელ ექიმს უნდა მიმართოს. ძალიან კარგი ინქებოდა ხალხსაც დაეხმარებით გამოკვლევაში და თერაპევტებისთვისაც კარგი ინქბეა თუ სწორედ დაგისვა დიაგნოზი. ელემენტარული თუ შეძლოა ადამიანის დახმარება გაუხარდება დარწმუნებული ვარ და იამაყებს კიდეც. დავუფასოთ გვერდში დგომა.

Posted by: BadbadGirl 12 Mar 2013, 22:03
გამარჯობა,
ძალიან გთხოვთ მირჩიოთ კარგი თერაპევტი ისანი სამგორის რაიონში, დედასთვის მინდა.
სასურველია რომ გამოცდილება ქონდეს ცოტა ასაკიან ადამიანებთან.

წინასწარ დიდი მადლობა.

Posted by: sakheli 13 Mar 2013, 11:36
QUOTE (alkagolik1 @ 9 Mar 2013, 21:42 )
რამე კარგი დამამშვიდებელი მცენარეული საშვალება თუ იცით ვინმემ მირჩიეთ რა,ნევროზისთვის და უძილობისთვის. 50 წლის ასაკიან ადამიანზე "კარგად" რომ იმოქმედოს?

პერსენ ფორტე
შემადგენლობა:
კატაბალახას მშრალი ექსტრაქტი.
ბარამბოს მშრალი ექსტრაქტი.
პიტნის მშრალი ექსტრაქტი .

Posted by: taso 13 Mar 2013, 13:45
QUOTE
დაბალი ტემპერატურა (35-36) მაქვს რამდენიმე დღეა. არადა ჩემი ნორმალური ტემპერატურა - 36,6-ია. :/
არ ჩამქოლოთ ექიმს რატომ არ მიმართეო. მეშინია რატომღაც. :/
რისი ბრალი შეიძლება იყოს?

მიპასუხეთ რა :/

http://mkurnali.ambebi.ge/kitxva-pasuxi.html?view=question&id=67082

Posted by: taamo 13 Mar 2013, 20:51
ამ ბოლო დროს ვატყობ, რომ ყნოსვა გამიუარესდა. თუ ძალიან მკვეთრი არაა სუნი, ისე არ მცემს.. ქრონიკული რინიტი, ფრონტიტი, ჰაიმორიტი და მსგავსი დაავადებები შეიძლება ფარულად მიმდინარეობდეს? არ მაწუხებს არცერთი და რას შეიძლება ყნოსვის პრობლემა გამოეწვია?

Posted by: lovelorn 16 Mar 2013, 14:02
შემოდის ნეტავ აქ ექიმი?

ფეხმძემე ვარ 7 თვემდე, ფეხები მტკივა ძალიან, თითქმის ვეღარ დავდივარ, მსუქანი არ ვარ, ისე კი მოვიმატე წონაში,
ღამეც საშინლად მეკრუნჩხება, არ მაძინებს.
სანამ ექიმთან წავალ,
მირჩიეთ რამე, რისი ბრალია?
ან რა ანალიზები გავიკეთო? ვისთან მივიდე?

Posted by: Mrs_Zum 16 Mar 2013, 15:57
ამას ქვია ღამის კრემპები. ხშირია ორსულებში. მეც მქონდა. გირჩევდი კოაგულოგრამა გაგეკეთებინა. ექიმიც ამას გეტყვის, და დანიშნულებასაც მოგცემს. შენს მეან -გინეკოლოგს მიაკითხე ვისთანაც დადიხარ.

Posted by: givi9988 16 Mar 2013, 18:21
არ ვიცი სწორ ადგილას ვსვამ თუ არა კითხვას, მაგრამ იქნებ ვინმემ იცოდეთ სად და როგორ ხდება შარდოვანას განსაზღვრა? წინასწარ მადლობა!

Posted by: rock in rose 16 Mar 2013, 18:35
givi9988

აი ეს ნახეთ http://enmedic.ge/index.php?name=PagesG&op=page&pid=51

თითქმის ყველა ლაბორატორიაში ხდება მისი განსაზღვრა(სისხლში).

Posted by: givi9988 16 Mar 2013, 18:42
rock in rose

მადლობა ინფორმაციისთვის

Posted by: ვივიანა 19 Mar 2013, 21:22
ვერ ვნახე სათანადო თემა და მაიცნ აქ დავსვამ იქნებ მიპასუხოთ

რენტგენის გადაღება ვთქვთ კვირაში 4ჯერ როგორ მოქმედებს ორგანიზმზე ?

Posted by: sakheli 19 Mar 2013, 21:29
QUOTE (vivianna @ 19 Mar 2013, 21:22 )
ვერ ვნახე სათანადო თემა და მაიცნ აქ დავსვამ იქნებ მიპასუხოთ

რენტგენის გადაღება ვთქვთ კვირაში 4ჯერ როგორ მოქმედებს ორგანიზმზე ?

მემგონი 6 თვეში ერთხელ არის რეკომენდირებული და შენ რისთვის გინდა რამე საჩქარო თუ არის უნდა შეეკითხო შენს მკურნალ ექიმს.

Posted by: ქეთუზა 20 Mar 2013, 15:24
მოკლედ ვერც მე ვიპოვე სათანადო თემა და აქ ვიკითხავ: taxofit zink ვიტამინის დალევა დავიწყე რეცეპტის გარეშე,მითხრეს რომ იმუნურ სისტემასაც აძლიერებს.
ვინმემ თუ იცის როგორი პრეპარატია,ერთ თვიანი კურსით შედეგი მექნება?

Posted by: sakheli 20 Mar 2013, 16:40
QUOTE (ქეთუზა @ 20 Mar 2013, 15:24 )
მოკლედ ვერც მე ვიპოვე სათანადო თემა და აქ ვიკითხავ: taxofit zink ვიტამინის დალევა დავიწყე რეცეპტის გარეშე,მითხრეს რომ იმუნურ სისტემასაც აძლიერებს.
ვინმემ თუ იცის როგორი პრეპარატია,ერთ თვიანი კურსით შედეგი მექნება?

რა ვიტამინია და რასთან დაკავშირებით გინდა შედეგი უფრო დააკონკრეტე და გაცემენ დამაკამაყოფილებელ პასუხს. wink.gif

Posted by: ქეთუზა 21 Mar 2013, 12:33
sakheli
რა ვიტამინია და taxofit zink,გერმანულიაbiggrin.gif
რამდენიმე სახეობის გამოდის,ეს იყო კონკრეტულად იმუნური სისტემის გასაძლიერებელი.
შესაბამისად შედეგიც მინდა ამ კუთხით მქონდეს. mo.gif
ზოგადად კი ის უფრო მაინტერესებს ვინმეს თუ ჰქონია შეხება ამ პრეპარატთან

Posted by: cin_namon 22 Mar 2013, 01:36
ნევროლოგი არის აქ ვინმე??????????????????



Posted by: sakheli 22 Mar 2013, 13:10
QUOTE (play it fucking loud @ 22 Mar 2013, 01:36 )
ნევროლოგი არის აქ ვინმე??????????????????

აქ ნახე აბა მემგონი მანდ უფრო უნდა იყოს.

http://forum.ge/?showforum=15
* * *
QUOTE (sakheli @ 22 Mar 2013, 13:10 )
QUOTE (play it fucking loud @ 22 Mar 2013, 01:36 )
ნევროლოგი არის აქ ვინმე??????????????????

აქ ნახე აბა მემგონი მანდ უფრო უნდა იყოს.

http://forum.ge/?showforum=15

უკაცრავად შემეშალა არც არის კატეგორია მემგონი.

Posted by: cin_namon 22 Mar 2013, 15:24
sakheli
არადა მჭირდება

sad.gif


Posted by: sakheli 22 Mar 2013, 15:29
ახალი თემა გახსენი ნერვოლოგი მჭირდებაო და იქნებ გამოგეხმაურონ ან გირჩიონ კარგი ნერვოლოგიც შეიძლება რომ იყვენ აქ ადამიანები რომლებმაც იციან კარგი ნერვოლოგი და კმაყოფილებიც არიან თან ღცევას მოგცემე ტან გირჩევენ ვისთან უნდა მიხვიდე და უფრო კარგად დაკვალიანდები იქნებ დააწერო კიდეც რატომ გჭირდება ნერვლოგი გამოგეხმაურებიან აუცილებლად.

Posted by: rock in rose 22 Mar 2013, 15:29
play it fucking loud

ბავშვთა ნევროლოგი პოსტავს მშობლების კლუბში, თუ გინდა ნიკს გეტყვი და მას მიწერე.

ისე დიდების ნევროლოგი საერთოდ არც ყოფილა ფორუმზე.

Posted by: cin_namon 22 Mar 2013, 15:59
rock in rose
მითხარი აბა

:*


Posted by: rock in rose 22 Mar 2013, 16:04
play it fucking loud

ეს გახლავს smile.gif http://forum.ge/?showuser=97223

Posted by: Giorgi984 27 Mar 2013, 16:30
ახლობელს (25 წლის) ორი-სამი დღე აქვს სიცხე 37. სიცხის დამწევი წამალი ვითომ უწევს, მაგრამ მაინც ადის 37-ზე. ზურგში უკან ტკივილი აქვს ხანდახან და ახველებს ბრონქებიდან, აგრეთვე სისველე ცხვირში და სისუსტე. რას მირჩევთ, ვურჩიო წასვლა ფილტვების გასასინჯად? თუ ავაღებინო სისხლის ზოგადი ანალიზი? გთხოვთ მიპასუხეთ როგორც კი მოიცლით.

Posted by: ashley judd 27 Mar 2013, 17:00
მახველებს და კარგ ექიმს ვის მირჩევთ რო გავესინჯო?

Posted by: rock in rose 27 Mar 2013, 17:43
ashley judd

უბნის ექიმი საკმარისი იქნება იმისთვის რომ ფილტვებში მოგისმინოს, დამიჯერე smile.gif

Posted by: ashley judd 27 Mar 2013, 20:40
არ მინდა უბნის ექიმთან და იქნებ ნორმალური მირჩიოთ smile.gif

Posted by: keep_walking 29 Mar 2013, 23:16
მთEლი დღეა თავბრუ მეხვევა, რომ დავდივარ ვეხეთქები კედლებს
ლამის წავიქცე
ყავა დავლიე, ვერ მიშველა
წნევას არავინ მისინჯავს user.gif
რა ვქნა? მივიდე ექიმთან? ან რის ექიმთან?

Posted by: nino877 31 Mar 2013, 20:57
3 კვირაა მოწევას დავანებე თავი.
რაღაც სტრანი შეგრძნებები მაქვს
არ ვიცი ფსიქოლოგიურია თუ ჯანმრთელობის
ვგრძნობ რომ გული წამივა,
აი რომ უნდა დააცემინო და არ გაცემინებს.
ცხვირი მატყუებს -ო რომ ამბობენ.
(Iთქოს გული მიხდება ცუდად, თითქოს უფრო თვალებში მიბნელდება და გულის რევის შეგრძნება მაქვს , მაგრამ ეს ყველაფერი გრძელდება 1 წამი

Vინმეს გქონათ ამდაგვარი ან პრაქტიკა ან რამე?

Posted by: Ree 3 Apr 2013, 20:48
ძალიან გთხოვთ, მირჩიეთ, ნაღვლის ბუშტის ტკივილს რა ვუშველო? ბევრი ცხიმიანი ვჭამე და ალკოჰოლიც დავაყოლე და შავ დღეში ვარ.
ძლიერი ტკივილი არ არის, უბრალოდ სიარული და მოძრაობა დისკომფორტს მიქმნის. თითქოს გვერდში მჭრის.
ნოშპა დავლიე ორი ტაბლეტი და ოდნავადაც არ მშველია.
არადა ხვალ სამსახურში ვარ გასასვლელი.
რამე მირჩიეთ რა. sad.gif

Posted by: rock in rose 3 Apr 2013, 22:00
Ree

რაფაქოლინ ცე დალიე 2 აბი, ნაღველმდენი საშუალებაა.

სამომავლოდ კი თავი უნდა შეიკავო ცხიმიანი და ცხარე საკვებისაგან.

Posted by: Ree 3 Apr 2013, 22:20
rock in rose
მადლობა smile.gif

Posted by: Solveig 4 Apr 2013, 03:25
rock in rose

ასეთ რაღაცებს ნუ ურჩევ ფორუმით...კენჭი რომ ჰქონდეს, მერე რა უნდა ქნას?

Posted by: taamo 4 Apr 2013, 21:56
QUOTE
ამ ბოლო დროს ვატყობ, რომ ყნოსვა გამიუარესდა. თუ ძალიან მკვეთრი არაა სუნი, ისე არ მცემს.. ქრონიკული რინიტი, ფრონტიტი, ჰაიმორიტი და მსგავსი დაავადებები შეიძლება ფარულად მიმდინარეობდეს? არ მაწუხებს არცერთი და რას შეიძლება ყნოსვის პრობლემა გამოეწვია?


Posted by: tornikke 6 Apr 2013, 11:10
გამარჯობათ

უკვე მე3 კვირა მთავრდება რაც თითქოს გაციებული ვარ. ტემპერატურა არ მქონია, ნუ არც გამიზომია მაგრამ რამდენჯერმე შეიძლება მქონდა. სურდო მაქ. მაციებს ხოლმე. პირველი კვირა ასე ვიყავი ვსვავდი ნიმესილს, ქოლდრექსს, კიდევ სხვადასხვა ჩაის. როცა ერთ ჩაის რამდენიმე დღე ვსავდი და არ მშველოდა სხვაზე გადავდიოდი. + დღის განმავლობაში განუზომელ ლიმნიან ჩაის ვსავ. ბოლო ერთი კვირაა თვალების წვა დამეწყო, ცრემლი მომდიოდა, დილით დასიებული მაქვს ხოლმე. სულფაცილს ვიწვეთებ და ცოტათი გამიარა მაგრამ მაინც დილაობით ისევ დასიებული მაქ. ამდენი ხანი გაციება და გრიპი არ მქონია მგონი არასდროს. ზოგადად კი ადვილად ვცივდები მაგრამ მაინც. ეხლა ვფიქრობ რამე ალერგია ხომ არ არის? არადა არასდროს არ ვყოფილვარ ალერგიული არაფერზე. ჩემი ძმაც ასევეა დაახლოებით. რას მირჩევთ?

Posted by: anderson 6 Apr 2013, 19:24
იშემიური ინსულტი - ინგლისურად როგორ არის?

Posted by: MONTY 11 Apr 2013, 16:02

პლაზმოფერეზის სრული კურსი რა ჯდება ხო არ იცით ? პლაზმოფერეზის გაკეთება ღირს თუ სისხლის გადასხმა ?

Posted by: Solveig 12 Apr 2013, 01:49
anderson

ischemic stroke

Posted by: shaverdani65 15 Apr 2013, 15:30
გამარჯობათ! მე მაინტერესებს ესეთი რაღაც ჩემ ახლობელს აქვს კისერზე პრობლემა მოღრეცილი აქვს ასევთქვათ, ადრე იყო ექიმთან ეხოსკოპიები, სურათები გადააღებინეს და ვერაფერი ვერ გაიგეს, ხან ნერვიულ სისტემას დააბრალეს ხან რას... ხო ვერ მირჩევდით ყველაზე კარგ საავანტყოფოს და ექიმს რომ მივიყვანო?? დიდი მადლობა წინასწარ! კოორდინატები მომწერეთ ექიმის...

Posted by: qartvelasi 15 Apr 2013, 18:11
არ ვიცი რამდენად სწორ ადგილას ვწერ მაგრამ იქნებ რამე მირჩიოთ, დილით რომ გავიღვიძე ცხვირის დასაწყისის გასწვრივ თითქმის ლოყასთან თვალის უპემდე რამდენიმე სანტიმეტრით ადრე, აი მანდ მაქვს გასივებული, ხელის მიდებაზე მტკივა, კბილების ან ნუნების მხრიდან პრობლემა არ უნდა იყოს მემგონი და რისი ბრალი იქნება? საღამო მოვიდა და უფრო გამისივდა

Posted by: Green Earth 19 Apr 2013, 03:13
გამარჯობათ,

ამ ბოლო დროს პირში მწარე გემო მაქვს ხშირად და რისი ბრალი შეიძლება იყოს ?

Posted by: Ree 19 Apr 2013, 10:51
Green Earth
მწარე გემო ნაღვლის ბუშტის და/ან კუჭის დისფუნქციის ნიშანია. გამოკვლევა ჩაიტარე.

Posted by: mirkani 19 Apr 2013, 11:12
თუ იცით სად იღებს დათო ვაშაძე პაციენტებს? მისამართი ან ტელეფონის ნომერი ?

Posted by: cin_namon 21 Apr 2013, 20:12
ორი დღეა მამთქნარებს ვინმე მეტყვის რატომ?
უძილობა არაა მიზეზი ვიცი, არც გადაღლა...


უკვე ნერვებს მიშლის : ))

Posted by: rock in rose 21 Apr 2013, 21:29
play it fucking loud
QUOTE
ორი დღეა მამთქნარებს ვინმე მეტყვის რატომ?
უძილობა არაა მიზეზი ვიცი, არც გადაღლა...


სავარაუდოდ ჰიპოქსია

სისუსტესაც ხომ არ გრძნობ ან თავბრუსხვევას?

Posted by: cin_namon 21 Apr 2013, 23:26
rock in rose
სისუსტეს კი, თავბრუხვევას არა

და ჰიპოქსია რას ნიშნავს?

მდგომარეობა, როცა მთელი ორგანიზმი, ან მისი ნაწილი განიცდის ჟანგბადის ნაკლებობას. ის შეიძლება გამოწვეული იყოს ალკოჰოლის ხშირი მოხმარების, სუნთქვის შეკავების, ავადმყოფური მდგომარეობის, ატმოსფეროში ჟანგბადის მცირე შემცველობისა, ან ორგანიზმის სიკვდილის გამო. ჰიპოქსია სიცოცხლისთვის მნიშვნელოვან ორგანიზმებში გარდაუვალ ცვლილებებს იწვევს. ჟანგბადის უკმარისობის მიმართ ყველაზე მგძნობიარეა ცენტრალური ნერვული სისტემა, გული, თირკმლის ქსოვილი და ღვიძლი. ჰიპოქსიამ შეიძლება გამოიწვიოს გაუგებარი ეიფორიის შეგრძნება და თავბრუსხვევა

eek.gif


თავის ტვინში ჟანგბადის უკმარისობის მიზეზი შეიძლება იყოს?

ან კიდევ წამლების.

Posted by: gogoshvili94 22 Apr 2013, 10:30
3 დღეა გადაბმულად მახველებს, 4-5 დღის წინ სისუსტე ვიგრძენი, სიცხის აწევის წინ გრძნობა რომაა, გაჟრიალებს და რამე, ეგეთი გრძნობა მქონდა, მაგრამ სიცხემ არ ამიწია, სურდო მქონდა მარტო + სისუსტე, თვალები მიხურდა თითქოს... 3 საათზე ძლივს დავიძინე და 5-ზე გამეღვიძა და ვეღარ დავიძინე... იმ დღის მერე მახველებს, ხან ძლიერად, ხან გადამივლის, ღამით ძილის დროს მივლის ხოლმე,ძილშიც არ ვახველებ, ანუ როცა მძინავს კარგად ვარ.(ლაითად მძინავს, ხველება რომ ამიტყდეს გამეღვიძება ეგრევე).. დილით ვიღვიძებ და ხველება მეწყება ისევ თან ნახველი მაქვს თითმის ყველა დახველებაზე.
ეხლა შედარებით გამიარა, არ მახველებს, მაგრამ ისევ დამეწყება ალბათ ცოტა ხანში, სურდო აღარ მაქვს. ხოოდა რამე წამალი მირჩიეთ ხველების, გთხოვთ. ვიფიქრე 2-3 დღეში გამივლის მეთქი, მაგრამ არა და მომბეზრა რა და მოშორება მინდა. ექიმთან წასვლაც მეზარება.
ფარინგოსეპტი როა საწუწნი "წამალი", ეგ მოვწუწნე 3ცალი და მივხვდი რო ტრუხა იყო, კანფეტი უფროა, მგონი... რამე კარგი წამალი მირჩიეთ რა გთხოვთ... smile.gif
* * *
მგონი გამიარა, აღარ შეწუხდეთ. smile.gif

Posted by: rock in rose 22 Apr 2013, 19:12
play it fucking loud
QUOTE
თავის ტვინში ჟანგბადის უკმარისობის მიზეზი შეიძლება იყოს?

ან კიდევ წამლების.


კი, შესაძლებელია, განსაკუთრებით თავის ტვინში ჟანგბადის უკმარისობა, ეგაა ჰიპოქსია

Posted by: newsgirl 24 Apr 2013, 15:33
გაციებისას რა უნდა ქნა ადამიანმა? 2 დღეა გაციებული ვარ, სულ ჩაის ვსვამ და ტაიქოლდს. სიცხე მაქვს 37.

Posted by: Solveig 24 Apr 2013, 16:24
მე მაქვს ასეთი კითხვა. პერიოდულად მარცხენა მხარეს, ფერდში ან ოდნავ ქვემოთ-თეძოს ძვლის ქედთან ვგრძნობ კუნთის "ფეთქვას". რა შეიძლება, რომ იყოს? შეიძლება, აორტის პულსაცია გადაეცემოდეს?

Posted by: merabiHaker 24 Apr 2013, 16:46
სალამი მოდი აქაც ვიკითხვა...

ექიმა მარტივი ჰერპესი დამიდგინა. გამომიწერა კრემი ДЕКЛОСИД DECLOSID. ხოდა წამლის აღწერაში წავაწყდი:
ПРОТИВОПОКАЗАНИЯ:
простой герпес;

თქვენ რას მირჩევთ? შეიძლება ვიხმარტო?

Posted by: Solveig 24 Apr 2013, 19:04
merabiHaker

ჰერპესი თუ გაქვს, ეგ მალამო რაღატომ დაგინიშნა?

Posted by: mirkani 25 Apr 2013, 15:14
თუ ვინმემ იცით მარინა ვარდიმიადის კოორდინატები, ძალიან გთხოვთ მომწერეთ

Posted by: rainbow jeanne 26 Apr 2013, 18:22
გამარჯობა,
თქვენი რჩევა მჭირდება.
დედაჩემს აწუხებს მაღალი წნევები,200-მდე ადის ამ ბოლო დროს. დამარეგულირებლებს სვამს,მაგრამ ვფიქრობ რომ ექიმის კონსულტაცია მაინც საჭიროა.მირჩიეთ ნორმალური ექიმი,ვისთან მივიყვანო რა.

Posted by: mariooo 30 Apr 2013, 11:06
არ ვიცი კონკრეტულად რა პროფილის ექიმს უნდა დავუსვა ეს შეკითხვა და ამიტომ აქ დავწერ: 2 დღეა ვგრძნობ რომ თავზე -კისრის ზემოთ აქეთ-იქით რომ ძვლებია (ხელის დადებით სიმრგვალე რომ იგრძნობა ცოტა ) მარჯვენა მხარეს უფრო კოპივით გამიხდა და ხელს რომ ვიდებ მტკივასავით და მეშინია რა უნდა იყოს? sad.gif( სხვა არანაირი ჩივილი არ მაქვს უბრალოდ ძაან მეუცნაურება და მეშინია sad.gif

Posted by: datobaxutashvili 30 Apr 2013, 22:08
ესეიგი რა მაინტერესებს არ ვიცი იქ ვპოსტავ სადაც უნდა ვპოსტავდე მარა მაინც ვარ სიმაღლეში 174 სმ მარა მინდა სადღაც 185 მდე რო გავიზარდი და რა ხერხებს უნდა მივმართო ?? ააა ხო 16 წლის ვარ smile.gif და კიდე მშობლებზე ცოტათი მაღალიც ვარ და არის რაიმე შანსი რო მაქამდე გავიზარდო???

Posted by: bigmaster 3 May 2013, 18:20
გამარჯობათ, ზაფხული რო მოდის თავი მტკივდებას და მაინტერესებს ტემპერატურის მომატება წნევასთა კავშირში არის? ანუ ზაფხულში მაღლა იწევს წნევა თუ პირიქით? ან რამე გავლენა თუ აქვს მაინტერესებს

Posted by: rock in rose 3 May 2013, 19:48
bigmaster

კი, ტემპერატურის მატება სისხლის წნევას მატებს და შესაძლბელია თავის ტკივილი ამის გამოც იყოს.

Posted by: COCOA 15 May 2013, 12:51
უკაცრავად და პირში მუდმივად მაქვს უსიამოვნო სუნი, როგორც მითხრეს დიდი შანსია რომ ნაღველის ბრალია იყოს. ამ შემთხვევაში თერაპევტს უნდა მივაკითხო?? იქნებ მიპასუხოს ვინმე კომპეტენტურმა ძალიან მაწუხებს. მადლობა

Posted by: Delirium 16 May 2013, 04:00
ხშირად ვსუქდები და ვხდები და სტრიები გამიჩნდა. ვის უნდა მივაკითხო საკონსულტაციოდ drug.gif

Posted by: ninilion 17 May 2013, 12:26
ჭარბი ნერწყვდენა რამ იცის?ამ ბოლოდროს ლამის დავიხრჩო... sad.gif

Posted by: BadbadGirl 18 May 2013, 11:38
რკინის შემცველობა სისხლში რამდენია ნორმა?
წინასწარ მადლობა პასუხისათვის.

Posted by: ninilion 23 May 2013, 23:46
გამარჯობათ. ვვარჯიშობ და განსაკუთრებით სირბილის დროს ღვიძლის არეში ტკივილს ვგრძნობ და ჭარბი ნერწყვდენა მაქვს( ეხოსკოპიით ღვიძლში ჰემანგიომა მაქვს). შეიძლება დატვირთვამ რამე პრობლემა გამოიწვიოს?

Posted by: Solveig 24 May 2013, 02:35
BadbadGirl
http://en.wikipedia.org/wiki/Iron_tests
http://en.wikipedia.org/wiki/Serum_iron

ninilion
ლოგიკურად, ჭარბი ნერწყვდენა საჭმლის მომნელებელი სისტემის რაღაც პრობლემასთან ნამდვილად იქნება კავშირში.
http://www.mayoclinic.com/health/liver-hemangioma/DS01125
აბა აქ ნახე ღვიძლის ჰემანგიომაზე..
აქ წერია, როცა ტკივილი ახლავს, ექიმმა უნდა ნახოსო.
მე მგონი, აჯობებს, ცოტა ხანს თავი შეიკავო სირბილისაგან და ექიმს გაესინჯო.

Posted by: ninilion 24 May 2013, 11:48
QUOTE (Solveig @ 24 May 2013, 02:35 )

აბა აქ ნახე ღვიძლის ჰემანგიომაზე..
აქ წერია, როცა ტკივილი ახლავს, ექიმმა უნდა ნახოსო.
მე მგონი, აჯობებს, ცოტა ხანს თავი შეიკავო სირბილისაგან და ექიმს გაესინჯო.

გავესინჯე. ადრე სირბილის გარესშედაც მტკიოდა ისე რომ ვერ დავდიოდი საერთოდ და რაფაქოლინ-ც დამინიშნა,ღვიძლს მოეხმარებაო... არაფერი შედეგი არ ქონია.facepalm.gif არ ვიცი ვის მივმართო უკვე თუნდაც გასასინჯად. მადლობა ჩემო კარგო პასუხისთვის

Posted by: Solveig 24 May 2013, 12:45
ninilion

QUOTE
ადრე სირბილის გარესშედაც მტკიოდა ისე რომ ვერ დავდიოდი საერთოდ


და, ტკივილის მიზეზად ჰემანგიომა დასახელდა? იქნებ ნაღვლის ბუშტის პრობლემა გაქვს? გამოკვლეული ხარ?

რაფაქოლინი ნაღვლის გამომუშავების და დენის ნორმალიზაციას უწყობს ხელს, თუმცა მდგომარეობას გააჩნია.

* * *
ექიმს რაც შეეხება, გასტროენტეროლოგი გჭირდება და თემა არის მაგაზე. გადახედე, იქნებ იპოვო ვინმე....

http://forum.ge/?f=43&showtopic=33563200&st=270

Posted by: Natuka NGN 24 May 2013, 20:24
ninilion
ღვიძლის ჰემანგიომა გარკვეული დროის სემდეგ საოპერაციოა თუ მას ტკივილი ახლავს თან. ჩემი ემგობარი ექიმი ჩამოავ ამ დღეებში დავეკითხები და გადაგამისამართებ სად უნდა მიხიდე.

Posted by: rock in rose 24 May 2013, 23:27
ninilion
QUOTE
ჭარბი ნერწყვდენა რამ იცის?ამ ბოლოდროს ლამის დავიხრჩო...


დიდი რაოდენობით, თხიერი ნერწყვის გამოყოფა პარასიმპატიკური ნერვული სისტემის დარღვევამ იცის.

Posted by: ninilion 25 May 2013, 20:01
QUOTE (rock in rose @ 24 May 2013, 23:27 )
ninilion
QUOTE
ჭარბი ნერწყვდენა რამ იცის?ამ ბოლოდროს ლამის დავიხრჩო...


დიდი რაოდენობით, თხიერი ნერწყვის გამოყოფა პარასიმპატიკური ნერვული სისტემის დარღვევამ იცის.

მაგას რით ვუმკურნალო? eek.gif მადლობა გამოხმაურებისთვის

Posted by: Solveig 25 May 2013, 21:04
ninilion
QUOTE
მაგას რით ვუმკურნალო?


ჯერ მიზეზი უნდა დადგინდეს. ასე ხომ ვერ იმკითხავებ კაცი.

ნერწყვდენა რეფლუქსის (ანუ, როცა გარკვეულ მიზეზთა გამო კუჭის და საყლაპავის კუნთების პერისტალტიკა უკუღმა მიდის) დროსაც არის ხოლმე. მაგალითად, ღებინების წინ ადამიანს ყოველთვის აქვს ჭარბი ნერწყვდენა. ასევე, ჭიებით ინვაზიასაც ახასიათებს.

რაკი ტკივილიც ერთვის ამ ყველაფერს, ფაქტია, საჭმლის მომნელებელი სისტემის რაღაც პრობლემა გაქვს. შეიძლება, შენს ჰემანგიომასთანაც იყოს კავშირში. შეიძლება, არც იყოს. მოკლედ, წესიერი გასტროენტეროლოგი გჭირდება და გამოკვლევები.

Posted by: Natuka NGN 26 May 2013, 02:31
ninilion
რა ზომებია შენი ჰემანგიომა? უ სეგიძლია მიტხარი ზუსტი ზმნები და მდებარეობა.

Posted by: ninilion 26 May 2013, 12:03
Solveig
კარგი, გავითვალისწინებ. მადლობა kiss.gif
Natuka NGN
არჩილ ახმეტელთან გავიკეთე 2007 ში და დაწერა : მარჯვენა წილში მეშვიდე სეგმტში ფიქსირდება ჰიპერექოგენური წარმონაქმნი- ჰემანგიომა დიამ: 16 მმ. მაშინ არ მაწუხებდა დიდად ღვიძლი, ეხლა მაწუხებს უფრო. 2012წლის დეკემბერში კლინიკა კურაციოში გავიკეთე და იქ დაწერეს კეროვანი დაზიანება მერვე სეგმენტში,მკაფიოდ კონტურირებული,ერტგვაროვანი სტრუქტურის უბანი ზომით 19 მმ- კაპილარული ჰემანგიომა

Posted by: kotora1993 27 May 2013, 12:02
დილით როგორც გავიღვიძებ , (უძილობას არ ვუჩივი) ვრგძნობ კისრის დაჭიმულობას , წნევის აწევას (ვატარებ დაბალ წნევას) როცა ცუდად ვგრძნობ თავს ვიზომავ წნევას და მაქვს 140 , 90 ზე ვარ 36 წლის მამაკაცი . რას შეიძლება ეს გამოეწვია იქნებ დამეხმაროთ ვინმე?

Posted by: Antea 2 Jun 2013, 22:23
გამარჯობათ, დეიდაშვილს (ბიჭია) 2 წლის განმავლობაში სამჯერ ჰქონდა ფილტვების ანთება და ბრონქიტი, ვხვდები რომ ეს არ არის კარგი, სამივეჯერ ექიმმა ერთიდაიგივე დანიშნულება მისცა, რა გართულება შეიძლება მოჰყვეს ამ სიტუაციას?

Posted by: sulana 7 Jun 2013, 20:28
მუდმივი ძილიანობა დაღლილობა სახსრების ტკივილი მოთენთილობა თავის ტკივილი რისი ბრალი შეიძლება იყოს ?
(არ ვმუშაობ მთელი დღე სახლში ვარ რა უნდა მღლიდეს )

Posted by: rock in rose 8 Jun 2013, 11:24
sulana

ფარისებრ ჯირკვალს თუ შეიმოწმებდით კარგი იქნებოდა.

წონაშიც ხომ არ მოგიმატიათ?

Posted by: sulana 8 Jun 2013, 17:16
QUOTE (rock in rose @ 8 Jun 2013, 11:24 )
sulana

ფარისებრ ჯირკვალს თუ შეიმოწმებდით კარგი იქნებოდა.

წონაშიც ხომ არ მოგიმატიათ?

არა პირიქით რაც ესე ვარ იმის შემდეგ 3-4 კილო მაქ დაკლებული

Posted by: felmina 8 Jun 2013, 20:22
გამარჯობათ
ასკორუტინზე იცით რამე?
რატო ნიშნავენ? ანუ როგორ კატეგორიას უნიშნავენ მისაღებად?

Posted by: rock in rose 8 Jun 2013, 22:54
sulana

ჯობია მიხვიდეთ ენდოკრინოლოგთან

felmina

ასკორუტინი არის იგივე C და P ვიტამინი, ოღონდ კომბინაციაში. სისხლძარღვის კედელს ამაგრებს და სისხლდენებისგან იცავს.

Posted by: felmina 9 Jun 2013, 00:44
QUOTE
felmina

ასკორუტინი არის იგივე C და P ვიტამინი, ოღონდ კომბინაციაში. სისხლძარღვის კედელს ამაგრებს და სისხლდენებისგან იცავს.

მადლობა smile.gif

Posted by: Summer_Rain 9 Jun 2013, 19:38
ერთი უცნაური კითხვა მაქვს

ორსულად ვარ და
ჭიპი რაც ღიპა მეზრდება ნელ-ნელა გადის სიგანეში,
და დღეს შევამჩნიე რაღაც ჭუჭყივით ჩამჯდარი და მოვიცილე რაც შეიძლება ფაქიზად
მერე სპირტით და ეწ ზელიონკით დავიმუშავე
მაგრამ მაინც შემეშინდა რაღაცნაირად
ინფექციას ხოარ შევიოტანდი
ბაოს არ ავნოს რამე

Posted by: Delirium 9 Jun 2013, 21:05
ტუჩის მარცხენა ქვედა ბოლოში რაღაც გაიბუშტა, არც მტკივა არც წყლულია. მეგობარმა გაურეცხავი რამე თუ ჭამე შეიძლება მაგის ბრალი იყოსო. მეშინია ჰერპესი არ იყოს. როგორ უნდა მივხვდე ჰერპესია თუ არა?

Posted by: Solveig 9 Jun 2013, 22:26
Summer_Rain

მე მგონი, ტყუილად ნერვიულობ.

Delirium
QUOTE
ტუჩის მარცხენა ქვედა ბოლოში რაღაც გაიბუშტა, არც მტკივა არც წყლულია.

ექიმს აჩვენე. შეიძლება, ჰერპესი იყოს, შეიძლება-ქეილიტი , ანუ ტუჩის კუთხის ანთება, რომელიც ვიტამინების/მინერალების ნაკლებობას მოსდევს ხოლმე.
თუმცა, საბოლოოდ ორივე წყლულია, ბუშტუკი არაა...

Posted by: Summer_Rain 10 Jun 2013, 10:07
Solveig

მადლობა smile.gif
მოგვიანებით
წყალბადის ზეჟანგიც ჩავისხი მაგრამ არ აშუშხუნდა, ეი
ინფექცია არიყო ხომ?
ორსულობის დროს ხომ არაფერს უშლის წყალბადის ზეჟანგის გამოყენება?

Posted by: taamo 10 Jun 2013, 13:50
გამარჯობა. როგორც ვატყობ მაქვს ტერფის გამრუდება და მგონი ხერხემლისაც, რომელ ექიმს უნდა მივმართო?

Posted by: iakuna79 16 Jun 2013, 00:05
გამარჯობათ
ჩემმა მეგობარმა, რომელიც 36წლისაა და11წლის შვილი ყავს, 3-4 წლის წინ, ამოიღო სპირალი და მეორეს გაჩენა გადაწყვიტა, მაშინ დაეწყო სწორედ პრობლემები, 3-4 დაორსულდა მაგრამ ნაყოფი ვერ განვითარდა(ანემბრიონი-მგონი ასე ქვია), ციკლი დაერღვა, 5-6 თვე არ მოდის ხოლმე, აქვს სახსრების ტკივილი, განსაკუთრებით ფეხების, ხშირი შარდვა . მოიარა უამრავი ექიმი smile.gif და თითქოს ყველაფერი წესრიგში აქვს-ორგანული არაფერი-მკერდი,საკვერცხე-საშვილოსნო,ღვიძლი,ფარისებრი ყველაფერი წესრიგში. ჰორმონები-TSH ,FT4,პროლაკტინი ნორმაში აქვს, კალიუმი ნომაში მხოლოდ დენსიმეტრიამ მინერალების ნაკლებობა აჩვენა ფეხის ძვლებზე, კუაგულოგრამა-ნორმაში, ინსულინი და ჰომო ინდექსი აქვს ცოტა მომატებული, რევმატიული სინჯები ზღვართან ახლოს მაგრამ ნორმაში, თირკმლის ფუნქციებში ცოტა დარღვევა - საერთო ცილა იყო დაბალი და კრიატინინი, დანარჩენიც ქვედა ზღვართან ახლოს ნორმაში. სისხლის საერთო ანალიზმა დაბალი ლეიკოციტები(3.5) და მაღალი ეიზოფილები (8)
ახლა რა მინდა გკითხოთ, ბოლო მისი ექიმი არის ენდროკრინოლოგ-გინეკოლოგი. რა შეგიძლიათ მითხრათ ამ ანალიზებთან დაკავშირებით და ვის უნდა მივმართოთ? გთხოვთ შემეხმიანეეეთ

Posted by: klounada 16 Jun 2013, 15:58
რა მინდა გკითხოთ: ჩემს შვილს,7 წლის ნიკუშას,იანვრის შემდეგ აწუხებს მშრალი ხველა,ქრონიკული სახით(ხშირად ახველებს ან უფრო სწორად ჩაახველებს ხოლმე,თითქოს ყელში რაღაცამ შეაწუხა).დღეს საჭმლის ჭამის დროს გადაყლაპვა გაუჭირდა,წყალი მიაყოლა და ძალიან შევშინდი.ხვალ მინდა წავიყვანო ექიმთან და ზუსტად ვერ ვხვდები ვისთან უნდა მივიყვანო...ვინმე ხომ არ ხართ ამ საკითხში გამოცდილი?გთხოვთ მირჩიეთ რამე...

Posted by: mtasiqiteli 19 Jun 2013, 20:33
თერაპევტებო ასეთი სიმპტომები მაქვს:
რამდენიმე დღეა ხშირი შარდვის მოთხოვნილება, ყრუ ტკივილი ქვედა არეში,
გადავიღე ექოსკოპია და პროსტატის ანთება გამოირიცხა,
შეიძლება რომ შარდის ბუშტის ანთება იყოს?


Posted by: midnight-madness 20 Jun 2013, 05:32
დამეხმარეთ რა სასწრაფოდ :|
მეორე დღეა, სიცხე მაქვს, 37,5-38,5 გრადუსი. ყველა კუნთი მტკივა, სუსტად ვარ და ვერაფერს ვაკეთებ. საღამოობით არ მაქვს სიცხე და დილიდან ხელახლა იწყება..

ამას აიტანდა კაცი, მაგრამ გუშინ საღამოდან რაღაც ბუშტუკები შევამჩნიე, უფერო სითხეა. თავიდან კისერზე მქონდა, ახლა მთელ ტანზე გამივრცელდა და რაც ყველაზე დიდი უბედურებაა, სახეზეც.. სკდება და საშინლად მეწვის, წითელ წინწკლებს მიტოვებს.. ვიცი უაზრობაა დილის 6ის ნახევარზე აქ ამის წერა, მაგრამ იქნებ ვინმე კეთილი ადამიანი გამოჩნდეს და რამე მირჩიოს, რა ვქნა, სანამ გათენდება და ექიმთან წავალ..

Posted by: simebi 25 Jun 2013, 18:11
რამე ძლიერ საძილე საშუალებას ვერ მირჩევთ? ძილის რეჟიმიდან ამოვვარდი და ჩემით ვერ ჩავდექი ვერანაირად.

Posted by: Salome_L 26 Jun 2013, 16:55
სისხლის ანალიზი გავიკეთე თან ინგლისურად მომცეს პასუხები, რატო ვერ გავიგე.

4 პარამეტრი ამომიგდო ნორმის იქით იქნებ მითხრათ რას ნიშნავს ესენი..


MCH 33.5 ნორმა 28-33

MCHC 36,3 ნორმა 33-36

neutrophils 48 % ნორმა 50-75

lymphocytes 41,7 % ნორმა 25-40



Posted by: kethrin-eka 27 Jun 2013, 14:53
midnight-madness
ეგ ჩუტყვავილა იქნება, ექიმს მიმართეთ, ალბათ უკვე მიმართეთ biggrin.gif

taamo

ორთოპედთან... yes.gif

Posted by: midnight-madness 3 Jul 2013, 13:24
kethrin-eka

კი, ჩუტყვავილა იყო smile.gif
რომ მახსენდება, რა სასოწარკვეთილმა დავწერე ეს პოსტი, ოოო biggrin.gif
გადამიარა უკვე თითქმის, მადლობა გამოხმაურებისთვის smile.gif

Posted by: Domaci 6 Jul 2013, 16:47
1 კვირაა ყელში თითქოს რაგაც მაქვს, ღამე რომ ვიძინებ ხანდახან თითქოს მაწვება და სუნთვას მიკეტავს, დახველების მერე ცოტახანს გამივლის. ნერწყვის გადყლაპის დროს ვგრძნობ რომ რაღაც არის. რა შეიძლება იყოს, მწეველი ვარ

ან სად მივიდე სად გავესინჯო?

Posted by: rock in rose 6 Jul 2013, 17:54
Domaci

მხოლოდ ღამე გაწუხებს ძილის წინ?


Posted by: keen 6 Jul 2013, 17:56
რბილი ქსოვილების ისეთ ონკოლოგს თუ მიმასწავლის ვინმე, ვინც დიაგნოსტიკის და მკურნალობის ცივილურ(თანამედროვე) მეთოდებს იყენებს? უფრო კონკრეტულად ხალების დიაგნოსტიკის...

Posted by: Domaci 6 Jul 2013, 18:12
rock in rose

დღის განმავლობაში ძაან იშვიათად რომ შემაწუხოს , თითქოს ყელი ძალიან გამიშრაოო აი რაღაც თითქოს რომ გეღიტინება და გიდნა რომ ჩაახველო . ღამე ერთი 2-ჯერ წამომაგდო ლოგინიდან, თითქოს გადამიკეტა სასუნთქი გზა

Posted by: nukabadzi 6 Jul 2013, 19:57
რაღაც ძალიან უცნაური შEგრძნება მაქ არ მტკივა მარა ძალიან შემეშინდა არ ვიცი რა არი
აი მარცხენა მხარეს სირბილის მერე რომ იცის ''ჭვალის ამოვარდნა'' ისე მაქ თითქოს მაძიმებული მაქ რაღაც ადგილი შიგნით და სიარულისას მაწუხებს sad.gif

Posted by: rock in rose 6 Jul 2013, 21:23
Domaci

ძალიან ზოგადი სიმპტომია, ბევრ რამეს შეიძელება ახასიათებდეს, ამიტომ კარგ თერაპევტთან მიდი smile.gif

Posted by: Gentle_Girl 6 Jul 2013, 21:35
გამარჯობა smile.gif
"ლამინარიას" ზუსტი დოზა და მიღების წესი როგორია?
ინსტრუქციაზე გაურკვევლად წერია, 1-2 ტაბლეტი დღეში ჭამის წინ, მხოლოდ ეს წერია.
http://www.aversi.ge/annotations.php?id=73&lang=geo&drug_id=9669


პ.ს. შემეშალა ვითომ თემა? smile.gif

Posted by: bfjribhs 8 Jul 2013, 23:29
ქოლეცისტიტი მაქვს
ფიზიკურად რო ვიტვირთები მოწამვლის შეგრძნება მიჩნდება ხოლმე და რატო ხდება ასე ხო ვერ მეტყვით

Posted by: buub 11 Jul 2013, 17:03
უმიზეზოდ ტემპერატურის მომატება რამ შეიზლება გამოიწვიოს 40 გრადუსამდე (რათქმაუნდა მიზეზი ყველაფერს აქვს მაგრამ ამ შემთხვევაში გამოვლენილი არაფერი არააა)? არც სურდო მაქ არც ხველა მეორე დღეა სახსრები მტკივა ძალიან და ეს ტემპერატურაც არ იწევს (ფილტვები სუფთაა,გამონაყარი არ მაქ ანუ წითელა არაა) sad.gif

Posted by: bagda 17 Jul 2013, 19:50
გამარჯობათ! ხალხო! დღეს სიცხე მომცა არ ვიცი რატომ (კონდენციონერს ვაბრალებ, პრავაზე მივედი ცხელი ქუჩიდან უცებ სიგრილეში), ხოდა რო დავბრუნდი ოდნა გულისრევის შეგრძნება და კეფის ნაწილში ტკივილი, მცირე ფეთქვასავით ვიგრძენი. სიცხე 38 მქონდა. მერე დავაბრალე ამ მზეს და კონდენციონერს ცოდა ცოტა წამოვწექი და ცივი ბინტები დავიდე შუბლზე. სიცხემ 37,5-მდე დამიწია. მერე ჩამეძინა და სტუმარმა უცებ გამაღვიძა და სიცხე 38,2 მქონდა ად უცებ რო წამოვდექი კეფის არეში მფეთქავი ტკივილი მქონდა. ნუ ეს ტკივილი მალე შესუსტდა, მარა სიცხე არა. ხოდა პარაცედამოლი და ციტრამონი დავლიე. ახლასიცხემ დმიწია თავიც აღარ მტკივა, მარა საღამოს ისევ რო მტკივდეს რა დავლიო ან თქვენი ვარაუდით რა შეიძლება იყოს? მზის დარტყმა ან გაციება? გებ მითხრათ 2kiss.gif

Posted by: bfjribhs 17 Jul 2013, 20:41
რისი სიმპტომია ფიზიკური დატვირთვის დროს ტოქსიკოზი.
აააააბა ვინ მეტყვის.

Posted by: atila92 21 Jul 2013, 17:27
ორი კვირაა უკვე ბრონქიტი მაწუხებს, ექიმმა მედიკამენტები დამინიშნა და მაინტერესებს ვირისტერფას ნაყენის მიღება თუ შეიძლება მედიკამენტებთან ერთად?

Posted by: audrey-girl 22 Jul 2013, 10:17
bfjribhs
რაში გამოიხატება მოწამვლის შეგრძნება?

Posted by: bfjribhs 22 Jul 2013, 12:00
audrey-girl
ცუდად ვარ სუსტად. მეტი არაფერი. მერე ბევრ წყალს ვსვამ და მივლის ნახევარ-1 საათში. როგორც ზემოთ დავწერე ქოლეცისტიტი მაქვს. ჩემი დილეტანტური აზრით რომ ვვარჯიშობ გაფუჭებული ნაღველი გამოიდევნება ბუშტიდან და იმიტომაც ვიწამლები. არა?

Posted by: ninininiko 23 Jul 2013, 23:32
გამარჯობათ.დამეხმარეთ რა.
ესენი რას ნიშნავს? ASLO anti-TPO PRL FSH LH f-hesto ,homaეს კვლევები პაციენტი დაუნიშნეს დედას.მე არ ვარ თბილისში და მაინტერესებს და ისტორიაში ეს რას ნიშნავს tbc

Posted by: misha-htc 25 Jul 2013, 08:47
გამარჯობათ მაქვს ჭინჭრის ციება (დერმატოლოგმა მითხრა გაქვსო და უნდა დავუჯერო მეტი გზა არ მაქვს) ჰოოდა როგორ ჯობია დერმატოლოგთან გავაგრძელო მკურნალობა თუ ალერგოლოგს მივმართო? რომლის კომპეტენციაა ეს დაავადება? დიდი მადლობა პასუხისთვის!

Posted by: rock in rose 25 Jul 2013, 18:09
misha-htc

მე გირჩევდი ალერგოლოგისთვის მიგემართა

Posted by: bfjribhs 27 Jul 2013, 16:45
ოეეეეეეეეეეეეეეეეეეეეეეე!!! sa.gif

Posted by: rock in rose 28 Jul 2013, 13:38
bfjribhs

მწარე გემოს შეგრძნება ხომ არ გაქვს პირში?

ქოლეცისტიტს მკურნალობ?

Posted by: Imp_ 29 Jul 2013, 17:34
გამარჯობა, ერთი კითხვა მაქვს ექიმთან და კონკრეტულად რომელს ვკითხო არ ვიცი, და ამიტომ თერაპევტს მივმართავ, ვარ 23 წლის გოგო, მეტნაკლებად ჯანმრთელი, ყოველ დილას ვსვამ ყავას, რომ არ დავლიო ვერ ვფხიზლდები და მემგონი დამოკიდებულებაც გამიჩნდა, მაგრამ როგორც ვიცი ყავას გამოაქვს კალცი ორგანიზმდან და კალციუმის ტაბლეტები რომ ვსვა ხოლმე თუა რეკომენდირებული? ან კალციუმ დე სამი რომაა ქალებისთვის? ან რა დოზით შეიძLება ამ კალციუმის მიღება?

Posted by: rock in rose 30 Jul 2013, 13:43
Imp_

პროფილაქტიკის მიზნით შეიგიძლია დალიო კალციუმ D3 ნიკომედი, დღეში 2 ცალი, დილა-საღამოს ჭამის დროს. ან დაღეჭე ან გადაყლაპე ცოტა წყალთან ერთად.

ეს 1 თვე მიიღე, ამის მერე კი ეცადე კალციუმით მდიდარი პროდუქტი მიირთვა დღის განმავლობაში, მაგალითად ხაჭო, ყველი, რძე.

Posted by: GOAT 30 Jul 2013, 18:16
აუ სად უნდა ვიკითხო არ ვიცი და თემის გახსნა მეზარება.
სიარულისას და მარჯვენა ფეხის მოძრაობისას მტკივა მუცლის უკიდურესი ქვედა მარჯვენა ნაწილი, ლამის საზარდულთან. ძალიან არა მაგრამ თIთო ნაბიჯზე თითო ტკივილი, ნემსის გაყრასავით, შემაწუხებელია. ხოდა რის ბრალი შეიძლება იყოს?

წოლის, ან დგომის, ან ჯდომის დროს არ მტკივა.

Posted by: rock in rose 30 Jul 2013, 19:22
GOAT

ხელის დაჭერით გტკივა?

Posted by: Imp_ 30 Jul 2013, 22:21
rock in rose

დიდი მადლობა, ჩავიტარებ ერთ თვიან კურს, რძის ნაწარმს ვიღებ კი, და გავგრძელებ smile.gif

Posted by: GOAT 30 Jul 2013, 23:47
QUOTE (rock in rose @ 30 Jul 2013, 19:22 )
ხელის დაჭერით გტკივა?

პრინციპში არა.
სულ ოდნავ აშვებისას.

Posted by: rock in rose 31 Jul 2013, 11:08
GOAT

არის აპენდიქსის მტკივნეულობის შესამოწმებელი წერტილები, ქირურგთან მიდი რომ აპენდიციტი გამოირიცხოს.

Imp_

არაფრის 2kiss.gif

Posted by: Sargerass 1 Aug 2013, 23:38
Mეგობრებო არ ვიცი სად ვიკითხო დამეხმარეთ რა. Xომ ვერ მეტყვით საიდან ჩნდება ჰემანგიომა? ჩემს 22 დღის პაწაწინას დაბადებიდან ააქვს ჭიპთან.ან რა შეიძლება გაკეთდეს რომ არ გაიზარდოს?


Dიდი მადლობა წინასწარ.

Posted by: eksdo 2 Aug 2013, 02:18
QUOTE (Sargerass @ 1 Aug 2013, 23:38 )
Mეგობრებო არ ვიცი სად ვიკითხო დამეხმარეთ რა. Xომ ვერ მეტყვით საიდან ჩნდება ჰემანგიომა? ჩემს 22 დღის პაწაწინას დაბადებიდან ააქვს ჭიპთან.ან რა შეიძლება გაკეთდეს რომ არ გაიზარდოს?


Dიდი მადლობა წინასწარ.

პირველ რიგში დარწმუნდით, რომ ნამდვილად ჰემანგიომაა და რა ფორმისაა. ის არცთუ იშვიათად ეშლებათ სხვადასხვა წარმონაქმნებთან. ჰემანგიომა სისხლძარღვოვანი კეთილთვისებიანი სიმსივნეა. ძალიან ხშირია ბავშვებში. არ არის საშიში დაავადება თუ დროულად და სწორად მიმართავთ ექიმს და მანამდე გაუფრთხილდებით. ეცადეთ ამ ადგილის ტრავმირება არ მოახდინოთ. კარგია, რომ სასიცოცხლო ორგანოებთან ახლოს არ არის. მკურნალობის რამდენომე მეთოდი არსებობს. ეს უკვე მკურნალმა ექიმმა უნდა გადაწყვიტოს.

ბავშვი პედიატრთან მიიყვანეთ და ის დაგაკვალიანებთ.
* * *
QUOTE (Imp_ @ 30 Jul 2013, 22:21 )
rock in rose

დიდი მადლობა, ჩავიტარებ ერთ თვიან კურს, რძის ნაწარმს ვიღებ კი, და გავგრძელებ smile.gif

რომ იცოდეთ რძის ნაწარმით კალციუმის მიღება რთულია. ჩვენი ორგანიზმი მხოლოდ ძალიან მცირე ნაწილს ითვისებს. მით უმეტეს თუ ნატურალური რძე არ არის. ასე, რომ აუცილებელიც კია დამხმარე საშუალებებით კალციუმის მიღება. მითუმეტეს თუ მისი ნაკლებობის ნიშნები გაქვთ. ფრჩხილები, თმა და კბილები ხილულად რეაგირებენ მის ნაკლებობაზე. თუ რაიმე მკვეთრი პრობლემა არ გაქვთ ამ ორგანოებთან დაკავშირებით, მაშინ ისეთი საშიშიც არაფერია. მაგრამ პროფილაქტიკა ძალიან კარგია smile.gif

Posted by: rock in rose 2 Aug 2013, 11:27
eksdo

გასაგებია, მაგრამ სულ კალციუმის პრეპარატს ხომ ვერ მიიღებს, მისი მიღება რაღაც პერიოდი შეძლებს, შემდეგ კი რძის პროდუქტებს უნდა მიეძალოს.

Posted by: Sargerass 2 Aug 2013, 15:00
eksdo
დიდი მადლობა.. 2მა პედიატრმა გვითხრა, რომ ჰემანგიომაა.. დააკვირდით თუ გაიზარდა გვითხარიო, მარა ბავშვი,რომ გაიზრდება და კანიც ისიც ხომ გაიზრდება..

აი 3 დღის იყო, რომ გადავუღე. კარგით, აუცილებლად მივიყვან კიდევ პედიატრთან. ძალიან დიდი მადლობა გამოხმაურებისთვის.

http://radikal.ru/fp/dba8fea8c88647a4b296cbba73ccd1d4

Posted by: NewsAvatar 2 Aug 2013, 19:19
http://forum.ge/?f=43&showtopic=34558894

Posted by: eksdo 2 Aug 2013, 22:11
QUOTE (Sargerass @ 2 Aug 2013, 15:00 )
eksdo
დიდი მადლობა.. 2მა პედიატრმა გვითხრა, რომ ჰემანგიომაა.. დააკვირდით თუ გაიზარდა გვითხარიო, მარა ბავშვი,რომ გაიზრდება და კანიც ისიც ხომ გაიზრდება..

აი 3 დღის იყო, რომ გადავუღე. კარგით, აუცილებლად მივიყვან კიდევ პედიატრთან. ძალიან დიდი მადლობა გამოხმაურებისთვის.

http://radikal.ru/fp/dba8fea8c88647a4b296cbba73ccd1d4

დარწმუნებული ვარ თქვენი მკურნალი ექიმი ჩემზე კარგად ერკვევა ამ საკითხში და მას დაუჯერეთ. გაზრდაზე იმიტომ გითხრათ, რომ პროგრესირება არ მოხდეს დაავადების. ისე ჰემანგიომას უნარი შესწევს გაიწოვოს. ოღონდ ეს პროცესი ცოტა დროში გაწელილია. ასე რომ თქვენი ვალია ყურადღებიანი მშობელი იყოთ.

Posted by: eksdo 3 Aug 2013, 23:25
QUOTE (rock in rose @ 2 Aug 2013, 11:27 )
eksdo

გასაგებია, მაგრამ სულ კალციუმის პრეპარატს ხომ ვერ მიიღებს, მისი მიღება რაღაც პერიოდი შეძლებს, შემდეგ კი რძის პროდუქტებს უნდა მიეძალოს.

ამას არც არავინ უარყოფს smile.gif რძე კარგია. მაგრამ განა მხოლოდ რძიდან იღებს ადამიანი კალციუმს?! არ დაგვავიწყდეს ხორცი, მწვანილი, ხილი და ა.შ. ადამიანი ახვადასხვა საჭმლიდან იღებს დღიურ ნორმას და არა მხოლოდ რძიდან. მაგრამ როცა კალციუმის დანახარჯი უფრო დიდია ვიდრე მიღება? როცა მისი ნაკლებობა აშკარაა? ასეთ შემთხვევაში აუცილებელია დამატებით პრეპარატების მიღება.

პროფილაქტიკა კაგია smile.gif მეც ვაპირებ მივიღო კალლცი დე მიუხედავად იმისა რომ ყავას საერთოდ არ ვსვამ smile.gif

Posted by: maia24 4 Aug 2013, 14:52
გამარჯობათ იქნებ მიშველოთ ,2 კვირის წინ ხელი ამტკივდა მარჯვენა იღლიიდან დაჭიმულობა მქონდა,მეგონა რამე მძიმე ავწიე და დამეჭიმა მაგრამ მერე იდაყვის ზემოთ რაგაც გამომივიდა თავი არა აქვს,მაგრამ ჯირკვალივიტაა გამაგრებული და მტკივნეულია,არ ვიცი ნაკბენია თუ ისე რამეა კარგა მოზრდილია ცენტრში კი რაგაც გაწიტლებულია ქირურგტან ვიყავი და მიტხრა ლიმფური ჯირკვლის ანტებააო და 5 ნემსი პეო და რაგაც წამალი დიგოლი დამინიშნა და სხივები 5 უკვე გავიკეტე იქნებ დამეხმაროტ?არაფერი მიშველა უფრო მტკივა

Posted by: eksdo 4 Aug 2013, 16:10
QUOTE (maia24 @ 4 Aug 2013, 14:52 )
გამარჯობათ იქნებ მიშველოთ ,2 კვირის წინ ხელი ამტკივდა მარჯვენა იღლიიდან დაჭიმულობა მქონდა,მეგონა რამე მძიმე ავწიე და დამეჭიმა მაგრამ მერე იდაყვის ზემოთ რაგაც გამომივიდა თავი არა აქვს,მაგრამ ჯირკვალივიტაა გამაგრებული და მტკივნეულია,არ ვიცი ნაკბენია თუ ისე რამეა კარგა მოზრდილია ცენტრში კი რაგაც გაწიტლებულია ქირურგტან ვიყავი და მიტხრა ლიმფური ჯირკვლის ანტებააო და 5 ნემსი პეო და რაგაც წამალი დიგოლი დამინიშნა და სხივები 5 უკვე გავიკეტე იქნებ დამეხმაროტ?არაფერი მიშველა უფრო მტკივა

კიდევ ერთ ექიმთან მიდით და იმის აზრიც მოისმინეთ.

Posted by: Calvados 10 Aug 2013, 11:20
7 თვის ბაია მყავს და სოფლის წიწილი თავის ძვლით რომ მოვუხარშო შეიძლება?

Posted by: rock in rose 10 Aug 2013, 20:01
Calvados

ეგ ამ თემაში იკითხეთ, პედიატრი შემოდის და გაგცემთ პასუხს

http://forum.ge/?f=97&showtopic=34546867&st=105

Posted by: lila2009 19 Aug 2013, 00:31
რამდენიმე თვის წინ ვიმშობიარე მეორედ.
მტკივა ბარძაყის თავი, მენჯებთან. განსაკუთრებულად დილით წამოდგომისას მიჭირს ფეხის გადმოდგმა და სიარული, და ასევე მთელი დღის განმავლობაში, თითქოს ვკოჭლობ.

რასთან შეიძლება იყოს დაკავშირებული? ვის მივმართო და რა გამოკვლევები დამჭირდება ? sad.gif


Posted by: Just_smile 23 Aug 2013, 00:52
კასანორმზე გაგიგიათ რამე? user.gif

Posted by: stm41 24 Aug 2013, 14:43
გამარჯობათ, მაინტერესებს დეფორმირებულ და ბილირეალური ნალექით სავსე ნაღვლის ბუშტს რა სიმპტომები შEიზლება ქონდეს?? ვინც იცის ძალიან გთხოვთ მომწერეთ

Posted by: EuRoPean 24 Aug 2013, 15:18
მოგესალმებით. რკინაზე გავიკეთე ანალიზი და მაინტერესებს ფერინტინი გამოჩდება? ტუ ფერიტინზე ცალკე კეთდება ანალიზი. ანუ ანემია არ მაკვს მაგრამ ხომ შეიძლება ანემიის გარეშე მიდიოდეს რკინად დეპიციტი და ეგ რომელ ანალიზში გამოჩნდება? ფერინტინის ანალიზში თუ რკინის? და რაგნსხვავება ამ ორ ანალიზში?
* * *
ანუ უფრო კონკრეთულად რო ვთკვა სიდეროპენული რკინა დეფიციტის დადგენის მიზნით, რომელი ანალიზი კეთდება?

Posted by: rock in rose 24 Aug 2013, 16:25
EuRoPean

არა ფერიტინზე ცალკე კეთდება ანალიზი. ფერიტინი არის რკინის შემნახველი ცილა, ზოგჯერ მისი დეფიციტია ხოლმე.


QUOTE
ანუ უფრო კონკრეთულად რო ვთკვა სიდეროპენული რკინა დეფიციტის დადგენის მიზნით, რომელი ანალიზი კეთდება?


რკინა და ფერიტინი ორივე

Posted by: EuRoPean 24 Aug 2013, 16:37
rock in rose
მარტო რკინის ანალიზში არ გამოჩნდება დეფიციტურია თუ არა ?
* * *
ანუ თუ სისხლის საერთო ანალიზი ჰემოგლობინი, ლეიკოციტები და ასე შმედეგ. ნორმაშია ფერიტინზე მაინც საჩიროა? მარტო რკინით ვერ შემოვიფარგლებით?

Posted by: rock in rose 24 Aug 2013, 17:05
EuRoPean

საჭიროა, მე გირჩევთ რომ გაიკეთოთ, მაშინ ნამდვილად სრული სურათი გექნებათ.

Posted by: EuRoPean 24 Aug 2013, 17:28
rock in rose
მადლობა რჩევისთვის. )

Posted by: rock in rose 24 Aug 2013, 17:31
EuRoPean

რისი მადლობა smile.gif

Posted by: elene9999 25 Aug 2013, 23:45
გამარჯობათ იქნებ დამეხმაროთ თითქმის 2 თვეა მაქვს საღამოს ესე 8 საათისთვის სიცხე 37.4 მაქსიმუმი თითონვე მიწევს დაბლაა სხვა არაფერი მაწუხებს თავიდან უბრალოდ ფაღარათი მქონდა და მაქედან დამეწყო სიცხეები ეს ფაღარათი აღარ მაქ ვიყავი ექიმებთან ფილტვებზეც სისხლიც შარდიც ყველაფერი ნორმაშია.. ძალიან ვნერვიულობ და მეშინია უკვე აღარ ვიცი დეპრესია მაქვს ეს 1 წელია პრობლემები მქონდა დაა ბევრი ვინერვიულეე მთელი წელი და შესაძლებელია მაგის ბრალი იყოს? ამ ინტერნეტში რავი რას აღარ ვკითხულობ შიდსმაც იცისოო და ვნერვიულოობ გათხოვილი ვარ და სისხლის საერთო ანალიზში არ იქნებოდა რამე ცვლილება ?

Posted by: N2O 30 Aug 2013, 12:28
არ ვიცი სწორ თემაში ვწერ თუ არა მაგრამ, კანდიდოზის დასადგენად რა სპეციალობის ექიმს უნდა მივმართო? და რომელ კლინიკაში იქნებ მითხრათ

Posted by: rock in rose 30 Aug 2013, 18:43
N2O

რისი კანდიდოზია გააჩნია

Posted by: eksdo 31 Aug 2013, 00:11
თუ კანის პრობლემაა დერმატოლოგს. თუ პირის ღრუს ან ტუჩების სტომატოლოგს. თუ ყურების ოტო-რინო-ლარინგოლოგს.

Posted by: maxuna14 4 Sep 2013, 22:52
დედა 80 წლისარი წევს რეანიმაციასჰი მარტის ტვესჰი ქონდა ინსულტი ჰემორარგიული და ივლისსჰი დაემარტო ფილტვების ანტება და მოვატავსეტ რეანიმაციასჰი რადგან სუნტქვის პრობლემა ქონდა ფილტვის პრობლემა აგარ არის და აპარატიდან ვერ იხსნება რა უნდა ვქნატ?

Posted by: GABRIELA 4 Sep 2013, 23:36
საშIნელი ქავილი მაქვს ხშირად კანზე მთელ სხეულზე ხანდახან.რისი Bრალია?

Posted by: Lacrimosa_ 7 Sep 2013, 14:06
გამარჯობათ,

ერთი-ორი თვეა, ზოგჯერ დილით მარცხენა თვალი თითქოს დაპატარავებულია და გარე კუთხით ქვევით არის დაქაჩული (შეგრძნება მაქვს ასეთი) და რამოდენიმე დღეა, ესეც დილით, ტუჩის მარჯვენა კუთხეს თითქოს ისე ფართოდ ვერ ვაღებ საუბრის დროს, როგორც ჩვეულებრივ.

არაფერს ვუჩივი, ვარ 30 წლის, მდედრობითი სქესის.
გასულ ზამთარს თხილამურებით სრიალისას დავეცი და თავი დავარტყი, თუმცა არც კი მტკენია (თვალები მიბრჭყვიალებდა დაახლოებით 10 წუთი), ექიმთანაც დამავიწყდა მისვლა. ახლა მეშინია, მაშინ თავი ხომ არ დავიზიანე


rock in rose
eksdo

Posted by: rock in rose 7 Sep 2013, 14:25
Lacrimosa_

ნევროლოგს უნდა ეჩვენოთ აუცილებლად

Posted by: Lacrimosa_ 7 Sep 2013, 14:33
rock in rose

მადლობა ოპერატიული პასუხისთვის
ნერვოზზე გაქვთ ეჭვი? smile.gif
ყველაზე კარგი გამოსავალია ჩემთვის smile.gif
იოლად მოვუვლი smile.gif

Posted by: rock in rose 7 Sep 2013, 14:52
Lacrimosa_

არა, ნევროზზე არა, თავი რომ დაარტყით შესაძლოა 12 წყვილი ნერვიდან, რომლებიც თავის ტვინიდან გამოდიან, ერთ-ერთი დაზიანდა. ნევროლოგი კი გასინჯვით მიხვდება რასთანა აქვს საქმე. საშიში არ არის smile.gif

QUOTE
მადლობა ოპერატიული პასუხისთვის


2kiss.gif

Posted by: Lacrimosa_ 11 Sep 2013, 20:14
rock in rose

ოჯახის ექიმმა გამიშვა ნევროლოგთან, ნევროლოგმა ელექტრომიოგრაფიულ კვლევაზე. 99%-ით დარწმუნებული ვარ რომ არაფერიაო, მაგრამ მაინც გამოვრიცხოთო.
ახლა პარასკევს ჩავეწერე და ვნახოთ smile.gif
ცუდადაა ჩემი საქმე? smile.gif

Posted by: rock in rose 11 Sep 2013, 20:55
Lacrimosa_

არა, პირიქით ძალიან კარგად ყოფილა საქმე რახან ასე თქვა ნევროლოგმა smile.gif

კვლევის მერე იტყვის დიაგნოზს

Posted by: Shine On 12 Sep 2013, 17:07
ჩემს მეგობარს ფრჩხილი გადატყდა გუშინ.
ფოტოზე, შავ ჩარჩოში ჩავსვი მონაკვეთი, სადაც ჩანს გუშინ გადატეხილი ადგილის კვალი (გაუფერულებული მონაკვეთი.
გადატეხისას სისხლი წამოუვიდა საშუალო რაოდენობის, მაგრამ რამდენად ბოლომდე იყო გადამტყდარი აღარ დავაკვირდით.

დღეს დილით შეამჩნია, რომ ამ ადგილას მხოლოდ გუშინდელი კვალი დარჩა ფერმკრთალი ზოლივით, რომელსაც ხედავთ ფოტოზე , თვითონ ფრჩხილი კი მთელია.

ორი ვარიანტი არსებობს.

1) ფრჩხილი გამთელდა (რაც ნაკლებსავარაუდოა, მაგრამ მაინც მინდა დაზუსტება)
2) მოგვეჩვენა რომ გადატყდა, მაგრამ ასეთ შემთხვევაში სისხლი წამოვიდოდა? ერთი დაკვირვებით აშკარად გადამტყდარი შეიმჩნეოდა.

მოკლედ არ ვიცით რა ვიფიქროთ.

იქნებ სპეციალისტები დაგვეხმაროთ.


Posted by: eksdo 12 Sep 2013, 17:14
ფრჩხილი რომ გადატეხილიყო მონატეხი ფრაგმენტი მოძვრებოდა ან თუ მთლიანად არ მოტყდა მაშინ ხელის შეხებით იმოძრავებდა smile.gif
ეტყობა აიყარა და სისხლიც იმიტომ წამოვიდა. საშიში არაფერია...

Posted by: Lacrimosa_ 13 Sep 2013, 15:07
rock in rose

არაფერი საინტერესო არ გამოავლინა ელექტრომიოგრაფიულმა კვლევამ (ანუ ნერვებიდან კუნთებს ინფორმაცია კარგად გადაეცემაო).

ეს გამორიცხავს თავის ტვინთან დაკავშირებულ პრობლემას? რატომღაც თავის ტვინის სიმსივნის მეშინია საშინლად

Posted by: newsgirl 13 Sep 2013, 20:32
რამე ვირუსია ქალაქში გავრცელებული? მეგობრისგან გადმომედო რაღაც. ცემინებით დაიწყო, სასუნთქი გზები გამეჭედა, ყელი მტკივა, მშრალი მაქვს, ვერ ვახველებ, სიცხე 38. სახსრების და თავის ტკივილი. ყველაზე ცუდი ისაა რომ ამოხველება მიჭირს. ვერც ვლაპარაკობ. წუხელ გავათენე სიცხისგან არ მძინებია.რამე რჩევა გაქვთ?

Posted by: eksdo 13 Sep 2013, 21:46
QUOTE (Lacrimosa_ @ 13 Sep 2013, 15:07 )
rock in rose

არაფერი საინტერესო არ გამოავლინა ელექტრომიოგრაფიულმა კვლევამ (ანუ ნერვებიდან კუნთებს ინფორმაცია კარგად გადაეცემაო).

ეს გამორიცხავს თავის ტვინთან დაკავშირებულ პრობლემას? რატომღაც თავის ტვინის სიმსივნის მეშინია საშინლად

ანუ სახის სხავადასხვა მხარეს გაქვთ ესეთი შეგრძნება?

Posted by: Lacrimosa_ 14 Sep 2013, 11:14
eksdo

გმადლობთ გამოხმაურებისთვის.

დიახ. შეგრძნებები სახის სხვადასხვა მხარეს მაქვს.
ტუჩზე სულ 2-3-ჯერ მქონდა მსგავსი შეგრძნება.
თვალს რაც შეეხება, ნევროლოგმა გასინჯვის დროს თავდაპირველად ვერ შენიშნა ასიმეტრია, ფიზიკური დატვირთვის (10 ბუქნი) შემდეგ კი ისევ ცოტათი დამივიწროვდა თვალი.
გუშინ დილით ცოტათი გაწითლებულიც მქონდა თვალის თეთრი გარსის სისხლძარღვები

Posted by: nina1992 18 Sep 2013, 00:34
სისხლსძარღვების გასაფართოებლად რა შეიძლება რომ მივიღოთ?
ანუ ხილი, ბოსტნეული...რომელია ამ შემთხვევაში დამხმარე...წალმების გარდა ანუ....

Posted by: STUNTER 19 Sep 2013, 18:36
გამარჯობათ მაინტერესებს თიაქარის ოპერაცია რა ჯდება ? მაინტერესებს სასწრაფოდ...

Posted by: coroline 21 Sep 2013, 15:56
არ ვიცი სწორად თემაში ვსვავ თუ არა კითხვას. თითქმის ერთი თვეა მაწუხებს ძლიერი თავის ტკივილები. ყოველდღე თუ არა ყოველმეორე დღეს. მეხსიერებაშიც ჩავარდნები მემართება. ვერ ვიხსნებ ზოგიერთ სიტყვას. სახელებს. თავიდან გადაღლას ვაბრალებდი, მაგრამ აშკარად არ არის გადაღლის ბრალი. აქამდე თუ ჩემი მესხიერებით ვაოცებდი ხოოლმე ხალხს და ყველაფერი დეტალურად მამახსოვრდებოდა ახლა უკვე დეტალები კი არა მთავარ მოვლენებსაც ვერ ვიხსენებ. მირჩიეთ რა გავაკეთო

ხო კიდევ მადა მაქვს გაძლიერებული , განსაკუთრებით ტკბილეულზე , რომელიც აქამდე არ მიყვარდა. მგონია რომ მშია და იმიტომ ვარ სუსტად , შევჭამ და მაინც ესე ვარ

Posted by: Lacrimosa_ 23 Sep 2013, 23:58
rock in rose
eksdo

დილაობით ცალი თვალი რომ მეჭუტება და შეგრძნება რომ მაქვს, რომ ქვემოთ იქაჩება, ის პაციენტი ვარ smile.gif
ელექტრომიოგრაფიულმა კვლევამ, ნორმის ფარგლებში გაქვს ყველაფერიო.
ტომოგრაფიაც გავიკეთე და არაფერი გამოავლინა.
ვანებებ გამოკვლევებს თავს და ჩაის ნაყენს ვიდებ თვალზე smile.gif
ხომ კარგი გადაწყვეტილებაა? smile.gif

Posted by: rock in rose 24 Sep 2013, 16:34
Lacrimosa_

თუ ნევროლოგიური ამბები გამოირიცხა კი ბატონო, მაშინ იქნებ თვალი იყოს დამნაშავე. ყოველშემთხვევაში არაფერს არ გავნებს ჩაის საფენები smile.gif

აქ თუ დაწერ მერე შედეგს კარგი იქნება smile.gif

Posted by: Lacrimosa_ 24 Sep 2013, 18:07
rock in rose

დღეს გამოვიტანე ტომოგრაფიის დასკვნა sad.gif
წერია:
მარცხენა ჰაიმორის წიაღში ისახება (1,0 სმ) ზომის ჰომოგენური შიგთავსის კისტური ჩანართი - მუკოიდური კისტა.
პერივენტიკალურად, გვერდითი პარკუჭის წინა რქის მიმდებარედ ისახება მომატებული - T2 სიგნალის მცირე (0.3 სმ) კერა, პერიფოკალური შეშუპების გარეშე.
ვილიზის წრის სისხლძარღვები თანაბრად მომატებული ინტენსიობის სიგნალებით ვლინდებიან დეფორმაციის და ექსტრავაზალური კომპრესიის ნიშნების გარეშე. მარცხენა განივ, სიგმოიდრ სინუსებში და საუღლე ვენაში სიგნალი სუსტია (ჰიპოპლაზია), ნაკადი კომპენსირებულია კონტრალატერალურად.

კვლევით ვლინდება მარჯვენამხრივი მცირე პერივენტრიკულური ლაკუნა.

ცუდადაა ჩემი საქმე? sad.gif

Posted by: rock in rose 24 Sep 2013, 21:09
Lacrimosa_

აი აქ მე ვერაფერს გეტყვით, ეს ნევროლოგმა უნდა ნახოს

Posted by: Lacrimosa_ 24 Sep 2013, 22:33
rock in rose

ვიყავი ექიმთან და არაფერი არ გჭირსო smile.gif მოკლედ, გადავრჩი smile.gif დიდი მადლობა ყველას smile.gif

Posted by: In_private 26 Sep 2013, 10:00
იქნებ დამაკვალიანოთ რა, სანამ მივალ ექიმთან როგორ მოვიქცე ან საერთოდ რომელ ექიმთან უნდა მივიდე 2 კვირაა დაახლოებით თავბრუსხვევა მაწუხებს ხან ძლიერი ხან მსუბუქი, პერიოდულად თავის ტკივილი და ეხლა 2 დღეა დაემატა თავში თუ ყურებში გუგუნი და არა შუილი . თან ასე მგონია თავი პრესში მაქვს მოყოლილი ყველა მხრიდან მიჭერს საშინელი შეგრძნებაა sad.gif სისხლის მიმოქცევის ბრალი შეიძლება იყოს ? ან საერთოდ რას ახასიათებს ძირითადად ასეთი სიმპტომები.

Posted by: isset 28 Sep 2013, 00:53
მოკლედ ესეთი სიტუაცია, ახლობელმა გაშუქა ფილტვები სხვდასხვა ადგილას, ერთგან დიგნოზი უთხრეს წყალია ცოტა პილტვშიო, მეორეგან არფერი გაქვსო, ბუნებრივია მნკურნალობაც სხვადსხვა დაინიშნა, რომელს დაუჯეროს როგორ უნდა მოიქცეს ამ შემთხვაში ადამინი?

იმასც დავამტებ რომ, იქ სადც გადაიღო ის ექიმი ძალიან კაი ექიმია დიდიხნის, მაგრამ არ არის ცნობილი კლინიკა, დღემდე ადმინი არ ყავს მომჩივარი ამ ექიმს,

ასევე შეგნებულად დარენდგენა ფილტვები ცნობილ კლინიკაში, სადაც უთხრეს წყალი გაქვსო. კლინიკის სახელს შეგნებულად არ ვასახლებ, საკმაოდ ცნობილი კლინიკა, ახლაი აპარტებია და რავი..... მოკლედ ვინმეს შულია, სწორი მიმრთულება მოცეს ამ გაურკველობაში, და არმარტო მე მსგავსი ბერისგან მსმენია, უბრალოდ ამ ცნობილ კლინიკაში იმტო გაშუქა რო უახლესი აპრატურებურა, თუმცა არც იმ უბრალო კლინიკაშია ძველი აპრატურეა,
რას დაუჯეროს უახლეს აპრატურას?, თუ გამოცდილ ექიმის დიგნოზს?

Posted by: Salome_L 29 Sep 2013, 10:36
გამარჯობა, იქნებ დიაგნოზი დამისვათ. რამდენიმე პრობლემა მაქვს.

უკვე რამდენიმე თვეა მკერდის ქვეშ კანი მექერცლება. ანუ სავარაუდოდ ჩახურებისგან გამოწვეული ქავილი, შედეგად კი მუქი შინდისფერი გადატყავებული კანი , პიმაფუცინის მალამო წავისვი და ცოტა ამილაგა, მაგრამ მერე ისევ თავიდან იწყება. ბიუსტჰალტერიც შევცვალე ...
ამის გარდა ყურებთან ზედა ნაწილში მაქვს ნახეთქები, ამ წუთას მორჩენილი მაქვს და არაფერი მეწვის, მაგრამ ესეც ხანგრძლივად არ მირჩება ხოლმე. ხელს რომ გადავატარებ, მიმხმარი კანი მძვება. ასეთივე ნახეთქი მაქვს მშობიარობის მერე (უკვე სამი წელი გავიდა) კუდუსუნის მიდამოში. აქაც ესე წვითა და აქერცვლით.
ასევე ეს ბოლო პერიოდი მიღიზიანდება ჭიპი (პატარა ბავშვივით ვისხამ პერეკისს) მაგრამ მაინც არ მირჩება. ეს ყველაფერი ერთმანეთთან კავშირში არის, თუ ყველა სხვადასხვა და ცალკე პრობლემაა? რამე ვიტამინი ხომ არ მაკლია, ან რამე კონკრეტული დაავადება ქვია მსგას სიმპტომებს?

მხოლოდ ჰიგიენის დაცვას ნუ მირჩევთ, რადგან ამ ყველაფრის მიუხედავად მჭირს ზემოთჩამოთვლილი.
* * *
გამარჯობა, იქნებ დიაგნოზი დამისვათ. რამდენიმე პრობლემა მაქვს.

უკვე რამდენიმე თვეა მკერდის ქვეშ კანი მექერცლება. ანუ სავარაუდოდ ჩახურებისგან გამოწვეული ქავილი, შედეგად კი მუქი შინდისფერი გადატყავებული კანი , პიმაფუცინის მალამო წავისვი და ცოტა ამილაგა, მაგრამ მერე ისევ თავიდან იწყება. ბიუსტჰალტერიც შევცვალე ...
ამის გარდა ყურებთან ზედა ნაწილში მაქვს ნახეთქები, ამ წუთას მორჩენილი მაქვს და არაფერი მეწვის, მაგრამ ესეც ხანგრძლივად არ მირჩება ხოლმე. ხელს რომ გადავატარებ, მიმხმარი კანი მძვება. ასეთივე ნახეთქი მაქვს მშობიარობის მერე (უკვე სამი წელი გავიდა) კუდუსუნის მიდამოში. აქაც ესე წვითა და აქერცვლით.
ასევე ეს ბოლო პერიოდი მიღიზიანდება ჭიპი (პატარა ბავშვივით ვისხამ პერეკისს) მაგრამ მაინც არ მირჩება. ეს ყველაფერი ერთმანეთთან კავშირში არის, თუ ყველა სხვადასხვა და ცალკე პრობლემაა? რამე ვიტამინი ხომ არ მაკლია, ან რამე კონკრეტული დაავადება ქვია მსგას სიმპტომებს?

მხოლოდ ჰიგიენის დაცვას ნუ მირჩევთ, რადგან ამ ყველაფრის მიუხედავად მჭირს ზემოთჩამოთვლილი.

Posted by: keen 29 Sep 2013, 12:14
ახლობელს (მოზარდს) აქვს შაკიკი და რა საშუალებაც არ სცადეს, არაფერი უმსუბუქებს მდგომარეობას, არა თუ უყუჩებს sad.gif საკმაოდ რთულ ფორმაში აქვს და ნუთუ არ არსებობს რაიმე საშუალება ამ სპაზმის მოსახსნელად? თუ შემოდის აქ ვინმე უცხოეთში მომუშავე მედიკოსთაგან, იქნებ გვირჩიოს რამე? აქ ყველა რესურსი ამოწურეს...

Posted by: Rabbit 30 Sep 2013, 11:01
გამარჯობათ, კითხვა მაქვს, კისრის მიდამოში მკიღებული ტრავმის ან დაზიანების სამკურნალოდ თუ შეიძლება რომ გარეგანი კათეტერი გამოიყენებოდეს? ან საერთოდ, ყელზე შემოხვეული კატეტერი თუ არსებობს? biggrin.gif საყელოსავით რომ იყოს

Posted by: BMWWMB 16 Oct 2013, 00:25
გამარჯობა, მარცხენა ხელი გათეთრდა და გამიჩერდა
ვერ ვხმარობ ფაქტიურად, რისი ბრალი შეიძლება იყოს? sad.gif
გაციებული არ მაქვს

Posted by: СAФO 24 Oct 2013, 09:03
ვარ 26 წლის გამომივლინდა ჩუტყვავილა რას მირჩევთ რომ გავაკეთო როგორ მოვიქცე ეს იგივე ბატონებია? რაიმე წამალი ან მაზი ან ასეთი რამ არსებობს საამისოდ? თავის ქალიდან დამეწყო და გუშინ შავი ღვინო დამალევინეს გამოვლინდებაო და ეხლა ტანზეც გამიჩნდა წითელი ლაქები. რამდენად მისაღებია ამდროს ნერვიულობა ან ჩხუბი ან ასეთი რამ. გამიგია არ შეიძლებაო.რის თუ არა რეკომენდირებული სპირტის წასმა დეზინფექციის მიზნით დაავადებულ ტერიტორიებზე?

Posted by: J-Time 4 Nov 2013, 00:28
ვსვამ წამლებს:

ცერებროვნი
როტალუდი
ტანიკანი
ართრა

მაინტერესებს თუ შეიძლება ალკოჰოლი.

Posted by: ich-liebe-ihn 9 Nov 2013, 20:20
გამარჯობა,

ჩემს ახლობელს ჰქონია თურმე თვეებია ფილტვების (უსიცხო) ანთება. ეხლა 5 დღით დააწვინეს სასწრაფოდ საავადმყოფოში. ფილტვებში რაღაც ჩრდილებიაო უთხრეს და 3-4 კვირაში რენდგენს რომ გადაგიღებთ და მაშინ ვნახოთ რაც ხდებაო. და რა შეიძლება ეს ჩრდილები რომ ნიშნავდეს? sad.gif მადლობთ წინასწარ პასუხისთვის.

Posted by: DIAMOND_G 20 Nov 2013, 15:02
საერთოდ კუჭში გასვლა არ მინდება.
სწორ ნაწლავში არ ვგრძნობ არანაირ მასებს.
მხოლოდ კუჭს აწვება ქვემოდან და ცუდად მხდის. გულის რევის შეგრძნება მაქვს და ზურგის, წელის და თავის ტკივილი.
სენადექსინს ვსვამ გუშინს მერე მაგრამ არ მშველის.
სხვა წამლებიც გამომიწერა ექიმმა, სამი დღეა ვსვამ მაგრამ არანაირი შედეგი არ ააქვს.

შეკრულობები ხომ არსებობს სხვადასხვა სახის და იქნებ ამას სხვანაირი მკურნალობა უნდა?
ზემოთ არის რაღაც დაგუბებული თითქოს, კუჭს აწვება , ქვემოთ არ ჩამოდის, კუჭში გასვლის მოთხოვნილება არ მაქვს.

მირჩიეთ რა რამე, რა უნდა ქნა ასეთ დროს, ან როგორი მკურნალობა ჭირდება, რისი ბრალია?

Posted by: rock in rose 20 Nov 2013, 18:40
DIAMOND_G

სენადექსინის გარდა რა გამოგიწერათ კიდევ ექიმმა?

Posted by: eksdo 20 Nov 2013, 21:44
ნაწლავების პერისტალტიკაა დარღვეულიო?

Posted by: DIAMOND_G 23 Nov 2013, 15:40
rock in rose
QUOTE
სენადექსინის გარდა რა გამოგიწერათ კიდევ ექიმმა?

ფერმენტალი და კასანორმი.

წარმოიდგინეთ კუჭს მიშლის, მაგრამ ტვალეტში გასვლა არ მინდება. ზემოთ კუჭთან ბუყბუყებს, ქვემოთ მსხვილ ნაწლავთან არ ჩამოდის არაფერი.
საერთოდ ვერ ვგრძნობ ტვალეტში გასვლის მოთხოვნილებას.

ასეთი რაღაცა არსებობ პრაქტიკაში თუ მარტო მე მჭირს? sad.gif

eksdo
QUOTE
ნაწლავების პერისტალტიკაა დარღვეულიო?

არ ვიცი, არ უთქვია.
რას ნიშნავს ეგ? განკურნებადია?

Posted by: eksdo 23 Nov 2013, 17:20
პერისტალტიკა ნაწლავების თანმიმდევრული ტალღისებური შეკუმშვებია რომელიც უზრუნველყოფს მასში არსებული მასის გადაადგილებას სწორედ ამ შეკუმშვების ხარჯზე.დარღვევა შეიძლება ადგილობრივი იყოს, შეიძლება სისტემური. მიზეზი ბევრია. ასე რომ კარგად გამოიკვლიეთ და პასუხი გაგვაგებინეთ.
სენადექსინი იმითაა კარგი რომ მასას ათხელებს და შედარებით ადვილია მისი გადაადგილება, მაგრამ არა მგონია თქვენი პრობლემა ეგ იყოს.

Posted by: DIAMOND_G 23 Nov 2013, 17:30
eksdo
ხო და არ ვიცი სად წავიდე და ვისთან გამოვიკვლიო, თორემ მეც მინდა. : (
თქვენ რას მირჩევთ, რომელ საავადმყოფოში ჯობია მისვლა გამოსაკვლევად?

Posted by: All Shall Perish 11 Dec 2013, 17:03
თირკმელებში თუ კენჭებია 5მილიმეტრამდე შეუძლია მუცლის შებერილობის და გაზების წარმოქმნა?

Posted by: konkretula 12 Dec 2013, 20:36
არავინ იცით რაიმე წამალი რომელიც მადას მომიმატებს?
რა ვიტამინი აღარ დავლიე მაგრამ ამაოდ, ძვირიანიც, იაფიანიც, რავიცი ათასნაირი და შედეგი მაინც არ მაქვს.
ერთადერთი რამაც რეალურად მადა მომცა იყო ასკორუტინი, ისიც რამდენიმე თვის მანძილზე ვსვი.
ვინმემ მირჩიეთ რამე რა... sad.gif

Posted by: ქეთუ 13 Dec 2013, 00:51
იქნებ მითხრათ მუდმივად მცივა, ასევე ვარ სულ ცივი და რისი ბრალია?

გემოგლობინის?

Posted by: natata25 19 Dec 2013, 19:19
ცოტა უცნაური რაღაც მჩირს.ამ ბოლო პერიოდში მარვენა ხელ-ფეხის ნეკათითები მეყინება და მიბუჟდება,ასევე მარჯვენა თვალი მისივდება ხომ არ იცით რის ექიმს უნდა მივმართო? sad.gif

Posted by: eksdo 22 Dec 2013, 01:40
QUOTE (natata25 @ 19 Dec 2013, 19:19 )
ცოტა უცნაური რაღაც მჩირს.ამ ბოლო პერიოდში მარვენა ხელ-ფეხის ნეკათითები მეყინება და მიბუჟდება,ასევე მარჯვენა თვალი მისივდება ხომ არ იცით რის ექიმს უნდა მივმართო?  sad.gif

და რის შემდეგ დაგეწყოთ ეს ჩივილები შეგიძლიათ გაიხსენოთ?

Posted by: natata25 22 Dec 2013, 20:57
eksdo
დაახლოებით 2 თვეა რას ესე მაქვს, მაგრამ პერიოდულად მაწუხებდა
ეხლა უკვე ზალია ნ დიდ დისკომფორტი მაქვს ძირითადათ დილაობით შესამჩნევად მაქვს მარჯვენა თვალი და თითები შეშუპებული.დღის განმავლობაში კი მეყინება ნეკა თითები.ზოგადად ხერხემალი მაწუხებს , მაგრამ თვალი რა შუაშია ვერ ვხვდები sad.gif sad.gif

Posted by: eksdo 24 Dec 2013, 20:11
ნევროლოგი გჭირდებათ აუცილებლად. შეიძლება კისრის მალების დაზიანება იყოს. ხერხემლის ტრავმა გაქვთ?

Posted by: natata25 24 Dec 2013, 21:12
eksdo
მალთაშუა თიაქარი მაქვს 4-5 მალებზე, ასევე კისერზეც მაქვს პრობლემა. სავარაუდოდ კისრის ოსტეოხონდროზის ბრალი იქნება ხო? ცოტა არ იყოს მეშინია სხვა რამე უფრო სერიოზული არ მჩირდეს cry.gif

Posted by: eksdo 25 Dec 2013, 00:06
ეგ კისრის მალებია. გასაგებია. თქვენი ეს სიმპტომები ზუსტად ამასთანაა დაკავშირებული.

Posted by: konkretula 4 Jan 2014, 00:08
მგონი სწორ ადგილას ვსვავ კითხვას.

ბოლო რამდენიმე წელია ყელი მიჩირქდება სისტემატიურად. ზამთარში ხშირად და ზაფხულში ერთხელ მაინც, თან სიცხესაც მაძლევს. ანტიბიოტიკების და სავლებებით მკურნალობა კარგად მოქმედებს მაგრამ რაც დრო გადის უფრო ხშირად მიჩირქდება და უფრო რთულად ექვემდებარება მკურნალობას.
ბაქტერიოფაგში ავიღე ანალიზი მაგრამ ჩემმა ექიმმა მითხრა ეს განკურნებადი არააო, რაღაც ბაქტერიაა ყელში და ეგ არ მოშორდებაო.

აღარ ვიცი რა ვქნა, ვის მივმართო? ძალიან დავიტანჯე sad.gif

Posted by: Tauriel 4 Jan 2014, 14:52
konkretula
ქრონიკული ფარინგიტი?

კავშირშია ჰორმონალურ დარღვევებთან და გულის ან ღვიძლის დაავადებებთან.

p.s. რა ყველაფერი შენ გჭირს ისე?

Posted by: super fast jellyfish 6 Jan 2014, 18:13
არ ვიცი სწორად ვკითხულობ თუ არა smile.gif

კონცენტრაცია მიჭირს, აზრები მეფანტება, მეხსიერება საერთოდ აღარ მივარგა sad.gif

რამე პრეპარატი არის რაც დამხმარება?

Posted by: Utopian 7 Jan 2014, 15:38
სელენის დალევა ექიმის დანიშნულების გარეშე შეიძლება?
1 კოლოფი უკვე დავლიე სადღაც 2 თვის წინ.. ჯიპისიში ფარმაცევტს ვკითხე და შეგიძლია დალიოო და თქვენი აზრიც მაინტერესებდა
ანოტაციაში ეწერა და აფთიაქშიც მითხრეს რომ წელიწადში რამდენჯრმე შეიძლება დალევა
ტაქსოფიტი ® სელენზე მაქვს საუბარი


Posted by: CORDOBA 7 Jan 2014, 21:34
გამარჯობათ ფორუმელებო, მრავალ შობას დაესწარით

კითხვა მაქვს, (არ ვიცი სად დამეწერე და თუ თემასთან საერთო არ არის მე მომიტევეთ)

ფილტვებზე მამაჩემა გადაიღო რენდგენი, მასში აღმოჩდა რაღაც ლაქები ფილტვებში,
ამის შემდეგ ექიმა მოითხოვა ტომოგრამის გადაღება ფილტვებზე


რა შეიძლება იყოს შავი ლაქები ? რა დაავადება ?

Posted by: rock in rose 7 Jan 2014, 21:53
CORDOBA

გამარჯობათ, მეც გილოცავთ შობას smile.gif

ექიმმა არაფერი თქვა რისთვის უკეთებს ტომოგრაფიას?

ან მამათქვენს რა ჩივილები ჰქონდა?

Posted by: CORDOBA 7 Jan 2014, 22:06
rock in rose

ფილტვების ანთება გადაიტანასავით ისე რომ არაფერი უთქვამს

ექიმმა მითხრა რომ ტომოგრამას დაველოდოთო, წინასწარ არაფრის თქმა არ შემიძლიაო,

გასკდა თავი ისე ვნერვიულობ უკვე, რა შეიძლება იყოს ეს ლაქები ?

Posted by: rock in rose 7 Jan 2014, 22:21
CORDOBA

რახან ანთება გადაიტანა, თუ გაქრონიკულდა, მაშინ ანთების კერაში ფიბროზული ქსოვილი ჩანაცვლდებოდა და სავარაუდოდ ამის დასადგენად მოითხოვა ექიმმა ტომოგრაფია. ნუ გეშინიათ, ეს არაა საშიში smile.gif

ისე კი ტომოგრაფიას სხვა შემთხვევებზეც უკეთებენ რა თქმა უნდა, მაგალითად, პარაზიტზე ეჭვის შემთხვევაში(ექინოკოკზე),
ასევე მეტასტაზების არსებობისას, მაგრამ თქვენს შემთხვევაში ესენი არაა ამ კვლევის მიზეზი. არ ინერვიულოთ.

Posted by: CORDOBA 7 Jan 2014, 23:00
rock in rose

რამე კიბოს მსგავსი შეიძლება იყოს ? ანუ სიცოცხლისთვის საშიში ? (რამე უკურნებელი ან ამდაგვარი)

Posted by: sakheli 7 Jan 2014, 23:16
რამდენი წლის არის და რა სიმტომები აქვს ეგ დაწერე , დამშვიდდი როდის უკეთებთ ტომოგრაფიას?

Posted by: CORDOBA 7 Jan 2014, 23:25
sakheli

ზეგ უკეთებ ტომოგრამას

სიმპტომები არ ვიცი, ცოტას ახველებდა, და ფილტვების ანთება გადაიტანა ჩუმათ, არავის გითხრა

Posted by: sakheli 7 Jan 2014, 23:32
QUOTE (CORDOBA @ 7 Jan 2014, 23:25 )
sakheli

ზეგ უკეთებ ტომოგრამას

სიმპტომები არ ვიცი, ცოტას ახველებდა, და ფილტვების ანთება გადაიტანა ჩუმათ, არავის გითხრა

რამე გაციება ან ვირუსი ხომ არ შეხვდა ვირუსმაც იცის ხოლმე ანთების დატოვება გამიგია ეგეთი რამეც მე არ ვარ ექიმი და სხვა რამე სიცხეები აქვს ხველება ღამე ოფლიანობა ან რამე აწუხებს საერთოდ რატომ გადაიღო საერთოდ სურათი უბრალოდ ხველების გამო?

Posted by: CORDOBA 7 Jan 2014, 23:40
sakheli
სურათი რატომ გადაიღო ? ალბათ მიხვდა რომ ფილთვების ანთება ქონდა, და ძლიერი ხველება, (ადრე რამდენჯერმა აქვს გადატანილი ფილტვების ანთება)

არ სვავს სასმელს, სიგარეტს ცოტას ეწევა,

ვირუსი რომ ქონოდა და გართულებოდა, ეს ლაქები შეიძლება იყოს სიცოცხლისთვის საშიში ?

Posted by: Lacrimosa_ 11 Jan 2014, 15:59
მეგობრებო, 3 იანვრიდან მაქვს სიცხეები, 37.8-მდე. ორი ექიმი მკურნალობს ერთდროულად smile.gif მაგრამ დღევანდელ ვიზიტზე მითხრა, ისევ ისეთი გაქვს სუნთქვა ბრონქებშიო.
როტაცეფს ვიკეთებდი 4 დღე.
და ყველაფრისგან დასვენებული ვარ smile.gif
რამე უჯიშო ვირუსია გამოშვებული?

Posted by: womanly 12 Jan 2014, 12:02
გამარჯობათ ... რამოდენიმე შეკითხვა მაქვს ომეგა 3 მინდა დავალევინო ბავშვს ანუ 12 წლს ბიჭს და ექიმის კონსულტაცია აუცილებელია თუ არააა? ომეგა 3- 700 მგ


კიდევ ერთი შეკითხვა მაქვს ასაკოვან კალბატონს ტკივა ფეხის უკანა მხარე , ამბობს რომ გაუცივდა და მგის შედეგად ტკივა თავიდან ძლიერი იყო ტკივილი და დიკლოფეროლის ნემსი გაუკეთეთ 3 დღეში ერთხელ მაგრამ მაინც ტკივააა... მირჩიეთ ექიმი ვისთან მივიყვანოო?

Posted by: 7903251 17 Jan 2014, 15:24
გამარჯობათ,

ლეიკოციტი ერთი თვის წინ მქონდა 3.30; ნეიტროფილი 1.83; ჰემოგლობინი; 116;

მაგის შემდეგ გავიკეთე 9 ინექციქცია და დავლიე 25 ტაბლეტი ფერო ფოლგამა თითო აბი დღეში.
გუშინ გავიკეთე სისხლის საერთო: ლეიკოციტი კვლავ: 3.30; ნეიტროფილი 1.72, ჰემოგლობინი 119.

ამჟამადვმკურნალობ ექსუდაციური პერიკარდიტის (უცნობი ეტიოლოგიის) დიაგნოზით.

Posted by: goga-mariami 18 Jan 2014, 12:03
მოწიტალო-მოვარდისპრო ნახველი და შარდი რისი ნიშანი შეიძლება იყოს? ( გაციების დროს)

Posted by: mcenare-99 22 Jan 2014, 12:18
გამარჯობათ 4 დღეა სიცხე მაქ ეგ რავ ვიცი, მესამე დღეა 37 დაბლა არ ჩამომდის 37 - 37,5 ეგრე მითამაშებს სულ 1 კვირაა გაციებული ვარ მაგარი, გუშინ სისხლიანი ნახველი ამომქონდა ხან, რას მირჩევთ?

Posted by: benito777 22 Jan 2014, 18:36

მე-4 დღეა გრიპი მაქვს, იშვიათად მახველებს მაგრამ როცა მახველებს ბრონქები მტკივა და ამომდის შავი ნახველი. (6 თვეა სიგარეტს თავი დავანებე და მაგის ბრალი არ უნდა იყოს მგონი ხომ? spy.gif )

სიცხე არ მაქვს , თავს ოდნავ სუსტა ვგძნობ, თან მაციებს ხანდახან

გადავრჩები? user.gif

Posted by: sakheli 22 Jan 2014, 20:57
QUOTE (mcenare-99 @ 22 Jan 2014, 12:18 )
გამარჯობათ 4 დღეა სიცხე მაქ ეგ რავ ვიცი, მესამე დღეა 37 დაბლა არ ჩამომდის 37 - 37,5 ეგრე მითამაშებს სულ 1 კვირაა გაციებული ვარ მაგარი, გუშინ სისხლიანი ნახველი ამომქონდა ხან, რას მირჩევთ?

მიმართე ექიმს აუცილებლად რენდგენი გადაიღე არ შეიძლება ეგრე ყურება და ლოდინი. ქოშინი ხომ არ გაქვს?

Posted by: perkusia 23 Jan 2014, 12:59
იცით რა მინდა ვიკითხო. როგორც მე ვიცი ერითროციტი CO-ს უფრ ადვილად უკვშირდება ვიდრე ჟანგბადს, ამიტომაა ის მხუთავი აირი. და მაინტერესებს ვთქვათ 1 ერითროციტი ფილტვებში დაუკავშირდა CO-ს, სად მოხდება მისი განთავისუფლება მისგან? ის ურედში CO-ს ჩაანაცვლებს CO2-ით თუ ბოლოს სისხლის დეპო ორგანებში მასთან ერთად დაიღუპება?

Posted by: rêveur 23 Jan 2014, 16:29
დროდადრო მაწუხებს სისუსტე, რომელსაც თან ახლავს გულისრევისა და გულისწასვლის შეგრძნება, რა დროსაც აუცილებლად უნდა დავჯდე და თავი ჩავხარო (მაგიდაზე დავდო ან რავი..) ან უნდა წამოვწვე. როგორც დავაკვირდი, მხოლოდ დილისა და დღის საათებში მემართება ასე, უზმოზეც და ნაჭამზეც ყოფილა.
ბავშვობაშიც მაწუხებდა(იმ დროს უჭმელობას ვაბრალებდი. მაგ.: საღამოს რომ არ მეჭამა დილით ცუდად ვიყავი, მაგრამ ახლა, როცა ნაჭამი ვარ მაშინაც მემართება). სკოლის პერიოდშიც, რამდენჯერმე კლასიც დამიტოვებია... ხან ცოტა წყალი უნდა დავლიო და მშველის, ხან მაგსაც ვერ ვსვამ.
მერე ერთი ხანი აღარ მაწუხებდა. ეს ბოლო დროა ისევ დამეწყო.
ჰორმ.წამალს ვიღებ(ბოლო 2 წელია), თავიდან მაგას დავაბრალე და ექიმმა შემიცვალა წამალი, მაინც მომდის ასე.
ხო, თუ გადავიღალე, ვინერვიულე მაშინ კიდე უარესია, მაგრამ დასვენებულზეც რომ ყოფილა?
ვერ გავიგე რის გამო მემართება ასე, თან მუდმივად არაა და...

რა შეიძლება იყოს მიზეზი? რისი გამოკვლევაა საჭირო?

აი, ახლაც ცუდად ვარ, მოვედი სახლში, დავწექი და გადამიარა. წამოვჯექი რომ დამეწერა ამ თემაში და ისევ ცუდად გავხდი.

* * *
ხილის ტკბილი წვენი დავკლიე და მიშველა.
ბავშვობაშიც შაქრიანი ჩაი მინდებოდა ხოლმე მხოლოდ ამ დროს.

Posted by: tamttamta 28 Jan 2014, 23:47
rêveur
შAქარი ფარისებრი გადაიკონტროლე
ისე ნერვიზულმაც იცის ასეთი წამოვლითები და მაინც მიდი ექიმთან

Posted by: mammolog 2 Feb 2014, 13:09
Gamarjobat. Tu sheudzlia terapevts an ptiziatrs rom mipasuxos: anambezshi filtvis tuberkuliozi.amjamad filtvis qronikuli obstruqcili bronqiti ramdenad rekomendirebulia seretidis danishvna?

Posted by: konkretula 3 Feb 2014, 16:37
მადის მოსამატებელ რაიმე წამალს ვერ მირჩევთ?

Posted by: goga-mariami 7 Feb 2014, 00:34
აქ არის ვინმე ტირკმელების ექიმი? რამოდენიმე კითხვა მაქვს სასწრაფო

Posted by: Natuka NGN 7 Feb 2014, 02:35
goga-mariami
თირკმელების ექისმ ჰქვია ნეფროლოგი.როგორც ვიცი ფორუმზე არ შემოდის. თუ რამე კიტხვა გაქვს დასხვი ამ თემაში დათუ იციან პასუხი გიპასუხებენ.

Posted by: MIT 13 Feb 2014, 15:10
გამარჯობათ,

ჩემს შევარებულს (25 წლის) აწუხებს კუჭი, ყოველი ჭამის მერე ან ტკივა, ან შებერილობა აქვს, ან ფაღარათი აწუხებს. ჩემი აზრით ეს მხოლოდ კუჭის არა, ნაწლავების (დისბაქტერიოზი) პრობლემაც არის.

კითხვა: რის ექიმთან უნდა მივიდეთ? საიდან უნდა დავიწყოთ ამ პრობლემის გამოკვლევა?

მადლობა წინასწარ!

Posted by: rock in rose 13 Feb 2014, 16:53
MIT

გამარჯობა.

გასტროენტეროლოგთან უნდა მივიდეს და ის ეტყვის რა გამოკვლევები იქნება ჩასატარებელი.

Posted by: ich-liebe-ihn 15 Feb 2014, 19:24
კარგი ფილტვების სპეციალისტი ვინ არის თბილისში? სასწრაფოდ მჭირდება sad.gif ერთი რევაზ გაგუა გამიგია, სად მუშაობს ან ტელ. ხომ არ გაქვთ ვინმეს?

Posted by: Kapitan kusto 20 Feb 2014, 13:40
მოგესალმებით. ჩემს ნათესავ ბავშვს 4 თვის განმავლობაში აქვს დაბალი სიცხეები. წამდაუწუმ საავადმყოფოებში ატარებენ. ბავშვი 1 წლის გახდება მალე. დღეს ყავდათ ხუდადოვზე, ტუბდისპანსერში. იქ უთხრეს რომ ვირუსოლოგთან მიეყვანათ და არ იყო მათი პაციენტი. სავარაუდოდ ციტომეგალოვირუსი ექნებაო. ხომ ვერ გვირჩევთ ვის მივაკითხოთ. ან რა ღირს გამოკვლევა, ანალიზები და თუ შეიძლება დაზღვევით გაკეთდეს ეს ყველაფერი.?


idea.gif
* * *
მოგესალმებით. ჩემს ნათესავ ბავშვს 4 თვის განმავლობაში აქვს დაბალი სიცხეები. წამდაუწუმ საავადმყოფოებში ატარებენ. ბავშვი 1 წლის გახდება მალე. დღეს ყავდათ ხუდადოვზე, ტუბდისპანსერში. იქ უთხრეს რომ ვირუსოლოგთან მიეყვანათ და არ იყო მათი პაციენტი. სავარაუდოდ ციტომეგალოვირუსი ექნებაო. ხომ ვერ გვირჩევთ ვის მივაკითხოთ. ან რა ღირს გამოკვლევა, ანალიზები და თუ შეიძლება დაზღვევით გაკეთდეს ეს ყველაფერი.? იქნებ მიპასუხოს ვინმეეემ sad.gif

Posted by: sakheli 20 Feb 2014, 22:36
QUOTE (Kapitan kusto @ 20 Feb 2014, 13:40 )
მოგესალმებით. ჩემს ნათესავ ბავშვს 4 თვის განმავლობაში აქვს დაბალი სიცხეები. წამდაუწუმ საავადმყოფოებში ატარებენ. ბავშვი 1 წლის გახდება მალე. დღეს ყავდათ ხუდადოვზე, ტუბდისპანსერში. იქ უთხრეს რომ ვირუსოლოგთან მიეყვანათ და არ იყო მათი პაციენტი. სავარაუდოდ ციტომეგალოვირუსი ექნებაო. ხომ ვერ გვირჩევთ ვის მივაკითხოთ. ან რა ღირს გამოკვლევა, ანალიზები და თუ შეიძლება დაზღვევით გაკეთდეს ეს ყველაფერი.?


idea.gif
* * *
მოგესალმებით. ჩემს ნათესავ ბავშვს 4 თვის განმავლობაში აქვს დაბალი სიცხეები. წამდაუწუმ საავადმყოფოებში ატარებენ. ბავშვი 1 წლის გახდება მალე. დღეს ყავდათ ხუდადოვზე, ტუბდისპანსერში. იქ უთხრეს რომ ვირუსოლოგთან მიეყვანათ და არ იყო მათი პაციენტი. სავარაუდოდ ციტომეგალოვირუსი ექნებაო. ხომ ვერ გვირჩევთ ვის მივაკითხოთ. ან რა ღირს გამოკვლევა, ანალიზები და თუ შეიძლება დაზღვევით გაკეთდეს ეს ყველაფერი.? იქნებ მიპასუხოს ვინმეეემ sad.gif

ბავშვს ექიმი პედიატრი არ ყავს? კითხეთ მას ჯობია პოლიკლინიკაში სადაც არის ბავშვი რეგისტრირებული. აქ ესე ვერავინ რცევას ვერ მოგცემთ მაშინ მემგონი ჯობია ახალი თემა გახსნა ვირსოლოგი მირჩიეთ და ისე უფრო ბევრი გამოხმაურება იქნება შეიძლება ნებისმიერს გონოდა მსგავში შემთხვევა და გამოცდილებიდან გირჩევენ. წარმატებები.

Posted by: Natuka NGN 21 Feb 2014, 12:22
sakheli
არარის საჭიროამისტვის ახლაი თემა.
Kapitan kusto
მშჴბელთა კლუბში არის თემა შეკითხვა პედიატრს, იქ უფრო დაგაკვალიანებენ.მე ჩემი მხირვ, გირცევდით, ირაკლის, გვარი ზუსტადარ მახსოვს, "მრჩეველის" დირექტორია და საკმაოდ კარგი ექიმი.

Posted by: Leo woman 25 Feb 2014, 12:35
გამარჯობა,
რამდენიმე დღეა გაგიჟებამდე მტკივა თეძოდან ფეხის ბოლომდე ,ცალ მხარეს.
არვიცი ნერვის ანთებაა თუ რა არის??
მითხრეს რომ კუნთის დაჭიმულობა შეიძლება იყოს
ძლიან გთხოვთ მირჩიეთ როგორ მოვიქცე,,,რა წამალი შეიძლება მივიღო ასეთ დროს ??????
ექიმტან წასვლა არ მინდაsad.gif

Posted by: happy dreamer91 26 Feb 2014, 21:18
ერთ თვეზე მეტია სიცხე მაქვს, 37 ხან 37.4 მდე ადის. დავდივარ ექიმებთან და ვერაფერი დავადგინე. ფილტვები ორ ექიმს მოვასმენინე და საერთოდ არაფერი ისმისო და არ გადამიღია. ქირურგთანაც ვიყავი ბრმანაწლავზე გავესინჯე, გინეკოლოგთანაც ვიყავი, თირკმელებზე გადავიღე, ყელ-ყურის ექიმთან ვიყავი და არსად არაფერია ისეთი რომ სიცხეს იწვევდესო. სისხლში მაქვს ანტისტრეპტოლიზინი მომატებული და ანტიბიოტიკი გავიკეთე 7 დღე. სიცხე ისევ მაქვს. ახლა განმეორებითი ანალიზი უნდა გავიკეთო და ხომ ვერ მეტყვით კიდე ვისთან მივიდე, რა შეიძლება იწვევდეს ასეთ სიცხეს.

Posted by: sakheli 26 Feb 2014, 21:33
QUOTE (happy dreamer91 @ 26 Feb 2014, 21:18 )
ერთ თვეზე მეტია სიცხე მაქვს, 37 ხან 37.4 მდე ადის. დავდივარ ექიმებთან და ვერაფერი დავადგინე. ფილტვები ორ ექიმს მოვასმენინე და საერთოდ არაფერი ისმისო და არ გადამიღია. ქირურგთანაც ვიყავი ბრმანაწლავზე გავესინჯე, გინეკოლოგთანაც ვიყავი, თირკმელებზე გადავიღე, ყელ-ყურის ექიმთან ვიყავი და არსად არაფერია ისეთი რომ სიცხეს იწვევდესო. სისხლში მაქვს ანტისტრეპტოლიზინი მომატებული და ანტიბიოტიკი გავიკეთე 7 დღე. სიცხე ისევ მაქვს. ახლა განმეორებითი ანალიზი უნდა გავიკეთო და ხომ ვერ მეტყვით კიდე ვისთან მივიდე, რა შეიძლება იწვევდეს ასეთ სიცხეს.

რენდგენი გადაირე და შემდეგ ტომოგრაფია გაიკეთე თუ გაქვს საშვალება.

Posted by: happy dreamer91 26 Feb 2014, 21:42
sakheli
ხო რენდგნის გადაღებას ვაპირებ. და ტომოგრაფია რატო, ყველაფრი ჩანს? კარგად ვერ ვერკვევი.

Posted by: sakheli 26 Feb 2014, 21:55
QUOTE (happy dreamer91 @ 26 Feb 2014, 21:42 )
sakheli
ხო რენდგნის გადაღებას ვაპირებ. და ტომოგრაფია რატო, ყველაფრი ჩანს? კარგად ვერ ვერკვევი.

ჯერ რენდგენი გადაიღე სხვარამე გაწუხებს რამე სიმპტომები გაქვს ან ხველება ღამით ოფლიანობა?

Posted by: shota1811 27 Feb 2014, 17:26
გამარჯობა, ე.ი. 3 დღეა მაქვს სურდო და ტემპერატურა 36,8–37,2 მერყეობს. არ მახველებს, მაღალი ტემპერატურა არ დაფიქსირებულა. სურდოს შეიძლება თან სდევდეს დაბალი ტემპერატურა? თუ უმჯობესია ექიმს მივმართო რომ სხვა უფრო სერიოზული დაავადება გამოირიცხოს?

Posted by: ninilion 27 Feb 2014, 22:28
10 დღე განუწყვეტლივ მაღალი ვენური წნევა, არითმია, უძილობა, აგზნებადობა, ადვილი დაღლა და ქოშინი, თავის ტკივილი.. რა დაავადება შიეძლება იყოს ან პირველად დახმარებად რის მიღება ჯობია?

Posted by: sakheli 27 Feb 2014, 23:45
QUOTE (ninilion @ 27 Feb 2014, 22:28 )
10 დღე განუწყვეტლივ მაღალი ვენური წნევა, არითმია, უძილობა, აგზნებადობა, ადვილი დაღლა და ქოშინი, თავის ტკივილი.. რა დაავადება შიეძლება იყოს ან პირველად დახმარებად რის მიღება ჯობია?

აუცილებლად მიმართე ექიმს ჯობია.

Posted by: rock in rose 28 Feb 2014, 09:28
ninilion

ვენური წნევა როგორ გაიზომეთ?

Posted by: qeqoo 1 Mar 2014, 19:25
გამარჯობა smile.gif თუ იცით კალცი დ3, დგის რომელ მონაკვეტში ისმევა?? დილით უზმოზე, შუადღით, თუ ღამე ძილის წინ??? smile.gif

Posted by: rock in rose 1 Mar 2014, 19:34
qeqoo

უმჯობესია შუადღისას, ჭამის დროს და არა ჭამამდე ან მის შემდეგ.

Posted by: Masked 2 Mar 2014, 05:10
ninilion
ეფტე4 და ტეესჰა ანალიზი, კორტისოლ, გლუკოზე. შეიძლება არის ჰიპატჰირეოზე.

Posted by: tamuska27 4 Mar 2014, 02:33
Aრ ვიცი თერაპევტს ეხება თუ არა....
2012 ში მქონდა 26 Dვიტამინის მაჩვენებელი.
Dამინიშნეს D3 და ვსვავდი 1 წელზე მეტი.
Eხლა მაქვს 202 მაჩვენებელი.
Yველგან წერია რომ ძალიან მაღალია და არის რამე გზა რომ სასწრაფოდ დაიწიოს ამ მაჩბენებელმა თუ არა?

Posted by: gramatikikina 4 Mar 2014, 03:20
ვინმემ იქნებ მითხრათ, რა დავლიო
ყელში საყლაპავთან ვგრძნობ პერიოდულად რაღაც გაღიზიანებას რამდენიმე დღეა და იძულებული ვარ ვახველო არანორმალურად. სანამ თავისით არ გაივლის, არაფერი შველის. ალერგიას ჰგავს მგონი, მაგრამ ალერგიული არ ვარ

Posted by: bfjribhs 4 Mar 2014, 17:05
ფიზიკურად რომ ვიტვირთები, თუნდაც მსუბუქად, ცუდად ვხდები, ისე თითქოს მოწამლული ვიყო.
ყოველთვის როცა რაღაცას გავაკეთებ ბოლოს მიჩნდება მოწამვლის შეგრძნება. არადა არანაირ საჭმელს არ ვიღებ მაგ დროს და არც ვსვამ და რამ უნდა მომწამლოს.
არ ვსვამ, არ ვეწევი.

Posted by: Lacrimosa_ 5 Mar 2014, 14:00
გამარჯობა მეგობრებო

ჩემს ძმას სიცხე აქვს (37,2). თვითონ ამბობს, რომ გაციებულია, მაგრამ ორი დღის წინ ბაკურიანში თხილამურებზე დაიმტვრა (ტალახზე) და გვარიანად იყო მოსვრილი მიწით.
მეშინია, სისხლი არ მოეწამლოს.
პანიკიორი ვარ ზოგადად და არ მიჯერებს, რომ ექიმთან წავიდეთ (არადა 100%-იანი დაზღვევა აქვს).
რას მირჩევდით?

Posted by: gramatikikina 5 Mar 2014, 19:18
gramatikikina
QUOTE
ვინმემ იქნებ მითხრათ, რა დავლიო
ყელში საყლაპავთან ვგრძნობ პერიოდულად რაღაც გაღიზიანებას რამდენიმე დღეა და იძულებული ვარ ვახველო არანორმალურად. სანამ თავისით არ გაივლის, არაფერი შველის. ალერგიას ჰგავს მგონი, მაგრამ ალერგიული არ ვარ


კლარიტინი დალიე ერთი აბი და მომენტალურად მოგიგვარდება პრობლემა up.gif

Posted by: ruska32 5 Mar 2014, 21:09
გთოხოვთ არ დამბლოკოთ თუ კითხვა არ ეკუთნის ამ განყოპილებას. მაქვს ჩიყვი და დამემათა ბაზადოვის დაავადება თვალზე. თუ ხართ კომპეტენტური ,შეგიძლიათ მიპასუხოთ. მკურნალობით ,უბრუნდება თვალის გუგა თავის ადგილს?. ცნობისთვის ოდნავ მაქვს გადიდებული.
წინასწარ მადლობთ.

Posted by: rock in rose 5 Mar 2014, 21:56
ruska32

კი, უბუნდება თავის ადგილას smile.gif

Posted by: ruska32 5 Mar 2014, 22:11
////////////////////////////////////////////////////////////////////////////////////
თქვენ სპეციალისტი ხართ?

Posted by: bird*mad*girl 7 Mar 2014, 22:58
თერაპევტი ვინც ხართ კითხვა მაქვs და იქნებ დაწეროთ რა smile.gif

Posted by: Perigee 14 Mar 2014, 10:30
ენის უკანა მხარის სიყვითლეს რა ეშველება?

spy.gif

Posted by: nolt83 19 Mar 2014, 20:21
მოგესალმებით ზუსტად არ ვიცი სად დავსვა ეს კითხვები და აქ დავწერ და ძალიან გთხოვთ მიპასუხეთ ან ლინკი დამიდეთ სადაც ვიკითხავ, მადლობა წინასწარ.
ადამიანის რომელ ორგანოს არ მიეწოდება სისხლი და ეს ორგანო საიდან იღებს ჟანგბადს ? და
ადამიანის რომელი ორგანო ვერ გრძნობს ტემპერატურის ცვალებადობას?
აი ჩემი კითხვები

Posted by: SebastiAn9 21 Mar 2014, 22:48
ხალხო იქნებ თქვენ მაინც დამეხმაროთ,

ძალიან მშიშარა და პანიკიორი ვარ, რასაც ვშოულობ(ბოლო თვეებია) თითქმის ყველაფერს ექიმებთან და წამლებში ვხარჯავ, ვარ 24 წლის, მგონი რომ ყველაფერი მე მჭირს, ვკითხულობ აბსოლუტურად ყველაფერს სამედიცინო ლიტერატურაში და თითქმის ყველაფერი მგონია რო ჩემ თავს ხდება,

1 თვის წინ დამეწყო ზურგში ტკივილები და მეგონა რომ რაღაცა უბედურება მჭირდა წავედი გადავიღე რენდგენი და ყველაფერი წესრიგში გაქვსო, მერე კუჭნაწლავი ამტკივდა იქაც გავადაღებინე და ნუ რაღაცა დიეტა და წამლები დამინიშნეს სერიოზული არაფერია უბრალოდ რეჟიმის დარღვევის და არასწორი კვების ბრალიაო...

ერთი 10 დღის წინ გავცივდი და მახველებდა, მივედი ექიმთან რომელმაც მითხრა რომ ფილტვები ისევ წესრიგში მაქვს( და გადაღებას აღარ გირჩევ რადგან უკვე გადაიღე რენდგენი და ამდენჯერ არაა კარგიო) დამინიშნა წამლები, რის შემდეგაც მესამე დღეა ხველება მომირჩა თუმცა სიცხე მაქვს(დაბალი), ძალიან პანიკაში ვარ, - წაკითხული რახან მაქვს რომ დაბალი სიცხემ შეიძლება ფილტვებიდან მოდიოდეს და სერიოზული რამე იყოსო სულ იმაზე ვფიქრობ რომ კიდე წავიდე და გადავიღო..
მთელი დღეებია და ღამები არ მძინავს და სულ ამაზე ვფიქრობ, იქნებ დამეხმაროთ... ნერვული სისტემის ბრალია თუ რა ხდება...

მადლობა წინასწარ,

Posted by: rock in rose 21 Mar 2014, 23:17
SebastiAn9

ხო, სწორი უთქვამს ექიმს, რენტგენის გადაღება ასე ხშირად არ შეიძლება.
ჩაამთავრე მკურნალობა, თუ სიცხეები ისევ დაგრჩა, მერე მიაკითხე ისევ ექიმს და მან გადაწყვიტოს გადაგიღოს თუ არა რენტგენი.

QUOTE
მთელი დღეებია და ღამები არ მძინავს და სულ ამაზე ვფიქრობ, იქნებ დამეხმაროთ... ნერვული სისტემის ბრალია თუ რა ხდება...


არ გაქვს არაფერი სანერვიულო დამიჯერე. smile.gif

Posted by: SebastiAn9 21 Mar 2014, 23:24
rock in rose
მადლობა რჩევისთვის,

მესამე დღეა უკვე ვსვავ წამლებს, ხველის მიუხედავად სიცხე მაქვს(დაბალი), და სულ 7 დღიანი მკურნალობაა რამე პროგრესი ხოუნდა მქონდეს sad.gif სისხლის ანალიზიც ნორმაში მაქვს ეგეც გავაკეთე...

QUOTE
არ გაქვს არაფერი სანერვიულო დამიჯერე.

ეჰ რავიცი მესამე დღეა გაშეშებული სახით დავდივარ

Posted by: rock in rose 22 Mar 2014, 09:51
SebastiAn9

ჩაიტარე ეგ მკურნალობა ბოლომდე, დაბალი სიცხე ყოველთვის არ ნიშნავს, რომ ფილტვებში პრობლემებია.

Posted by: Eirene 22 Mar 2014, 13:14
თავი მივარტყი მაგრად 2 დღის წინ
2 დღეა ძალიან მტკივა, გაბრუებული ვარ
ნუ ღებინება არ მქონია, დიდად არც თავბრუსხვევა, თვალებში ვერ ვიყურები უბრალოდ კარგად

მივიდე ექიმთან თუ გამივლის? დღეს მინდოდა, მაგრამ ჩEმი ექიმი დაკავებული იყო.. საშიშია რამე?

Posted by: ninilion 23 Mar 2014, 13:14
გამარჯობათ, masked- მა რაღაც წამლების სია ჩამომიწერა და ზოგი ვერ გავარკვიე და იქნებ დამეხმაროთ? იოდ 100 აუგ, სელემ 100აუგ, ცინკი 5მგ ც ვიტამინთან(ეს გასაკეთებელია ნემსად?) ლიმფომიოზოტი 3 ამპულა, ენგისტოლი 3 ამაპულა, ტრაუმელი 3 ამპულა, კოენციმ კომპოზიტუმი 2 ამპულა ესენი გაიკეთე ერთადო და ესენი თითო ამპულა ავურიო და გავიკეთო?

* * *
QUOTE (Masked @ 2 Mar 2014, 05:10 )
ninilion
ეფტე4 და ტეესჰა ანალიზი, კორტისოლ, გლუკოზე. შეიძლება არის ჰიპატჰირეოზე.

წერილების ყუთი სავსე გაქვთ და ვერ გიგზავნით მესიჯს

Posted by: Sargerass 30 Mar 2014, 00:18
მოგესალმებით

მეგობრებო იქნებ მირჩიოთ რამე... ჩემს მეუღლეს ხელზე ასე წინწკლებივით დააყარა.. თვეებია ასე აქვს,,(მგონი წლებიც კი) ხან გაუწითლდება, ხან გაუფერმკრთალდება მაგრამ არ ურჩება..

http://s006.radikal.ru/i214/1403/2b/1717b7b683a4.jpg

მივიდა დერმატოლოგთან, გამოუწერენ წამლებს რომლებიც 2 კვირა 10 დღე შველის და ვსო..მერე ისევ უღიზიანდება..ალერგიული თითქოს არაა..დიაგნოზი ვერ დასვეს..იქნებ გვირჩიოთ ან კარგი დერმატოლოგი ან რამე (( ამდაგვარად აყრის კიდევ კეფაზე უკვე 10 წელზე მეტია.

Posted by: 1715 1 Apr 2014, 18:18
ვარ 22 წლის გოგო , ამ ბოლო დროს მაწუხებს უძილობა და დაბალი წნევები , აი შეიძლება ღამე დამეძინოს და 2 საათში გამეღვიძოს ან საერთოდ არ დამეძინოს და მერე წნევა 70-50ზე 80-60ზე და ა.შ. მოკლედ რისი ბრალი შეიძლება იყოს ? ? და რომელ ექიმს უნდა მივმართო ? წინასწარ დიდი მადლობა

Posted by: lasha-17 7 Apr 2014, 11:33
მუცლის შებერილობის დროს გააქტივებული ნახშირი უნდა დავღეჟო, თუ გადავყლაპო? ჭამის შემდეგ უნდა მივიღო თუ მანამდე?

Posted by: ia_Baggins 10 Apr 2014, 13:23
ბოლო 2 თვეა ცხვირიდან სისხლი ჟონავს, არ ჩამომდის მარა ცხვირის მოწმენდის დროს ყოველთვის ცხვირსახოცს სისხლები მოყვება

რა ჯანდაბაა? ეხლა გაციებული ვარ და განსაკუთრებით შემაწუხა ამ სისხლიანმა გამონადენმა

Posted by: androtbt 18 Apr 2014, 02:13
QUOTE (ia_Baggins @ 10 Apr 2014, 13:23 )
ბოლო 2 თვეა ცხვირიდან სისხლი ჟონავს, არ ჩამომდის მარა ცხვირის მოწმენდის დროს ყოველთვის ცხვირსახოცს სისხლები მოყვება

რა ჯანდაბაა? ეხლა გაციებული ვარ და განსაკუთრებით შემაწუხა ამ სისხლიანმა გამონადენმა

აუცილებლად მიაკითხე ექიმს, და გირჩევ რენტგენიც გადაიღო თავის ქალაზე ყოველი შემთხვევისთვის, შეიძლე ბა სისხლი გაქვს ცოტა შეთხელებული, ცხვირის კაპილარები დასუსტებული რის გამოც ადვილად სკდება კაპილარი პატარა გაღიზიანებაზეც და სისიხლი გამოდის მარა დააკვირდი ცხვირის ნესტოდან მოდის თუ უფრო მაღლიდან, აუცილებლად მიდი ექიმთან სანამ რაიმე გართულება მოყვა.

Posted by: BadbadGirl 18 Apr 2014, 20:24
ნევროლოგის თემას ვერ ვპოულობ ამიტომ აქ ვიკითხავ.
დავით გუნიაზე რას მეტყვით?


წინასარ მადლობა ყველანაირი ინფოსთვის.

Posted by: rock in rose 18 Apr 2014, 20:50
BadbadGirl

მე დავით გუნიას ნეიროქირურგს ვიცნობ და ინგოროყვაში მუშაობს. თუ მასზეა საუბარი, მაშინ ძალიან კარგია ექიმია.

Posted by: BadbadGirl 18 Apr 2014, 21:44
rock in rose
კი მაგ ექიმზე ვაუბრობ. ძირითადად რა სახის ოპერაციებშია კარგი?

Posted by: rock in rose 18 Apr 2014, 22:29
BadbadGirl

თავის ტვინზე ჰემორაგიულ ინსულტებში და სიმსივნეებში.

Posted by: atila92 19 Apr 2014, 18:58
აქ ვიკითხავ
თითებს რომ ატკლაცუნებენ
ეგრე ვატკლაცუნებ ხელის თითებს, ფეხის თითებს, მაჯებს, კოჭებს, კისერს, მხრებს, ხერხემალს
ხოდა მითხრეს ნუ შვრებიო biggrin.gif
მარილები ჩაგიდგებაო თუ რაღაც

Posted by: webgirl77 23 Apr 2014, 10:41
QUOTE (Summer_Rain @ 3 Jan 2011, 12:34 )
სალამი!!

ცუდია, რომ არაა თემა "კითხვა გასტროლოგს"
მარტო ჩემს გარშემო იმდენი ვიცი, რომლებსაც აწუხებს კუჭი..


იქნებ აქ მაინც მიპასუხოთ ჩემს გასაჭირზე და პრობლემებზე.


დავიწყებ თავიდან.
ივლისის ბოლოს დამეწყო კუჭში სპაზმების მსგავსი, 2-3 დღე გამიგრძელდა, ნოშპამ და მცირე დიეტამ მიშველა.
აგვისტოს ბოლოს კი პამიდორმა მომწამლა,
მეორე დღეს ვიდიეტე
მესამე დღეს ხინკალი ვჭამე ისიც არ შემერგგო
მეოთხე დღეს ხაჭაპური, ისევ პირღებინება
შემდეგ 1 კვირიანი დიეტა..
შემდეგ აჯაფსანდალი მივირთვი, მაინც ვერ გადაამუშავა კუჭმა.

დავამატებ იმასაც, რომ როგორც კი გულისრევის შეგრძნებას ვიგრძნობდი ეგრევე ნერვებს ვერ ვთოკავდი
მთელი სხეული მეჭიმებოდა. და ასანამ არ მოვიცილებდი არასასურველ საკვებს კუჭიდან განუწყვეტლივ მაკანკალებდა..
შემდეგ რის ვაი ვაგლახით ვახერხებდი ჩაძინებას.. sad.gif

გადავწყვიტე გასტროლოგთან მისვლა, ზონდი არ გადამაყლაპა, მითხრა ვეგეტო-ნევროზი გაქვსო, რომელიც გულის რევითაა გამოხატულიო და გამიშვა ნევროპათოლოგთან.
ნევროპათოლოგმაც ნევროზის ნიშნებიაო და გამომიწერა წამლები, ერთმა მადა დამიკარგა, მეორემ გული ამიჩქარა. ისევ სარაჯიშვილის სიროფი ავიღე და ვსვამდი საღამოობით.

სექტემბრიდან ჩემით დიეტა დავიწყე სერიოზული, ოქტომბრის ბოლომდე ვიდიეტე, თითქოს უკეთ ვგრძნობდი თავს, მაგრამ კუჭში სიმძიმე ისევ მქონდა და საღამოობით გულისრევის შეგრძნება ისევ მაწუხებდა.

ამიტომ გადავწყვიტე ზონდი გადამეყლაპა, იქ სადაც ვარ დაზღვეული, აისი გრუპის სადაზღვევოში, გავიკეთე გასტროფიბროსკოპია.
ექიმმა კუჭის ანთებაო, ნაღვლსი მომატებული რაოდენობაო და მაღალი მჟავიანობაო დამისვა დიაგნოზად. ზედაპირული გასტროდუოდენტი (მგონი სწორად მახსოვს)

ნოემბრის დასაწყისიდან დავიწყე დიეტა, კვება გრაფიკით, დილით ულსეპანს ვღებულობდი, სიმალგელის სიროფს ჭამის წინ, 40 დღის მერე ლანსოლით ჩავანაცვლე ულსეპანი. ბევრად უკეთესად ვგრძნობდი თავს, თუ არ გავითვალისწინებთ იმას რო კვირაში ერთხელ მაინც მაწუხებდა საღამოობით გულისრევის შეგრძნება. და ამავდროულად ძალიან მოვიკელი.












მივედი კითხვამდე
მოვრჩი მკურნალობას.
გავიდა 2 თვე
და 30-ში ღამით მაინც ცუდად გავხდი
არადიეტური არაფერი მიჭამია არადა
ისევ კუჭში სიმძიმე მაქვს
წვა დამეწყო
თითქოს რაღაც მიჭერს კუჭზე და მექაჩება




ძალიან ვნერვიულობ
ნერვებსაც ვეღარ ვთოკავ
მგონია რაღაც საშინელება მჭირს, რატომ ვერ გამოვჯანმრთელდი sad.gif
ოდნავ ცუდად ვიგრძნო თავი და სულ ცუდე ვფიქრობ.





მაინტერესებს
ასეთი ჩივილებით რას მირჩევთ
მივიდე სხვა გასტროლოგთან
კიდევ ნევროპათოლოგთან?
თუ რავქნა?


ძალიან განვიცდი უკვე
დეპრესიის ნიშნებიც მაქვს
ეს 3-4 დღეა წესიერად არაფერი მიჭამია, იმდენად მეშინია რო შემაწუხებს
ისედაც მეწვის და მექაჩება კუჭი და რაღააც რო ვჭამო მგონია უარესად მაწყენს.

მოკლედ გელოდებით კომპეტენტურებო
მირჩიეთ რამე.

Natuka NGN

მგონი შენ უფრო შეგიძლია კითხვაზე მიპასუხო
იქნებ კარგი გასტროლოგის ან ნევროპათოლოგის კოორდინატები მომცე და
მირჩიო როგორ მოვიქცე.


მადლობა მას, ვინც ამდენი წაიკითხა smile.gif

zustad aseti simptomebi mkonda,kuchis tkivili,simzimis shegrzneba,gulis revis shegrzneba,gulzmarva,mokled kidagam gadavdeki chkuidan,damewyo nevrozi,shishebi,ramac gaauaresa mdgomareoba. reebi ar vipikre,kuchis kibo,shidsi,mokled gavreke lol.gif ,sxva ragaceebis amtkivda am nevrozul ponze. mokled chavylape zondi da agmomachnda zedapiruli antraluri gastriti,seriozuli araperi.
Me girchev nekxiums,magan mishvela, 20 mg. dilas uzmozze,20 mg. zilis win plius dieta da simalgeli chamis shemdeg. wesit unda gishvelos . war.gif

Posted by: ავგაროზი 25 Apr 2014, 23:03
ამერია : )) უკაცრავად

Posted by: Lacrimosa_ 29 Apr 2014, 13:00
მეგობრებო, მოგესალმებით.

ყავის დალევის შემდეგ ხელები მიკანკალებს.
გადავწყვიტე აღარ დავლიო ყავა, თუმცა არ ვფიქრობ, რომ ეს პრობლემის მოგვარების გზაა.
ფაქტია, რომ რაღაც დარღვევაა.
ძალიან მაინტერესებს, რატომ ახდენს ასეთ გავლენას და რასთან შეიძლება იყოს კავშირში.
ხომ ვერ მეტყვით?

Posted by: LOTUSi 30 Apr 2014, 01:05
კარგი რევმატოლოგი ან ნორმალური, არ იცით?

მარცხენა ფეხი მტკივდება ხოლმე ღამით, ან საღამოობით. ტკივილი იმდენად ხშირად არაა, რამდენადაც სიმძიმის და დაჭიმულობის შეგრძნებისმ მსგავსი. უფრო მუხლთან მაგრამ სახსრებიც მტკივა. პლიუს მარცხენა ფეხზე წითელი ლაქები მიჩნდება, ეხლა ძაან მომემატა, ნარკომანივით მაქვს :@ კონტრაცეპტივებს უნდა შევეშვა და რევმატოლოგთანაც ვაპირებ მისვლას.

Posted by: Lacrimosa_ 30 Apr 2014, 12:38
ეს რა აქტიური განყოფილება ყოფილა smile.gif

Posted by: giorgil599 2 May 2014, 08:59
შეკითხვები ინტერნისტებს(თერაპევტებს)

Posted by: BadbadGirl 3 May 2014, 10:50
გამარჯობა
ანალიზის მიხედვით, ერითროციტების დალექვის სიჩქარე 25 თუა (ნორმა 5-15), საშიშია რამე? 83 წლის ბებო მყავს შეშინებული და მე არ ვიცი ეს რარაცეები რომ ვუპასუხო.

გმადლობ წინასწარ.

Posted by: Solveig 3 May 2014, 14:19
LOTUSi
QUOTE
პლიუს მარცხენა ფეხზე წითელი ლაქები მიჩნდება, ეხლა ძაან მომემატა,

QUOTE
კონტრაცეპტივებს

ჰმ...ლაქებში რა იგულისხმება?


QUOTE
ყავის დალევის შემდეგ ხელები მიკანკალებს.

კოფეინის ბრალია.
ეცადე, მაგარი ყავა არ სვა.

Posted by: Lacrimosa_ 3 May 2014, 16:54
Solveig
QUOTE
QUOTE
ყავის დალევის შემდეგ ხელები მიკანკალებს.


კოფეინის ბრალია.
ეცადე, მაგარი ყავა არ სვა.


დიდი მადლობა kiss.gif

მე იმის მეშინია, რაიმე სერიოზულთან ხომ არ არის კავშირში. რატომ უნდა მოქმედებდეს ეგრე საშინლად.


Posted by: Moby-Dick 8 May 2014, 13:47
......................................................................................

Posted by: qalbatoni_inkognito 10 May 2014, 14:20
იქნებ ვინმემ მიპასუხოთ,ჩემს 16 წლის შვილს დღის მეორე ნახევარში სტაბილურად აქვს სიცხეები 37,2-37,4, უკვე მეხუთე თვეა.პრაქტიკულად ყველაფერი გამოირიცხა.სისხლის საერთო ანალიზში ცვლილებები არ აღენიშნება. რა შეიძლება იყოს ამის მიზეზი?

Posted by: Lacrimosa_ 12 May 2014, 15:34
ჩემს 55 წლის დედიკოს წნევა "უთამაშებს" უკვე ერთი კვირაა.
ადის 140-160-180-ზე და ჩამოდის ისევ ხან წამლით ხანაც წამლის გარეშე.
გულის წნევაც შესაბამისად იმატებს.

ანალიზები აიღო და არაფერში არა აქვს ცვლილება.
ფარისებრი, თირკმელები, ყველაფერი გამოირიცხა.
არადა ფაქტია, რომ ქალი საჭმლის ჭამის დროსაც კი იღლება და მაღლა ასდის წნევა.
ადრე სუპერ ენერგიული იყო ყოველთვის და ახლა არაფრის ძალა არა აქვს.

რა ვუყო ან ვისთან მივიყვანო, ვერ ვხვდები

Posted by: happy dreamer91 13 May 2014, 11:21
Lacrimosa_
ყავის შემდეგ არა, მაგრამ ალკოჰოლის მიღების შემდეგ მიკანკალებს ხოლმე, ზოგჯერ რამდენიმე დღეც მიმყვება და არ მომწონს ეგ ამბავი. ახლა რომ დავაკვირდი ცოტა ახლაც მიკანკალებს. საინტერესოა რა იწვევს.
* * *
qalbatoni_inkognito
მე 22 ვარ და ეგრე ვარ, უკვე სადღაც 4-5 თვეა, თავიდან დავდიოდი ექიმებთან და არსად არაფერი არ იყო, ზოგჯერ არ მაწუხებს, ზოგჯერ კი ვგრძნობ. და რა ჯანდაბაა ვერ გავიგე sad.gif

Posted by: qalbatoni_inkognito 14 May 2014, 09:16
happy dreamer91
ექიმების ერთერთი ვერსიით,გარდამავალი ასაკის ბრალია,მაგრამ თქვენი შემთხვევაც ადასტურებს,რომ ასაკი არაფერ შუაშია. რა ვქნა,რა ვიღონო,არ ვიცი. ზოგად ანალიზებში ცვლილებები არ აღენიშნება,და მივყვე და ყველა ანალიზი გავუკეთო,რაც კი არსებობს??? ისე,იქნებ სპეციალისტი გამოგვეხმაუროს ვინმე

* * *
Lacrimosa_
გინეკოლოგთან იყავით? შეიძლება ეს კლიმაქსის ბრალი იყოს.

Posted by: LLL_DDD 14 May 2014, 10:34
ხველების საწინააღმდეგო წამალი შეგიძლიათ მირჩიოთ რაიმე? რასაც ექიმის დანიშნულება არ სჭირდება,ლაითი?

გრიპი მქონდა 2-3 დღე, რომელიც მალევე მორჩა (ფერვეკსები და მსგავსი ჩაი არ მშველის).

თნქს wink.gif

Posted by: BadbadGirl 14 May 2014, 12:25
ირაკლი კაციტაძეზე რას მეტყვით? ფსიქონევროლოგიაო როგორც გავიგე.

Posted by: LuciDiTy 14 May 2014, 14:07
გამარჯობათ, მაქვს მწვავე ბრონქიტი. ექიმმა დამინიშნა პრეპარატი ბრონქოიმუნალი. როგორც ანოტაციაში ვნახე, ეს პრეპარატი რელევანტურია, თუ გამომწვევი ბაქტერიაა და მე ლაბორატორიული გამოკვლევა არ გამიკეთებია. მაინტერესებს, კვლევის გარეშე, შეუძლია, თუ არა ექიმს რომ მიხვდეს ბაქტერიულია თუ ვირუსული?
კიდევ საქართველოში სადმე ნახველის ვირულოგიას აკეთებენ (ეგეთი რამე თუ არსებობს smile.gif )?

Posted by: happy dreamer91 14 May 2014, 16:13
qalbatoni_inkognito
18 წლის ასაკშიც ასე ვიყავი დაახლოებით ერთი წელი გაგრძელდა და თავისით გამიარა. ახლაც ძიროთადი გამოკვლევები გავიკეთე და არაფერი იყო და რავიცი ველოდები იქნებ გამიაროს. ერთი რენტგენი უნდა გადავიღო. ორმა ექიმმა მიმისმინა მასIნ და არაფერი იყო ფილტვებში და აღარ გადავიღე და ახლა მაინც ვაპირებ რომ დავმშვიდდე საბოლოოდ.

Posted by: qalbatoni_inkognito 15 May 2014, 10:47
QUOTE
happy dreamer91

გისურვებთ გამოჯანმრთელებას! როგორც ვხედავ,კომპეტენტური არავინ პასუხობს აქ sad.gif

Posted by: BadbadGirl 16 May 2014, 22:10
სახელი და გვარი შემშლია.
მამუკა მაკარიძე - ზე მაინტერესებს რას მეტყვით? წინასწარ მადლობა.

Posted by: UzerNeim 21 May 2014, 02:24
სად უნდა ვიკითხო არ ვიცი და თუ რამე გადაიტანეთ ჩემი შეკითხვა შესაბამის განყოფილებაში.

მოკლედ, ასეთი პრობლემა მაქვს:

ქუთუთოს ქვემოთ მაქვს ნატკენივით, აი წითელი მუწუკივით, ოღონდ პატარა, საიდანაც სულ გამოდის რაღაც სითხე, მცირე რაოდენობით, რომელიც ხმება და იმას გავს, აი აღალგაღვიძებულზე რომ აქვთ ადამიანებს, რა ქვია, არ ვიცი.
თითებს რომ მივაჭერ, ეს სითხე და ცოტა სისხლიც გამოდის. ჩირქს არ გავს.
რა შეიძლება იყოს? უკვე ძალიან დიდი ხანია ასეა, 2-3 თვეა. აღარ უნდა შეხორცებულიყო და დამცხრალიყო? ან ის მოყვითალო სითხე რატომ გამოდის მუდმივად, რომ ხმება მერე.

Posted by: Just_smile 21 May 2014, 20:55
ოქსალინი მენინგიტისგან იცავს?rolleyes.gif

Posted by: kolia84 22 May 2014, 02:13
არ ვიცი სად დავწერო მარა რამე ისეთი წამალი არ არსებობს, კოშმარები რომ არ დაგესიზმროს?


sad.gif

Posted by: უვიცი 22 May 2014, 16:00
არც ვიცი სწორ ადგილზე თუ ვწერ, ან ვინმე თუ არის შემომსვლელი sad.gif
თერაპევტი მირჩიეს, ეს მაქვს:
ყელის ტკივილი ყლაპვისას და ისეც (უფრო დილისკენ), თავის ტკივილიც, სიცხე (ტაი–ქოლდი, სხვა დროს ნიმესილი დავლიე და შემიმსუბუქა) კისერთან ჯირკვლები გამკვრივებული მეჩვენება biggrin.gif (როცა ვეხები)
ყურის არისკენ უფრო ეგ ტკივილები, შიგნით. ცხვირიდან–ყელში თითქოს სითხე გადამდის ხოლმე რაღაც, პირიდან სისხლი გამომივიდა ნახველივით

Posted by: bfjribhs 27 May 2014, 12:58
რამე ტკბილს რო ვჭამ (მურაბა,შაქრიანი ჩაი ან უბრალოდ წყალი, პავიდლო და ა.შ.) მტკივდება პანკრეასი და რისი ბრალია?
ისე ალერგია მაქვს თუ გამოგადგებათ გარკვევისთვის
რძეს ან ჰისტამინის შემცველ რამე პროდუქტს რო ვიღებ ასევე მკივდება პანკრეასი

Posted by: bfjribhs 28 May 2014, 14:08
ადმინ დახურე ეს თემა მაინც არავინ არ გვცემს პასუხს
რამდენი კითხვაც გინდა დასვი აზრი არ აქვს

Posted by: kolia84 6 Jun 2014, 13:25
ვინმემ უცებ მიპასუხეთ რაა, ვალერიანის მაქსიმალური დოზა რამდენი შეიძლება?

ან რამე კარგი დამამშვიდებელი რა ვიყიდო?

Posted by: Patagonia 8 Jun 2014, 11:15
საჭმლის შემდეგ რამდენ ხანში უნდა დავაყოლო წყალი?
ზოგი ამბობს ეგრევეო, ყელს საკვები ნაწილაკები რომ არ მიეკრას და კიბო რომ არ გამოიწვიოს საბოლოო ჯამშიო.
ზოგი ამბობს 20 წუთშიო, ზოგიც საათნახევარშიო. კუჭის წვენს საკვების მონელება უნდა დავაცადოთო და ცივი წყლით ხელს ვუშლითო.

სიმართლე სად არის?

Posted by: super fast jellyfish 9 Jun 2014, 11:04
Patagonia
მე ექიმმა მითხრა ჭამამდე ნახევარი საათით ადრე და ჭამის მერე 1 სთ უნდა გავიდესო.
შავი ღვინო შეგიძლია ჭამისას სვა მეტი არაფერიო smile.gif



კითხვა მაქვს ასეთი, მე3 დღეა თავის არეში ტკივილი მაქვს მარჯვენა მხარეს. ტკივილი ზედაპირულია და თან მოვლითი.
გეგონება რაღაცა მაჭერს თავზეო. ვფიქრობ რომ გამიცივდა, შეილება სხვა მიზეზი იყოს?

Posted by: Patagonia 9 Jun 2014, 17:10
super fast jellyfish
QUOTE
მე ექიმმა მითხრა ჭამამდე ნახევარი საათით ადრე და ჭამის მერე 1 სთ უნდა გავიდესო.

აჰა, მეც მასე ვფიქრობდი smile.gif

Posted by: Piqush 10 Jun 2014, 16:09
გამარჯობა, ატარაქსთან დაკავშირებით მჭირდება ინფორმაცია, უფრო სწორად ზედოზირებასთან...
მოკლედ, ამას წინათ ფსიქოთერაპევტმა დამინიშნა ატარაქსი ძილის წინ (ძილის პრობელემბი მაქვს, დღე-ღამე მაქვს არეული), ჰოდა 25მგ-იანი ატარაქსის ნახევარი უნდა მიმეღო ძილის წინ 30 საათით ადრე. აღმოჩნდა რომ ამ წამალზე რეაქცია არ მქონდა და ვერ მაძინებდა, ამიტომ შევეშვი... გუშინ მოვინდომე რომ 12საათზე დამეძინა და დავლიე ატარაქსის ერთი მთლიანი ტაბლეტი და კიდევ ნახევარი, სამი საათი ვიწრიალე და ვერ დავიძინე, ოდნავადც არ მეძინებოდა, ავდექი და მეორე პატარა ანხევარიც მივაყოლე, ანუ, ჯამში 2 25მგ-იანი ტაბლეტი დავლიე და ცოტა ხანში კი ჩამეძინა. დღეს ძლივს გავიღვიძე, ახლაც გაბრუებული ვარ, + მტკივა თავი, კეფის არეში მარჯვენა მხარეს მოძრაობის დროს, საჭმელს ვერ ვჭამ, საყლაპავთან რაღაც მიშლის ხელს, სითხეებ სვსვამ რომ იქნებ მალე მეშველო... თვალებში დაბნელებული არ მაქვს, მაგრამ მსგავსი რამ მაინც აღმენიშნება :დ
მოკლედ, საშიშია რამე? კიდევ უნდა ველოდო რამე ეფქტს? ან როდოს გაივლის? და რეგიდრონის მიღება თუ შეიძლება ახლა??

Posted by: Tissier 10 Jun 2014, 16:53
გუშინ მსუბუქად გავცივდი დღეს უკვე 38 და რამოდენიმე ხაზი მაქვს სიცხე,
ნიმესილი დავლიე ორჯერ უკვე და მიშველის ვითომ ?

თუ მომიწევს ექიმთან მისვლა და რეცეპტები : (

Posted by: Mr Adam 13 Jun 2014, 11:46
როგორ გავიგი რომელი ვიტამინები მაკლია?
რა ანალიზები უნდა გავიკეთო?

Posted by: taamo 13 Jun 2014, 12:17
ამ ბოლო დროს ხშირად მაწუხებს თავბრუსხვევა, ამასთან ერთად ბოლო 3 დღეა თითქოს თვალებიც მტკივა, რისი ბრალი შეიძლება იყოს?

Posted by: kolia84 14 Jun 2014, 02:19
გულისრევის წინააღმდეგ რა დავლიო?
* * *
აქ ხო კაციშვილი არ შემოდის................

Posted by: lika_law 17 Jun 2014, 20:10
გამარჯობა
მეუღლეს პირის ღრუში უკვე წელიწადზე მეტია უჩნდება თეთრი ბუშტუკი სტომატიტივით, მტკივნეულია საკმაოდ როგორც თვითონ აღწერს, გაუვლის, ისევ გაუჩნდება, სხვადასხვა ადგილებში, ეხლა აიღო ნაცხი და ამოითესა ,,სტრეპტოკოკუს დურანს" უხვი რაოდენობით და ,,ე.კოლი" ზომიერი ზრდით...იქნებ გაგვარკვიოთ რა არის? მადლობა წინასწარ

Posted by: Sargerass 19 Jun 2014, 16:46
შემოიხედეთ თერაპევტებო რაა biggrin.gif

ბებიაჩემს ძალიან აწუხებს ვენები..ფეხები სულ ლურჯი აქვს ისე ეტყობა...ექიმმა დაუნიშნა წამლები ქვეყნის, 4 თვეა შედეგი 0-ი აქვს.. პირიქით..ეხლა უკვე თითქმის ვეღარ დადის, არადა საკმაოდ ენერგიული ქალია.. 70 წლისაა.. ეუბნებიან ექიმები ასაკის ბრალიაო.. აღარაფერი ეშველება? იქნებ მითხრათ ვისთან მივიდეთ, რომელ ექიმს ეკუთვნის ან კარგი ექიმი ხომ არ იცით...ძალიან მეცოდება, მარტო ცხოვრობს .

Posted by: rock in rose 19 Jun 2014, 17:50
Sargerass

მამუკა ბოკუჩავასთან მიიყვანეთ თბილისის ცენტრალურ საავადმოყოფოში(იგივე რკინიგზის საავადმყოფოა ყოფილი).

Posted by: Sargerass 19 Jun 2014, 23:55
rock in rose
გაიხარეთ.. დიდი მადლობააა 2kiss.gif

Posted by: keandra 22 Jun 2014, 09:43
სირბილის დროს (თუნდაც ვარჯიშის) და ისედაც მარჯვენა მკერდში მჭვალავს ხშირად, გულის მოპირდაპირე მხარეს. ზოგჯერ აუტანლადაც კი. საღამოობის დავრბივარ, ვვარჯიშობ. მეგონა გადამივლიდა დროთა განმავლობაში და არ გადის. რომელ ექიმთან უნდა მივიდე? ვის გავესინჯო ვერ მირჩევთ?

Posted by: angelochka_ 28 Jun 2014, 18:10
Bავშვს Aქვსს კალცინატები ,კოტეტიშვილმა არ უნდა Aრაფერიო,წამლები ა. IნიშNებაა???რამეს Xომ რ გამოიწვევს???

Posted by: ემპეწრი 3 Jul 2014, 19:25
სად ვიკითხო ეს არ ვიცი
ჰოდა
ყურებში სიჩუმეში რაღაც შიშინი რომ მესმის ყრუდ
რას მივაწერო?
ყურსასმენებს?
დაყრუების პირველ სტადიას? biggrin.gif

Posted by: Keti/Keti/Keti 7 Jul 2014, 00:46
ჩაიტარეთ გამოკვლევა ,,ტონალური აუდიომეტრია" ნებისმიერ ყელ-ყურ ცხვირის კლინიკაში. თუ სმენის დეფიციტი გამოვლინდა აუცილებლად მიმართეთ ექიმს.


ამერიკაში ჩატარებული კვლევის მიხედვით, მოზარდების 12-15%-ს აღენიშნება სხვადასხვა ხარისხის სმენის დაქვეითება. უხშირესად ეს დაქვეითება შეუქცევადია და დაკავშირებულია ყურსასმენებთან, მიტომ განსაკუთრებით მნიშვნელოვანია ზრუნვა სმენის დაქვეითების პრევენციაზე.

ხმამაღლობის ერთეულად მიჩნეულია 1 ბელი ანუ 10 დეციბელი. 85 დეციბელზე მეტი ხმაური ჯანმრთელობაზე ცუდად მოქმედებს. შეეცადეთ გააკონტროლოთ, რომ მუსიკალურ აპარატურას ხმა 85 დეციბელზე მეტი არ ჰქონდეს.

Posted by: Greenie 10 Jul 2014, 00:27
გამარჯობათ smile.gif
კათეტერის ამოღების (ვენიდან) შემდეგ ჩემს მეგობარს ხელი გაუყვითლდა, გალურჯდა & ტკივა ცოტათი. ეს გალურჯებული ადგილი 7-9 სმ-ის დიამეტრისაა.
რისი ბრალი უნდა იყოს? არასწორი ამოღების? თავიდან არ ჰქონდა ესე, ბევრი სისხლი წამოუვიდა კათეტერის ამოღების შემდეგ, მაგრამ არ გალურჯებია 1-2 დღე.
ვიცი, რომ აქ ზუსტ პასუხს ვერ მივიღებ & ჯობია, ექიმს გაესინჯოს, მაგრამ საქმეც ისაა, რომ ვიყავით ბევრთან & ნორმალური ექიმი ვერ ვიშოვნეთ.
ხოდა, იქნებ, ნორმალური რჩევა აქ მაინც მივიღო ან ვინმემ დამაკვალიანოთ სად მივიდეთ.
მადლობა წინასწარ.

Posted by: მადამ ფისუნია 11 Jul 2014, 20:08
ერთ პატარა ჩეკუშკასთან ერთად დალეული 10 აბი მწვანე ვალერიანი ტრანკვილიზატორივით იმოქმედებს და გამთიშავს?

Posted by: Kaxab 14 Jul 2014, 13:47
ფეხბურთის თამაშისას გავაკეთე მკვეთრი მოძრაობა, მარცხენა ფეხი გავიქნიე მარჯვნივ, ტანი კი პირიქით შევატრიალე და მეტკინა მარჯვენა მხარეს ღვიძლი როა იქ. იმ თამაშში ორჯერ მომიხდა ასე. ახლა ღამით ლოგინში გადაბრუნებისასა ან ოდნავ მკვეთრი მოძრაობისას და ხელის დაჭერისას მტკივა ღვიძლთან ჭვალივით.
რა პროცედურები უნდა ამის მკურნალობას?
დღეს მესამე დღეა ფეხბურთის თამაშიდან და თითქოს მიკლო, მაგრამ მაინც მაინც მაწუხებს.

Posted by: konkretula 17 Jul 2014, 11:13
მადის მოსამატებელი წამლის დანიშნა რა ანალიზების საფუძველზე ხდება?

Posted by: konkretula 18 Jul 2014, 11:53
გთხოვთ სასწრაფოდ მიპასუხოთ ვინმემ!..
დღეს დილით რომ გავიღვიძე იღლია მტკიოდა და ვიფიქრე დაჭიმული მაქვს ცუდად მედო თქო... მერე სამსახურშ რომ წავედი უფრო და უფრო მომემატა... დილით რომ შევიმოწმე არაფერი მჭირდა, ახლა ხელს რომ ვისვავ პატარა მრგვალივით მაქვს. რისი ბრალი შეიძლება იყოს?

რომელ ექიმს მივმართო?
დღეს ვერსად ვეღარ მივალ სამსახურში ვარ და შაბათს სადმე მუშაობენ ექიმები?

Posted by: marto 18 Jul 2014, 13:03
არ ვიცი სად დავსვა ეს კითხვა
3 თვის წინ დავეცი და ეხლა თავის ტომოგრაფია გადავიღე და მიწერია

სუსტად კალციფიცირებულია ეპიფიზი და გვერდითი პარკუჭთა ქოროიდული წნულები
პარანაზალური წიაღები პნევმატიზირებულია
დიდი ზომის კალცინატი ფიქსირდება ფალქსის წინა ნახევარში
ვლინდება ფრონტალური კორტიკალური ატროფიის ნიშნებიო

ეს ბოლო რომ დავგუგლე რაღაც საშინელი შედეგები მოყვება ამ დიაგნოზს

ვინ ნდნა ვნახო ნერვოლოგი თუ ნევროპათოლოგი თუ რა ვქნა საერთოდ

ან ვინ არის ნორმალური ექიმი ეგეც არ ვიცი

: ((((((((((((((((((


Posted by: zy_ra 18 Jul 2014, 14:22
ორწლინახევრის ბავშვს ჩავუტარეთ სისლის საერთო ანალიზი და გემოგლობინი ამოუვიდა 96 და ნორმა კი არის 110 დან 160 მდე ხოლო ლიმფოციტები 57 და ნორმა არის 20 დან 40 მდე, ხო ვერ მეტყვით ეს ნორმალურია თუ სახიფათოა და რაიმე დამატებითი გამოკვლევაა ჩასატარებელი?

ყველა დანარჩენი ანალიზი ნორმაშია!

Posted by: marto 18 Jul 2014, 22:39
ვინმე მიპასუხებს ნეტავ? : (((((((((((((

Posted by: an apple 21 Jul 2014, 00:43
rock in rose
მიპასუხეთ რა, გთხოვთ.
3 თუ 4 დღის წინ, რომ გავიღვიძე, სისუსტე და ცუდად ყოფნა ვიგრძენი. ცოტა ხანში ამტკივდა სახსრები და თითქმის ყველა ძვალი და სიცხე მქონდა 38.
მას შემდეგ, სიცხე ოდნავ მიწევს და დღის მეორე ნახევარში სისუსტე მემართება. ხველების და სხვა ნიშნების გარეშე, ფილთვების ანთება შესაძლებელია? კიდევ თირკმელის მხარეს მაქვს ყრუ ტკივილი განუწყვეტლივ.

Posted by: pigeon2 22 Jul 2014, 21:06
მაინტერესებს წონაში მომატების დროს მეტაბოლურ პარამეტრებზე კვლევა რა ანალიზებს მოიცავს?

Posted by: >>levani9<< 22 Jul 2014, 21:48
გამარჯობათ ეზოს კატამ გამკაწრა 3 დღეა ხელზე. და ვისთან მივიდე ან უნდა მივიდე საჭიროა მისვლა ვაქცინაზე?

Posted by: ილონენ 23 Jul 2014, 22:53
>>levani9<<
სად მოახერხე ლევან biggrin.gif

მე სულ დაკაწრული არ დავდიოდი ჩემი კატის გადამკიდე, მარა იმას იმდენი აცრა და წამალი ქონდა ნაყლაპი აქეთ ჩემგან რამე არ აეკიდა მეშნოდა biggrin.gif

ანტირაბიული აცრები გაიკეთე სანამ გაგვიცოფდი ეს ჯანმრთელი კაცი biggrin.gif

ისე მამენტ შეიძლება არც არაფერი მოგივიდეს მარა მაინც რისკია, მერე გვიანი არ იყოს

Posted by: Solveig 24 Jul 2014, 01:09
>>levani9<<
ანტირაბიულ კაბინეტში მიდი აუცილებლად!

ქუჩის კატა და საერთოდ ნებისმეირი გარეული ან გაგარეულებული ცხოველი საშიშია.

Posted by: pigeon2 24 Jul 2014, 12:30
QUOTE (pigeon2 @ 22 Jul 2014, 21:06 )
მაინტერესებს წონაში მომატების დროს მეტაბოლურ პარამეტრებზე კვლევა რა ანალიზებს მოიცავს?

იქნებ მიმითითოდ ეს კითხვა ამ განყოფილებაში უნდა დამესვა თუ სხვაგან რადგ აქ არავინ პასუხს არ მცემს

Posted by: infuzoria 24 Jul 2014, 15:26
კისტა შეიძლება სიმსივნეში გადაიზარდოს?

Posted by: rock in rose 24 Jul 2014, 22:29
an apple
QUOTE
მიპასუხეთ რა, გთხოვთ.
3 თუ 4 დღის წინ, რომ გავიღვიძე, სისუსტე და ცუდად ყოფნა ვიგრძენი. ცოტა ხანში ამტკივდა სახსრები და თითქმის ყველა ძვალი და სიცხე მქონდა 38.
მას შემდეგ, სიცხე ოდნავ მიწევს და დღის მეორე ნახევარში სისუსტე მემართება. ხველების და სხვა ნიშნების გარეშე, ფილთვების ანთება შესაძლებელია? კიდევ თირკმელის მხარეს მაქვს ყრუ ტკივილი განუწყვეტლივ.


ძალიან დიდი ბოდიში გვიან რომ გპასუხობთ, დასასვენებლად ვარ წასული და ახლა შემოვედი ფორუმზე.
ახლა როგორ ხართ? ფილტვების ანთება ყველაზე ნაკლებ სავარაუდოა, ვირუსულ ინფექციას გავს.

Posted by: >>levani9<< 25 Jul 2014, 17:32
Solveig
ილონენ
მადლობა პასუხისთვის. ვიყავი მისული. სამი დღის უკან. ავუხსენი სიტუაცია და მაინც უნდა გაიკეთოვო. ეზოს კატა იყოვო თუნდაც სახლის ყოფილიყოვო მაგაზეც უნდაო.
და დამბომბეს ნემსებით biggrin.gif

Posted by: Keti/Keti/Keti 26 Jul 2014, 00:32
infuzoria
95% შემთხვევებში კისტა არ გადაიქცევა ქსოვილოვან წარმონაქმნად, თუმცა მნიშვნელოვანია მისი ანატომიური მდებარეობა smile.gif, რადგან კიბომ შესაძლოა მოგვიანებით წარმოქმნას ცისტები
* * *
pigeon2
პირველ რიგში გამოთვალეთ თქვენი სხეულის მასის ინდექსი, http://www.fitni.ge/?page_id=323
თუ წონის დეფიციტი გაქვთ შემდეგ იწყება კვლევა მიზეზისა, რომელიც შესაძლოა იყოს ალიმენტური (არასაკმარისი კვება და ენერგიის ჭარბად ხარჯვა) ან ნივთიერებათა ვცლის დარღვევა, რომლის მიზეზებიც საკმაოდ ბევრია, ამიტომ უნდა მიმართოთ ექიმს, გირჩევთ თქვენით არ მიიღოთ მედიკამნტები smile.gif

Posted by: pvm 28 Jul 2014, 21:07
--როგორ უნდა განისაზღვროს ადამიანს ინსულინრეზისტენტობა/ჰიპერინსულინემია აქვს თუ დიაბეტი?

---ინსულინრეზისტენტობის დროს ჰომას გამოთვლას რამდენად აქვს აზრი? (გლუკოზა-ინსულინის მიხედვით განსაზღვრა).

---ინსულინრეზისიტენტობა/ჰიპერინსულინემიის დროს გალვუს მეტის ( 50/1000 ) დანიშვნა რამდენად სწორია?

ინსულინი 28 იყო და გახდა 38, გლოკოზა ვენიდან 90 და ამჟამად 103 (1 წელი გალვუს მეტის მიღების მიუხედავად)! ანუ 1წლიან მკურნალობა უშედეგო აღმოჩნდა.


გთხოვთ მიპასუხეთ, წინასწარ დიდი მადლობააააააა

Posted by: Kobalty 29 Jul 2014, 17:34
Keti/Keti/Keti
QUOTE
რადგან კიბომ შესაძლოა მოგვიანებით წარმოქმნას ცისტები

სად წარმოშობს ხოლმე, თავისივე ადგილას?
ანუ რის სიმსივნეც არსებობს, იქვე შობს კისტებს თუ სხვაგანაც?

Posted by: pigeon2 1 Aug 2014, 00:27
Keti/Keti/Keti

ჩემ შემთხვევაში არის ზედმეტი წონის პრობლემა და ეს გამოთვლამაც დაადასტურა სხეულის მასის ინდექსი BMI- 25 აჩვენა ნუ ჩატარებული მაქვს სისხლის საერთო ანალიზი, FT-3, FT-4, TSH, პროლაქტინი, და ეხლა კიდევ ვეღარ ვიხსენებ კიდევ რაღაცის ანალიზები მაქვს გაკეთებული პასუხები არის ყველაფერის ნორმის ფარგლებში მაგრამ რატომ ვიმატებ წონაში და ან რატომ ვერ ვიკლებ ვერ ვხვდები ვიკვებები ისევე როგორც ყოველთვის და შეიძლება უფრო ნაკლებადაც თქვენი რჩევა მჭირდება ამიტომ მაინტერესებდა ,,მეტაბოლურ პარამეტრებზე კვლევა,, რა ანალიზებს მოიცავსთქო რო იქნებ კიდევ რამე გამორჩა ჩემს ექიმს და დამეხმაროთ

Posted by: Female User 1 Aug 2014, 23:03
გამარჯობა. ვარ 29 წლის ქალი. მაქვს მჯდომიარე სამსახური, აღარ ვეწევი, შვილები არ მყავს, ვიკვებები ხშირად არაჯანსაღად და რეჟიმიც არ მაქვს საუკეთესო. გენეტიკურად მაქვს მიდრეკილება დიაბეტისა და სიმსივნისაკენ . განსაკუთრებულად არაფერი მაწუხებს. ჩემი შეკითხვა კი ასეთია: ამ ყველაფრის გათვალისწინებით რამდენ ხანში ერთხელ უნდა ჩავიტარო ხოლმე პროფილაქტიკური სამედიცინო შემოწმება და კონკრეტულად რის ექიმებთან, რა გამოკვლევები და ანალიზები. გმადლობ წინასწარ

Posted by: Keti/Keti/Keti 2 Aug 2014, 11:42
pvm
ინსულინორეზისტენტობა არის მდგომარეობა, რომლის დროსაც სისხლში დარღვეულია ინსულინისა და გლუკოზის კონცენტრაციის თანაფარდობა. ამ მდგომარეობის დიაგნოსტიკა ხდებოდა შემდეი მაჩვენებლებით (გლუკოზის ტოლერანტობის ტესტი, ინსულინის სუპრესიის/დათრგუნვის ტესტი, ინსულინის ტოლერანტობის ტესტი) და ამ მონაცემებთან ასოცირებული HOMA-ს ინდექსით (მათემატიკური ფორმულით გამოითვლება).
ბოლო წლების მონაცემებით ინსულინორეზისტენტობის დიაგნოსტიკაში მეტი მნიშვნელობა ენიჭება კლინიკურ გამოვლინებებს (ანუ ჩივილებს და გამოვლენილ ნიშნებს). ასე რომ HOMA ინდექსის გამოთვლა აღარ არის ისე აქტუალური და არ გამოიყენება რუტინულად !!!!!!

მნიშვნელოვანია მოგვიანებითი შედეგები ინსულინორეისტენტობისა, რაც ასოცირდება: დიაბეტი ტიპი2, გულსისხლძარღვთა სისტემის დაავადებების და გარკვეული სისტემების ავთვისებიანი ახლაწარმონაქმნების განვითარებასთან. ამიტომ თუ დიაგნოსტირებულია ინსულინრეზიტენტობა თავისი გამოვლინებებით აუცილებელია შესაბამისი რეკომენდაციების გატარება.

რაც შეეხება გალვუს-მეტს ეს კომბინირებული პრეპარატია, რომელიც ძირითადად ინიშება დიაბეტი ტიპი-2 დროს, ხოლო ინსულინორეზისტენტობის შემთხვევაში მოწოდებულია მხოლოდ მეთფორმინი (გალვსუმეტის ერთ-ერთი კომპონენტი), რაც ამცირებს დიაბეტის განვითარების რისკს 32% შემთხვევებში.
* * *
Kobalty
კანცეროფობია ძალიან ცუდი პრობლემაა, აკონტროლეთ კისტები 6 თვეში ერთხელ და დაანებეთ ამაზე ფიქრს თავი
* * *
pigeon2
წონაში მატების ფორმულა ძალიან მარტივია smile.gif მიღებული კალორაჟი აღემატება დახარჯულს, როგორც თქვენი მონათხრობიდან ვხვდები მეტაბოლური პრობლემა არ უნდა გქონდეთ, კიდევ ერთხელ გადახედეთ კალორაჟს და გაზარდეთ ფიზიკური აქტოვობა. აუცილებლად დაგეხმარებათ კლებაში.
* * *
Female User
ყველწლიურად (გლუკოზა ლიპიდები, წნევის კონტროლი, პაპ-ტესტი (თუ სქესობრივად აქტიური ხართ), . სარძევე ჯირკვლის გამოკვლევა 40 წლიდან (რენტგენომამოგრაფია). მნიშვნელოვანია გენეტიკური წინასწარგანწყობა კიბოს მიმართ (რომელი სისტემის კიბოზეა საუბარი??) თუ ჭარბი წონა ან სიმსუქნე აღინიშნება გასატარებელია შესაბამისი პრევენციული ღონისძიებები

Posted by: Female User 2 Aug 2014, 13:52
Keti/Keti/Keti
დიდი მადლობა ქეთი. აუცილებლად გავითვალისწინებ. თუ ვინმე რამე დაამატებთ, დიდი მადლობაsmile.gif

Posted by: pigeon2 2 Aug 2014, 15:42
Keti/Keti/Keti
დიდი მადლობა ყურადღებისთვის
* * *
მინდა ავღნიშნო რომ fohow-ს სამკურნალო პრეპარატებით მომიწია მკურნალობა 2 წლის წინ გარკვეული პრობლემები მქონდა ჯანმრთელობასთან და ჩემმა მეგობარმა მირჩია ამ ფირმაში მისვლა რადგან მასაც გაგებული ჰქონდა ვითომდა მისი კარგი შედეგების მეც მივედი ექიმმა სრული დამაჯერებლობით პირობაც მომცა რომ სრულიად გამოვჯამრთელდებოდი და მეც დავუჯერე საკმაოდ სოლიდური თანხა გადავიხადე მათ პრეპარატებში და დანიშნულების მიხედვით შევუდექი მკურნალობას როცა დავამთავრე სრული კურსი ანალიზები გადავამოწმე და აი მაშინ კი მივხვდი ვის ხელშიც აღმოვჩნდი როცა საერთოდ არანაირი შედეგი არ მქონდა არაფერზე ძალიან ვინანე მაგრამ უკვე ძალიან გვიანი იყო მას შემდეგ სხვაგან მომიწია მკურნალობა სადაც მართლა შედეგი უკვე ერთ კვირაშივე დავინახე და იმ თანხის მეოთხედიც არ დამიჯდა რაც ამ ფირმაში fohow-ში გადავიხადე.
ამ ფირმის პირად ექიმსაც დაველაპარაკე და ანალიზის პასუხებიც ვანახე პასუხი მოვთხოვე მაგრამ იცით რა მიპასუხა ძალიან რთული მდგომარეობაა და კიდევ უნდა გავაგრძელოთ მკურნალობაო თანხის დაბრუნებაზე ხომ საუბარიც ზედმეტია მე ძალიან უკმაყოფილო ვარ და არ არიან სანდო ასეთი ფირმები

Posted by: pvm 6 Aug 2014, 19:39
QUOTE (Keti/Keti/Keti @ 2 Aug 2014, 11:42 )
pvm
ინსულინორეზისტენტობა არის მდგომარეობა, რომლის დროსაც სისხლში დარღვეულია ინსულინისა და გლუკოზის კონცენტრაციის თანაფარდობა. ამ მდგომარეობის დიაგნოსტიკა ხდებოდა შემდეი მაჩვენებლებით (გლუკოზის ტოლერანტობის ტესტი, ინსულინის სუპრესიის/დათრგუნვის ტესტი, ინსულინის ტოლერანტობის ტესტი) და ამ მონაცემებთან ასოცირებული HOMA-ს ინდექსით (მათემატიკური ფორმულით გამოითვლება).
ბოლო წლების მონაცემებით ინსულინორეზისტენტობის დიაგნოსტიკაში მეტი მნიშვნელობა ენიჭება კლინიკურ გამოვლინებებს (ანუ ჩივილებს და გამოვლენილ ნიშნებს). ასე რომ HOMA ინდექსის გამოთვლა აღარ არის ისე აქტუალური და არ გამოიყენება რუტინულად !!!!!!

მნიშვნელოვანია მოგვიანებითი შედეგები ინსულინორეისტენტობისა, რაც ასოცირდება: დიაბეტი ტიპი2, გულსისხლძარღვთა სისტემის დაავადებების და გარკვეული სისტემების ავთვისებიანი ახლაწარმონაქმნების განვითარებასთან. ამიტომ თუ დიაგნოსტირებულია ინსულინრეზიტენტობა თავისი გამოვლინებებით აუცილებელია შესაბამისი რეკომენდაციების გატარება.

რაც შეეხება გალვუს-მეტს ეს კომბინირებული პრეპარატია, რომელიც ძირითადად ინიშება დიაბეტი ტიპი-2 დროს, ხოლო ინსულინორეზისტენტობის შემთხვევაში მოწოდებულია მხოლოდ მეთფორმინი (გალვსუმეტის ერთ-ერთი კომპონენტი), რაც ამცირებს დიაბეტის განვითარების რისკს 32% შემთხვევებში.
* * *
Kobalty
კანცეროფობია ძალიან ცუდი პრობლემაა, აკონტროლეთ კისტები 6 თვეში ერთხელ და დაანებეთ ამაზე ფიქრს თავი
* * *
pigeon2
წონაში მატების ფორმულა ძალიან მარტივია smile.gif მიღებული კალორაჟი აღემატება დახარჯულს, როგორც თქვენი მონათხრობიდან ვხვდები მეტაბოლური პრობლემა არ უნდა გქონდეთ, კიდევ ერთხელ გადახედეთ კალორაჟს და გაზარდეთ ფიზიკური აქტოვობა. აუცილებლად დაგეხმარებათ კლებაში.
* * *
Female User
ყველწლიურად (გლუკოზა ლიპიდები, წნევის კონტროლი, პაპ-ტესტი (თუ სქესობრივად აქტიური ხართ), . სარძევე ჯირკვლის გამოკვლევა 40 წლიდან (რენტგენომამოგრაფია). მნიშვნელოვანია გენეტიკური წინასწარგანწყობა კიბოს მიმართ (რომელი სისტემის კიბოზეა საუბარი??) თუ ჭარბი წონა ან სიმსუქნე აღინიშნება გასატარებელია შესაბამისი პრევენციული ღონისძიებები

ტოლერანტობის ტესტი არასდროს გამიკეთებია. Pირველად გლოკოზა მკონდა 141 და ინსულინი სადგაც 28 , ამ. Mიხედვით ექიმმა დამიდგინა რეზისტენტობა და ჰHიპერინსულინემია. უამრავჯერ ვკითხე, დიაბეტი ხო არ დამემარტებათKო და არაო, პირიქით ანტიდიაბეტი გაქვსო. Hომას ყოველთVის ითვლის ექიმი.


Dიდი მადლობა გამოხმაურებისთVის
* * *
Gალვუსი კი არის დიაბეტიკების წამალი, მაგრამ რეზისტენტობაზე და დიაბეტზე ერთNაირი წამლები ინიშNება. თAვიდან მეტფოგამა დავიწყე და გამიორმაგდა ინსულინი. Mერე გალვუსი ზაან მომკხდა, მაგრამ ეს დასაწყისშI. საერთOდ უშEდეგო აღMოჩNდა ცემთVის

Posted by: Kobalty 7 Aug 2014, 00:11
Keti/Keti/Keti
QUOTE

კანცეროფობია ძალიან ცუდი პრობლემაა, აკონტროლეთ კისტები 6 თვეში ერთხელ და დაანებეთ ამაზე ფიქრს თავი

გმადლობთ.
და რა სახისკონტროლს გულისხმობთ?
და რანაირად ვაკონტროლო ხოლმე?

Posted by: Solveig 8 Aug 2014, 00:36
lika50

სარეკლამო პოსტი წაიშალა.

Posted by: marto 10 Aug 2014, 14:19
საძილე აარტერია რომ გამოჩნდეს რა გადავიღო? ანგიოგრაფია თუ როგორ ჰქვია

სად გადავიღო და ანგიოლოგს უნდა ვაჩვენო?
* * *
ანუ სისხლი თუ კარგად მოძრაობს და რამე ანთება ან კედლის გასქელება რომ მიჩვენოს

Posted by: BadbadGirl 10 Aug 2014, 14:31
marto
QUOTE
საძილე აარტერია

გარეთა საძილე არტერია Carotid Ultrasound
შიდა საძილე არტერია - ანგიოგრაფია.

დედაჩემმა გადაიღო - თანამედროვე სამედიცინო ტექნოლოგიების ჰოსპიტალში.

Posted by: marto 10 Aug 2014, 19:55
გასაგებია, მადლობთ




მოკლედ ჩემს სიტუაციაზე უნდა დავწერო და იქნებ ვინმე კომპეტენტურმა მიპასუხოს

2 თვის წინ უცბად დამეწყო თავბრუსხვევები და თავის ტკივილები. რადგან იქამდე 1 თვით ადრე წავიქეცი და კეფა დავარტყი, ვიფიქრე იმას არ გამოეწვია რამე პრობლემათქო და გადავიღე კომპიუტერული ტომოგრაფია, რომლის დასკვნაშიც დამიწერეს კორტიკალური ატროფიის ნიშნები. მერე მივედი ერთ ნევროპათოლოგთან და მითხრა რომ არანაირი ატროფია არ მქონდა და დამინიშნა ანტიდეპრესანტი - რომ ვნერვიულობდი ჩათვალა რომ შფოთვა მქონდა. (ტესტი გამაკეთებინა ბოლო 2 კვირა როგორც ხარ ისე უპასუხეო და ბოლო 2 კვირა ამ დასკვნის გამო ვნერვიულობდი და ისედაც შეუძლოდ ვგრძნობდი თავს).

ანტიდეპრესანტი არ დამილევია.

შემდეგ მივედი ნეიროქირურგთან, რომელმაც დაადასტურა კი არის პატარა ატროფიის კერები მაგრამ საშიში არაფერიაო. : | და სისხლძარღვები მაინტერესებსო და რაღაცეები დამინიშნა და სექტემბერში მოდიო და მერე მაგნიტო რეზონანსული გადაიღე და ის მანახეო.

მე ვერ მოვითმინე ისე ვნერვიულობბდი და გადავიღე მაგნიტო რეზონანსული და იქ ატროფია არ დაფიქსირდა, თან ამიხსნეს კომპიუტერული ტომოგრაფიის დროს ცოტა სხვა რაკურსია და ამიტომ გამოჩნდა ისეო. მოკლედ რა.. არადა ერთსა და იმავე კლინიკაში გადავიღე.

მივუტანე ნეიროქირურგს და კიო ატროფიები არისო ცოტაო თავისას მიაწვა ისევ: ( მაგრამ მოკლედ სისხლძარღვებს რომ დახედა აი საძილე არტერიაზეო მე რაც მეგონა ზუსტად ისააო მეორე ექიმს უთხრა მაგრამ მე ვერაფრით დავაცდენინე რა პრობლემა იყო. ბოლოს ვერტებრო ბაზილარული უკმარისობაო ეს მითხრა დიაგნოზი.

ანგიოგრამა ხომ არ გადავიღო საძილე არტერიის თუ რა ვქნა. ძალიან მეშინია და ვნერვიულობ.

ისეთი შეგრძნება მაქვს თითქოს გონებას ვკარგავ და თავს ძალას ვატან რომ არ დავკარგო.


Posted by: Pompiero 12 Aug 2014, 17:16
წნევა მქონდა 170/110 ერთი საათის წინ

დავწექი და თანდათან ჩამოვიდა 110/70

წამოვჯდები და ისევ ნელ-ნელა ადის. დავწვები და ჩამოდის


ეს მარტო დღეს არ ხდება. წლების განმავლობაში მივხვდი, რომ წნევის წამლის სმას აზრი არ აქვს. როდესაც ხანგრძლივად ვსვამ წნევის დამწევებს ჰიპითიაზიდთან კომბინაციაში ან მის გარეშე, წნევა როგორც წესი, ძალიან დაბლა ჩამოდის, მაგრამ ფიზიკური დატვირთვისას ან რამდენიმე საათს ზედიზედ ფეხზე ყოფნიდსას აუცილებლად მატულობს და თავისით ჩამოდის დაწოლის შემდეგ. მოკლედ, წამალი მაინც ვერაფერს ცვლის და ვერ მიცავს კრიზისგან. მაგრამ სადაა ეგეთი ცხოვრება, რომ ყოველ ერთ საათში წამოწვე?

ვის გაგიგიათ მსგავსი რამ და რა შეიძლება იყოს? ყველა ვერსია მაინტერესებს, რაც გაქვთ ექიმებს ან ვისაც მსგავსი რამ სჭირს

Posted by: NEIRON 16 Aug 2014, 00:39
პოლიკლინიკაში ექიმები (ამბულატორია) მუშაობენ შაბათს?

Posted by: mdinaradze 18 Aug 2014, 10:34
გამარჯობათ. სისხლში მონოციტები მაქვს მომატებული 2-8 არის ნორმა და 9.2 მაქვს. რა შეიძლება იყოს ამის მიზეზი? მაქვს დაბალი სიცხეები. როე არ არის მაღალი არც ლეიკოციტები არ მაქვს მომატებული ნორმაშია.

Posted by: ciania 18 Aug 2014, 10:42
გამარჯობათ. იქნებ დამეხმაროთ, გამარკვიოთ sad.gif( უეცარი კანკალი, მოდუნება, სიცხის მაღლა ავარდნა რისი სიმპტომებია? ქალბატონმა გაიკეთა ყველანაირი ანალიზი, ფილტვებზე რენტგენიც და კუჭნაწლავის ექოც, შაქარზეც თითქოს ანალიზში არაფერია, არადა კვირაში ერთხელ მაინც ხდება აასე ცუდათ... ამისთანა სცხეში აკანკალებს და მთელი ორგანიზმი თითქოს ხტუნავსო... გრძელდება სადღაც 1 საათი... მერე უბრუნდება ნორმალურ მდგომარეობას, მაგრამ რჩება საერთო სისუსტე. დაუნიშნეს აუგმენტინი 1 გრ და ფლუიმუცილი 600 მგ, კიდევ ვაგოსტაბილი. ამ წამლების სმის ფონზეც ჰქონდა ეს შეტევა (მესამედ). რა იწვევს გაურკვეველია, თერაპევტმაც ვერაფერი თქვა... იქნებ დამეხმაროთ რამეში გამარკვიოთ ძალიან გთხოვთ (((( მადლობა

Posted by: rock in rose 18 Aug 2014, 11:15
ciania

გაგიმარჯოთ.
ფარისებრი ჯირკვალი შემოწმებული აქვს? TSH-ის ანალიზი არ გააკეთებინეს?

Posted by: ciania 18 Aug 2014, 11:41
არა.. ენდოკრინოლოგი საერთოდ არ ჩაურთავთ...

Posted by: rock in rose 18 Aug 2014, 14:03
ciania

TSH-ის ანალიზი გაუკეთეთ აუცილებლად და შედეგი დაწერეთ.

Posted by: ciania 18 Aug 2014, 14:43
rock in rose

აუცილებლად 2kiss.gif ეხლა გადავიკითხე ფარისებრის დარღვევების სიმპტომები და ძალიან მენიშნა გააააააააიიიიიიიხაააააააარეეეეეეეეეეთ


სად ჯობია ხომ ვერ დამაკვალიანებთ, დიღომში ვიცი, მთელი ცენტრია, ვინმეს ხომ ვერ მირჩევდით smile.gif


Posted by: rock in rose 18 Aug 2014, 16:10
ciania

დიახ, თირეოტოქსიკოზს გავდა და მეც იმიტომ გირჩიეთ.

ბევრგან აკეთებენ კარგად, ენმედიცში, მედეაში, ავერსში, მრჩეველში. ესენი ძირითადები გირჩიეთ და ნამდვილად კარგად აკეთებენ smile.gif

Posted by: ciania 18 Aug 2014, 16:17
rock in rose
2kiss.gif 2kiss.gif 2kiss.gif

Posted by: MoniBell 24 Aug 2014, 11:36
გამარჯობათ. რადიოლოგის დასკვნა: " გულმკერდის ყაფაზი კასრისებრი ფორმის, ფილტვები ემფიზემატოზური, პნევმოსკლეროზული, ინფილტრაციული ცვლილებების გარეშე მარცხენაში ისახება მკვრივი კეროვანი ჩრდილი ( გონის კერა ) ჰილუსები უსტრუქტურო, მარცხენაში აღინიშნება პეტროფიკატები გულის ჩრდილი მარცხენა პარკუჭის ჰიპერტროფიით. სინუსები არ იშლება. "
პაციენტი 71 წლის. მიჩიეთ რამე, გთხოვთ.

Posted by: an apple 25 Aug 2014, 23:23
rock in rose
კიდევ ერთხელ მოგმართავთ smile.gif
ვიკვებები არასწორად, აქ არ მოვიყვან მაგალითებს smile.gif
ბოლო ერთ წელიწადში, რამოდენიმეჯერ დამემართა სპაზმი კუჭზე , შეტევა მქონდა ანუ. ძლივს გამოვედი მდგომარეობიდან. მაქვს დისკომფორტი ხშირად. ეჭვი მაქვს კუჭის წყლულზე. რა ნიშნები უნდა მქონდეს ამ შემთხვევაში?

Posted by: rock in rose 26 Aug 2014, 08:23
an apple

თუ წყლული არის თორმეტგოჯა ნაწლავზე, მაშინ მას ახასიათებს მშიერი ტკივილები, ანუ როცა მშიერი ხარ მაშინ გტკივდება და საკვების მიღებისთანავე გეხსნება. ასევე იცის ღამის ტკივილები.
თუ კუჭშია წყლული, მაშინ ჭამის დამთავრებისთანავე გეწყება ტკივილი.
ზოგადად ნებისმიერმა წყლულმა ისევე როგორც გასტრიტმა იცის ღებინება და გულძმარვა. თუმცა მე არამგონია მთლად წლული ჩამოყალიბებულიყო, ალბათ უფრო გასტრიტი იქნება ანუ კუჭის ლორწოვანი გარსის ანთება.

Posted by: an apple 26 Aug 2014, 13:15
rock in rose
მადლობა smile.gif
ჭამის შემდეგ მეწყება რამოდენიმე საათში . მაქვს მოწამვლის ნიშნები. გულისრევა და გულძმარვა.
გუშინ მქონდა შეტევა მაგალითად და მდგომარეობიდან ვერ გამოვედი, მეორე დღეა. არ მშველის ომეპრაზოლი და მსგავსი პრეპარატები smile.gif

Posted by: rock in rose 26 Aug 2014, 14:04
an apple

ეგ უფრო გასტრიტს გავს, აუცილებლად უნდა გაიკეთო გასტროსკოპია. და დიეტა უნდა დაიცვა. დიეტის გარეშე მედიკამენტების სმას არ აქვს აზრი.
არ გადადო, აუცილებლად მიხედე smile.gif


Posted by: Solveig 26 Aug 2014, 14:13
MoniBell
QUOTE
გონის კერა

ტუბზე გამოკვლეულია?

Posted by: an apple 3 Sep 2014, 20:41
rock in rose
ჩემო გულისხმიერო ადამიანო, სანამ ბორჯომიდან ჩამოვალ და თავს მივხედავ,კიდევ ერთხელ გკითხავ smile.gif
კიდევ დამემართა შეტევა და უფრო კარგად დავაკვირდი მოვლენებს. ცოტა ზედმეტი მომივიდა მირთმევა (ცომეული და შემვარი საკვები) და 4 საათში გავხდი ძალიან ცუდად. მთლიანად ამოვიღე ყველაფერი, მქონდა ციებ-ცხელების შეგრძნება. აუტანელი კუჭის ტკივილი. არ მიშველა არაფერმა. მას შემდეგ ვჭამ მხოლოდ ხილს და ბოსტნეულს (შიშით) და ვარ არაჩვეულებრივად .
პ.ს გასტროსკოპია ზონდთან კავშირშია? თუ არისალტერნატიული საშუალება ,რაც გამოკვლევისას ზონდს ამაცილებს?
უღრმესი მადლობა წინასწარ smile.gif

Posted by: rock in rose 3 Sep 2014, 22:39
an apple

აუცილებლად უნდა მიხვიდეთ ექიმთან. მიხედვა სჭირდება თქვენს კუჭ-ნაწლავს. ადრე წყლულს არ ვფიქრობდი თქვენს შემთხვევაში, ახლა კი უკვე ვეღარ გამოვრიცხავ.

QUOTE
პ.ს გასტროსკოპია ზონდთან კავშირშია? თუ არისალტერნატიული საშუალება ,რაც გამოკვლევისას ზონდს ამაცილებს?


გასტროსკოპია იგივე ზონდის "გადაყლაპვაა", ყველაზე კარგ მეთოდად ეს ითვლება გამოკვლევისთვის ამ შემთხვევაში, თუმცა თუ თქვენ ვერ შეძლებთ და უარზე იქნებით, მაშინ ბარიუმის ფაფა დალიეთ, ოღონდ ეს მეთოდი უფრო ნაკლებ ინფორმატიულია, მაგალითად ეროზიები ნაკლებად ჩანს ბარიუმის ფაფის შემთხვევაში. ისე კი თანამდეროვე კლინიკებში ისე აკეთებენ გასტროსკოპიას, რომ დისკომფორტი მინიმუმამდეა დაყვანილი.


QUOTE
უღრმესი მადლობა წინასწარ


რისი მადლობა
2kiss.gif

Posted by: an apple 3 Sep 2014, 22:50
rock in rose
QUOTE
ადრე წყლულს არ ვფიქრობდი თქვენს შემთხვევაში, ახლა კი უკვე ვეღარ გამოვრიცხავ.

რატომღაც მგონია,რომ წყლულია.მხოლოდ გასტრიტი ასეთ შეტევას ვერ გამოიწვევდა,ჩემი აზრით.
QUOTE
ისე კი თანამდეროვე კლინიკებში ისე აკეთებენ გასტროსკოპიას, რომ დისკომფორტი მინიმუმამდეა დაყვანილი.

თუ არ შეგაუხებთ,რომელიმე მათგანი მირჩიეთsmile.gif
2kiss.gif

Posted by: rock in rose 4 Sep 2014, 12:17
an apple

ვოვა შენგელიძე რესპუბლიკურში მუშაობდა და ძალიან კარგი იყო, ახლაც იქაა თუ არა არ ვიცი, მემგონი არ უნდა წასულიყო არსად.

ასევე ირაკლი ხმალაძე, ზუსტად სად მუშაობს არ ვიცი, აგერაა მისი კოორდინატები. http://www.vidal.ge/doctors/1315

აუცილებლად დაწერეთ რას გეტყვით ექიმი smile.gif

წარმატებებს გისრუვებთ და მალე გამოკეთდით 2kiss.gif

Posted by: zeppelin3 7 Sep 2014, 11:38
გამარჯობა, სამი დღის წინ, ღამის 5 საათზე ცუდად გავხდი, დამეწყო კანკალი სადღაც 40 წუთი გაგრძელდა, წინა დღეს დაბადების დღეზე 2 ჭიქა ღვინო დავლიე, ცხიმიანი საკვებიც მივიღე. პოლიკლინიკაში ვიყავი,მორიგე ექიმმა არაფერია, შეიძლება მცირედი ინტოქსიკაციის ბრალია ან სტრესისო, რინგერის ხსნარი გადამისხეს. დიეტა არ დამიცავს. წუხელ ისევ 5 საათზე დამეწყო კანკალი, უფრო ნაკლები სიძლიერის და ხანგრძლივობის. შეიძლება კუჭ-ნაწლავი იყოს მიზეზი? ცოტათი გაღიზიანებული მაქვს, ბ ჰეპატიტიც მაქვს და ინტერნეტში წავიკითხე კანკალი ღვიძლის მწვავე უკმარისობამაც იცისო და შემეშინდა. რომელ ექიმს მივმართო მირჩიეთ.ვარ 30 წლის სქესი- მდედრობითი.

Posted by: an apple 8 Sep 2014, 16:45
rock in rose
მოვედი დასკვნით smile.gif
"მრავლობითი სქელი და არასქელი ეროზიები" დასკვნის მიხედვით , ლორწოვანი გარსის მცირე პროლაბსი. ნაღველი ერევა კუჭს ასევე და კიდევ მჟავიანობა ძ მაღალი .
შემამოწმეს ჰელიკობაქტერიაზე და საკმაოდ მაღალი ტიტრით დამიდასტურდა.
ერთი შეკითხვა მაქვს თქვენთან, თიაქარია მითითებული კიდევ და რა თიაქარზე შეიძლება იყოს საუბარი?
დანიშნულებას ხვალ მივიღებ, მუცლის ღრუს ექოსკოპიის შემდეგ .

Posted by: rock in rose 8 Sep 2014, 19:14
an apple

წყული არაა, ეს უკვე ძალიან კარგია.

QUOTE
ერთი შეკითხვა მაქვს თქვენთან, თიაქარია მითითებული კიდევ და რა თიაქარზე შეიძლება იყოს საუბარი?


საყლაპავის თიაქარზე იქნება საუბარი.

არაა საგანგაშო მდგომარეობა, ჰელიკობაქტერიებზე ანტიბიოტიკებს დანიშნავენ და მოგვარდება. სხვა დანარცენზეც ასევე, არაა რთულად მოსაგვარებელი სიტუაცია smile.gif დიაგნოზად გასტრიტი გამოიტანეს თუ ჯერ არაფერი?

Posted by: an apple 8 Sep 2014, 23:50
rock in rose
საბოლოო დასკვნის და დანიშნულების დასადებად ნაღვლის გაშუქებაა საჭიროო.
ეროზიულ ჩანართებს მკურნალობა არ ჭირდება?
რა საინტერესეოა, არასოდეს მქონია ყლაპვასთან დაკავშირებული რაიმე პრობლემა და თიაქარი ნამდვილი "სიურპრიზი" იყო smile.gif

Posted by: rock in rose 9 Sep 2014, 09:08
an apple

QUOTE
ეროზიულ ჩანართებს მკურნალობა არ ჭირდება?


კი, სჭირდება, კომპლექსური მკურნალობა დაინიშნება.



QUOTE
რა საინტერესეოა, არასოდეს მქონია ყლაპვასთან დაკავშირებული რაიმე პრობლემა და თიაქარი ნამდვილი "სიურპრიზი" იყო


თუ მხოლოდ გასტროსკოპია გაგიკეთეს, სხვა არაფერი დ ამის ფონზე თქვეს თიაქარიაო, მაშინ საყლაპავისა თიაქარზე იქნება საუბარი, რადგან გასტროსკოპიით ვიზუალიზდება, და ძალიან ხშირად სიმპტომები არც აქვს, ისე კი ძირითადად გულძმარვა და ბოყინი ახასიათებს, ასევე ტკივილი გულმკერდში.
თუ მუცლის ღრუსა და რბილი ქსოვილების ექოსკოპიაც გაკეთდა, მაშინ თეთრი ხაზის ან ჭიპის თიაქარი შეიძლება იყოს,

Posted by: chupa82 9 Sep 2014, 12:02
მოგესალმებით, მყავს 52 წლის დედა, რომელსაც ხშირად აქვს მაღალი წნევა, თან უშუპდება მარცხენა ფეხი ( კოჭის ქვემოთ). მაინტერესებს რომელ ექიმთან უნდა მივიყვანო, სათანადო ანალიზები რომ გაუკეტოს და + მკურნალობა დაუნიშნოს? თერაპევტი მჭირდება თუ ენდოკრინოლოგი?

Posted by: rock in rose 9 Sep 2014, 12:27
chupa82

QUOTE
თერაპევტი მჭირდება თუ ენდოკრინოლოგი?


კარდიოლოგი, კარდიოლოგთან მიიყვანეთ.

Posted by: marto 9 Sep 2014, 12:27
არსებობს რაიმე ასეთი ანალიზი რომ გაიკეთო და გამოჩნდეს რა ინფექცია გაქვს ორგანიზმში?


და კიდევ

ტესტი გლუტენის აუტანლობაზე სადმე კეთდება თბილისში?

Posted by: an apple 9 Sep 2014, 15:06
rock in rose
ნაღველი სუფთაა smile.gif
დიაგნოზი -გასტრიტი და ჰელიკობაქტერია.
ვიწყებთ შეტევას ანტიბიოტიკებით . smile.gif
თიაქარი ,საყლაპავის თიაქარს გულისხმობდა თურმე. ეგ შეგრძნება, თითქოს გულზე გადგას რაღაც სიმძიმე, მქონდა ნამდვილად .

Posted by: rock in rose 9 Sep 2014, 16:25
an apple

ნუ გეშინია, არაფერი სანერვიულო არაა მაგ დიაგნოზში smile.gif მედიკამენტები+დიეტა და გამოჯანმრთელდები.

Posted by: California_Dreamin 16 Sep 2014, 17:27
არვიცი აქ ვიკითხო თუ არა ანუ სწორად ვსვამ აქ კითხვას თუ არა.


თავის შიდა ქალის წნევა მაქვს, რაღაცა პერიოდი დამეწყება აქტიურად, რაღაცა პერიოდი გამივლის.

ხოდა ახლა გულიც მერევა ხოლმე თავი როცა მტკივა.

რამე ცუდის მანიშნებელია?
როგორც კი გამიჩერდება რამე გამაყუჩებლით გულიც აღარ მერევა.

განგაშის ზარი ჩამოვცე და წავიდე ექმითან? sad.gif

Posted by: ashley_tko 17 Sep 2014, 21:50
მიპასუხეთ რაა ძაან გთხოვთ. 4 წლის ბავშვს ლიმფოციტები აქვს ძაან შემცირებული. მინიმუმ 25 უნდა ქონდეს და 13 აქვს, ნეიტროფილები კი პირიქით, მომატებული. 55 ია მაქსიმუმი და 70 აქვს. რას ნიშნავს ეს პასუხი და რა გამოკვლევები ჭირდება? ათასი საშინელება წავიკითხე ინტერნეტში და ძალიან ვნერვიულობ. მონოციტები და ბაზოფილებიც აქვს სულ ოდნავ მომქტებული,სხვა ყველაფერი რიგზეა. მიპასუხეთ რაა.
* * *
Keti/Keti/Keti
rock in rose

Posted by: rock in rose 17 Sep 2014, 22:55
ashley_tko

http://forum.ge/?f=97&showtopic=34546867&st=795 ამ თემაში იკითხეთ, პედიატრი შემოდის და გიპასუხებთ.

ისე კი ყველას აჯობებდა თუ თქვენს პედიატრს ესტუმრებოდით ან დაურეკავდით მაინც.

Posted by: ashley_tko 18 Sep 2014, 00:32
rock in rose
ძალიან დიდი მადლობა, გადავიტანე კითხვა.
ხვალ მივდივართ პედიატრთან, მაგრამ მანამდე ძაან ვნერვიულობ, ერთი სული მაქვს როდის გათენდება..

Posted by: ხუციშვილი 18 Sep 2014, 18:13
ფეხი ლურსმანს დავაბიჟე, ჟანგიანი იყო, ჰოდა მივედი ექიმთან და რაღაც აცრა გამიკეთა, არც ნახა ფეხი ისე.

თუ გასიება დაიწყო სადაც შემერჭო იმ ადგილას როგორ დავამუშავო, ან რა იქნება კარგი რო გავუკეთო?

Posted by: Solveig 18 Sep 2014, 21:57
ხუციშვილი

ქირურგთან მიდი და გაასუფთავებინე ჭრილობა.

შენით შეგიზლია, სპირტით გაიწმინდო, იოდი წაისვა და სუფთა ნახვევი გაიკეტო, მაგრამ თუ ღრმაა, მაშინ აჯობებს, რომ ექიმმა ნახოს. ისე, წესით იგოვე ექიმს უნდა ექნა ეგ, ვინც აცრა და შრატი გაგიკეთა.

Posted by: ხუციშვილი 18 Sep 2014, 22:05
Solveig
სპირტით გავიწმინდე, იოდი წავისვი, მეზობელმა რაღაც მაზი შემომიტანა, ცოტა ხნის წინ იმასაც ეგეთი რაღაც მოუვიდა და ეგეც წავისვი. იქ მისვლას აღარ ვაპირებ, არც დამსიებია, უბრალოდ ჭრილობის გარშემო გამიმაგრდა

Posted by: ashley_tko 29 Sep 2014, 15:26
ჩემს ქმარს ყელზე, ხორხთან გაუჩნდა მოზრდილი ბურთულა,რა შეიძლება იყოს? ეხოზე უნდა წავიდეთ თუ როგორ მოვიქცეთ პირველ რიგში?

Posted by: gachumdii 29 Sep 2014, 17:00
ტემპერტურის დაცემამ (35°C მდე) თავის ტკივილები იცის?

Posted by: konkretula 2 Oct 2014, 16:30
სტომატიტები მაქვს ძალიან ხშირად, ერთი რო გამივლის მეერე მეორე მეწყება და ა.შ. დავიტანჯე... თან ერთდროულად რამდენიმე. რისი ბრალი შეიძლება იყოს? ან ვის მივმართო, რომელ ექიმს?

Posted by: ილონენ 9 Oct 2014, 14:42
თემა ვერ ვნახე ნევროლოგისთვის რომ დამესვა კიტხვა და მაშინ აქ დავწერ

ზოგჯერ ტვინის ერთ მხარეს სიცივეს ვგრძნობ ხოლმე... აი ახლა ვიგრძენი და დამაინტერესა რა არის მეთქი აქამდეც ხშირად მიგრძვნია, რამოდენიმე წამი და მერე ვეღარ ვგრძნობ
ახლა მაგალითად მარჯვენა მხარეს ზემოთ ვიგრძენი და ახლა სითბოც დაემატა მარცხენა ქვემოთ მხარეში
რა უნდა იყოს?

Posted by: ილონენ 14 Oct 2014, 23:29
სანამ კინოს მე4 სტადია არ მექნება არ მიპასუხებთ? biggrin.gif

Posted by: rock in rose 15 Oct 2014, 19:03
ილონენ

არ შემოდის თერაპევტი, უფრო სწორედ აღარ, სამწუხაროდ.

Posted by: sallli1 18 Oct 2014, 22:10
გამარჯობა არ ვიცი კითხვას სწორ ადგილას ვსვავ თუ არა
ბებიაჩემს საკერავი ნემსი შეერჭო ხელზე ძარღვში და გაუსივდა მაშინვე და საშიშია რამე?

Posted by: Sargerass 19 Oct 2014, 23:25
მეგობრებო გამარჯობათ..

ჩემი მეზობლის შვილს ბავშვობაში ჰქონდა ჰიდროცეფალია..ექიმებს უთქვიათ მერე რომ ვსო მორჩა, აღარ არის წყალი ჯანმრთელი ბავშვიო უბრალოდ მერე წლების შემდეგ შეამოწმეთო.. ეხლა ის ბიჭი უკვე დიდია მგონი, არ ვიცი არ მინახსვს..25 წლის უნდა იყოს.. და უნდათ შემოწმება თავის ისევ.. ტომოგრაფია უნდა გადაიღონ? ხომ ვერ მეტყვით სად აკეთებენ და დაახლოებით რა ჯდება?

თვალიდან მოსდის თურმე ჩირქი სულ და არაფერი შველის..მაგიტომ ფიქრობენ ისევ მაგას.

Posted by: ashley_tko 20 Oct 2014, 17:00
სისხლის საერთო ანალიზზე ცალკე თემა არ გვაქვს??
მაინც ვიკითხავ,იქნებ მიპასუხოს ვინმემ sad.gif
სისხლის საერთო ანალიზში ერითროციტები,ჰემოგლობინი და ჰემატოკრიტი არის დაბალი. დანარჩენი მონაცემები ნორმაშია. რაზე შეიძლება ეჭვის მიტანა და რა უნდა გამოიკვლიოს პაციენტმა ამ ანალიზის მიხედვით??

Posted by: Eirene 31 Oct 2014, 13:56
QUOTE
სისხლის საერთო ანალიზზე ცალკე თემა არ გვაქვს??

მეც მაინტერესებს რაღაც

ლიმფოციტების რაოდენობა მაქვს დაბალი. ნორმა წერია25.0 - 50.0 % და მე მაქვს 15.3
მერე დავგუგლე და შიდსსზე ამომიგდო სტატიები

ვინმე კეთილმა დამამშვიდეთ რამით
ექიმმა ყურადღებაც არ მიაქცია მაგ ნაწილს, სხვა რაღაცაზე დამინიშნა მკურნალობა

Posted by: cunc 5 Nov 2014, 22:49
მაინტერესებს ვინმემ თუ იცით რა განსხვავებაა ატფ-ს და გტფ-ს შორის

Posted by: OiOiOi 9 Nov 2014, 19:52
მეგობრებო ერთი შეკითხვა მაქვს, რამდენად სწორად ვწერ თემაში არ ვიცი, მაგრამ თქვენი გულისხმიერების იმდი მაქვს.. ოჯახის წევრს რომელიც არის 74 წლის აქვს ფილტვების ანთება, ცოტა რთული ფორმით ანუ საშიშროებაა რომ წაყალი ჩადგეს, ექიმმა გამოუწერა ცეფამედი დილა საღამოს 1.0 -იანი შვიდი დღის განმავლობაში, აფტიაქში მითხრეს რომ ააქვთ აბსოლიტურად ერთანირი მოქმედების ცეფამედი: 1. იტალიური ეკონომშეფუთვით, რომელიც ღირს 7 ლარი და 2. ეგვიპტური ინდივიდუალური შეფუთვით, რომელიც ღირს 17 ლარი.. ასევე მითხრა, რომ ააქვთ ერთნაირი მოქმედება და ფასთაშორისი სხვაობა გამოწვეული ამხოლოდ და მხოლოდ შეფუთვის გამო.. იქნეგბ ვინმემ იცოდეს მართლა ამის გამოა სხვაობა თუ მოქმედების მხრივაც განსხვავდებიან ??

Posted by: annaninio 10 Nov 2014, 22:32
ართრიტზე თემა ვერ ვნახე და აქ ვიკითხავ თუ შეიძლება. ჩემ ნათესავს აქვს პოლიართრიტი და ვიღაცამ ურჩია წყალბადის ზეჟანგის, იგივე პერეკისის 10 წვეთის წყალში გახსნა და უზმოზე ყოველ დილას მიღება. მე რაღაც ძალიან საშIნლად მეჩვენება ეს ამბავი და ვინმეს გაგიგიათ თუ არა რამე ამის შესახებ. უკვე 10 დღეა აკეთებს და ვნერვიულობ.

Posted by: giorgevich23 11 Nov 2014, 00:23
რამდენიმე დღეა -დღეში რმდენჯერმე ვგრძნობ რომ სიცხე მაქვს-ვიზომავ და 37 გრადუსია-ეს გრძელდება დაახლოებით ნახევარი ერთი საათი და მერე ისევ 36 გრადუსზე ჩამოდის-გაცივებული არ ვარ და ვირუსიც არაფერი მაქვს(არ მაცემინებს და ცხვირში სისველე არ მაქ) არც კლიმაქსის ასაკში ვარ smile.gif რისი ბრალი შეიძლება იყოს??

Posted by: Hu_man 13 Nov 2014, 23:26
გამარჯობა, დაფენთხვისას საძინებელში გამიტყდა თერმომეტრი სხვა დროს ბურთულების სახით ვაგროვებდი ახლა იატაკსა და კარადაზე საღებავივით წაესვა. იატაკი სველი სარეცხის ფხვნილიანი ტილოთი გავწმინდე (დასაწყისში უფრო მეტად მოედო იატაკს), კარადა მხოლოდ წყლით. გავაღე ფანჯარა. მაინტერესებს რატომ არ შეგროვდა ვერცხლისწყლისთვის დამახასიათებელი ბურთულების სახით? (არ ვარ დარწმუნებული რომ ყველგან ავიღე) რამდენი ხანი არ დავიძინოთ საძინებელში? რამდენ ხანში გავა ოთახიდან? ძალიან ვნერვიულობ, რამდენად საშიშია? წინასწარ მადლობა. გთხოვთ გამომეხმაუროთ.

Posted by: giorgevich23 14 Nov 2014, 00:24
Hu_man
სავარაუდოდ უხარისხო ვერცხლის წყალი ესხა,თერმომეტრში არაა ის რაოდენობა ადამიანი მოწამლოს მისმა ანაორთქლმა
ბავშვობაში თერმომეტრის ყიდვას ვერ აუდიდნენ სახლში-კარებში მოვაყოლებდი -გავტეხავდი და დავდევდი ვერცხლის წყალს ასაღებად,ძალიან ხშირად ისე გაიფანტებოდა ვერ ვპოულობდი ,არავის არაფერი დაგვშავებია

Posted by: Hu_man 16 Nov 2014, 02:44
დიდი მადლობა პასუხისთვის , ცოტა დავმშვიდდი smile.gif smile.gif

Posted by: ^____^ 18 Nov 2014, 16:59
გამარჯობათ, პერიოდულად 1-2 დღეში ერთხელ გული მიჩქარდება, ხელები მიოფლიანდება და თვალებშიც მიბნელდებასავით, რისი ბრალი შეიძლება იყოს? sad.gif წნევა, ხან 100\110 - 65\70 მაქვს ხოლმე

და კიდევ, საშინლად მგრძნობიარე გავხდი ყველა ცუდი ინფორმაციის მიმართ, ოდნავ დაძაბულობა და უკვე ნერვები მეწეწება sad.gif( არადა არ გამოვხატავ საერთოდ

Posted by: erfiro 20 Nov 2014, 17:55
მაღალ წნევას რომელი ექიმი მკურნალობს?

მოკლედ, ვისი ეპარქიაა - ნევროლოგების, თერაპევტების, კარდიოლოგების თუ ვინმე სხვისი? drug.gif

ვის და სად მივმართოთ? rolleyes.gif

Posted by: Hu_man 28 Nov 2014, 02:56
QUOTE (^____^ @ 18 Nov 2014, 16:59 )
გამარჯობათ, პერიოდულად 1-2 დღეში ერთხელ გული მიჩქარდება, ხელები მიოფლიანდება და თვალებშიც მიბნელდებასავით, რისი ბრალი შეიძლება იყოს? sad.gif წნევა, ხან 100\110 - 65\70 მაქვს ხოლმე

და კიდევ, საშინლად მგრძნობიარე გავხდი ყველა ცუდი ინფორმაციის მიმართ, ოდნავ დაძაბულობა და უკვე ნერვები მეწეწება sad.gif( არადა არ გამოვხატავ საერთოდ

ფარისებრი ჯირკვლის პრობლემა ხომ არ გაქვს ?
* * *
QUOTE (erfiro @ 20 Nov 2014, 17:55 )
მაღალ წნევას რომელი ექიმი მკურნალობს?

მაღალი წნევა კარდიოლოგის სფეროა smile.gif))

Posted by: konkretula 29 Nov 2014, 11:02
რამდენიმე დღეა ყურში რაღაც წუილი მსემის, ბოლო თვეები კი (2-3 თვე) თავი ხშირად მტკივა . თვალებზე მაწვება შიგნიდან რაღაცნაირად და მგონია რომ თავის ქალა მაქვს დაჭიმული .

მივედი გუშინ ყურის ექიმთან, გამიშვა მერე სადღაც ანალიზზე რომელიც ორშაბათს უნდა გავიკეთო - სმენის...

სავარაუდოდ ნერვზე გჭირს რაღაცაო და როგორ ფიქრობთ შეიძლება რამე სერიოზული იყოს? ქალმა ისეთი სახე მიიღო მეგონა ვკვდები და ეგაა თქო sleep.gif

Posted by: Lacrimosa_ 29 Nov 2014, 21:19
მეგობრებო, დაბალი სიცხე რატომ უნდა ჰქონდეს ადამიანს?

ჩემს დედიკოს აქვს დაბალი სიცხეები (36.1-36.2), თავიდან მეგონა, ცუდად იდებდა თერმომეტრს, მაგრამ ფაქტია, ასე დაბალი აქვს.
პარალელურად წნევა აწუხებს ხოლმე, უცებ გულისრევის შეგრძნება აქვს, გავუსინჯავთ და მაღალია წნევა, მერე უცებ, 10 წუთში, ყოველგვარი წამლის გარეშე კარგია.

ფილტვებში ყველაფერი რიგზეა (რენტგენი გადავიღეთ),
სისხლის საერთო, ქოლესტერინი და ა.შ. ანალიზები ორშაბათს იქნება.

გულის ექიმთან ჩაწერილები ვართ.

სხვა რა შეიძლება კიდევ?

Posted by: Leo woman 1 Dec 2014, 01:03
Dეზინფექციის საშUალება სახლის პირობებშI რ. Aრის გარდა ოთAხის განიავებისა ?
DავუშVათ თU არსებობს რამე სპრეს სახით რომ ჰაერშI გავუშVათ

Posted by: Amon`Ra 9 Dec 2014, 09:36
გამარჯობათ, არ ვიცი რამდენად წსორად ვსვავ თემაში კითხვას.. იმედია მიპასუხებთ ვარჯიშობ ზოგჯერ დღეში ორჯერ და ვარჯიშის მერე აუცილებლად ვბანაობ თუმცა მაინც მაქვს ხოლმე ოფლის უსიამოვნო სუნი და მაინტერესებს რისი ბრალია შეიძლება იყოს ან რომელ ექიმს მივმართო დამეხმარეთ რა

Posted by: gramatikikina 21 Dec 2014, 18:02
მარცხენა ხელის ნეკი და არათითი მაქვს გაბრუჟებული 3 დღეა და რამის ნიშანია თუ იცით?help.gif ნეკი თითქმის მთლიანად და არათითის წვერი მარცენა მხრიდან
Amon`Ra
გიპასუხებენ ალბათ და პოლ ბრეგს უწერია, რომ შიმშილობით შეიძლება სუნის გაქრობა. კვირაში 1 დღე, მარტო წყალი. 1 კოვზი თაფლი და 1 ლიმონი უნდა შეურიო, რომ სიხისტე დაკარგო წყალმა. და მეორე დღე უნდა დაიწყო გახეხილი სტაფილოთი.

Posted by: VAMPIRE_W 23 Dec 2014, 20:10
ბრონქები მქონდა გაციებული ძალიან, მახველებდა. ვიმკურნალე და ახლა ასეთი რაღაც დამიტოვა, დღეში ერთხელ ან ორჯერ დამახველებს მარა სასუნთქი გზა იკეტება და ვერ ვსუნთქავ რამოდენიმე წამი, მერე ნელ ნელა იწყებს ჰაერის გატარებას.
რისი ბრალია? ან საშიში ხომ არ არის?

Posted by: pihtona1 10 Jan 2015, 14:31
მოგესალმებით
ერთი სიტყვით სპირტიან სასმელს რომ ვიღებ არაქვს მნიშვენობა რა სახის იქნება სასმელი 1-2 ჭიქაც საკმარისია მეწყება სახის გაწითლება არ მყოფნის ქაერი გულის აჩქარება ეს სიმპტომები ყოველთვის არ მაქვს მაგრამ ხშირად მაქვს.....

არ მაქვს ნაღვლის ბუშტი.
ინტერნეტში როგორც მოვიძიე ამნაირი სიმპტომები ახასიათებს ადამიანს რომელსაც აქვს ალერგია ალკოქოლის მიმართ ამ ალერგიას კი იწვევს ფერმენტის ნაკლებობა რომელიც ამუშავებს ალკოქოლს (алкогольдегидрогеназа - ნალკებობა).

რა შეგიძლიათ მითხრათ ამასთან დაკავშირებით და არსებობს თუ არა რაიმე პრეპარატი ამ ფერმენტის შემცველობით?

წინასწარ დიდი მადლობა....

Posted by: OKUPANT 23 Jan 2015, 13:45
სამიოდე დღის წინ გავცივდი. უფრო. სწორად კი ანგინა გამისივდა , მქონდა დაბალი სიცხე, ამას დაემატა თვალების ჩასისხლიანება.
ანგინამ გამიარა არც სიცხე მაქვს თვალები კი ისევ ისე დამრჩა :-/

Posted by: DEATHNESS 31 Jan 2015, 16:46
არ ვიცი სად დამეწერა და აქ დავწერ საჩმელს რო ჩვამ მუცელში მარჯვენა მხარეს ნეკნის ცოტა დაბლა მტკივდება და ყოველ ჩამაზე უფრო იმატებს ტკივილი სანამ მეორე დგე არ მოვა მანამდე მტკივა

Posted by: ashley judd 10 Feb 2015, 21:15
კოფეინის ნაკლებობა არსებობს? biggrin.gif
აქამდე წელიწადში მაქსიმუმ 2ჯერ დამელია ყავა და ახლა ერთ თვეში უკვე მეოთხედ ვსვამ. კოკა-კოლა და შოკოლადიც ყოველდღე მინდება, არც მახსენდებოდა ხოლმე შოკოლადის ყიდვა. ჩაისაც ცოტას ვსვამ.
ამის ფონზე შეიძლება განვითარდეს კოფეინის ნაკლებობა? თუ არ აქვს ორგანიზმისთვის მნიშვნელობა მაგ ნივთიერების შემცველობას?

Posted by: blowers daughter 15 Feb 2015, 11:15
ashley judd

QUOTE
კოფეინის ნაკლებობა არსებობს? biggrin.gif


კოფეინის ნაკლებობა არ არსებობს არსებობს კოფეინზე დამოკიდებულება


QUOTE
ამის ფონზე შეიძლება განვითარდეს კოფეინის ნაკლებობა? თუ არ აქვს ორგანიზმისთვის მნიშვნელობა მაგ ნივთიერების შემცველობას?



კოფეინს არ შეიცავს ბუნებრივად ადამიანის ორგანიზმი და მის გარეშეც უპრობლემოდ იარესებებ
უბრალოდ არის მეორე მომენტი , როდესაც შენი ნერვული სისტემა სტიმულიზირდება კოფეინით და ასე ვთქვათ ტონუსში მოდის და ამას ეჩვევა,
მერე შენ ფსიქოლოგიურად შეგექმნება ის მომენტი თითქოს რაღაც აკლია შენს ორგანიმზს ...
მაგრამ გარკვეული პერიოდის მერე გადაგივლის


................................................
ყოველ დილით მაწუხებს თავის ტკივილი , რომ ვიღვიძებ ასე ვთქვათ თავის ტკივილით მეღვიძება
შეიძლება ეს ზედმეტი კოფეინის და ნიკოტინის ბრალი იყოს?

Posted by: ashley judd 17 Feb 2015, 01:54
blowers daughter
ეგ მეორე მომენტი არ მაქვს. მიწერია, რომ არ ვსვამ ხოლმე.

Posted by: ZUCHA 17 Feb 2015, 09:21
გამარჯობათ....რა მინდა გკითხოთ, იქნებ რამე მირჩიოთ....მარცხენა ხელის მტევნის ზედა მხარე ამტკივდა და ოდნავ შეწითლებულიც მაქ, ძირითადად მოძრაობაში მტკივა ანუ თითების მოხრა-გაშლისას...კომპიუტერთან მიწევს ხშირი ჯდომა, სამსახურში მერე სახლშიც, და ალბათ მაგის ბრალია?! 1-2 დღის მერე ნაკლებად მტკივა მარა რა შეიძლება იყოს? ექიმთან ღიირს მისვლა ან მკურნალობა?

Posted by: Summer_Rain 9 Mar 2015, 15:27
გამარჯობა!
მაინტერესებს, სალონში მანიკურის გაკეთებისას კანის დაზიანება გაჭრას რა ინფექციებით დაავადების რისკი შეიძლება ჰქონდეს?
და რამდენ ხანში შეიძLება გადამოწმება მოხდა თუარა დაინფიცირება.

რამდენადაც ვიცი ასე გადადის შიდსი, ჰეპატიტები
კიდევ?

Posted by: aura 13 Mar 2015, 13:35
გამარჯობა, რჩევა მინდა თქვენგან, მოკლედ დედას აწუხებს ფეხები, ხშირად ეკვანძება, ამბობს რომ თითქოს ძვალი მექვანძებაო.. აქვს სიმძიმის შეგრძნება , 58 წლისაა საკმაოდ მძიმე წონით . ბოლო დროს აღნიშნავს რომ აქვს დილაობით თავბრუსხვევის შეგრძნება და არის გაბრუებული... წნევა მაღალი სადღაც 160-მდე... იყო ექიმთან სისხლის ანალიზები გაუკეთეს თითქოს ყველაფერი ნორმაში აქვს. რა გამოკვლევები უნდა ჩაიტაროს კიდევ, და თუ იცით ნორმალური კლინიკა , უკვე აღარ ვიცით სად მივიდეს?

Posted by: sofia1234 2 Apr 2015, 22:01
გამარჯობათ იქნებ დამეხმაროთ.
ბებია არის 65 წლის, განიცდის ძლიერ სპაზმურ ტკივილს, სიგრძეზე ძალიან სტკივდება, როგორც მიხსნის ძალიან გავს ძარღვის გაკვანძას, მაგრამ განსხვავებულიაო, ბევრად მტკივნეულიაო, დაახლოებით თვეში ორჯერ ემართება, ვენების გამოსაკვლევად ვიყავით და ყველაფერი ნორმაშია, კოაგულოგრამაც, იქნებ მირჩიოთ რა შეიძლება იყოს მიზეზი, ორივე ბარძაყის უკანა მხარეს, ერთი და იგივე მხარეს ემართება.

Posted by: jokeri777 6 Apr 2015, 21:26
გამარჯობათ, ლაბორანტთან მაქვს შეკითხვა და ამ საიტზე თუ არის ვინმე ლაბორანტი? ან სად დავსვა ლაბორანტთან დაკავშირებული კითხვა იქნებ დამილინკოთ

Posted by: Solveig 6 Apr 2015, 21:51
jokeri777

ცალკე თემა არ გვაქვს. გააჩნია, მედიცინის რომელ დარგს შეეხება კითხვა...მოძებნე შესაბამისი თემა (ენდოკრინოლოგი, ონკოლოგი, გასტრიენტეროლოგი, გინეკოლოგი, და ა.შ.) და იქ დასვი...

Posted by: beqaG 12 Apr 2015, 13:40
5 დღე გაციებული ვიყავი, სიცხეებით. პარალელურად ვიღებდი ნაზივინს და ნაზოლს, ცხვირით რომ მესუნთქა.
უკვე ერთ კვირაზე მეტია ვეღარ ვანებებ ამ საშუალებებს თავს, ცხვირი ისეთი გამომშრალი მაქვს რომ ყოველ 6 საათში ერთხელ რომ არ ჩავიწვეთო შანსი არ არის.
ანუ დამოკიდებული გავხდი, რით ჩავანაცვლო ნაზოლი? რამე ბუნებრივი საშუალება არ არის? აკვა მარისი ვერ მიხსნის ცხვირს თორემ ეგ კაი საშუალებაა.

თან ამდენი წვეთებისგან თვალები დამძიმებული მაქვს და თითქოს თავიც რაღაც გაბრუებული.

Posted by: fsiqea 14 Apr 2015, 20:04
ფერმენტალის მიღება როდის შეიძლება, ჭამის წინ, დროს თუ შემდეგ??

Posted by: iase 16 Apr 2015, 22:56
მოგესალმებით.
კითხვა მაქვს ვინმემ თუ იცის ექოსკოპიის სურათის წაკითხვა?
თუ ვინმემ იცით დავდებ და ძალიან მოკლედ მინდა მითხრათ რა განსხვავებაა ორ პერიოდში გაკეთებულ ექოსკოპიაში.
საკითხი ეხება თავის ექოსკოპიას.


პირველი ექოსკოპიის ფოტო არის ახალი ხოლო მეორე და მესამე არის ძველი, იანვარში აკეთებული.
წინასწარ მადლობას მოგახსენებთ.


* * *
QUOTE (iase @ 16 Apr 2015, 23:56 )
მოგესალმებით.
კითხვა მაქვს ვინმემ თუ იცის ექოსკოპიის სურათის წაკითხვა?
თუ ვინმემ იცით  ძალიან მოკლედ მინდა მითხრათ რა განსხვავებაა ორ პერიოდში გაკეთებულ ექოსკოპიაში.
საკითხი ეხება თავის ექოსკოპიას.
წინასწარ მადლობას მოგახსენებთ.

http://www.radikal.ru
http://www.radikal.ru


http://www.radikal.ru
* * *
მოგესალმებით.
კითხვა მაქვს ვინმემ თუ იცის ექოსკოპიის სურათის წაკითხვა?
თუ ვინმემ იცით დავდებ და ძალიან მოკლედ მინდა მითხრათ რა განსხვავებაა ორ პერიოდში გაკეთებულ ექოსკოპიაში.
საკითხი ეხება თავის ექოსკოპიას.

Posted by: ZUCHA 18 Apr 2015, 17:33
ამ თემაში მარტო კითხვები ისმება პასუხის გამცემი არავინაა და რა აზრი აქ ამ თემას?! biggrin.gif

Posted by: Elephant 19 Apr 2015, 17:55
გამარჯობათ....... არ ვიცი სად დავსვა კითხვა და აქ ვიკითხავ თქვენის ნებართვით smile.gif ესეიგი ყველაფერს რაც მეტალისააა ხელს მოვკიდებ თუ არა ნაპერწკლებს ყრის, თუ დიდი ხანი დავრჩი რკინასთან ახლოს იმუხტება და რომც არ შევეხო ვგრძნობ ამ მუხტს გარკვეულ დისტანციაზე და რატო მემართება ესე?

Posted by: sandrikia 22 Apr 2015, 23:11
გამარჯობათ. მაინტერესებს რას ნიშნავს მდგრადი ხველა ანუ როგორ გავიგო როგორია მდგრადი ხველა..

Posted by: sandrikia 26 Apr 2015, 20:46
????????????????????????????????????????????????????????????????????????????????????????????????????????????

Posted by: ciyvi77 30 Apr 2015, 19:33
ხომ არ იცით ნაწოლების პრევენციისთვის რა გამოიყენება ლოგინში გასაფენად და სად შეიძლება ინახოს?

Posted by: Summer_Rain 30 Apr 2015, 21:53
Xალზე თმის მოძრობა, ხელით მოპუტვა რამდენად საშიშია?

Posted by: Lаki 30 Apr 2015, 21:58
Summer_Rain
QUOTE
Xალზე თმის მოძრობა, ხელით მოპუტვა რამდენად საშიშია?


სერიოზულად საშიშია!

Posted by: Summer_Rain 30 Apr 2015, 23:01
რატომ რა შეიძლება გამოიწვიოს?
?????Lаki

* * *
Lаki

შენმა პასუხმა შემაშინა. ამიტომ უკეთ ავხსნი და იქნებ მიპასუხოთ კომპეტენტურებმაა


ტუჩს ზემოთ მაქვს პატარა ხალი, ამობზეკილია ის ადგილი რაზეც ხალია.
მუქი ფერის არ არის, შუაში კი ძაან პატარა წერტილი უფერული აქვს.
ერთი ღერი თმა მქონდა ამ ხალზე და 2 ღერი გვერდით.
ვიფიქრე გვერდითას მოვიძრობ ხელითთქო და ეტყობა ხალზე არსებულ თმაზეც მომიხვდა და ისიც მოძვრა.
ძალიან მარტივად მოძვრა, დიდი ძალდატანების გარეშე.
ერთი დღე გავიდა მას მერე და თითქოს დაჭიმული მაქვს ის ადგილი.

მითხარით რამდენად საშიშია ეს და აუცილებელია სპეციალისტთან ვიზიტი?
თუ კი, ვის მირჩევთ?

Posted by: sandrikia 3 May 2015, 22:05
გამარჯობათ. უკვე 15 დღეა ნაქ სიცხე 37/ 37.1 გაციებული ვიყავი მაგრად. 2 ჯერ შემიბრუნა ეხლა აღარ ვარ გაციებული მარა ნახველი დამიტოვასავით. და ცოტა აი ხელში შიგნით გიღიტინებს სავით და გეგონება უნდა ამოახველოო რომ განთავისუფლდეო. ნუ ეხლა შედარებიტ უკეთ მაქ ყელი. მარა 8 საათის მერე ტემპერატურა ხდება 37. აი გეგონება დანიშნულ დროსო. ხოლო ესე 11 საათისკენ ისევ იწევს ტემპერატურა დაბლა. წავედი მოვასმენინე ფილტვები. არაფერი. მერე გავეშუქე არაფერი. გავიკეთე სისხლის ანალიზი საერთო ყველაფერი წესრიგშ იყო გარდა ალერგიული ფონისა ანუ უნდა ყოფილი 5 და არის 6. რა შეილება იყოს ამის მიზეზი. ნერვოზმა შეილება სიცხეები იცოდეს?

Posted by: keTili Zia 5 May 2015, 11:38
გამარჯობა.
მგონი აქ უნდა ვიკითხო...

ცივი წყლით დაბანვისას ფეხზე დამემართა ნერვის ანთება... ვისმევ მალამოს, გავიკეთე 3 ნემსი... კიდევ მაწუხებს.
ფიზიკური აქტივობა (ველო, ლაშქრობა) ხელს შეუწყობს მორჩენას? თუ, გავაგრძელო ნემსების კეთება?
მაწუხებს, როცა სკამზე დიდხანს ვზივარ, ადგომისას. 15-20 წამი მტკივა და მერე ვეღარ ვგრძნობ. სიარულისას ხელს არ მიშლის.

Posted by: Lizikuna3 7 May 2015, 11:56
keTili Zia
ფეხი რა ადგილზე გტკივათ? რამდენი ხანია რაც გტკივათ? რა ნემსები გაიკეთეთ და რა მალამოს ისმევთ? ტკივილის ხასიათი როგორია?

Posted by: mzeko 9 May 2015, 14:47
ვაიმე ხახლო მიშველთ რაა ეს რა ექიმები გვყავს გამაგიჟეს უკვე სამი წელზე მეტია, მოკლედ სამი წლის წინ ვმუშაობდი ტუბ დისპანსერში ხოდა იმ ზაფხულს თავი რომ დავანებე სამუსაოს გავხდი ცუდად თავიდან ყურადღება არ მივაქციე მეგონა უბრალო გაციება იყო რადგან სურდოთი და ყელის ტკივილით დამეწყოო და ცოტახანი სიცხეებიც მქონადა, მერე რომ არ გამიარაა ამ ყველაფერმა გადავწყვიტე ექიმთან მისვლაა თუმცა დიდ შეღავათი არაფერი. რამოდენიმე ხნის შემდეგ შევამჩნიე რომ ყელის მიდამოში ჯირკვლები გამეზარდა თან ტკივილებიც მქონდა ამ ჯირკვლების გამო, რომ იგებდნენ ტუბში ვმუშოაბდი ეგერევ იწყებდნენ ტუბი ხომ არ აგქვს, მე კი მიკვირდა სისუსტეს არ ვგრძნობდი, სიცხეები არ მქონდა უმადობა არ მჭირდა, მოკლედ გადავიღე ეს ფილტვები,ჩავაბარე ანალიზები გავიკეთე გულმკერდის მრავალშრიანი ტომოგრაფია და მეუბნებიან რომ ყველაფერი სუფთა და არ მჭირს ეს ტუბი. მერე მითხრეს რომ იქნებ გლანდების ბრალია გავიკეთე ამ გლანდების ოპერაციაც დაა მანც მაქვს ჯირკვლები თან მტკივა ამასთანავე. თან საინტერესო ის არის რომ ზომაში არ იზრდება ეს ჯირკველიბი და პუნქციასაც ვერ მიკეთებენ რადგან არ არის საკმარისად დიდი ზომისო. ვიყავი ჰემატოლოგთანაცა და მითხრა რომ არ არის საშიში ეს ჯირკველბი და თავისით გამივლის.

დღეისათვის მაქვს ასეთი სიმპტომები:
1. ჯირკველბი ყელის მიდამოში და მტკივა ამსათანავე
2. ყელში ბგრძნობ სისხლისა და ცუდ გემოს
3. მეუბნებიან რომ ხანდახან ამომდის სუნი პირის ღრუდან
4. მაქვს თბაროს ხვევები ოღონ გამუდმებით არა შიგადაშიგ.
5. მაქვს მკლავებისა და ფეხების ტკივილი


მოკლედ რას მირჩვეთ ვის მივაკითხო და რა ვქნაა?

Posted by: westsider 12 May 2015, 13:59
სისხლის ანალიზის წაკითხვა შეუძლია ვინმეს?

ალანინი იგივე გპტ 52 U/L (მაქსიმუმ 50 მდე)
მონოციტი 10% (ნორმა მაქსიმუმ 9%)
ბილირუბინი 2.53 მგ/დლ ( ნორმაა 1.10)

ვინმეს შეუძლია ამ მონაცემებიდან რამე გაიგოს?

Posted by: Leo woman 15 May 2015, 21:03
მაქვს სწრაფი მეტაბოლიზმი და წონაში ვერ ვიმატებ.მირჩიეთ რამე პრეპარატი რაც საკვების უკეთ შეწოვას უწყობს ხელს???
წინასწარ გიხდით მადლობას

Posted by: keTili Zia 19 May 2015, 11:22
QUOTE (Lizikuna3 @ 7 May 2015, 11:56 )
keTili Zia
ფეხი რა ადგილზე გტკივათ? რამდენი ხანია რაც გტკივათ? რა ნემსები გაიკეთეთ და რა მალამოს ისმევთ? ტკივილის ხასიათი როგორია?

თავიდან მეგონა კუნთის ტკივილი და მეთქი თავისით გამივლის... ტკივილმა რომ მოიმატა, მერეღა მივხვდი ნერვი რომ იყო.
წოლისას ან სიარულისას არ მაწუხებდა. რომ დავჯდებოდი, ადგომისას მიჭერდა ფეხს... მტკიოდა საჯდომის კუნთი, ბარძაყის კუნთი და მუხლს ქვემოთ კუნთი.
აფთიაქში მირჩიეს ბერინფერ ინგელჰაიმის ნემსი და ფებროფუდის მალამო.
გავიკეთე 3 ნემსი, ვისმევდი მალამოს და შედეგი არ იყო... მომენტალურად გამიარა და ისევ დამეწყო ტკივილი.
გავიკითხე და Anodyne-Dual მოგარჩენსო... ესეც 3 ცალი გავიკეთე. 2 ნემსმა შედეგი იქონია, თითქოს გამიარა, მაგრამ 4-5 დღეში მოიმატა კუნთის ტკივილმა (დახველებისას თუ ვიჯექი, მტკიოდა საჯდომის კუნთი და ადგომისას მუხლს ქვემოთ კუნთი, მაგრამ შედარებით მსუბუქად) და მესამეც გავიკეთე გასული კვირის ხუთშაბათს.
ამჟამად, რომ ვზივარ, ფეხი მაქვს დამძიმებული, თითქოს 15 კმ გაიარაო... ადგომისას, მუხლის სახსარში ვგრძნობ ოდნავ უსიამოვნო ტკივილს. რომ გავივლი 10-15 ნაბიჯს, ყველა უსიამოვნო შეგრძნება მივლის, ვეღარ ვგრძნობ.
აქტიურ ცხოვრებას რომ არ მივსდევდე, არაფერი პრობლემა, დაველოდებოდი მოთმინებით...
მინდა ვარჯიში გავაგრძელო, ველოთი გავიდე, მთაში წავიდე და რა ვქნა, დაველოდო, თუ სხვა რამეს მირჩევთ?
მეშინია გართულების... გუშინ, ანძიდან ფეხით ჩამოვედი და ოდნავ მოიმატა "დაღლილობამ".

მადლობა გამოხმაურებისთვის.

Posted by: newsgirl 19 May 2015, 23:12
მამაკაცებში ანალური მეჭეჭების მიზეზი აუცილებლად პაპილომა ვირუსია?

Posted by: mask- 20 May 2015, 16:48
ელექტრული ველის მახასიათებლები მედიცინაში ამის პრეზენტაცია მაქვს გასაკეთებელი და ვერ ვასწრეებ ვინმემ თუ იცით მზა პრეზენტაცია სად არის ან თქვენ თვითონ გაქვთ გამომიგზავნეეთ რაა :/

Posted by: torckiko 20 May 2015, 18:57
გავიკეთე მუცლის ღრუს ეხოსკოპია და ექიმს არ მოეწონა ღვიძლი მირჩია კატეს გადაღება, დიაგნოზი ასეთია: ღვიძლი არ სცილდება ნეკნთა რკალს, მარჯვენა წილის საგიტალური ზომა -133მმ (ნორმა <140მმ.), მარცხენა წილის საგიტალური ზომა 56მმ. (ნორმა <75მმ.). ღვიძლის კიდეები სადა, მკაფიო, პარენქიმის ექოგენობა მომატებული. ექოსტრუქტურა მკვეთრად დიფუზურად არაერთგვაროვანი, მარჯვენა წილის ექოსტრუქტურა მსხვილმარცვლოვანი. ღვიძლის ვენები და სანაღვლე სადინარები არ არის დილატირებული. კარის ვენი სდიამეტრი 13მმ. (ნორმა 8- 14მმ.) არის აქ რამე ისეთი რომ აუცილებელი იყოს კომპიტერული ტომოგრაფია? ვინც ერკვევით იქნებ მიპასუხოთ რაააა.

Posted by: Lizikuna3 21 May 2015, 16:00
torckiko
ღვიძლის ფუნქციები AST, ALT, GGT, ვირუსულ ინფექციებზე ანალიზები გაკეთებული გაქვთ? ეს სურათი, ერთი შეხედვით (თვენი ავადმყ. ისტორია არ ვიცი) კტ კვლევას არ საჭიროებს.

Posted by: torckiko 21 May 2015, 19:16
QUOTE (Lizikuna3 @ 21 May 2015, 16:00 )

ღვიძლის ფუნქციები AST, ALT, GGT, ვირუსულ ინფექციებზე ანალიზები გაკეთებული გაქვთ? ეს სურათი, ერთი შეხედვით (თვენი ავადმყ. ისტორია არ ვიცი) კტ კვლევას არ საჭიროებს.

არ მაქვს გაკეთებული, ეხლა ვაპირებ და რამე სერიოზული რომ იყოს ეხოზე ხომ გამოჩნდებოდა? ტკივილები კი მაქვს მარჯვენა მხარეს და კენჭი მეგონა, მაგიტო გადავიღე ეხო.

Posted by: Lizikuna3 21 May 2015, 20:07
torckiko
მითუმეტეს კტ კვლევის აზრს ვერ ვხედავ თუ ეს ანალიზები არ გაქვს გაკეთებული.

Posted by: torckiko 21 May 2015, 20:37
QUOTE (Lizikuna3 @ 21 May 2015, 20:07 )

მითუმეტეს კტ კვლევის აზრს ვერ ვხედავ თუ ეს ანალიზები არ გაქვს გაკეთებული.

ამ ანალიზებს გავიკეთებ აუცილებლად და დიფუზურად არაერთგვაროვანი რამე სერიოზულზე მიუთითებს? მაგას გაუსვა ხაზი ექიმმა.

Posted by: Lizikuna3 21 May 2015, 23:40
ალბათ უფრო ჰეპატიტებზე. თუმცა ანალიზების გარეშე, წინასწარ დიაგნოზებზე ფიქრი არ ღირს

Posted by: torckiko 22 May 2015, 11:13
Lizikuna3

ძალიან დიდი მადლობა, ამ დღეებში გავიკეთებ ანალიზებს და ვნახოთ რა იქნება.

Posted by: Lizikuna3 22 May 2015, 11:42
torckiko
არაფრის, წარმატებები!

Posted by: lurdes 26 May 2015, 20:38
გთხოვთ, ამიხსნათ და მირჩიოთ:
სისხლის საერთო ანალიზი გავიკეთე საკუთარი ინიციატივით
ყველა მაჩვენებელი ნორმაშია, გარდა:
ერითროციტები – 4.07-ია და ჰემატოკრიტი – 35.5
რამდენად საშიში მდგომარეობაა და რა აგვაკეთო, ვის მივმართო

წინასწარ დიდი მადლობა
გთხოვთ მირჩიეთ

Posted by: torckiko 26 May 2015, 21:37
QUOTE (Lizikuna3 @ 21 May 2015, 16:00 )

ღვიძლის ფუნქციები AST, ALT, GGT,

გავიკეთე ეს ანალიზები, ყველაფერი ნორმის ფარგლებშია, სისხლის საერთოშიც მარტო ლიმფოციტებია მომატებული 50 და როე 18 , სხვა რა გამოკვლევის ჩატარება არის შესაძლებელი? საოპერაციო ვარ საკვერცხეზე და სანამ ღვიძლზე არ გაირკვევა რა ხდება ისე ოპერაციას არ გავაკეთებთო.

Posted by: Lizikuna3 27 May 2015, 14:32
torckiko
ვირუსულ ინფექციებზე ანალიზები გაიკეთეთ? ც, ბ ჰეპატიტი?
* * *
lurdes
სულაც არ არის საშიში, უბრალოდ ნორმის ქვედა ზღვარზეა. დასვენება, ჯანსაღი ჰაერი, რკინით მდიდარი საკვები.

Posted by: lurdes 27 May 2015, 21:31
Lizikuna3
დიდი მადლობა გამოხმაურებისთვის
შემეშინდა
შევეცდები, რაციონი გავამდიდრო ამ მხრივ

Posted by: Leo woman 29 May 2015, 09:59
Lizikuna3
გამარჯობაsmile.gif
ძალიან გთხოვთ მირჩიოთ ანტიბიოტიკი ბრონხების ანთებისას.
ვსვამ გრიპის საწინააღმსეო პრეპარატებს მაგრამ სიცხე 37 გრადუსი ვერ დავწიე.
ფილტვის ანთება ამქვს გადატანილი და მეშინია.
წიანსწარ დიდი მადლობა.

Posted by: Lizikuna3 29 May 2015, 10:13
Leo woman
რატომ გგონია რომ ბრონქიტი გაქვს? 37 გაქვს სიცხე მხოლოდ? კიდევ რა სიმპტომები გაქვს?
როდის გაქვს ფილტვების ანთება გადატანილი?

Posted by: Leo woman 29 May 2015, 10:46
Lizikuna3
ჩემი სუსტი ადგილია ბრონხებიsmile.gif
უკვე კარგად ვიცნობ სიმპტომებსsmile.gif
ხველება მწვავე და ხშირი არ მაქვს ამჯერად იმიტომ რომ როგორც კი შევატყვე ვცივდები მაშინვე დავიწყე მიღება ჩაის სახით გრიპის საწინააღმდეგო პრეპარატის. იოლ ფორმებშია მაგრამ რსმე ეფექტური თუ არ მივიღე ვატყობ გამირთულდება.
ფილტვის ანთება დაახლოებით სამი წლის წინ მქონდა.
ახლა ვიღებ ა ფერინ ფორტეს და კარგია ეს?აფთიაქში მირჩიეს.
ხო სიცხე 37 და 2 ხახი მაქსიმუმ.საერთოდ მაღალი სიცხე არ მქონია წლებია.

Posted by: Lizikuna3 29 May 2015, 11:16
ა ფერინ ფოტე, ნორმალური მედიკამენტია. მე ანტიბიოტიკის მიღებას არ გირჩევ, მით უფრო მაშინ რომ ტემპერატურა მხოლოდ 37-ია და შესაბამისი სიმპტომატიკასაც დიდად არ აღინიშნავ.

Posted by: Leo woman 29 May 2015, 14:49
Lizikuna3
ანთებითი პროცესია ხომ ცხადია და თან არ მშველის ეს მსუბუქი ანტიბიოტიკუ.
მეოთხე დღეა უკვე და მემგობი საყურადღებოა.
საერთო სისუსტე,ტემპერატურა,იშვიათი ხველა არ არისსაკმარისი??
აზიმაკი გამომიწწრა ადრე როგორ მახსოვს და ხომ არ მივიღო ეგ???? მეშინია ძალიანsad.gif
არც მაქვს ექიმთან მისვლის თავი.

Posted by: Lizikuna3 31 May 2015, 11:41
Leo woman

თუ ასე გინდათ მიიღეთ აზიმაკი 500მგ დღესი 1ჯერ, 3 დღე. (რეცეპტით გაიცემა)

Posted by: miau 1 Jun 2015, 18:16
დედაჩემს აქვს ტროფიკული წყლული, საშინელი ტკივილები აქვს და კეტონალის ინექციაც არ შველის. იქნებ რამე ძლიერი საშუალება მირჩიოთ sad.gif

Posted by: Lizikuna3 2 Jun 2015, 08:31
miau

ყველაზე ძლიერად მაინც ტორადოლი ითვლება.

Posted by: torckiko 2 Jun 2015, 19:01
QUOTE (Lizikuna3 @ 27 May 2015, 14:32 )
ვირუსულ ინფექციებზე ანალიზები გაიკეთეთ? ც, ბ ჰეპატიტი?


ჰეპატიტები შევამოწმე და უარყოფითი აჩვენა, ანალიზები რომ ნორმაშია კომპიუტერის გადაღებას აქვს აზრი?

Posted by: gio_cxel 2 Jun 2015, 19:14
ბაბუაჩემს კუჭ-ნაწლავის პრობლემა აქვს. მოკლედ ყაბზობაო. თან დაერთო საყლაპავი მილის სიმშრალე და ხველა . დაუნიშნეს წამლები, იმკურნალა, მარა არაფერი შედეგი. კიდევ მივიდა ექიმთან, კიდევ დაუნიშნეს წამლები და შეღავათი არ არის დიდად..

რა მაინტერესებს. ეს ნორმალურია 80 წლის ასაკში, რომ მუდმივად ყაბზობა გაწუხებდეს ადამიანს, თუ მალევე უნდა განიკურნოს, თუ რეალურად ეგ არის?? სიმსივნე არ ქონდეს, ან განვითარდეს ამდენი დროც რომ მიდის..
ვის შეიძლება მივმართოთ?? კუჭ-ნაწლავის ულტრაბგერითი კვლევა ხომ არ გავაკეთოთ? ან თუ კი, სად მირჩევთ??
სხვა რითი შეიძლება გამოკვლევა გაკეთდეს??

მადლობა ყველას.

Posted by: daxatuli888 3 Jun 2015, 14:09
1 წლის წინ გადავიტანე ბრონხიტი, ხველა და უჰაერობა დიდხანს გამყვა, გაშუქებისას კი ფილტვის 9 სეგმენტთან გამკვრივებული კერა დაფიქსირდა და ეს ანთების დატოვებული კვალიაო როგორც მითხრეს... მაინტერესებს ბრონხის ანთების მერე ფილტვზე რატო დარჩა ეს კერა? არის თუ არა საყურადღებო და გასაკონტროლებელი 1 წლის შემდეგ? ეხლა ფეხმძიმედ ვარ და ისევ შემახსენა თავი ამ შეგრძნებებმა ,მაწუხებს პერიოდულად უჰაერობა და ღიტინის შეგრძნება და სიმძიმე ზურგის მხარეს და გულმკერდთან ჭვალივით…. ასთმა გამოირიცხა სპირომეტრიით....

Posted by: Lizikuna3 3 Jun 2015, 16:28
torckiko
მაშინ ამ შემთხვევაში უნდა გაიეკთოთ კტ კვლევა, რადგან ყველაფერი ნორმაშია და ვირუსული ინფექციებიც არ აღინიშნება. რადგან ღვიძლის ფუნქცია ნორმალურია უნდა ინახოს შრეობრივად რა ხდება.

gio_cxel
რას ეძახით ყაბზობას? რატომ გაქვთ ეჭვი სიმსივნეზე?

daxatuli888
რაც ეს სიმპტომები დაგეწყოთ იყავით მას შემდეგ ექიმთან?

Posted by: daxatuli888 3 Jun 2015, 19:03
Lizikuna3
არა ესიმისთვის ამჯერად არ მიმიმართავს...3 დღეა რაც ეს ღიტინის შეგრძნება და სიმძიმე მაქვს, ხველებით არ მახველებს, არც ხიხინს ვგრძნობ, არც სისუსტე მაქ, უბრალოდ უჰაერობა მაკ და თითქოს ლოდი მადევს გულზე, და მეჭვენება რო ჭვალივით მაქვს ხანდახან გულის მხარეს და ზურგზეც....ფეხმძიმედ რო ვარ შეილება უჰაერობა მაგას დავაბრალო მაგრამ სხვა არვიცი..... თუ ექიმი ხართ იქნებ მითხრათ ეგ 1 წლის წინანდელი გამკვრივებული კერა როგორც კტ ზ\ე აჩვენა მისახედია წლის შემდეგ თუ იწოვება თავისით?

Posted by: Pac. 3 Jun 2015, 19:26
ტვინის პრობლემები , მეხსიერების დაქვეითება, კონცეტრაციის დაკარგვა
არასწორი ცხოვრების წესის გამო...
რომელ ექიმს უნდა მივმართო? smile.gif ვერ ვიპოვე ესეთი თემა სადაც დავსვავ კითხვას

Posted by: Lizikuna3 3 Jun 2015, 20:54
daxatuli888
თქვენი ჩივილები ზოგადია და ობიექტურ შეფასებას საჭიროებს. რაც შეეხება "გამკვრივებულ კერას" არ ვიცი რას გულისხმობთ, ან რატომ დაგჭირდათ ბრონქიტის გამო ფილტვების კტ კვლევა? მოკლედ უნდა მიმართოთ ექიმს, რომელიც შეძლებს თვენი მდომარეობის შეფასებას, მე სხვა რამის თქვა ამ ეტაპზე არ შემიძლია.
Pac.
ტვინის პრობლემებში და არასწორ ცხოვრების წესში, რას გულისხმობთ, დააკონკრეტეთ.

Posted by: Pac. 3 Jun 2015, 21:34
Lizikuna3
ალკოჰოლო, ნარკოტიკები... კვირაშI ერთხელ ან ორჯერ ღამის გათენება (უძილობის პრობლემა არ მქონია) ბოლო რამოდენიმე წელიწადი ამ რეჟიმში ვიყავი, ეხლა რო დავიწყე გამოდზილება მეხსიერების დაქვეითებამ და კონცენტრაციის ყურადღების გაფანტულობამ შემაწუხა smile.gif ხანდახან როცა გადაგლილი მაქვს გონება მუშაობისგან მომენტებში წანიერად არაფხიზელი მგონია ხოლმე თავი...

Posted by: Lizikuna3 3 Jun 2015, 21:42
Pac.
ფსიქიატრის კონსულტაცია დაგჭირდებათ.

Posted by: Pac. 3 Jun 2015, 21:44
Lizikuna3
რა ჯდება კონსულტაცია და სად მირჩევდით მისვლას?
ჩემით რამე ბუნებრივი მეთოდებით ვერ დავეხმარები საკუთAრ თავს ,?
ვარჯიშI ვიტამინები , კითხვა
biggrin.gif

Posted by: daxatuli888 4 Jun 2015, 15:57
QUOTE (Lizikuna3 @ 3 Jun 2015, 20:54 )
daxatuli888
თქვენი ჩივილები ზოგადია და ობიექტურ შეფასებას საჭიროებს. რაც შეეხება "გამკვრივებულ კერას" არ ვიცი რას გულისხმობთ, ან რატომ დაგჭირდათ ბრონქიტის გამო ფილტვების კტ კვლევა? მოკლედ უნდა მიმართოთ ექიმს, რომელიც შეძლებს თვენი მდომარეობის შეფასებას, მე სხვა რამის თქვა ამ ეტაპზე არ შემიძლია.
Pac.

ჩემი ინიციატივით გავიკეთე მაშინ კტ, რადგან დიდხანს გამყვა უჰაერობა და მშრალი ხველა... დაახლოებით 2 ტვის მერე იკო რომ კტ\ზე წივწივაძემ მითხრა გაცკრიალებული გაქ ფილტვებიო და 9 სეგმენტზე გამკვრივებული კერა, სხვა რაიმე კერობრივი დაზიანება არ ფიკსირდებაო და როგორც ვიკითხე ეგ კერა ანთების დანატოვარიაო.... და მაინტერესებს თუ იწოვება, ან ეგრე რჩება?

Posted by: gio_cxel 4 Jun 2015, 16:29
Lizikuna3
QUOTE
რას ეძახით ყაბზობას? რატომ გაქვთ ეჭვი სიმსივნეზე?

შეკრულობა აქვს, პერიოდულად მარჯვენა მხარეს იტკიებს მუცელზე. თან დაერთო პერიოდული ხველა. საყლაპავი მილიდანო თვითონ ამბობს. ფილტვები სუფთაა..

ეჭვი არ მაქვს, უბრალოდ გაუვალობის მიზეზი შეილება სიმსივნეც იყოს ხომ სწორ ნაწლავში ??
სიმსივნე რომ იყოს, სისხლდენა აუცილებლად უნდა ქონდეს დეფეკაციის დროს ?


Posted by: Lizikuna3 5 Jun 2015, 00:14
gio_cxel
რამდენ დღეში ერთხელ აქვს კუჭის მოქმედება? დეფეკაციის დროს სისხლი, ტკივილი ან რაიმე ჩივილი ააქვს? მარჯ. მხარეს ბევრი რამ არის და ამიტომ ასე ვერ გეტყვით, შესაძლოა ნაღვლის ბუშტის ბრალიც იყოს და ბევრი სხა რამისაც. შესაძლოა ხველა, მართაც რეფლუქსით იყოს გამოწვეული, რასაც მკურნალობასთან ერთად კვების რეჟიმის სერიოზული დაცვა უნდა. (ღამით, საღამოს არ მიირთვას, კვების მერე არ წამოწვეს, მიირთვას მცირე ულუფებით).
ყაბზობა და გაუვალო სხვადასხვა რამ არის.

Posted by: gio_cxel 5 Jun 2015, 19:27
Lizikuna3
იშვიათად, მგონი კვირაში ერთხელ თუ ორჯერ. ზუსტად არ ვიცი . ტკივილი ბოდიში, მარცხნივ აქვს და არა მარჯვნივ. პერიოდულად კუჭის მიდამოებშიც იტკიებს ხოლმე.
დეფეკაციის დროს ტკივილი და სისხლდენა არ აქვს. პირველად ნაღვლის ბუშტის ანთება და ყაბზობა უთხრეს... იმკურნალა, მაგრამ მარტოა და დიდად კვებით ვერ უწყობს ხელს და მაინც აწუხებს ეს გაუვალობა...

QUOTE
რეფლუქსი

გადავიკითხე და მემგონი ეგ აწუხებს, იმიტომ რომ ბოყინიც აქვს ხოლმე და ყელის ტკივილიც. ტკბილი უყვარს და ტკბილეულს რომ შეეშვა, თითქოს შვება იგრძნო და აღარ მახველებსო...
ესეც ცალკე სამკურნალოა??
ვის შეიძლება ნორმალურ ექიმს მივმართოთ?? რაღაც არ მომწონს სადაც დადის, საყოველთაო დაზღვევით...
მადლობა პასუხისთვის..

Posted by: Lizikuna3 6 Jun 2015, 10:23
gio_cxel
პმ მოგწერეთ და შეგიძლიათ ნახოთ.

Posted by: lila2009 6 Jun 2015, 15:00
მენჯების არეში ,წელთან და ფეხებში მაქვს ძლიერი ტკივილი.
რაც სიცხეები დაიწყო ახლა გამიძლიერდა ტკივილი.
ვის მივმართო? ზოგადად რამ იცის ასეთი ტკივილი

Posted by: OKUPANT 6 Jun 2015, 16:44
არ ვიცი ვის ეხება ეს მაგრამ დავწერ აქ.
გაციების რამე ვირუსი დადის?

Posted by: gio_cxel 6 Jun 2015, 16:54
Lizikuna3
მადლობა smile.gif
----------------

Posted by: Lizikuna3 6 Jun 2015, 19:02
lila2009
დააკონკრეტეთ,

OKUPANT
ეგეთი ვირუსი ყოველთვის დადის biggrin.gif

Posted by: lila2009 6 Jun 2015, 19:28
Lizikuna3
რა დავაკონკრეტო , მკითხ3თ.
თაბროს ხჳევებიც მაქვს , დაბრუჟებული ხელები.
გული აჩქარებული.

სადაზღვეოს კლინიკაში მივედი ეხლა, არ მიმიღეს, გადაუდებელი არ ხარ, ჩაეწერეთო. ალბათ ერთი ორი კვირის მერე ექნებათ ადგილი

Posted by: annaninio 6 Jun 2015, 19:35
Lizikuna3
7 წლის ბავშვს სტკიოდა ფეხი მენჯ-ბარძაყის არეში, რენგენზე არაფერი ჩანდა, ექოსკოპიამ აჩვენა სახსრის ანთება და მცირე რაოდენობით სითხე, ედს აუვიდა 55 , სიცხეები აქვს 37.5 მდე, სიარულისას კოჭლობს, გინდაც გამაყუჩებელი ქონდეს დალეული.მოგვიანებით ასტკივდა წელიც. დღეს გაუკეთეს ტომოგრაფია, დიაგნოზის ფურცელს ორშაბათამდე არ აძლევენ, ორი სიტყვით უთხრეს ზურგის ტვინის ანთება და გულმკერდის მალების შეშუპება. ინტერნეტში ვერ მოვიძიე ინფორმაცია თუ როგორ ხდება განკურნება, დიანოზი რომ სერიოზულია გასაგებია. გთხოვთ მომწეროთ მკურნალობაზე, ან სავარაუდოდ რა შედეგებით ხასიათდება მკურნალობა.მადლობა წინასწარ.

Posted by: Lizikuna3 6 Jun 2015, 20:01
lila2009
რამდენი ხანია რაც გტკივათ? უკავშირებთ ტკივილის დაწყებას რამეს? ტკივილის ხასიათი როგორია? ჯერ წელი აგტკივდათ, მენჯები თუ ფეხები? რამდენი წლის ხართ?

annaninio
ვერაფერს გეტყვით, ვფიქრობ პედიატრის კომპეტენციაა.

Posted by: lila2009 6 Jun 2015, 20:31
Lizikuna3
ბოლო სამი დღეა ინტენსიურად მტკივა, ყველაფერი ერთად ამტკივდა თითქოს.
ვუკავშირებ ფიზიკურ გადაღლას. მაგრამ დასვენებაც არ მშველის.
საერთო სისუსტესაც ვერაფერს ვუხერხებ.
არაფრის თავი მაქვს ;(

სასწრაფო მიშველის რამეს?

Posted by: Leo woman 15 Jun 2015, 01:04
ექიმმა დინამოგენი გამომიწერა ჯერ ათი დღე დალიეო რომ გავაგრძელო არ შეიძლება???? და მაქსიმუმ რამდენი ხანი შეიძლება მიღება????ერთ ფლაკონს ვსვსმ ერთხელ დღეში.

Posted by: Lizikuna3 17 Jun 2015, 00:08
Leo woman
რა მიზეზით გამოგიწერათ? რატომ გინდათ რომ მეტი მიიღოთ?

Posted by: Leo woman 17 Jun 2015, 00:35
Lizikuna3
მომატება მინდა,ძალიან სუსტი ვარ

Posted by: 死海. 17 Jun 2015, 01:38
თუ იცით ძილის ან შეშინების დროს სისხლში შაქარის დონე იცვლება?
უდროო დროს გაღვიძებისას, განსაკუთრებით მაშინ როცა შეშინებულს მეღვიძება, ვარ ძალიან ცუდად და მხოლოდ ტკბილის მიღების შემდეგ გამოვდივარ მდგომარეობიდან rolleyes.gif

Posted by: Lizikuna3 17 Jun 2015, 10:21
Leo woman
რეკომენდირებული არის 10 დღე მიღება დღეში 2ჯერ. მაგრამ ვინაიდან თქვენ დღეში ერთხელ იღებთ 20 დღეც შეგიძლიათ.
* * *
死海.
გლუკოზის დონე სისხლში სტაბილური არ არის და სხვადასხვა მდგომაროების დროს იცვლება. რასაც თქვენ ამბობთ ჯობია რომ გამოიკვლიოთ გლუკოზა უზმოდ და ჭამის შემდეგ.

Posted by: Leo woman 17 Jun 2015, 10:45
Lizikuna3
მადლონაsmile.gif
ასეც მოვიქცევიsmile.gif

Posted by: qetishotiko 19 Jun 2015, 22:23
გამარჯობა, არ ვიცი ვის მივმართო ან სად დავსვა კითხვა, ჩემი ახლობელი არის ალკოჰოლზე დამოკიდებული, უნდა რომ დავეხმაროთ თანახმაა ყველაფერზე მაგრამ ჩვენ არ ვიცით რამდენად შეძლებს ტავის შეკავებას, ყველაზე ოპტიმალური მეთოდი რა არის, მხოლოდ ფსიქოლოგი ვერ შველის ამპულა რომ ჩავუდგათ (არ ვიცი სწორად ვწერ თ არა) რომ დალიოს რა შედეგით შეიძლება დასრულდეს და თუ არის ალტერნატივა, ის რაიონშია და რაიონებში კეთდება ეს? ან რამდენად ეფექტურია, ჩვენ დამატებით რითშეგვიძლია დავეხმაროთ ბოდიში თ შესაბამის განყოფილოებაში არ ვწერ იქნებ იცოდეთ რამე sad.gif

Posted by: desuzapro 26 Jun 2015, 09:54
გამარჯობათ რამდენად საშიშია თრომბოციტების საშუალო მოცულობა 11.20 როცა ნორმა 10 ვარ 18 წლის სხვა ყველაფერი ნორმაში მაქვს წარმოადგენს თუ არა ეს საფრთხეს თრომბებისათვის
* * *
გამარჯობათ რამდენად საშიშია თრომბოციტების საშუალო მოცულობა 11.20 როცა ნორმა 10 ვარ 18 წლის სხვა ყველაფერი ნორმაში მაქვს წარმოადგენს თუ არა ეს საფრთხეს თრომბებისათვის

Posted by: desuzapro 28 Jun 2015, 20:41
გამარჯობათ რამდენად საშიშია თრომბოციტების საშუალო მოცულობა 11.20 როცა ნორმა 10 ვარ 18 წლის სხვა ყველაფერი ნორმაში მაქვს წარმოადგენს თუ არა ეს საფრთხეს თრომბებისათვის
* * *
გამარჯობათ რამდენად საშიშია თრომბოციტების საშუალო მოცულობა 11.20 როცა ნორმა 10 ვარ 18 წლის სხვა ყველაფერი ნორმაში მაქვს წარმოადგენს თუ არა ეს საფრთხეს თრომბებისათვის

Posted by: Salome ♥ 8 Jul 2015, 14:06
მოკლედ შეიძლება ბევრს ეს შეკითხვა სულელურად მოეჩვენოს მაგრამ მე მართლა მაინტერესებს სხველი მაპროვოცირებელი თუ არის მზესუმზირა ან ხველის დროს მზეუმზირის ჭამა თუ შეიძლება. ვიყავი თერაპევტთან და გამომიწერა რაღაცეები. ეხლაც და მანამდეც მიშლიას სახლში მზესუმზირის ჭამას არ შეილებაო და მართალია?

Posted by: pleita 10 Jul 2015, 12:15
lila2009
ვაიმე იგივეს დასაწერად შემოვედი. ბოლო რამოდნეიმე დღეა ამებასავით დავიზლაზნები, საშინელი განწყობა, ძილიანობა, უმოძრაობა მჭირს.ეს რა უბედურებაა საოცრად ენერგიული ვარ ყოველთვის.. არაფრის თავი მაქვს საერთოდ. მინდა ვჭამო და ვიძინო sad.gif

Posted by: Sargerass 19 Jul 2015, 19:22
გამარჯობათ მეგობრებო..

ჩემს მეუღლეს გაუსივდა ერთი თითი ხელზე sad.gif რამოდენიმე დღეა ასე აქვს..ალბათ რამე მძIმე მეჭირა და მას მერე გასივდაო,არც ახსოვს ზუსტად როდის დაეწყო..რა თქმა უნდა აწუხებს და ვიზუალურადაც ცოტა დიდია თითი სხვებთან შედარებით..რა ვუშველო? sad.gif

Posted by: bagda 20 Jul 2015, 22:20
ხალხო მთელი დღეა სუსტად ვარ. დილით დაბალი წნევა მქონდა (90/60), მერე ყავა დავლიე და ცოტათი ამეწია (95/65). მერე სიცხე მომცა 37,5 და ვგრძნობდი, რომ მემატებოდა. დავლიე პარაცეტამოლი და დამეწია სიცხე. თუმცა ახლაც სუსტად ვარ და ზოგადი სიპტომები დამრჩა. ესენია: წელის არეში დისკომფორტი, სისუსტე, მადის ნაკლებობა, ძილისკენ მიდრეკილება, ხშირი შარდვა.

თავიდან ვიფიქრე აკლიმატიზაციის ბრალი ხომ არაათქო (გუშინ ჩამოვედი მთიდან), მარა გუშინ მთელი დღე კარგად ვიყავი. შემდეგ, ვიფიქრე, ხომ არ გავცივდითქო, ან კონდენციონერმა ხომ არ გამოიწვიათქო, მარა იქ ყოფნისას თითქმის 4 დღის განმავლობაში, არ მქონია არანაირი ჩივილები. საჭმელ-სასმელიც კარგი იყო და ინტოქსიკაცია ამ მხრივ, ვგონებ, გამორიცხულია. გრიპი არ მაქვს.

შეიძლება, აკლიმატიზაცია ასეთ რთულ ფორმებში გამოიხატოს?

Posted by: kotora1993 20 Jul 2015, 23:08
გამარჯობათ , მაინტერესებს რეტაბოლილი შეიცავს თუ არა დოპინგს და თუ შეიცავს რამდენი ხანი რჩება ორგანიზმში?

Posted by: Leo woman 28 Jul 2015, 22:10
გამარჯობაsmile.gif
ფრჩხილზე მაქვს სოკო და ვხმარობ ექსოდერილს უკვე ერთი წელი,გამოსწორების გზაზე კი არის როგორც ვამჩნევ მაგრამ მინდა ორალურად მივიღო რამე საშუალება მეტი ეფექტურობისთვის.
გთხოვთ მირჩიოთ მედიკამენტი.

Posted by: VAMPIRE_W 30 Jul 2015, 12:03
გამარჯობათ, ბოდიშით რომ გაწუხებთ, მაგრამ ერთი კვირაა დამიწყო ყელის მიდამოში და ბრონხებთან თიტქოს ღიტინის შეგრძნება და ამის ნიადაგზე მახველებს ცოტ ცოტა. ნახველს ვერ ვგრძნობ, ექიმმა გასინჯა ფილტვები და ასე თქვა ფილტვები სუფთაადა არც ბრონხიტი გაქვსო, ყელისგან იქნება ხველებაო და ანთების წამალი გამომიწერა. ვსვამ სამი დღეა მაგრამ თითქოს უფრო იმატა ხველამ და ღიტინმა. რისი ბრალი შეიზლება იყოს? თან ხვალ გავდივარ საქართველოდან და მაგაზეც ვნერვიულობ საშიში ხომ არ არის რომ გამირთულდეს გზაში.
მადლობა ყურადღებისთვის.

Posted by: chuvak_lexo 4 Aug 2015, 15:26
გამარჯობათ 23 წლის ვარ, ბეჭები (მარცხენა განსაკუთრებით) მტკივა, ეხლა მკერდი (შუაში, ძვალი მტკივა) ამტკივდა - სუნთქვის დროსაც, მოძრაობისასაც და ხელის შეხებითაც

რამე მალამო მირჩიეთ რა თორე ვეღარ ვვარჯიშობ :/

Posted by: პროფაზა 5 Aug 2015, 16:32
მოწამვლიდან რამდენ დღეში შეიძლება ალკოჰოლის დალევა შეგიძლიათ მითხრათ?

Posted by: Scoorp 7 Sep 2015, 18:17
კალცი D3 ნიკომედ ფორტეს ვსვამ დილა საღამოს
კი არადა ვღეჭავ
უბრალოდ რა დროს უნდა მივიღო მითხარით რა
ჭამის შემდეგ ჭამის დროს თუ აქვს მაგას მნიშვნელობა საერთოდ?

Posted by: keen 20 Sep 2015, 15:30
ნორმალური თერაპევტი თუა ამ ქვეყანაში, რომ მისვლის არ შეგეშინდეს? sad.gif

Posted by: roknaldo 23 Sep 2015, 12:35
ნეფროლოგი არის აქ..............................................................................

Posted by: ninonino2007 3 Nov 2015, 16:04
გამარჯობა! თუ შეიძლება მითხრათ, სად შეიძლება გავიკეთო ტესტი ფოლის და მორელის მეთოდით.
წინასწარ გმადლობთ!

Posted by: Sttylex 8 Nov 2015, 14:35
გამარჯობათ , კუჭის პრობლემა მაქვს , დაწითლებულია , რაღაც სითხე გამოდის .. რა ვქნა ? სერიოზულია რამე ?

Posted by: Leo woman 10 Nov 2015, 14:24
ბრონხიტი მაქვს და ვსვამ სამი დღეა ბრონქიდიაზინას და პროსპანს...
ანტიბიოტიკი არ გამოუწერია და უკეთესობა არ მაქვს,
ანტიბიოტიკის გარეშე გაქრება????

Posted by: pvm 24 Nov 2015, 14:58
71 წლის ადამიანს აღმოაჩნდა თორმეტგოჯას წყლული (კიდევ კუჭის ლორწოვანის ანთება და რეფლუქსი). სავარაუდოდ ამის გამო ჰემოგლობინი ქონდა 82. პაციენტი იმყოფება ანტკიკოაგულანტებზე- ვარფარინი+ პლავიქსი. ანტიკოაგულანტები შეუწყვიტეს ეგრევე, შეწყვეტიდან მესამე დღეს ჰემოგლობინი გახდა 97.

ჩაიტარა 10 დღიანი მკურნალობის კურსი (ანტიბიოტიკებით), ჰემოგლობინი კი ისევ დაბალია - 99. მაინტერესებს, არ არის ეს მდგომარეობა საშიში? ექიმის აზრით ყველაფერი კარგად არის, მთავარია მკურნალობის მერე არაფერმა მოიკლო და იგივე მდგომარეობა უკვე ძალიან კარგიაო.

Posted by: qeseramovich 1 Dec 2015, 18:06
ესეიგიი სადღაც ერთი კვირის წინ თავი დავარტყი ძლიერად, გონება არ დამიკარგავს მაგრამ რაღაცეები კარგად არ მახსოვდა მეორე დღეს. მთელი კვირაა მსუბუქიი თავბრუსხვევა, მაქვს გულის რევის შეგრძნება მეორე და მესამე დღეს მქონდა მაგრამ არც ისე ძლიერი. ეს თავბრუსხვევა ნაბახუსევს დავაბრალე და მთელი კვირა მაინც ვიარეე არ დავწოლილვარ. დღეს დავურეკე ექიმს და რაღაც შარდმდენი და დამაწყნარებელი დამინიშნა. მინიმუმ ერთი კვირა სიარულს მოერიდეო მითხრა. მაინტერესებს ამ ერთი კვირა სიარულის გამოო რამე გართულება ხო არ მოყვება და რამე სერიოზულში ხომ არ გადაიზრდება? მადლობა წინასწარ 2kiss.gif

Posted by: gio_cxel 1 Dec 2015, 18:42
ვერ დავიმახსოვრე, ორსიტყვიანი კრიზი (დიაგნოზი) . სიმპტომები : თავბრუსხვევა მოძრაობის დროს და მხედველობის გაუარესება (დაბნელება პერიოდულად), ზოგადი სისუსტე. დაინიშნა ერთ თვიანი მკურნალობა. სავარაუდოდ ნერვიულ ფონზე დაემართა პაციენტს, მაგრამ მაინტერესებს რამდენად სერიოზულია ეს ყველაფერი ? რაიმე გადაღება, ტომოგრაფია და ა.შ. ამ ეტაპზე არ ჩათვალა ნევროპათოლოგმა საჭიროდ.

Posted by: atila92 11 Dec 2015, 01:11
აქაც ვიკითხავ
ყურის ექიმთან ვიყავი და ჩასაწვეთებელი გამომიწერა. ყურში რომ ვიწვეთებ ყური მეგუბება. რომ გაიხსნას ყური თავი უნდა გადავაბრუნო და წამალი კი გამოდოს მაშინ უკან. არასწირად ვიწვეთებ თუ ასე დაგუბებული უნდა მქონდეზ ჩაწვეთების მერე?

Posted by: lika_law 12 Dec 2015, 20:47
გამარჯობა, მახველებს სპაზმურად, თითქოს მაქვს ნახველიც, მაგრამ ვერ ამომაქვს, ხველა მიძლიერდება ღამით და ვერ ვჩერდები, ვსვამდი მუკალტინს, თბილ-თბილ სითხეებს, მაგრამ შედეგი არ მაქვს. რა მივიღო?

და კიდევ: დუნდულის ძირში მტკივა ფეხი, გადასცემს მთელს ფეხს და მიკავებს, სიარული მიჭირს, ვიყავი ორშაბათს ექიმთან (დაზღვევის), ნოპეინის ნემსი გამომიწერა, გავიკეთე, ტკივილი გადავიდა წელისკენ და ბოლოს გაიარა, ნუ მიშველა, მაგრამ დღეს ისევ ამტკივდა, სიარულისას მიკავებს ფეხს ძალიან, ნერვია თუ რა ჯანდაბაა ვერ გავიგე, ან რატომ არ მიშველა ნემსებმა?

Posted by: qeseramovich 20 Dec 2015, 02:49
რისთვის არის ეს თემა თუ მაინც ყველას კიდია და არავინ გვპასუხობს? რაღას ვწერთ კითხვებს ვაბშე ?

Posted by: XVTIURI 20 Dec 2015, 03:11
ვეძებთ ექიმებს ?

არადა უნდა ფუნქციონერობდეს ეს თემა
































Posted by: თალიბანი 7 Jan 2016, 15:34
მე ვიქნები თქვენი ახალი თერაპევტი biggrin.gif


Posted by: libertad 23 Jan 2016, 19:20
თუ იცით ვინმემ მენსტრუაციის დროს შეიძლება მაგნიტო-რეზონანსული ტომოგრაფიის გადაღება?რადიაციაა მაინც,სოლარიუმსაც კრძალავენ ამ დროს და მგონია რო ესეც არ იქნება დასაშვები

Posted by: Bobgure 24 Jan 2016, 20:51
libertad

კი შეიძლება, უსაფრთხოა.


Posted by: malaktavus 31 Jan 2016, 09:06
ანუ ეს ორი დღეა მაცემინებს........... და გუშინ სარამოდან. რო მაცემინებს. ვიწვები ეგრევე მთელ ტანში. თითქოს "კიპიტოკი" დამასხეს!

რატომა ესე? სიცხეც მაქ?

Posted by: ckupee 15 Feb 2016, 00:30
გამარჯობაა...მოკლეედ დავიწყებ სულ თავიდაან მოკლედ პატარა ვიყავიი თვეების რომ გამირთულდა ხველ ბატონები და ლარინგიტი თუ რაღაც ეგეთი მჭირდა,მოკლედ ძლივს გადამარჩინეს...მერე გავიზარდე და 2-3 ჯერ მეტჯერ თუ არა ფილტვწბის ანთება მქონდა.ნუ მერე და მერე უკეთ ვიყავი..20 წლის ვიყავი მოწევა რომ დავიწყეე ეხლა ვაარ 26 ის და მოკლეედ ძაან ბევრს ვეწევიი...ორსულობის დროსაც ვერ შევეშვი 2 წელია თითქმის სულ მახველებს და რახაც ხიხინიც მესმის ზოგჯერ სპაზმური ხველა მააქვს..უკვე იმდენჯერ გადავიგე. ფილტვებზე რენტგენი რომ ყველა მეუბნება არ შეიძლებაო...არაფერი მააქვს ფილტვებში..ვერავინ ვერაფერი მითხრა..ერთადერთი ერთმა ექიმა რაც მითხრა ქრონიკული ბრონქიტიაო და არც მკურნალობა დამინიშნ.იქნებ რამე მირჩიოთ...'ვიცი მოწევაა ძააან ცუდი. მაგრამ მკურნალობაააც ხომ.შეიზლებააა

Posted by: konkretula 16 Feb 2016, 18:50
რამდენიმე დღეა ზურგის მხარეს, ფილტვები სადაცაა (ბეჭების ქვემოთ დაახლოებით) შიგნიდან მტკივა მუდიმვად. რაღაც მომენტებში უცებ გამელვებს და მეტად მტკივა. სუნთქვაზე არაა დამოკიდებული აშკარად ეგ ტკივილი.

რისი ბრალი შეიძლება იყოს ან რა ექიმს მივმართო? სიცხე და რამე არ მაქვს :/

Posted by: Faniack 18 Mar 2016, 15:25
ckupee
პირველ რიგში მოწევას უნდა გაანებო თავი , სანამ გამღიზიანებელის ( სიგარეტის) მოქმედება იქნება მუდმივად გექნება ჰიპერსეკრეცია და ანთებითი ცვლილებები ბრონქის სანათურში შესაბამისად მწეველის ბრონქიტი. მკურნალობა მხოლოდ სიმპტომურ შედეგს თუ მოგცემს და გამწვავებების დროს ინიშნება ანტიბიოტიკოთერაპია ...


სიგარეტს შეეშვი არღირს ჯანმრთელობად კვამლის ყლაპვა


konkretula
გავს ნეკნთაშორის ნევრალგიას, მოძრაობისას თუ ძლიერდება ? ადგილობრივად დიკლოფენაკის ან რაიმე არასტეროიდული ანთების საწინააღმრდეგო გელი სცადე.

Posted by: pato-7 18 Mar 2016, 17:45
არამწეველ ადამიანს აქვს ფილტვის რენდგენზე დაჩრდილვა, შესაძლებელია, რომ ავთვისებიანი არ იყოს და კეთილითვისებიანი იყოს სიმსივნე?

Posted by: gramatikikina 19 Mar 2016, 04:04
:

Posted by: ckupee 19 Mar 2016, 17:34
Faniack
QUOTE
ckupee
პირველ რიგში მოწევას უნდა გაანებო თავი , სანამ გამღიზიანებელის ( სიგარეტის) მოქმედება იქნება მუდმივად გექნება ჰიპერსეკრეცია და ანთებითი ცვლილებები ბრონქის სანათურში შესაბამისად მწეველის ბრონქიტი. მკურნალობა მხოლოდ სიმპტომურ შედეგს თუ მოგცემს და გამწვავებების დროს ინიშნება ანტიბიოტიკოთერაპია ...

მადლოობთ

Posted by: Faniack 20 Mar 2016, 23:43
pato-7
დაჩრდილვა როგორც ასეთი მხოლოდ და მხოლოდ სიმსივნის ნიშანი არ არის.

მწეველობის გარდა ბევრი რამ შეიძლება იყოს რამაც შეიძლება ფილტვის ავთვისებიანი სიმსივნის წარმოქმნას შეუწყოს ხელი.

რენტგენოლოგის დასკვნით და სურათით გირჩევთ მიბრძანდეთ ფტიზიატრთან ,რომელიც თავის მხრივ თუ საჭიროდ ჩათვლის დამატებით კველვებს ჩაატარებს დიფერენციაციის მიზნით.

Posted by: gramatikikina 22 Mar 2016, 22:06
Faniack
უი ექიმი გვყოლია? ვერ შევამჩნიე და წავშალე:დ
ძალიან ცუდ დღეში ვარ მოკლედ. გაციებული არა ვარ არაფერი. ყელში ვგრძნობ ხოლმე მოულოდნელად წვას ერთ წერტილში. იმ მხარეს თვალიდან მომდის ცრემლი და ამ წვის გამო ისტერიულად მახველებს, მარა უაზროდ. რაღაც დრო უნდა, ეს წვა გაივლის და ვითომც არაფერი ნორმალურად ვარ შემდეგ შემთხვევამდე.
ფარისებრზე გავესინჯე და ორი პატარა კვანძი მაქვს. ბიოფსია ჯერ არ გინდაო.
მაგრამ ეგ წვა პირველად მქონდა ძალიან დიდი ხნის წინაც და მერე აღარ. ბოლო წლებია ზამთარში თუ გაზაფხულზე მაქვს ისევ და ეხლა ძალიან გართულდა.
ერთი ეგაა ვეწეოდი მაშინაც და ეხლაც
კეტოტიფენი დავლიე და შემიმცირდა შემთხვევები, მაგრამ მაინც დღეში რამდენჯერმე მემართება.
რით ვიმკურნალო ან სად ჯობია მისვლა იქნებ მირჩიო

Posted by: Faniack 25 Mar 2016, 02:57
gramatikikina
ნუ შენი სიმპტომებით და კეტოტიფენის მიღების შემდეგ დადებითი დინამიკით თუ ვიმსჯელებთ ალერგიული უნდა იყოს შენი პრობლემა + პერიოდი .

სხვამხრივ ალერგიული ხარ? მშობლების ან ბებია ბაბუისგან ალერგიული ფონი გაქ?

1) სიგარეტი ისევ და ისევ როგორმე უნდა გაანებო თავი.

2) მკურნალობას ასე დაუზუსტებელ დიაგნოზზე არავინ დაგიწყებს ამიტომ ალერგოლოგთან გირჩევ მისვლას

3) პირველრიგში ოჯახის ექიმთან , მოუყევი ჩივილებზე და გადაგამისამართებს თვითონ : )



Posted by: gramatikikina 25 Mar 2016, 05:48
Faniack
საგრძნობლად შემიმცირდა კეტოტიფენითup.gif თითქმის აღარ მაქვს. ალერგიული ზაფხულში ვარ ოდნავ და ისე არავინ მყავს ეგეთი
აქ პოლიკლინიკაში მიმსვლელი არა ვარ, არავინ ჰყავთ ნორმალური :დ მოვძებნი ვინმეს
დიდი მადლობა 2kiss.gif

Posted by: ქართლოსი 5 Apr 2016, 20:12
რამე პირბადე თუ არსებობს მანქანების გამონაბოლქვი რომ შეაკავოს?

Posted by: Faniack 6 Apr 2016, 19:13
ქართლოსი

O_ო სრულად შეკავებას რამე მაღალტექნოლოგიური ნიღაბი თუ შეძლებს რომელიც ასტრონომიული ფასი ეღირება smile.gif

უბრალო ნიღაბი ვერაფერს ვერ შეაკავებს

Posted by: _Moxuci_ 16 Apr 2016, 09:10
არ ვიცი შესაბამის თემაში ვწერ თუ არა, ალბათ არა :დ მაგრამ უკეთესი ასე უცებ ვერ ვიპოვე და იქნებ დამაკვალიანოთ ვინმემ...

საყოველთაო დაზღვევაზე მაქვს კითხვა:

მაგალითად თუ ვარ სტუდენტი და ჩემი დაზღვევა რეგისტრირებულია ზემო იმერეთის N ქალაქის კლინიკაში (სადაც ადრე ვცხოვრობდი), მაგრამ დღეს ვარ ქვემო იმერეთში და მინდა Z ქალაქის კლინიკაში მისვლა(სადაც ახლა ვცხოვრობ), მაგრამ არ მარეგისტრირებენ და ვერაფერი დაგიფინანსდება, რადგან იქ ხარ უკვე რეგისტრირებული და უნდა ამოეწეროო. შესაძლებელია თუ არა პირადად ჩასვლის გარეშე, უბრალოდ ოჯახის წევრი მივიდეს და მან მოახდინოს ამოწერა?

Posted by: Faniack 17 Apr 2016, 19:33
_Moxuci_
კი რამდენადაც მე ვიცი შეუძლია ოჯახის წევს რო ამოგწეროს იქიდან მაინც სამინისტროს ცხელ ხაზზე დარეკე და დაგაკვალიანებენ უფრო კომპეტენტურად

Posted by: Vano52 21 Apr 2016, 11:26
ესეგი ძაან ცუდად ვარ რაა, რაღაც ვირუსი შემხვდა რაც ყურების ანთებით გამირთულდა, ყურებში 1 კვირაა ოტიპაქსს ვიწვეთებ, ანთებამ გამიარა მაგრამ სუ დაგუბებული მაქ არაფერი მესმის,
დღეს კიდე თავზე გადავიღე რენტგენი და მარჯვენა ფრონტალური ღრუ საერთოდ არ ჩანს და რა მიზეზი შეიძლება იყოს დამეხმარეთ

Posted by: Faniack 21 Apr 2016, 18:11
Vano52
პირველ რიგში დავიწყოთ იმით რო მწვავე შუა ოტიოტი შეიძლება გამოწვეული იყოს ცხვირხახის ინფექციების დროს როგორც სავარაუდოდ შენ შემთხვევაშიც მოხდა , დაგუბება ოტიტის ერთერთი ტიპიური სიმპტომია

ფრონტალური სინუსის დაჩრდილვა ანთებითი პროცესით ანუ ფრონტიტით უნდა იყოს გამოწვეული.

მეგირჩევ რენდგენის ფირით ყელყურცხვირთან მიხვიდე კარგი მკურნალობის კურსით ორივე კურდღელს ერთდროულად მოკლავ

Posted by: medicine2016 21 Apr 2016, 22:43
მოკლედ გეტყვით რა ხდება.
ბოლო დროს წონაში მოვიმატე საგრძნობლად და უმოძროდ ვიყავი
ბოლო 2 თვეა მაწუხებს სახსრების მოვლითი ტკივილი
თითქოს ხელი დაღლილი მაქვს რომ ვწევ
ხელის თითები გული
დღის განმავლობაში ხან ერთი რამე მაწუხებს ხან მეორე
გაუსაძლისი და რამე ეგეთი არ არის
დავიწყე დიეტა და წონაშიც მოვიკელი და
რამე საშიში ხო არაა
მაგ. მეტასტაზები ყელში ან ძვლის
მაგრამ 2 თვე ასე თუ მაწუხებს ეხლა ცუდად ხო უნდა ვიყო
დამისვით რა კითხვები და გიპასუხებთ პანიკაში ვარ ამ ბოლო დროს

ვისთან მივიდე თერაპევთან, კარდიოლოგთან, რევმატოლოგთან, ონკოლოგთან თუ ვისთან
ღამე მძინავს მშვიდად
სიარულის დროს თითქოს ფეხი მტკივა შიგნიდან და დაჯდომა და მოხრა მინდა

გუშინ წვიმის დროს მარჯვენა ხელის სახსარი მტკიოდა ყოველ ნახევარ საათშI ერთხელ
დღეს აღარ მტკივა
ხელის თითები შიგნიდან და გარედან არ ვიცი
რა
დღეს არაფერი მაწუხებს
თუ მეტასტაზურია დაუშვად ასეთი რამე შეიძლება იყოს რომ გადამიაროს თავისით? წონაც საკაიფოდ მაქვს?

სისხლის ანალიზებით არაფერი გამოჩნდა. ნევროპათოლოგმა დამინიშნა ქსანაქსი
ათი დღის წინ

აი, როცა ვდგავარ მგონია, რომ თითქოს შიგნიდან ჩატეხილი მაქ ძვალი, ხან ხელი წამიერად, როცა ვზივარ წამოვდგები უცბად მგონია რომ რაღაცა ჩამიწყდა
ეს წონის ბრალია?
ვისთან მივიდე? რა გამოკვლევები ჩავიტარო?

Posted by: J&J 23 Apr 2016, 12:56
Faniack
გამარჯობა. ნაღველში კენჭები მაქვს. ამოღება მირჩიეს. სამწუხაროდ წლის ბოლომდე ვერ მოვახერხებ ლაპარასკოპიით გაკეთებას. ტკივილი სულ მაქვს. ხან ძლიერი,ხან ყრუ. რამდენად სახიფათოა ოპერაციის გადადება და რა შეიძლება გავაკეთო მანამდე,როკ მდგომარეობა არ დამძიმდეს(დიეტაზე ვარ).
პ.ს. საქართველოში არ ვცხოვრობ. და უცებ რომ მომიწიოს მანდ შვებულებაში ყოფნისას გაკეთება,ხომ არ იცით რა დაჯდება და რამდენი ხანი არ შემეძლება სიმძიმის აწევა. წლამდე ბავშვი მყავს. ვერ დადის.11კილოა.

Posted by: ჟანრი51 24 Apr 2016, 00:07
საშინელი ხველა და ყელის ტკივილი მაქვს 3 კვირაზე მეტია. ღამ-ღამობით ხველება მახრჩობს. დამიდგინეს მწვავე ბრონქიტი და მწვავე ლარინგოტრაქეიტი. მკურნალობას შეგედი არა აქვს ჯერჯერობით. როგორ მოვიქცე?

Posted by: Faniack 24 Apr 2016, 22:33
medicine2016
გამარჯობა, მეტასტაზები საიდან მოიტანე? რაიმე სიმსივნური გაქ ან გქონდა ოდესმე?
სიმსივნურ წარმონაქმნს წონაში კლება ახასიათებს რამე რო იყოს და არა მატება.
ტკივილი წვრილ სახსრებში რევმატოიდულმა ართრიტმა იცის თუმცა უმოძრაობის გამოც შეიძლება ზომიერი ატროფია გქონდეს და ფიზიკურ დატვირთვაზე დისკომფორტი გქონდეს. თუ შემაწუხებელია რევმატოლოგს მიმართეთ და გამოგიკვლევთ.

მევფიქრობ რომ სანერვიულო არუნდა იყოს არაფერი smile.gif

J&J
გაგიმარჯოს დიეტა წამყვანია ნაღველკენჭოვანი დაავადებების დროს ტკივილის შემთხვევაში სპაზმოლიზური საშუალებები ( ნოშპა , სპაზმალგონი , დროტავერინი ) შეგიძლია იხმარო , ფასს რაც შეეხება ყველა კლინიკას თავისი კონკრეტული ფასი აქვს და ასე ზუსტად ვერ გეტყვი . სიმძიმის აწევაზე ოპერატორი თვითონ დაგაკვალიანებს.

ჟანრი51

ვინ დაგინიშნა მკურნალობა და რას იღებ კონრეტულად

Posted by: medicine2016 24 Apr 2016, 22:48
QUOTE (Faniack @ 24 Apr 2016, 22:33 )
medicine2016
გამარჯობა, მეტასტაზები საიდან მოიტანე? რაიმე სიმსივნური გაქ ან გქონდა ოდესმე?
სიმსივნურ წარმონაქმნს წონაში კლება ახასიათებს რამე რო იყოს და არა მატება.
ტკივილი წვრილ სახსრებში რევმატოიდულმა ართრიტმა იცის თუმცა უმოძრაობის გამოც შეიძლება ზომიერი ატროფია გქონდეს და ფიზიკურ დატვირთვაზე დისკომფორტი გქონდეს. თუ შემაწუხებელია რევმატოლოგს მიმართეთ და გამოგიკვლევთ.

მევფიქრობ რომ სანერვიულო არუნდა იყოს არაფერი smile.gif

J&J
გაგიმარჯოს დიეტა წამყვანია ნაღველკენჭოვანი დაავადებების დროს ტკივილის შემთხვევაში სპაზმოლიზური საშუალებები ( ნოშპა , სპაზმალგონი , დროტავერინი ) შეგიძლია იხმარო , ფასს რაც შეეხება ყველა კლინიკას თავისი კონკრეტული ფასი აქვს და ასე ზუსტად ვერ გეტყვი . სიმძიმის აწევაზე ოპერატორი თვითონ დაგაკვალიანებს.

ჟანრი51

ვინ დაგინიშნა მკურნალობა და რას იღებ კონრეტულად

დაწვრილებით გეტყვით როგორ არის
4 თვის წინ იანვარშI დამეწყო
მაშინაც მსუქანი ვიყავი და ეხლაც ვარ
აი ხელის თითები მიხურს ხოლმე და გულს გადაცემს, ხელის გულები მეფხანება, ტერფის არეში რამდენიმე წამით ნერვები მომიჭერს,
აი თითქოს სახსრებზე ნერვები მაწუხებს
რევმატულს ვერ დავარქმევ
უბრალოდ გაწამებული ვარ ამ დისკომფორტებით
თორემ არც სიცხე მოუცია, არც გამაყუჩებელი დამჭირვებია, ჩვეულებრივად მძინავს, უბრალოდ მეორე დღესაც რომ ვიცი ეგეთI რაღაცეები დამეწყება გაღვიძებისთანავე ცუდ მდგომარეობაში მაყენებს
აი ხელები ისე მაწუხებს თითქოს ართრიტი მაქვს და გაშლა და რამეზე დაყრდონობისაც მეშინია, რამე არ ამტკივდეს
სისხლის ანალიზი ერთI კვირის წინ გავიკეთე არაფერი არ მაქვს საგანგანშო
ნერვოპათოლოგმა კი დამინიშნა ქსანაქსი
პატარა ანერვიულებაზე კანი მწიწკნის ფეხებში და ხელებში ნერვიულად
არადა 23 წლის ბიჭი ვარ
და აქამდე ეგეთი არაფერი მაწუხებდა
შიშმა მომიცვა ერთი სიტყვით
სულ სარკეში ვიხედები ხომ არ დავიკელი წონაშI sad.gif
* * *
QUOTE (medicine2016 @ 24 Apr 2016, 22:48 )
QUOTE (Faniack @ 24 Apr 2016, 22:33 )
medicine2016
გამარჯობა, მეტასტაზები საიდან მოიტანე? რაიმე სიმსივნური გაქ ან გქონდა ოდესმე?
სიმსივნურ წარმონაქმნს წონაში კლება ახასიათებს რამე რო იყოს და არა მატება.
ტკივილი წვრილ სახსრებში რევმატოიდულმა ართრიტმა იცის  თუმცა უმოძრაობის გამოც შეიძლება ზომიერი ატროფია გქონდეს და ფიზიკურ დატვირთვაზე დისკომფორტი გქონდეს. თუ შემაწუხებელია რევმატოლოგს მიმართეთ და გამოგიკვლევთ.

მევფიქრობ რომ სანერვიულო არუნდა იყოს არაფერი smile.gif

J&J
გაგიმარჯოს  დიეტა წამყვანია  ნაღველკენჭოვანი დაავადებების დროს ტკივილის შემთხვევაში სპაზმოლიზური საშუალებები ( ნოშპა , სპაზმალგონი , დროტავერინი ) შეგიძლია იხმარო , ფასს რაც შეეხება  ყველა კლინიკას თავისი კონკრეტული  ფასი აქვს  და ასე ზუსტად ვერ გეტყვი . სიმძიმის აწევაზე ოპერატორი თვითონ დაგაკვალიანებს.

ჟანრი51

ვინ დაგინიშნა  მკურნალობა და რას იღებ კონრეტულად

დაწვრილებით გეტყვით როგორ არის
4 თვის წინ იანვარშI დამეწყო
მაშინაც მსუქანი ვიყავი და ეხლაც ვარ
აი ხელის თითები მიხურს ხოლმე და გულს გადაცემს, ხელის გულები მეფხანება, ტერფის არეში რამდენიმე წამით ნერვები მომიჭერს,
აი თითქოს სახსრებზე ნერვები მაწუხებს
რევმატულს ვერ დავარქმევ
უბრალოდ გაწამებული ვარ ამ დისკომფორტებით
თორემ არც სიცხე მოუცია, არც გამაყუჩებელი დამჭირვებია, ჩვეულებრივად მძინავს, უბრალოდ მეორე დღესაც რომ ვიცი ეგეთI რაღაცეები დამეწყება გაღვიძებისთანავე ცუდ მდგომარეობაში მაყენებს
აი ხელები ისე მაწუხებს თითქოს ართრიტი მაქვს და გაშლა და რამეზე დაყრდონობისაც მეშინია, რამე არ ამტკივდეს
სისხლის ანალიზი ერთI კვირის წინ გავიკეთე არაფერი არ მაქვს საგანგანშო
ნერვოპათოლოგმა კი დამინიშნა ქსანაქსი
პატარა ანერვიულებაზე კანი მწიწკნის ფეხებში და ხელებში ნერვიულად
არადა 23 წლის ბიჭი ვარ
და აქამდე ეგეთი არაფერი მაწუხებდა
შიშმა მომიცვა ერთი სიტყვით
სულ სარკეში ვიხედები ხომ არ დავიკელი წონაშI sad.gif

როგორც იტყვიან გაწამებული ვარ ტირილამდე მივდივარ
არც გამაყუჩებელი დამილევია წამებში გადის ან მაქვს მიხურს თითქოს ეგრე უნდა იყოს
ორგანოები ზურგი არაფერი მაწუხებს აი ხელები თითქოს მალე პარალიზებული გახდება ისეთი გრძნობა მაქვს როცა რამეს ვაკეთEბ

Posted by: Faniack 24 Apr 2016, 23:12
medicine2016
დამშვიდდი როგორც მე გატყობ ემოციური ხარ , არაფერი სანერვიულო არ გაქვს , მიიღე ნევროლოგის დანიშნულება , ივარჯიშე თუმცად სახლის პირობებში ემოციურად დაგამშვიდებს პლიუს სეროტონინის გამოყოფას უწყობს ხელს .




Posted by: J&J 25 Apr 2016, 13:21
Faniack
მადლობა პასუხისათვის. ეგ მედიკამენტები აქ არ არისმ ნოვამინსულფატს ვსვამ. ის მადარდებს , ოპერაციამდე სიმსივნე არ განვითარდეს. ნაღვლის შემთხვევაში მკურნალობას შედეგი არა აქვს.

Posted by: Faniack 25 Apr 2016, 15:49
J&J
არაფრის smile.gif იგივე პარაცეტამოლია ნოვამინსულფატი , თუ არასაკმარისია იბუპროფენი შეგიძლია იხმარო .

QUOTE
  ოპერაციამდე სიმსივნე არ განვითარდეს. ნაღვლის შემთხვევაში მკურნალობას შედეგი არა აქვს.

სიმსივნე მასე მარტივად არ ვითარდება მექანიკური სიყვითლის რისკი უფრო დიდია მანდ თუ სრული ოკლუზია მოხდება სადინრის.

Posted by: medicine2016 25 Apr 2016, 16:36
QUOTE (Faniack @ 25 Apr 2016, 15:49 )
J&J
არაფრის smile.gif იგივე პარაცეტამოლია ნოვამინსულფატი , თუ არასაკმარისია იბუპროფენი შეგიძლია იხმარო .

QUOTE
  ოპერაციამდე სიმსივნე არ განვითარდეს. ნაღვლის შემთხვევაში მკურნალობას შედეგი არა აქვს.

სიმსივნე მასე მარტივად არ ვითარდება მექანიკური სიყვითლის რისკი უფრო დიდია მანდ თუ სრული ოკლუზია მოხდება სადინრის.

ბოდიშIთ რომ გაწუხებთ, გასაგებია, რომ ექიმს უნდა მივენდო ბოლომდე, მაგრამ რა ანალიზიგავიკეთO სიმსივნის გამოსარიცხად მთელ ორგანიზმში ცალ-ცალკე არ მინდა გავიკეთო ტომოგრაფიები
ფულიც არაა პრობლემა
მინდა რომ დავმშვიდდე

Posted by: ჟანრი51 25 Apr 2016, 18:11
Faniack
QUOTE
ვინ დაგინიშნა მკურნალობა და რას იღებ კონრეტულად

ჟანრი51
QUOTE
საშინელი ხველა და ყელის ტკივილი მაქვს 3 კვირაზე მეტია. ღამ-ღამობით ხველება მახრჩობს. დამიდგინეს მწვავე ბრონქიტი და მწვავე ლარინგოტრაქეიტი. მკურნალობას შეგედი არა აქვს ჯერჯერობით. როგორ მოვიქცე?

მადლობა Faniack,

მკურნალობა დამინიშნა პულმონოლოგმა. მესამე დღეა ვიღებ - ინჰალიპტი, ზეფექსალი 180 მგ, ლუკამონტი 10 მგ, ბროვენსინი. სიცხეეს მაძლევს 37.3-მდე.

Posted by: J&J 25 Apr 2016, 22:52
Faniack
მექანიკური სიყვითლე რა არის? სადინარები თავისუფალია. ხვალაც მივდივარ კონტროლზე. აი ტკივილი საშინელებაა. სუნთქვაც კი მიჭირს ხოლმე. ერთია გულისრევა არა მაქვს.

Posted by: Faniack 25 Apr 2016, 23:05
medicine2016
სიმსივნის მარკერები შეგიძლია გაიკეთო თუმცა კიდე ერთხელ გიმეორებ შენი პრობლემა მხოლოდ ფსიქოლოგიურია და შენ ასაკში და ჩივილებში არანაირად არ შეესაბამება რაიმე სახის სიმსივნეს smile.gif ტყუილად ნერვიულობ .

ჟანრი51
ნორმალური დანიშნულებაა დრო სჭირდება უბრალოდ მიჰყევი მაგ დანიშნულებას , გამღიზიანებლებს მოერიდე სიგარეტს და ა.შ თუ ტემპერატურამ იმატა და ჩივილები გამწვავდა შეიძლება ანტიბიოტიკების სამომავლოდ ჩართვა ამ ეტაპზე არ ვთვლი არც მე საჭიროდ

J&J
მექანიკური სიყვითლე არის სიყვითლე რომელიც ნაღვლის სადინრის მექანიკური ობსტრუქციის გამო და ნაღვლის შეგუბების გამო შეიძლება განვითარდეს გააჩნია კონკრემენტი რამხელაა.გასაგებია არასასიამოვნოა თუმცა ოპერაციის გარდა სხვას ვერაფერს იზავ სიმპტომურად თუ შეიმსუბუქებ ჩIვილებს .

Posted by: medicine2016 25 Apr 2016, 23:08
QUOTE (Faniack @ 25 Apr 2016, 23:05 )
medicine2016
სიმსივნის მარკერები შეგიძლია გაიკეთო თუმცა კიდე ერთხელ გიმეორებ შენი პრობლემა მხოლოდ ფსიქოლოგიურია და შენ ასაკში და ჩივილებში არანაირად არ შეესაბამება რაიმე სახის სიმსივნეს smile.gif ტყუილად ნერვიულობ .

ჟანრი51
ნორმალური დანიშნულებაა დრო სჭირდება უბრალოდ მიჰყევი მაგ დანიშნულებას , გამღიზიანებლებს მოერიდე სიგარეტს და ა.შ

J&J
მექანიკური სიყვითლე არის სიყვითლე რომელიც ნაღვლის სადინრის მექანიკური ობსტრუქციის გამო და ნაღვლის შეგუბების გამო შეიძლება განვითარდეს გააჩნია კონკრემენტი რამხელაა.გასაგებია არასასიამოვნოა თუმცა ოპერაციის გარდა სხვას ვერაფერს იზავ სიმპტომურად თუ შეიმსუბუქებ ჩIვილებს .

და ეს მარკერები რა ღირს
როგორ გავიკეთე
ნორმები რამდენია ხო ვიცი, მაგრამ მოვისვენებ
რა ერთხელ და სამუდამოდ

Posted by: Faniack 25 Apr 2016, 23:12
medicine2016
რაღირს ნამდვილად ვერ გეტყვი შეგიძლია მიხვიდე დაუშვათ სინევოში და დაგაკვალიანებენ smile.gif მე მაინც ფსიქოლოგს გირჩევ . ზოგად სიმსივნური პროცესების დროს უმეტეს შემთხვევაში გარკვეული ცვლილებები სსა შიც იქნებოდა ედსიც მომატებულია ხშირშემთხვევაში . ოჯახის წევრს ან ვინმე ახლობელს ხოარ დაუდგინდა ბოლოპერიოდში რაიმე ონკოლოგიური დაავადება ? საიდან წამოვიდა ეს შიში რა საფუძველი აქვს საკუთარ თავს ამ სიტუაციაში შენვე უნდა აჯობო თუ ვერ შეძლებ მარტო გამკლავებას მიმართე ფსიქოლოგს მოგეხმარება ამ ყველაფერში. ავად ნამდვილად ხარ და შენ დაავადებას იპოქონდრია ქვია მეგობარო .

Posted by: medicine2016 25 Apr 2016, 23:16
QUOTE (Faniack @ 25 Apr 2016, 23:12 )
medicine2016
რაღირს ნამდვილად ვერ გეტყვი შეგიძლია მიხვიდე დაუშვათ სინევოში და დაგაკვალიანებენ smile.gif მე მაინც ფსიქოლოგს გირჩევ . ზოგად სიმსივნური პროცესების დროს უმეტეს შემთხვევაში გარკვეული ცვლილებები სსა შიც იქნებოდა ედსიც მომატებულია ხშირშემთხვევაში . ოჯახის წევრს ან ვინმე ახლობელს ხოარ დაუდგინდა ბოლოპერიოდში რაიმე ონკოლოგიური დაავადება ? საიდან წამოვიდა ეს შიში რა საფუძველი აქვს საკუთარ თავს ამ სიტუაციაში შენვე უნდა აჯობო თუ ვერ შეძლებ მარტო გამკლავებას მიმართე ფსიქოლოგს მოგეხმარება ამ ყველაფერში. ავად ნამდვილად ხარ და შენ დაავადებას იპოქონდრია ქვია მეგობარო .

ედსი 6 მქონდა ერთი კვირის წინ და ყველაფერი ნორმაში არც წინ და არც უკან
ბოლო პერიოდშI რომ დავფიქრდი მეთქი საიდან მოდის ეს დისკომფორტი მეთქი
1 წლის წინ არც ისე ახლობელი, მაგრამ მაინც ახლობელი გარდაიცვალა ამ დაავადებით.
სულ მესმოდა ეს მორფის დოზები როგორ უკეთებდნენ
გასვენებაშიც ვიყავი და ფსიქოლოგიურად იმოქმედა ჩემზე

Posted by: J&J 26 Apr 2016, 00:06
Faniack
დიდი მადლობა ყურადღებისათვის. იმედია წლის ბოლოს მაინც მოვახერხებ ოპ ს გაკეთებას და მანამდე არაფერი დამემართება.

Posted by: Kaifistka 26 Apr 2016, 01:11

აუ რაღაც უაზრობა მჭირს და არვიცი ვის ვკითხო.

მოკლედ მეგობარმა მიყიდა თბილი ელექტრონული ადიალა. რომ ჩავრთავ ათბობს.

ხოდა რომ გადავიფარებ და მუხლები მითბება მტკივა.

რატომ შეიძლება ადამაინს სითბოზე მუხლები ტკიოდეს?

არ მწვავას და არაფერი რა თქმა უნდა.

გუგლში ვერ ვიპოვე პასუხი თორემ არ შეგაწუხებდით.

Posted by: medicine2016 26 Apr 2016, 10:20
QUOTE (medicine2016 @ 25 Apr 2016, 23:16 )
QUOTE (Faniack @ 25 Apr 2016, 23:12 )
medicine2016
რაღირს ნამდვილად ვერ გეტყვი შეგიძლია მიხვიდე დაუშვათ სინევოში და დაგაკვალიანებენ smile.gif მე მაინც ფსიქოლოგს გირჩევ  . ზოგად სიმსივნური პროცესების დროს უმეტეს შემთხვევაში გარკვეული ცვლილებები სსა შიც იქნებოდა ედსიც მომატებულია ხშირშემთხვევაში . ოჯახის წევრს ან ვინმე ახლობელს ხოარ დაუდგინდა ბოლოპერიოდში რაიმე ონკოლოგიური დაავადება ? საიდან წამოვიდა ეს შიში რა საფუძველი აქვს საკუთარ თავს ამ სიტუაციაში შენვე უნდა აჯობო თუ ვერ შეძლებ მარტო გამკლავებას მიმართე ფსიქოლოგს მოგეხმარება ამ ყველაფერში. ავად ნამდვილად ხარ და შენ დაავადებას იპოქონდრია ქვია მეგობარო .

ედსი 6 მქონდა ერთი კვირის წინ და ყველაფერი ნორმაში არც წინ და არც უკან
ბოლო პერიოდშI რომ დავფიქრდი მეთქი საიდან მოდის ეს დისკომფორტი მეთქი
1 წლის წინ არც ისე ახლობელი, მაგრამ მაინც ახლობელი გარდაიცვალა ამ დაავადებით.
სულ მესმოდა ეს მორფის დოზები როგორ უკეთებდნენ
გასვენებაშიც ვიყავი და ფსიქოლოგიურად იმოქმედა ჩემზე

რისი ონკომარკერი გავიკეთო მივიდე და ვუთხრა მთელი სხეულის?

Posted by: Faniack 26 Apr 2016, 16:32
medicine2016
არაფრის ონკომარკერი არ გინდა კაროჩე ტყუილად ნერვიულობ არანაირი ობიექტური მიზეზი არ გაქ !

J&J
წარმატებები smile.gif

Kaifistka
სისხლძარღვების გაფართოებას იწვევს სითბო , ვენების ვარიკოზული პრობლემები თუ გაქვს თუმდაც უმნიშნველოდ რომელიც ჩვეულებრივ არ გაწუხებს სითბოს ზემოქმედებით უფრო მწვავედ გამოვლინდეს შეიძლება სიმძიმის განცდა ფეხებში , წვა,წვივის კუნთების დაჭიმულობა და ა.შ smile.gif

გამოსავალი მარტივია ნუ დაიფარებ წელქვევით ადიალას ეცადე მოერიდო ზედმეტად ცხელ აბაზანებს smile.gif

Posted by: ჟანრი51 26 Apr 2016, 22:03
Faniack
მადლობა.. გაიხარეთ!

Posted by: medicine2016 26 Apr 2016, 22:26
Faniack

აი ეხლაც ერთღI საათის წინ მარცხენა ხელ ირამდენიმე წუთით წაიღო ლამის
ნერვოზულია?
ეხლა გამიარა მარა პატარ პატარა შოკებივით მაქ სხვეულის სხვა ნაწილებზე ხელებზე და ფეხზე ვემოთ

Posted by: Faniack 26 Apr 2016, 22:58
medicine2016
ნევროპათოლოგთან ხო ხარ ნამყოფი? ეეგ გადაგიღო? ვენდობი ჩემ კოლეგას და ნევროლოგიურ პათოლოგიას გამოვრიცხავ შესაბამისად რჩება ფსიქოლოგიური და ნევროტული ფაქტორი აშკარაა ისედაც

გადაერთე სხვა რამეზე და ვარჯიში დაიწყე როგორც გითხარი

Posted by: medicine2016 27 Apr 2016, 00:30
QUOTE (Faniack @ 26 Apr 2016, 22:58 )
medicine2016
ნევროპათოლოგთან ხო ხარ ნამყოფი? ეეგ გადაგიღო? ვენდობი ჩემ კოლეგას და ნევროლოგიურ პათოლოგიას გამოვრიცხავ შესაბამისად რჩება ფსიქოლოგიური და ნევროტული ფაქტორი აშკარაა ისედაც

გადაერთე სხვა რამეზე და ვარჯიში დაიწყე როგორც გითხარი

სიხლის საერთო გავიკეთე არაფერი ისეთი

Posted by: Kaifistka 27 Apr 2016, 00:53
Faniack
QUOTE
სისხლძარღვების გაფართოებას იწვევს სითბო , ვენების ვარიკოზული პრობლემები თუ გაქვს თუმდაც უმნიშნველოდ რომელიც ჩვეულებრივ არ გაწუხებს სითბოს ზემოქმედებით უფრო მწვავედ გამოვლინდეს შეიძლება სიმძიმის განცდა ფეხებში , წვა,წვივის კუნთების დაჭიმულობა და ა.შ

გამოსავალი მარტივია ნუ დაიფარებ წელქვევით ადიალას ეცადე მოერიდო ზედმეტად ცხელ აბაზანებს


ვაიმეე დიდი მადლობა.
კი მაწუხებს ხანდახან აი დიდხხანს თუ ვდგავარ სამსახური მაქვს ეგეთი (8 საათი და მეტი მიწევს ფეხზე ხანდახან) და ცოტა ჭარბი წონა.

აი მუხლებში ვერ წარმოვიდგენდი.

ცხელი აბაზანები მიყვარს არადა აი რომ გწვავს კანს ლამის ისეთი. გადავეჩვევი.

სხვა რა გავაკეთო პროფილაქტიკისთვის რომ მერე ნაკლებად შემაწუხოს?
ალბათ წონაც ცუდია.

Posted by: ჟანრი51 27 Apr 2016, 01:47
QUOTE
საშინელი ხველა და ყელის ტკივილი მაქვს 3 კვირაზე მეტია. ღამ-ღამობით ხველება მახრჩობს. დამიდგინეს მწვავე ბრონქიტი და მწვავე ლარინგოტრაქეიტი. მკურნალობა დამინიშნა პულმონოლოგმა. მესამე დღეა ვიღებ - ინჰალიპტი, ზეფექსალი 180 მგ, ლუკამონტი 10 მგ, ბროვენსინი. სიცხეეს მაძლევს 37.3-მდე.  კურნალობას შეგედი არა აქვს ჯერჯერობით.  როგორ მოვიქცე?



Faniack
QUOTE
ნორმალური დანიშნულებაა დრო სჭირდება უბრალოდ მიჰყევი მაგ დანიშნულებას , გამღიზიანებლებს მოერიდე სიგარეტს და ა.შ თუ ტემპერატურამ იმატა და ჩივილები გამწვავდა შეიძლება ანტიბიოტიკების სამომავლოდ ჩართვა ამ ეტაპზე არ ვთვლი არც მე საჭიროდ


მადლობა..
ღამე ხველება მაღვიძებს და სული მეხუთება. ამოსუნთქვის საშუალებას არ მაძლევს. სერიოზულად მეშინია ღამე რომ მოდის, არ დავიხრჩო. იქნებ მირჩიოთ რა არის ამისი საშველი.. ტემპერატურამ მომიმატა და დღეს ანტიბიოტიკი დამინიშნა ექიმმა „როდინირი“.


Posted by: Faniack 27 Apr 2016, 10:41
Kaifistka
ფიზიკური ვარჯიში კუნთთა ტონუსმა რო მოიმატოს ფეხით სიარული იდეალურია მეტი ამეტაპზე არაფერი
ჟანრი51
ინჰალატორი ხომ არ გაქ სახლში ? ანტიბიოტიკი სწორად აქვს არჩეული , თუ გაქვს პულმიკორტის ინჰალაცია დრეში სამჯერ , ვენტოლინი საღამოს ძილის წინ შეგიმსუბუქებს ჩივილებს

Posted by: hikikomori 28 Apr 2016, 00:04
გამარჯობათ,

ბოლო პერიოდია ფილტების არეში მაწუხებს ტკივილი, ხან ერთზე ხან მეორეზე
რაღაც ჭვალივითაა თითქოს, ხანდახან მჩხვლეტსსავით

რენტგენი რო გადავიღო გამოჩნდება რამე თუ მჭირს?

სისხლის საერთო ანალიზი გავიკეთე 1 თვის წინ დაახლოებით და რამე სერიოზული რომ ყოფილიყო არ გამოჩნდებოდა?
მწეველი ვარ, ძააან ბევრს არ ვეწევი მარა ალბათ რაოდენობას ამ შემთხვევაში არ აქვს გადაწყვეტი მნიშვნელობა

Posted by: medicine2016 28 Apr 2016, 12:04
QUOTE (hikikomori @ 28 Apr 2016, 00:04 )
გამარჯობათ,

ბოლო პერიოდია ფილტების არეში მაწუხებს ტკივილი, ხან ერთზე ხან მეორეზე
რაღაც ჭვალივითაა თითქოს, ხანდახან მჩხვლეტსსავით

რენტგენი რო გადავიღო გამოჩნდება რამე თუ მჭირს?

სისხლის საერთო ანალიზი გავიკეთე 1 თვის წინ დაახლოებით და რამე სერიოზული რომ ყოფილიყო არ გამოჩნდებოდა?
მწეველი ვარ, ძააან ბევრს არ ვეწევი მარა ალბათ რაოდენობას ამ შემთხვევაში არ აქვს გადაწყვეტი მნიშვნელობა

სისხლის ანალიზზე ჩანს რამე სერიოზული დაავადებები თუ არის ბოლო სტადია, ან რამე ანთება
მაგრამ შეიძლება გაგიცივდა, ნერვოზულია ან სისხლი არ მიდის კარგად იმ ადგილას
რავიცი აბა
ფილტვზე რამე სერიოზული რომ იყოს ამ 1 თვეში მოგტეხავდა კაცს

Posted by: hikikomori 28 Apr 2016, 12:20
medicine2016

მადლობა გამოხმაურებისთვის,
ღია ფანჯარასთან კი ვზივარ ხოლმე სამსახურში და შეიძლება მაგის ბრალი რო იყოს?
ნუ ქარში და გრიგლში არა მარა კარგი ამინდი როა სულ ღია მაქვს ხოლმე
სიგარეტზე შემეშინდა ცოტა, შეიძლება მაგის ბრალიც რო იყოს?

Posted by: medicine2016 28 Apr 2016, 12:30
QUOTE (hikikomori @ 28 Apr 2016, 12:20 )
medicine2016

მადლობა გამოხმაურებისთვის,
ღია ფანჯარასთან კი ვზივარ ხოლმე სამსახურში და შეიძლება მაგის ბრალი რო იყოს?
ნუ ქარში და გრიგლში არა მარა კარგი ამინდი როა სულ ღია მაქვს ხოლმე
სიგარეტზე შემეშინდა ცოტა, შეიძლება მაგის ბრალიც რო იყოს?

როგორი ტკივილია? სიცხე გაქვს ხველება? წონა როგორი გაქვს
გამაყუჩებლები გჭირდება ასაკი?

Posted by: hikikomori 28 Apr 2016, 12:32
medicine2016

არა გამაყუჩებელი არ მჭირდება, ხანდახან მტკივა ხოლმე მუდმივად არა
აი რაღაც დაახლოებით ყრუ ტკივილია
ხანდახან ჩხვლეტასავით
არც მახველებს, არც სიცხე არ მაქვს

55 კილო ვარ- 25 წლის

Posted by: medicine2016 28 Apr 2016, 12:36
hikikomori

რავი სისხლის ანალიზის მონაცემები გახსოვს ძირითადი?

Posted by: hikikomori 28 Apr 2016, 12:37
medicine2016

კი, მახსოვს
რენტგენზე რო მივიდე გამოჩნდება რამე თუა?

Posted by: medicine2016 28 Apr 2016, 12:39
hikikomori

რამდენიმე მომწერე აბა მონაცემი
კი რენტგენზე გამოჩნდება ექოსკოპიაზეც თუ ფული გაქვს ტომოგრაფიაც გადაიღე რავი

Posted by: Faniack 28 Apr 2016, 15:35
hikikomori
გამარჯობა , რას უკავშირებ ჩხვლეტას ანუ ფიზიკურად რო დაიტვირთები მაშინ იწყება თუ მოსვენებულ მდგომარეობაშიც ? ადგილობრივად ხეჭის დაჭერით გტკივა ? ჩასუნთქვისას ხომარარის ?

არ გინდა რენტგენი ფილტვებს არ ახასიათებს ტკივილი მითუმეთეს მჩხვლეტავი .



medicine2016
რაშიც არ ხარ კომპეტენტური თავი შეიკავე.რჩევებისგან შეიძლება ვინმეს რაიმე ზიანი მიაყენო , თუმდაც მატერიალური...

Posted by: hikikomori 28 Apr 2016, 21:06
Faniack
გამარჯობა,

ვვარჯიშობ ხოლმე , მაგრამ ბოლო 2 კვირაა არ მივარჯიშია მაგრამ ტკივილი მაინც მაქვს , მოსვენებულ მდგომარეობაშიც ღამეც ხანდახან
არც ჩასუნთქვისას და არც ხელის დაჭერაზე
აი ჩვეულებრივად რომ ვსუნთქავ ხანდანხან მტკივა ხოლმე

შეიძლება ნერვოზის თუ ნევროზის ბრალი რომ იყოს?


Posted by: Faniack 28 Apr 2016, 22:32
hikikomori
წესით ნერვის ადგილობრივი რეაქცია უნდა იყოს პერიოდულად

შეგიძლია ადგილობრივად დიკლოფენაკის გელი წაისვა რამდენიმე დღე და ვნახოთ რა შედეგი გექნება


Posted by: hikikomori 28 Apr 2016, 22:55
Faniack

მადლობა გამოხმაურებისთვის და რჩევისთვის

Posted by: pvm 29 Apr 2016, 00:38
პაციენტს ამოეთესა მიკოპლაზმა პნევმონია და ქლამიდია პნევმონია, იმკურნალა ანტიბიოტიკებით, გაუმჯობესდა ზოგადი მდგომარეობა, მაგრამ მაინც აქვს ჩივილები. თერაპევტიც ვეღარ ამბობს ვერაფერს.

ვის უნდა მიმართოს ადამიანმა ასეთ დროს? და რამდენად საშიშია ეს ინფექციები?!


პ.ს. პაციენტ უჩივის საშინელ ხველას -მოხრჩობის შეგრძნებით, რომელიც ძალიან შეუმცირდა დიდხნიანი მკურანლობის შემდეგ, მაგრამ ხანდახან ისევ აქვს, უფრო დილით.

Posted by: Faniack 29 Apr 2016, 15:56
hikikomori
არაფრის smile.gif

pvm
განმეორებით გულმკერდის რენტგენი თუ კვლავ არის პნევმონია
განმეორებით ნახველის ბაქტერიოლოგია და პლიუს ანტიბიოტიკო გრამა გააკეთეთ მაგის მიხედვით აირჩევა შემდგომი ანტიობიოტიკოთერაპია

რას იღებდა კონკრეტულ რომელ ანტიბიოტიკს და რამდენი ხანი?


მწეველია პაციენტი? ანამნეზში რაიმე ქრონიკული ფილტვის პათოლოგია ხომ არ აქვს ?


Posted by: medicine2016 29 Apr 2016, 18:28
Faniack

კიდევ რომ გაწუხებთ უკაცრავად,
კიდურებში მონაცვლეობით ეგეთი რამე რეინო სინდრომს შეუძლია გამოიწვიოს?
აი გული მიფეთქავს მაგრად და ხანაც ვენები რომ ვაკვირდები სპაზმებია ეს თუ რა არის არ ვიცი
რომ ვწერ ეხლა მეშIნია ხელებში ისევ არ დამიწყოს შეტევები

Posted by: Faniack 29 Apr 2016, 20:07
medicine2016
დარწმუნებული ვარ გადაიკითხე უკვე ყველაფერი , არშეიძლება ასე ფსიქიატრიულში ამოყოფ ასე თავს გადაერთე სხვა რამეზე ! არაფერი სერიოზული არ გჭირს !


https://en.wikipedia.org/wiki/Hypochondriasis აი შენ რა გჭირს


https://ka.wikipedia.org/wiki/%E1%83%98%E1%83%9E%E1%83%9D%E1%83%A5%E1%83%9D%E1%83%9C%E1%83%93%E1%83%A0%E1%83%98%E1%83%90


ეცადე საკუთარ თავს მოერიო ! მიდი ფსიქოლოგთან მეათედ გიმეორებ უკვე მე და ვერც ვერავინ სხვა აქ ვერაფრით დაგეხმარება

Posted by: fdfdfd 30 Apr 2016, 17:16
Faniack

მოგესალმებით, როცა ვწევარ მგონია, რომ გული მაწუხებს და უკნიდან ბეჭში და ხელის გაყოლებაზე დენს მირტყამს
2 თვეა ასე ვარ
ვისთან მივიდე? რამე საშიშია?

Posted by: Faniack 1 May 2016, 12:39
fdfdfd
რამხელა ხარ ასაკით? დავუშვათ რომელ სართულზე შეგიძლია ახვიდე კიბით ისე რო არ დაიღალო? გულის მხრივ რაიმე პრობლემა გქონია ოდესმე?


ხერხემლის პრობლემა უნდა იყოს სავარაუდოდ , პირველრიგში რენტგენოსკოპია გაიკეთე და დასკვნა რო გექნება მაგის მიხედვით დავკვალიანდებით

Posted by: fdfdfd 1 May 2016, 12:43
QUOTE (Faniack @ 1 May 2016, 12:39 )
fdfdfd
რამხელა ხარ ასაკით? დავუშვათ რომელ სართულზე შეგიძლია ახვიდე კიბით ისე რო არ დაიღალო? გულის მხრივ რაიმე პრობლემა გქონია ოდესმე?


ხერხემლის პრობლემა უნდა იყოს სავარაუდოდ , პირველრიგში რენტგენოსკოპია გაიკეთე და დასკვნა რო გექნება მაგის მიხედვით დავკვალიანდებით

სქოლიოზი მქონდა ადრე რავი ხერხემალზე არაფერი მაწუხებს
აი მგონია, რომ მთელ ხელზე რაღაცა მიფეთქავს ხოლმე და გულიც მიბრაგუნებს
კიბეზე რავი არ მიცდია წონა მაქ საკაიფო
იქნებ ხელებზე და ფეხებზე აქვს დაწოლა
22 ის ვარ
აი მარცხენა ხელი ბეჭიდან მხრიდან მგონია რომ მიკანკალებს, მაგრამ სინამდვილეში არ კანკალებს მოკლედ რა smile.gif

Posted by: Faniack 1 May 2016, 23:07
fdfdfd
დაბუჟება გაქვს ხელში?


მიდი ოჯახის ექიმთან კარდიოგრამას გადაგიგებს , მე მაინც ვფიქრობ რომ ხერხემლიდან გამომდინარე უნდა იყოს პრობლემა სქოლიოზი კი გასაგებია მარა შეიძლება ნერვზე ზეწოლა იყოს თუმდაც მცირე რომელიც მსგავს სიმპტომებს მოგცემს...

Posted by: fdfdfd 2 May 2016, 10:33
QUOTE (Faniack @ 1 May 2016, 23:07 )
fdfdfd
დაბუჟება გაქვს ხელში?


მიდი ოჯახის ექიმთან კარდიოგრამას გადაგიგებს , მე მაინც ვფიქრობ რომ ხერხემლიდან გამომდინარე უნდა იყოს პრობლემა სქოლიოზი კი გასაგებია მარა შეიძლება ნერვზე ზეწოლა იყოს თუმდაც მცირე რომელიც მსგავს სიმპტომებს მოგცემს...

დაბუჟება არ მაქვს
თითქოს სისხლი არ მიეწოდება კიდურებს და რაღაც უსიამოვნო ჩხვლეტები მაქვსხოლმე
15 კილო რო დავიკლო შვებას არ მომცემს:?

Posted by: medicine2016 3 May 2016, 15:10
QUOTE (fdfdfd @ 2 May 2016, 10:33 )
QUOTE (Faniack @ 1 May 2016, 23:07 )
fdfdfd
დაბუჟება გაქვს ხელში?


მიდი ოჯახის ექიმთან კარდიოგრამას გადაგიგებს , მე მაინც ვფიქრობ რომ ხერხემლიდან გამომდინარე უნდა იყოს პრობლემა სქოლიოზი კი გასაგებია მარა შეიძლება ნერვზე ზეწოლა იყოს თუმდაც მცირე რომელიც მსგავს სიმპტომებს მოგცემს...

დაბუჟება არ მაქვს
თითქოს სისხლი არ მიეწოდება კიდურებს და რაღაც უსიამოვნო ჩხვლეტები მაქვსხოლმე
15 კილო რო დავიკლო შვებას არ მომცემს:?

კაი დავუშვათ თუ მოყოლილია ნერვი რა უნდა გავაკეთო
მეზარება ექიმთან მისვლა და ტომოგრაფიები
არაფერი მაწუხებს ხერხემალში და ისე გადასცემს კიდურებს?

Posted by: Masavem 10 May 2016, 09:54
არგაატაროთ რა user.gif



კაროჩე სირბილის დროს ტკივილი მეწყება გულმკერდის არეში და ძან შემაწუხებელია ადრე ასეთირამ არდამმმარყნია არვეწევი არვსვამ უბრალოდ რამდენიმე წელიწადია არ მირვენია ახლა დავიწყე და ეს რისი ბრალიუნდაიყოს ხომვერმეტყვით ?? sad.gif

Posted by: Faniack 10 May 2016, 10:01
Masavem
გამარჯობა, კარდიოლოგს მიმართე და ფიზიკური დატვირთვის ტესტს გაგიკეთებს სარბენ ბილიკზე , პირველ რიგში კარდიოლოგიური პრობლემა უნდა გამოირიცხოს , რამდენი წლის ხარ ? წონა რამდენიგაქ ? რაიმე პრობლემა კარდიოლოგიური გქონია ოდესმე ?

Posted by: Masavem 10 May 2016, 10:27
QUOTE (Faniack @ 10 May 2016, 10:01 )
Masavem
გამარჯობა, კარდიოლოგს მიმართე და ფიზიკური დატვირთვის ტესტს გაგიკეთებს სარბენ ბილიკზე , პირველ რიგში კარდიოლოგიური პრობლემა უნდა გამოირიცხოს , რამდენი წლის ხარ ? წონა რამდენიგაქ ? რაიმე პრობლემა კარდიოლოგიური გქონია ოდესმე ?

23 ს... 62 კგ..



არასოდეს არანაირი პრობლემა არმქონია ამ მხრივ sad.gif

Posted by: Pac. 10 May 2016, 15:12
ყელმა დამტანჯა და ვერ გამიგია რა არის....
ხორხთან დაახლოებით მგონია რო შიგნიდან რაღაც მაქვს გაჩხერილი და სუნთქვაშიც მიშლის ხელს და დიდი დისკომფორტია ძაან
სულ არ მჭირს , მაგრამ რო ჩავჯდები ხოლმე, აი სადღაც დაჯდომა წამოწოლის შუაში რა მდგომარეობაც არის ეგრე რო ვჯდები ან ვწვები ეგრევე მეწყება და მერე შეიძლება მთელი დღე შემაწუხოს
რავქნა? რას მირჩევთ ?
* * *
QUOTE
ყელმა დამტანჯა და ვერ გამიგია რა არის....
ხორხთან დაახლოებით მგონია რო შიგნიდან რაღაც მაქვს გაჩხერილი და სუნთქვაშიც მიშლის ხელს და დიდი დისკომფორტია ძაან
სულ არ მჭირს , მაგრამ რო ჩავჯდები ხოლმე, აი სადღაც დაჯდომა წამოწოლის შუაში რა მდგომარეობაც არის ეგრე რო ვჯდები ან ვწვები ეგრევე მეწყება და მერე შეიძლება მთელი დღე შემაწუხოს
რავქნა? რას მირჩევთ ?


ვისთან მივიდე , სად მივიდე
ეგ მაინც მირჩიეთ
დავიტანჯე რა

Posted by: Faniack 11 May 2016, 19:36
Masavem
არუნდა იყოს არაფერი თუმცა მაინც გირჩევ კარდიოლოგთან მიხვიდე და ფიზიკური დატვირთვის ტესტი გაიკეთო.


Pac.
ყელყურთან მიდი

რამდენიხანია დაგეწყო? რამე პრობლემა გქონია ოდესმე? გლანდები გაქ? ფარისებრზე რაიმე პრობლემა ხოარ გაქ?

ტემპერატურა ხველა?

Posted by: gvancula3007 11 May 2016, 22:49
4 wlis bavshs nadebi aqvs enaze periodulad.shemdeg yelis problems ewyeba da antibiotikit mtavrdeba.axla gsuchnda isev.sqeli nadebi.ekimtan ver mimyavs paraskevamde.fagio chavasxi.vfikrob mikrobuli xom araa rame. Nistatini ROM mivce davashaveb dzalian ? Gmadlobt

Posted by: Faniack 12 May 2016, 00:00
gvancula3007
გამარჯობა ანგინის სურათს აღწერ , ტიპურია ამ ასაკის ბავშვისთვის , არანაირი ფაგიო და ნისტატინი ეგ მხოლოდ ანტიბიოტიკების შემდგომი გართულების პროფილაკტიკისთვის .


ანგინა გამოწვეულია ბაქტერიული ფლორით 95% ში და ორგანიზმი თვითონვე მოერევა ხშირ შემთხვევაში საჭირო ხელშეწყობით

ყელი წითელიაქვს ბავშვს?

ნუშურა ჯირკვლები აქვს? შარშან ამ დროიდან რამდენჯერ ჰქონდა მსგავსი პრობლემა ანგინის სახით?



ამ ეტაპზე თუ ტემპერატურა 38 გრადუსამდეა არანაირი მედიკამენტი არ გჭირდებათ, წოლითი რეჟიმი , დიდი რაოდენობით სითხეები (ჩაი ლიმნით , ნატურალური წვენები ოთახის ტემპერატურის ) ხილი მაღალკალორიული კვება .
თუკი ტემპერატურა შემაწუხებელი გახდება ან 38 აცდება პარაცეტამოლი ნახევარი აბი



Posted by: gvancula3007 12 May 2016, 00:15
Mapatiet shrifti.AR aqvs sicxe .arc surdo.chamitac chams da xasiatzea.ASE nadebit sheizleba daiwyos angina? Bagshi muvedit da seqtembridan akamde 4-5 jer hjonda virusi yelit.wina tveshi miigo antibiotikebi da. Msgas vabraleb.

Posted by: Faniack 12 May 2016, 14:20
gvancula3007
რასაბრალებ ანტიბიოტიკებს?

კი შეიძლება დაიწყოს, ყელი წითელი არ აქვს?



Posted by: medicine2016 12 May 2016, 19:31
Faniack

ძარღვი გიფეთქავდეს ერთ ხელზე და მეორეზე რამდენიმე წამით ნორმალურია?

Posted by: Pac. 13 May 2016, 12:03
Faniack
ვიყავი, ფარინგიტი გაქვს მწვავეო
ვიმკურნალებ ამას და მერე ისევ თუ მექნება , ფარისებრზე შევემოწმები...
ადრე არ მქონია არანაირი პრობლემა
ეს ფარინგიტიც შემომეპარა საიდანღაც biggrin.gif

Posted by: Faniack 13 May 2016, 13:39
Pac.
ხოდა ეგეც მიზეზი smile.gif იმკურნალე კარგად იქნება ყველაფერი smile.gif

medicine2016
=]] კი

Posted by: burninghall 13 May 2016, 19:48
Faniack

სალამი
ამ ბოლო დროს იდაყვთან ხან ქუსლზე ხან თითზე მიხურდება რამდენიმე წამი, თითქოს დამწვარ ასანთის ღერს მადებენ
ნერვოზულია რამე? ვისთან მისვლას მირჩევთ?

Posted by: Faniack 13 May 2016, 22:44
burninghall
გამარჯობა

მოძრაობისას დისკომფორტი ხომარ გაქ? სახსარი ხოარ არის შეწითლებული ვიზუალურად ან მტკივნეული მოძრაობისას ? რამეს უკავშირებ?

რამხენა ხარ რა სქესის?

Posted by: burninghall 13 May 2016, 23:03
Faniack

20 ის ვარ მამაკაცი
უმოძრაო ცხოვრების წესს
წონის ჩქარი მომატება დაკლება შემიძლია
ნერვოზს რა ორგანოები არ მაწუხებს არც ხერხემლიდან მოდის
ტიპიური ნერვოზული ჩივილებიაო ექიმმა

მოძრაობისას არა რომ ვდგავარ ფსიქოლოგიურად მგონია, რომ ფეხში რაღაცა მაქვს და სიარულის მეშინია
ისე ავრბივარ კიბეებზე
სისხლში ქოლესტერინი ხო არ გავისინჯო?
შესქელებული რომ იყოს ან რამე არა მგონია
ნერვული მგონია მე მაინც
გულზე, ბეჭებზე, ხელებზე, ტერფზე წამიერად მირტყამს რაღაცა

Posted by: Faniack 14 May 2016, 00:38
burninghall
ვერაფერს გეტყვი ასე ზეპირად გირჩევ ჯანსაღ ცხოვრების წესს და ფიზიკური აქტივობის მატებას.


Posted by: fdfdfd 14 May 2016, 16:07
Faniack

სალამი ძილში რამდენი დეზოდორიც არ უნდა წავისვა ოფლიანი ვარ დილით
ხელები სულ სველი მაქვს
შეიძლება ნერვოზის ბრალი იყოს?
ფარისებრი გამოკვლეული მაქვს

Posted by: buska13 19 May 2016, 16:49
გამარჯობათ, ბოლო ერთი კვირაა ყელში ბურთივით ვგრძნობ, შეიძლება დღე გავიდეს არც იყოს მაგრამ ისევ განმეორდეს მეორე დღეს, ძირითადად მარცხენა მხარეს, ხანდახან ტკივილი გადადის ყურის არეში, რისი ბრალი შეიძლება იყოს, ნერვოზულიც მგონია

Posted by: Faniack 19 May 2016, 21:18
buska13
ყელყურ ცხვირთან მიდი გაგსინჯავს


რაიმე სხვა სიმპტომი გაქ?


Posted by: buska13 19 May 2016, 21:39
არა დანარჩენი არაფერი სიმპტომი არ მაქ, მეზარება ექიმთან წასვლა იქნებ გაიაროს ტქო

Posted by: lukako93 20 May 2016, 02:55
მოგესალმებით

3 დღეა რაღაც ვირუსი ავიკიდე და ხველა მაქვს საშინელი, ასევე სუნთქვის დროს მაქვს ხიხინი და ვიბრაციას გარედანაც კი ვგრძნობ ხელს რომ ვადებ. ძირითადათ ჩასუნთქვისას მაქვს ხიხინი და მახველებს ეგრევე. რამე სერიოზულია?

ჰო და კიდევ ერთი ფაქტი არ მაძლევს მოსვენების საშუალებას. შარშან ივლისში გადავიტანე ფილტვების ანთება მაგრამ არა წოლით რეჟიმში. სამსახურში დავდიოდი და ყოველსაღამოს ვიკეთებდი ანტიბიოტიკებს. იქიდან მოყოლებული დამიტოვა ხველა და რატომღაც უყურადღებოდ დავტოვე, არადა თითმის 1 წელია ინტენსიურად ვახველებ (ხიხინები ძალიან იშვიათად მქონდა).

1 კვირაში მივალ ექიმთან და მანამდე იქნებ მითხრათ რისი სიმპტომებია ეს ხიხინები და ანთების გადატანის შემდეგ გამოყოლებული ხველა

Posted by: Faniack 20 May 2016, 13:26
lukako93
გამარჯობა , ტემპერატურა გაქ ? მწეველი ხარ ? მანამდე მიდი ერთ კვირას ნუ გაწელავ ბრონქიტი უნდა იყოს წესით. ხველა პროდუქტიულია ?

Posted by: pvm 20 May 2016, 18:42
ძილის კვლევა –პოლისომნოგრაფია უნდა გავუკეთო დღეს ოჯახის წევრს და დამავიწყდა ექიმისთვის მეკითხა მაისურზე, ანუ გარედან უნდა შემოვახვიო აპარატის ქამარი თუ პირდაპირ ტანზე. თუ იცით მიპასუხეთ რა დღეს.

Posted by: Faniack 20 May 2016, 19:44
pvm
გამარჯობა.

გარედან მაიკაზე არაა პრობლემა

Posted by: lukako93 20 May 2016, 23:03
Faniack

კი, ეს 3 დღეა 37.5-37.8ზე მაქვს. საღამოს საათებში უფრო ვითენთები და მაშინ ვგრძნოიბ სიცხეს და მაშინ ვიზომავ. დილას ძალიან ცუდად არ ვარ და შესაბამისად სიცხესაც არ ვიზომავ

სიგარეტს ვეწევი 4 წელია

მაქვს ნახველი ძალიან ბევრი.

დღეს დილიდან ფილტვები მტკივა ხველებისას. ჯოჯოხეთად მაქვს ქცეული ყოველი ჩახველება და კომფორტულ პოზას ვეძებ და ისე ვახველებ რომ ნაკლებად მეტკინოს. საშინელი რაღაც მაქვს აკიდებული და აშკარად გამირთულდა იმიტო რომ ჩემს ირგვლივ ხალხმაც გადაიტანა ეს ვირუსი მხოლოდ ხველებით.

Posted by: Faniack 21 May 2016, 03:32
lukako93
დროზე მიდი ექიმთან ჯობია ასე ზეპირად მეტს ვერაფერს დაგეხმარები , ოჯახის ექიმთან მიდი მოგისმენს რენტგენს გადაგიღებს სისხლის საერთოს გაგიკეთებს და დაგინიშნავს მკურნალობას თუ საჭირო იქნება

Posted by: niki777 21 May 2016, 15:35
QUOTE (buub @ 3 Jan 2011, 01:13 )
[ჩვენი ტრადიციული მედიცინის შესახებ ხალხური მედიცინა უძველესიაა და არაერთი ადამიანი სარგებლობს მისი გამოყენებით. მაგალითად ხახვის ნახარში კარგია ხველებისთვის. იზორია ალეკო

ჩვენი ტრადიციული მედიცინის შესახებ ხალხური მედიცინა უძველესიაა და არაერთი ადამიანი სარგებლობს მისი გამოყენებით. მაგალითად ხახვის ნახარში კარგია ხველებისთვის. იზორია ალეკო

Posted by: chanelgirl999 25 May 2016, 14:31
ანტიბიოტიკ უნიდოქსს ვიღებ, 1 თვით მაქვს დანიშნული და 2 კვირა უკვე გავიდა და ამ პერიოდში ერთჯერადად ალკოჰოლის მიღება შეიძლება? კონკრეტულად ამ პრეპარატთან მიმართებაში მაინტერესებს

Posted by: Faniack 26 May 2016, 23:14
chanelgirl999
Ara arcert antibiotiktan ar girchev dalevas

Posted by: CL55AMG 28 May 2016, 17:32
გამარჯობათ, მგონი სწორად ვწერ.მოკლედ მესამე დღეა მაწუხებს დისკომფორტი მუცლის მარცხენა მხარეს(ჭიპის გასწვრივ) ამ ბოლო დროს ვჭამ ძალიან ბევრ ცომეულს(თითქმის ყოველდღე), კერძოდ ლობიანს, მაქვს არასწორი კვებითი რეჟიმი და შეიძლება ამას გამოეწვია ეს დისკომფორთი? სხვა არაფერი ახლავს, არც გულისრევა, დიარეა, მადაც მაქვს. მოკლედ ჩვეულებრივად ვარ მაგრამ აი რო ვწვები და ვეყუდები ბალიშს ან რამე შემდეგ მაწუხებს, არც ხელის დადებით მტკივა უბრალოდ დისკომფორტი მაქვს

Posted by: Faniack 28 May 2016, 19:10
CL55AMG
გაზები გაქვს? კოლიკას გავს რასაც ყვები smile.gif მითუმეტეს პარკოსნები ლობიო ჭარბად იწვევს გაზების დაგროვებას , შეგიძლია ესპუმიზანი ან რაიმე მსგავსი მიიღო

Posted by: Top-Gun 30 May 2016, 13:35
გამარჯობა,

მონელებასთან მაქვს პრობლემები და იქნებ დამაკვალიანოთ რა გავაკეთო.

ესე იგი, საჭმლის ჭამიდან დაახლოებით 1-1,5 საათში მეწყება შებერილობა, გულძმარვა, სუნთქვა მიჭირს და ასე შემდეგ.
ასეთი რამ მემართება თითქმის ყველა საჭმელზე გარდა ხილ-ბოსტნეულისა.

რისი ბრალი შეიძლება იყოს?

მე პირადად ვცადე ეუკარბონი - ვერაფერი ვერ შევატყვე, ესპუმიზამი - არაუშავს შედეგი ქონდა, მეზიმფორტე - ვერაფერი შედეგი ვერ ვნახე.
ასევე გამოვიწერე ამაზონიდან პრობიოტიკი, საკმაოდ კარგი ჩანდა, კარგი შეფასებებით და ასე შემდეგ, მაგრამ საერთოდ არანაირი შედეგი არ ქონია და ძალიან იმედგაცრუებული დავრჩი. 60 ლარამდე წყალში გადაყრილი გამომივიდა.

ასევე, რამდენიმე თვის წინ ვირუსი რომ მქონდა ექიმმა ანტიბიოტიკთან ერთად დამინიშნა რომელიღაც მსგავსი პრობიოტიკული საშუალება, სახელს ვერ ვიხსენებ სამწუხაროდ, და მაშინ მახსოვს კმაყოფილი ვიყავი შედეგით. ოღონდ ვერ ვიხსენებ რომელი იყო. 1 ის მახსოვს რომ 2 მლრდ ბაქტერიას შეიცავდა 1 კაფსულა.

თუ რამით დამეხმარებით დიდად მადლიერი ვიქნები smile.gif

Posted by: Faniack 31 May 2016, 03:09
Top-Gun
გამარჯობა გირჩევ ბაქტერიოფაგში მიხვიდე საჭირო ანალიზებს გაგიკეთებენ დიზბაქტერიოზია პირველრიგში გამოსარიცხი თუ არის თვითონვე დაგინიშნავენ მკურნალობას

Posted by: Top-Gun 31 May 2016, 11:07
Faniack
მადლობა. მივალ აუცილებლად smile.gif

Posted by: pvm 12 Jun 2016, 16:30
ძლიერი თერაპევტი ვინმემ თუ იცით მირჩიეთ რააააა

Posted by: Faniack 13 Jun 2016, 11:56
pvm
Raxdeba ram shegawuxa aba mitxari da dagakvalianeb

Posted by: pvm 13 Jun 2016, 16:28
QUOTE (Faniack @ 13 Jun 2016, 11:56 )
pvm
Raxdeba ram shegawuxa aba mitxari da dagakvalianeb

კრეატინკინაზამ მოიმატა ძალიან და მიზეზის დადგენას ვცდილობთ: პაციენტი იღებდა ქოლესტერინის დამშლელ წამალს და შეაწუხა კუნთების (ქვედა კიდურებში) ტკივილმა. შემოწმდა კრეატინკინაზა და იყო მომატებული. შეუწყვიტეს მერე სტატინი და კრეატინკინაზა ჩადგა ნორმაში. 7–8 თვე არ მიუღია არანანაირი სტატინი და თავიდან ფეხების ტკივილი თითქოს არ ქონდა და მერე ( ბოლო 1 –1,5 თვეა ალბათ) ისევ დაეწყო. დაახლოებით 1 თვეა რაც დაიწყო არტერინორმის მიღება და ძალიან არეული ქოლესტერინი მკვეთრად გაუმჯობესდა, თითქმის ნორმა გახდა. ამ წამლის გამო ყველა ექიმი გამორიცხავს კრეატინკინაზის მატებას. ამასთან ერთად ცოტა მოიმატა ღვიძლის ფუნქციებმაც და აქვს ერითროციტოზი ( ოღონდ ცოტა მაღალია ნორმასთან შედარებით, ჰემოგლობინიც შესაბამისად ოდნავ მაღალი).
პაციენტმა გადაიღო ქუსლზე რენტგენი აღმოაჩნდა წანაზარდი ( დეზი) და ფიზიპროცედურებმა ( სხვადასხვა მათ შორის კარიპაზიმი) შედეგი არ მისცა არანაირი.


ეხლა, ამ სტადიაზე მჭირდება გულმკერდის კომპიუტერული ტომოგრაფია ინგოროყვას კლინიკაში, თუ იცით ვინმე კარგი ექიმი ძალიან გთხოვთ მირჩიეთ რა....

დიდი მადლობა წინასწარ

Posted by: Faniack 13 Jun 2016, 16:41
pvm
გულმკერდის კატე რაშუაშია ? ვენები როგორ აქვს ქვემო.კიდურების რამხელაა პაციენტი ასაკით? სქესი ? წონა ?

თავისთავად სტატინის გვერდითი ეფექტიც არის კუნთების ანთებითი ცვლილებები მაგრამ სტატინის გარეშე განვითარებული ტკივილი ძალიან საყურადღებოა .
კრიატინკინაზა და წვივევის ტკივილი კუნთოვანი იშემიის მანიშნებელია მითუმეტეს თუ ქოლესტერინი მაღალიაქ ათეროსკლეროზული ცვლილებები ექნება მე გირჩევთ დოპლერი გაუკეთოთ ქვემო კიდურების და სისხლძარღვოვანი პრობლემა გამოვრიცხოთ პირველ რიგში

კარდიომაგნილს ან რაიმე ანტიაგრეგანტს იღებს ?

Posted by: pvm 13 Jun 2016, 18:14
ვენების პრობლემა არ აქვს არანაირად, იყო შარშან 2 ანგიოლოგთან. პაციენტი 64 წლის (მდედრობითი), აქვს ჭარბი წონა და ეხლახან აღმოაჩნდა დიაბეტი-ტიპი2. 2 კვირაა არის იანუმეტზე და შაქარი აქვს უზმოზე 90.

რაც შეეხება გულმკერდის კატეს-- ფილტვის რამე პრობლემის გამოსარიცხად დანიშნა ექიმმა. ერთმა ექიმმა დაუდგინა პნევმო ქლამიდია ( ანტისხეულები ქონდა ცოტა მომატებული, გულმკერდის რენტგენში ყველაფერი კარგად იყო). ქონდა საშინელი მოხრჩობის შეგრძნება, ხველა -ნახველით (ერთხელ ხველის დროს გონებაც დაკარგა და წაიქცა), დაუდგინდა ასევე ლარინგიტი, ფარინგიტი- ქრონიკული (ადრეც ქონდა). ექიმი კატეს ითხოვს ფილტვის ფიბროზის ან რამე უარესის გამოსარიცხად. პაციენტი არ არის მწეველი, მაგრამ სამწუხაროდ პასიური მწეველია.

მეორე ექიმი კი არ თვლის, რომ ეს არის პნევმო ქლამიდია, მისი აზრით ეს ოტოლარინგოლოგის პრობლემაა მხოლოდ. მაგრამ ამ ექიმის კონსტაციის დროს კრეატინკინაზა და ღვიძლის ფუნქციები არ იყო შემოწმებული.

პ.ს. ასაფენს იღებს მგონი ეხლა

Posted by: Faniack 13 Jun 2016, 18:36
pvm
ეგ დიაბეტიც შეიძლება სტატინის მიღების შემდგომ განვითარებულიყო ...

ჰქონდა და აღარ აქვს? თუ ახლაც აქვს მოხრჩობის განცდა? აქ მეორე ექიმს ვეთანხმები , ქლამიდია პნევმონიის ერთერთი გამომწვევი მიზეზია და რენტგენოლოგიურად ან მოსმენით მაინც იძლევა რაიმე სურათს . სისხლის საერთოში რა ხდება?
მოხრჩობის განცდა ზემო სასუნთქ გზებს საკმაოდ ხშირად ახასიათებს .


შარდვა როგორ აქვს?

კრეატინკინაზას და ღვიძლის ფუნქციებს ამშემთხვევაში მეორეხარისხოვანი მნიშვნელობა აქვს


გულზე რახდება უკმარისობა აქვს?


QUOTE
ასაფენს იღებს მგონი ეხლა

up.gif

Posted by: pvm 13 Jun 2016, 19:47
რამდენიმე დღეა ცოტას ისევ დაიწყო ხველა. ასევე გაკეთებული აქვს ხორხის ლარინგოსკოპია და სახმო იოგაზე აღმოაჩნდა უმნიშვნელო ზომის ფიბრომა, რომელიც შეიძლება საერთოდ გაქრესო და ამიტომ ისევ უნდა გადაამოწმოს ცოტა ხანში და თუ არ გაქრა, მაშინ მოიშორებს რათქმაუნდა. ონკოლოგიურში გაიკეთა ლარინგოსკოპია და იქ უთხრეს, რომ ამ უმნიშვნელო ზომის ფიბრომას პაციენტის სასუნთქ გზებთან დაკავშირებულ პრობლემებთან არანაირი კავშირი არ აქვს, მკურნალი ექიმები კი თვლიან, რომ გარკვეულწილად ფიბრომაც ცუდად მოქმედებს.

ასევე პაციენტს აქვს გასტრიტი, გასტროეზოფაგური რეფლუქსური დაავადება ( გასტროსკოპია გაიკეთა) და ჩაიტარა მკურნალობის კურსი, თუმცა ნექსიუმს მაინც იღებს.

რაც შეეხება გულის პრობლემას - აქვს გულის იშემიური დაავადება, არტერიული ჰიპერტენზია ( წამლების ფონზე მოწესრიგებული), უკმარისობა არა! დატვირთვის ტესტის და ქოლესტერინის მიხედვით ერთი კარდიოლოგის აზრია აუცილებლად გაიკეთოს კორონაროგრაფია, მეორე კარდიოლოგი კი თვლის, ვინაიდან გაუარესება არ არის ( 2 ჯერ ჩაუტარდა ბოლო 6 თვის მანძილზე დატვირთვის ტესტი) ჯერჯერობით არ არის საჭირო გაკეთება.

სისხლის საერთო ანალიზში კი ცოტა ცვლილებებია: ჰემოგლობინი 160, ერითროციტი 5,49, ლეიკოციტი 9,08.

პაციენტს ჩაუტარდა კომპიუტერული სპიროგრაფია და ასთმა არ აღმოაჩნდა. ექიმის აზრით, კატეს მიხედვით თუ არ დადასტურდა ჰიპოქსიური ერითროციტოზი, მაშინ უნდა წავიდეს კვლევა კიდევ სხვა მიმართულებით...

და ბოლოს - შარდის ანალიზში ქონდა ლეიკოციტები ( თუ არ ვცდები!) და გაიკეთა შარდის ბაქტერიოლოგიური ანალიზი. აღმოჩნდა ენტეროკოკი 10 000მიკრობი/მლ, დიაგნოზი: შარდის გზების ინფექცია დაუზუსტებელი ლოკალიზიაციის (მოშარდვის დროს ცოტა წვის შეგრძნება ქონდა). სანამ კვლევები არ დამთავრდება, ამასთან დაკავშირებით მკურნალობას არ უნიშნავენ ( არ არის მნიშვნელოვანი ამ სტადიაზეო).

Posted by: anianiheart 13 Jun 2016, 22:04
მოგესალმებით. გთხოვთ დამითმოთ 5 წთ და შემივსოთ კითხვარი: "ჰოსპიტალური სერვისების მიღების გამოცდილების მქონე მოსახლეობის შეხედულებების და დამოკიდებულების შესწავლა სამედიცინო სერვისების ხარისხთან დაკავშირებით". მადლობას გიხდით დახმარებისთვის!

კითხვარის შესავსებათ მიყევით ლინკს:
https://www.surveymonkey.com/r/LWBCN2R

Posted by: Faniack 14 Jun 2016, 01:37
pvm
ამწუთას კონკრეტულად რა აწუხებს ? ხველა და ტკივილი ფეხებში?


Posted by: barcabarcabarca 14 Jun 2016, 14:29
მოკლედ ხელი გამეჭრა რამდენიმე დღის წინ, დავეჯახე ჯიხურს biggrin.gif
ხოდა არაფერი რა ნაკაწრი მქონდა. ახლა ეს ნაკაწრი შემიშუპდა და მტკივა.
ხოდა, თან დრო და დრო მარჯვენა ყური, თავის მარჯვენა მხარე, მარჯვენა მხარი წამომტკივდება ხოლმე.
ხოდა, ახლა გაუსაძლისი ტკივილები კი არ მაქვს. მარა ნიტოა რა. მარჯვენა ხელი რო მაქვს მაწუხებს biggrin.gif
ნუ იოდი და ა.შ. არაფერი მიხმარია.
ხოდა, შეიძლება რამე კავშირი ჰქონდეს თუ ვიგონებ და ვფანტაზიორობ? : )

Posted by: Faniack 14 Jun 2016, 14:49
barcabarcabarca იოდი წაისვი ჭრილობაზე , ყბაც ხოარ გტკივა ? 48 საათი თუარ გასულა მიდი სადმე მიმღებში და აცრას გაგიკეთებენ

Posted by: barcabarcabarca 14 Jun 2016, 15:00
Faniack
არა, ყბა არა.
სამი დღე გავიდა, მეტიც მგონი.

Posted by: pvm 14 Jun 2016, 16:29
კუნთების ტკივილი განსაკუთრებით ქვედა კიდურებში და აქვს შეგრძნება თითქოს ჭიანჭველების დადიან. ხველა- მშრალი, იშვიათად. ერთ ფეხში, კერძოდ კოჭთან აქვს წანაზარდი და ასევე დეზიც.

პრობლემა არის კრეატინიკინიზის მომატება მხოლოდ!!!
* * *
სპირიმოეტრია გაიკეთა- ასთმა არ აქვს;

გულმკერდის კატე- ფილტვებში არანაირი პრობლემა;

კოჭ წვივის სახსრების ეხოსკოპია, ასევე რენტგენოგრაფია- აღმოჩნდა ოსტეოფიტი ( პატარა კოჭზე) და ქუსლზე დეზი ( დეზი ადრეც იცოდა და ფიზიო პროცედურები ჩაიტარა - სამწუხაროდ უშედეგოდ).

Posted by: pvm 18 Jun 2016, 00:33
QUOTE (pvm @ 14 Jun 2016, 16:29 )
კუნთების ტკივილი განსაკუთრებით ქვედა კიდურებში და აქვს შეგრძნება თითქოს ჭიანჭველების დადიან. ხველა- მშრალი, იშვიათად. ერთ ფეხში, კერძოდ კოჭთან აქვს წანაზარდი და ასევე დეზიც.

პრობლემა არის კრეატინიკინიზის მომატება მხოლოდ!!!
* * *
სპირიმოეტრია გაიკეთა- ასთმა არ აქვს;

გულმკერდის კატე- ფილტვებში არანაირი პრობლემა;

კოჭ წვივის სახსრების ეხოსკოპია, ასევე რენტგენოგრაფია- აღმოჩნდა ოსტეოფიტი ( პატარა კოჭზე) და ქუსლზე დეზი ( დეზი ადრეც იცოდა და ფიზიო პროცედურები ჩაიტარა - სამწუხაროდ უშედეგოდ).

ალფაფეტოპროტეინი და რკინა ნორმაშია, მაგრამ ფერიტინი არის მაღალი, დაახლოებით 250 მდე.
კვლევები გრძელდება...

* * *
მიპასუხეთ რაააააააა


მეგონა მიპასუხებდით ისე tongue.gif

Posted by: inamura7 18 Jun 2016, 13:52
ჰემოსკანირების შედეგები: აგრეგაცია, ფინი-7ც, ორთოფოსფორმჟავას კრისტალი, მსუბუქი ლიპიდი, ანთება, ბაქტერიები, თრომბოაგრეგაცია, ფიბრინოგენი, ერითროციტების ლანდები.

რა მჭირს იქნებ მითხრათ და ვის მივმართო?

Posted by: Faniack 18 Jun 2016, 16:34
pvm
რაგითხრა აბა ? კრეატინკინაზა სავარაუდოდ კუნთოვანი დისტროფიის გამოა ყველა ჩივილს ხსნის რაც ფეხებზე აქვს

ფერიტინი დეპონირებული რკინაა ორგანიზმში , ღვიძლის ფუნქციების მომატების გამოა ეგეც მომატებული


გყავთ მკურნალი ექიმი რომელიც იცნობს პაციენტის ანამნეზს ბოლომდე და გარკვეული მკურნალობის გეგმა აქვს ჯობია მიყვეთ მის მკურნალობას ასე ინტერნეტით ცოტა რთულია ამხელა ანამნეზის პაციენტის კონსულტირება

თქვენადგილას გამოვრიცხავდი გულისმხრივ პრობლემებს , ფარისებს ( ამანაც შეიძლება გამოიწვიოს კრეატინკინაზას მატება ) ღვიძლს გამოვიკვლევდი და კუნთოვანი დისტროფიას , სავარაუდოდ ყველა ზემოთჩამოთვლილმა თავისი წილი შეიტანა პაციენტის მდგომარეობაში მარა მაინც გასარკვევია წამყვანი რომელი პათოლოგიაა




inamura7
რაგაწუხებს კონრეკტულად და რატო გაიკეთე ჰემოსკანირება

Posted by: inamura7 18 Jun 2016, 18:21
Faniack
სისხლის მიმოქცევის პრობლემა მაქვს, პერიფერიული სისხლძარღვების სპაზმი (რეინო)
ხანდახან სივრცეში ორიენტაციას ვკარგავ

Posted by: konkretula 19 Jun 2016, 02:01
მადის მოსამატებლად რა წამალს მირჩევთ?
ანუ რომელია ყველაზე ეფექტური.

Posted by: Faniack 19 Jun 2016, 02:08
inamura7
და ეს ცოცხალი სისხლის ანალიზი ვინ გირჩიათ? რისი მომცემია თქვენს შემთხვევაში?


რეინო დადასტურებული გაქვთ? მკურნალობთ რამით?


konkretula
პოლივიტამინები ნებისმიერი , მიზეზი რაარის მადის დაქვეითების ?

Posted by: inamura7 19 Jun 2016, 11:43
Faniack

გაკეთებული მაქვს კოაგულოგრამა, კისრის მალების რენტგენი, ზემო კიდურების მაგისტრალური არტერიები დოპლეროგრაფია, და ლაზერულ-დოპლერული ფლუომეტრია.
დიაგნოზი: რეინოს ფენომენი, ვაზოსპატიური ფორმა. განზიდვის სინჯები ორივე მხარეს დადებითია, რენტგენოლოგიურად დამატებითი ნეკნი არ ფიქსირდება. ფიზიოლოგიური ლორდოზი გასწორებულია. ლაზერული ფლურომეტრიით გამოხატულია მცირე კალიბრის სისხლძარღვთა მდგრადი სპაზმი, სისხლძარღვთა ელასტიურობა დაქვეითებულია. სისხლში გამოხატულია ნორმოკოაგულემია.
ვიკეთებდი კლექსანს, აქტოვეგინს. ვსვამდი: დოქსი-ჰემს, აქტოვეგინს, კარდიო მაგნილს. მკურნალობის კურსი ჩავიტარე ზამთარში, მაგრამ შედეგი არ მქონდა.

ახლობელმა მირჩია ცოცხალი სისხლის ანალიზის გაკეთება და იქ მითხრეს, რომ სისხლში პარაზიტებია , რის გამოც შედედებულია სისხლი და სისხლი უნდა გაიწმინდოს. ასევე მითხრეს, რომ ტრადიციული მედიცინა აქ უძლურია smile.gif))

მადლობელი ვიქნები, რამეს თუ მირჩევთ smile.gif

Posted by: konkretula 19 Jun 2016, 13:15
Faniack
QUOTE
მიზეზი რაარის მადის დაქვეითების ?

ეგ რო ვიცოდე...
მთელი ცხოვრებაა უმადობა მაწუხბს, რამდენჯერაც ექიმთან მივედი სულ სხვა რამეები გამომიძებნა და მადის მომატებამდე ვერ მივედით.
ჯერ გულის ოპერაცია გავიკეთე, მერე გლანდების, მერე ჰორმონებზე ვიმკურნალე. დიეტოლოგიც მყავდა ))) მოკლედ მადას არაფერი ეშველა. უკვე აღარ ვიცი რა ექიმთან მივიდე და სიმართლე ვთქვა აღარც მჯერა ამ ექიმების. გამომიჩენენ კიდევ რამეს და დამაწყებინებენ იმაზე მკურნალობას.


Posted by: Takomakharadze 20 Jun 2016, 16:09
მოგესალმებით ძვირფასო მეგობრები,
უკვე რამოდენიმე დღეა, რაც ბატონ გია მაისაშვილის თანადგომასთან დაკავშირებით გავრცელდა ინფორმაცია და საბანკო ანგარიშის მონაცემები, ამ დღეების მანძილზე ვიგრძენით თქვენგან როგორც მორალური ასევე მატერიალური თანადგომა, რაზეც გიხდით უღრმეს მადლობას.
ვისაც გიყვართ და პატივს სცემთ ბატონ გიას და ვისაც გაგაჩნიათ ამისათვის მატერიალური შესაძლებლობა, ძალიან გთხოვთ შეეწიოთ მის ჯანმრთელობას. დავეხმაროთ სიმსივნის დამარცხებაში. მას სასწრაფოდ ესაჭიროება უცხოეთში გამგზავრება და მკურნალობა. გია მაისაშვილს წელიწადზე მეტია სიმსივნეს ებრძვის.
შემოდით გვერდძე და გთხოვთ არ დაიზაროთ გაზიარება.

https://www.facebook.com/helpgiamaisashvili/

გია მაისაშვილისადმი თანადგომისათვის თანხების ჩარიცხვა შესაძლებელია ყველა მოქმედი ბანკის მეშვეობით.
მიმღები ბანკის კოდი: KSBGGE22
მიმღები ბანკის დასახელება: ტერაბანკი
მიმღების ანგარიში: GE21 KS00 3601 0130 7995 01 GEL, USD.
მიმღების დასახელება: თამარ კაპანაძე - მეუღლე
____________________________________________________
თი ბი სი ბანკის TBC PAY - ს მომხმარებლებს საშუალება გეძლევათ თანხები ჩარიცხოთ თი ბი სი ბანკის სწრაფი გადახდის აპარატებიდან -TBC PAY
შემდეგი მონაცემების მეშვეობით.
პირადი ნომერი: 01005000006
დაბადების თარიღი: 12.10.1978
მიმღები: თამარ კაპანაძე
ანგარიში: მიმდინარე GEL
თი ბი სი ბანკიდან ან ბანკინგიდან თანხის გადარიცხვის შემთხვევაში:
მიმღები ბანკის კოდი: TBCBGE22
მიმღები ბანკის დასახელება: თი ბი სი ბანკი
მიმღების ანგარიში: GE22TB7961136010300010 GEL.
მიმღების დასახელება: თამარ კაპანაძე - მეუღლე
უცხოეთიდან თანხების ჩარიცხვა შესაძლებელია ქვემოთ მოცემული საბანკო მონაცემების მეშვეობით.
გთხოვთ არ დაიზაროთ და გააზიაროთ ეს ინფორმაცია,
წინასწარ გიხდით დიდ მადლობას თანადგომისათვის.
კონტაქტი +995 577 577717 / +995 595 008885

უცხოეთიდან თანხების ჩარიცხვა შესაძლებელია ქვემოთ მოცემული საბანკო მონაცემების მეშვეობით, კიდევ ერთხელ მადლობას გიხდით თანადგომისათვის.

Posted by: Librocubicularist 21 Jun 2016, 14:31
გამარჯობა

არ ვიცი, სად ვიკითხო და დამაკვალიანეთ თუ შეიძლება . სანამ მივალ ექიმთან, ცოტა მაინც რო გავერკვიო.

სამი დღეა რაც მუცლის მარცხენა ნაწილში მაქვს ისეთი შეგრძნება თითქოს რაღაც ფეთქავს შიგნით, რომ ვაკვირდები თვითონ მუცლის ზედაპირიც მოძრაობს ამ დროს. ხანდახან ბუშტების გახეთქვის მსგავსი შეგრძებაა ინტერნეტ სივრცეში რა ინფორმაციაც მოვიძიე, ძირითადად ორსულობაზეა, რაც გამორიცხულია ჩემს შემთხვევაში. ზოგადად მაქვს გასტროენტეროლოგიური პრობლემები (კოლიტი, გასტრიტი) და შეიძლება დაკავშირებული იყოს?

Please help user.gif

მადლობა წინასწარ

Posted by: gubaz_merve 21 Jun 2016, 16:34
გამოცდა იწყება...smile.gifსისხლის ფორმულა ?

Posted by: Breaky 27 Jun 2016, 16:02
ჰემოგლობინის ნორმა 12.8 - 16.8 რომ წერია, ეგ ძველი "მერკებით" რამდენია? რა ფორმულით გადადის?

Posted by: Solveig 28 Jun 2016, 10:07
inamura7
აშკარა ბოლია ეგ ცოცხალი სისხლის ანალიზი და არც რეინოსთანაა კავშირში (მე ვერ ვხედავ).

პარაზიტები და მათგან გაწმენდა საერთოდ სასაცილოა.

აქტოვეგინზეც ბევრ ამბობს, რომ ბოლია.

Breaky
ეგ ერთეული გამოიყენება უკვე ბევრი წელია. რაღა ძველს გულისხმობ?

აგერ ვნახე რაღაც გამომთვლელი და ალბათ, გამოგადგება.
* * *
http://www.scymed.com/en/smnxpf/pfxdq210_c.htm

Posted by: inamura7 28 Jun 2016, 10:53
Solveig

ჰო. პარაზიტებთან დაკავშირებით მეც მასე ვფიქრობ. smile.gif)) თან მითხრეს, ძაღლები რომ გყავს, იმიტომ დაგემართა მასეო biggrin.gif

რა ვქნა, ვერავინ მირჩევს? რა დავლიო, ვის მივმართო?


Posted by: Breaky 28 Jun 2016, 11:08
Solveig
არაა მხოლოდ ეგ, სხვებიც არის ხოლმე, მაგალითად 120 რომ ქვედა ზღვარია. ხოდა ვიბნევი მერე. ძველ მერკებს ვადარებ ხოლმე ყოველთვის, ანუ 68 როა ქვედა ზღვარი.

Posted by: Solveig 28 Jun 2016, 19:05
Breaky
თანამედროვე ლაბებში გამოიყენება გრამი ლიტრზე, გრამი დეცილიტრზე ან მილიმოლი ლიტრზე.

120 რომაა ქვედა ზღვარი, ეგ არის გრამი ლიტრზე ერთეულით. 1 ლიტრში თუ გაქვს 120 გრამი, 1 დეცილიტრში იქნება 12 (10-ჯერ ნაკლები) გრამი.

შენ მგონი პროცენტებს და სალის ჰემომეტრის კოეფიციენტებს გულისხმობ, რომელსაც სსრკ-ს დროს იყენებდნენ.

აქ წერია, როგორც ითვლება

https://otvet.mail.ru/question/95767599

აღარ გამიგია, რომ მაგას ვინმე იყენებდეს ახლა. ვერ ვხვდები დაბნეულობის მიზეზს smile.gif რამე სპეციფიკური მიზეზია, რომ მაინცდამანც ძველი მეთოდი გჭირდება?


Posted by: Breaky 28 Jun 2016, 22:22
Solveig
მადლობა :*

გრამი ლიტრზეს მივეჩვიე, სადაც ვიღებ ხოლმე, იქ სულ მასე წერია და ქვედა ზღვრად წერია ხოლმე 120.

ახლა დამჭირდა ბავშვისთვის სახლში გამოძახება და პასუხი მოვიდა დეცილიტრებში, ოღონდ ქვედა ზღვრად წერია 12.8 ხოდა ამიტომ დავიბენი. 12 რომ ეწეროს, გადავიყვანდი მეთვითონ მარტივად, ანუ მივხვდებოდი.

ბავშვს აქვს 12.3 და მითითებულია, რომ დაბალიაო, თუმცა ინტერნეტი რომ გადავაქოთე, ამის ასაკზე ნორმის ქვედა ზღვარი 12 წერია ყველგან. ხოდა. იფიქრე, რომ რამე სხვა სისტემა ხომ არაათქო.

Posted by: Solveig 29 Jun 2016, 01:11
Breaky

ბავშვის ნორმები ცოტა განსხვავებულია ხოლმე. 12 ქალებში არის ქვედა ზღვარი.

http://www.laborlexikon.de/Lexikon/Tabellen/17-Blutbild_Kinder.htm

აგერ წერია, რომ 2 დან 6 წლამდე პატარებისათვის 10,8-14,3გრამი/დეცილიტრია ნორმა. 7-დან 13 წლამდე-11,3-14,9. 13 წლის ზემოთ უკვე იგივეა, რაც ზრდასრულებში.

მალე გამოკეთებულიყოს ბავშვი smile.gif

Posted by: Breaky 29 Jun 2016, 10:08
Solveig
ხოდა ჯდება მაგ ნორმებში ლაბის პასუხი. თუმცა მათი ნორმები 12.8-დან იწყება biggrin.gif
ისე გავუკეთე, საკონტროლოდ, ხოდა დავიკიდე. კიდევ როცა გვაკონტროლებ, მერე ვნახოთ რა იქნება

Posted by: keandra 30 Jun 2016, 10:47
გულ-მკერში მჭვალავს სირბილის დროს, ზოგჯერ საკმაოდ ძლიერად. რა ტიპის გამოკვლევა უნდა გავიკეთო?

Posted by: keandra 4 Jul 2016, 09:41
ორგანიზმის სრული გამოკვლევა თუ გაგიკეთებიათ და სად მაინტერესებს კიდევ.

Posted by: B l a c k J a c k 5 Jul 2016, 21:39
თირკმლების პრობლემაზე იმედია სწორად ვპოსტავ. რას ნიშნავს: აღსანიშნავია პირამიდული სისტემის კარგი ვიზუალიზაცია და ექოგენობის დაქვეითება? თირკმელზე არის საუბარი sad.gif

Posted by: annaninio 7 Jul 2016, 10:15
57 წლის ქალს მოულოდნელად დაეწყო რაღაც ფსიქოლოგიური პრობლემები,ჯერ სისუსტეს უჩიოდა,შემდეგ ორიენტაციის დარღვევას ,მახსოვრობის დაქვეითებას, გაღიზიანებას.წაიყვანეს სფიქოლოგთან,შემდეგ ნევროლოგთან, გადაუღეს თავზე და არაფერი საყურადღებო არ აღმოჩნდა,ზოგმა ნაადრევი ალცჰეიმერის დაავადება დაუდგინა, ზოგმა ძლიერი დეპრესია, გამოწერილი მკურნალობა საერთოდ არ მოქმედებს.ამ ერთი თვის განმავლობაში მდგომარეობა ისე დაუმძიმდა რომ თავისსით ვეღარც ჭამს, ვეღარც დადის,აქვს თითების კრუნჩხვა,გეგონებე ეხლა შეტევა უნდა მოუვიდესო.ხანდახან ემართება ისტერიული შეტევები.ხშირად იმეორებს თავის სახელს.ეხლა მხოლოდ დამამშვიდებლებს იღებს.რა შეიძლება იყოს?თუ ფსიქიქტრიული პაციენტია, ეს კრუნჩხვა და ორიენტაციის დარღვევა რაღა შუაშია.თითქოს ფეხი საით გადადგას არ იცის.საწოლზე გვერდით თუ არავინ უწევს ვარდება.ოჯახი სასოწარკვეთილია,აღარ იცის ვის მიმართოს...

Posted by: pvm 10 Jul 2016, 16:12
46 წლის პაციენტმა გაიკეთა ვიტამინ D3 25(OH) ანალიზი და აღმოჩნდა ძალიან დაბალი 5,85.
რამდენად საშიშია ასეთი დაბალი მაჩვენებელი იქნებ იცით ვინმემ?!

Posted by: S7777777 10 Jul 2016, 17:07
Gamarjobat iqneb vinmem icodet.18 tvis go go as sxeulze aqvs yavisferi tandayolili laqebi.xom ar yofila tqvens irgvliv msgavsi shemtxveva da xom ar icit raime?

Posted by: inamura7 15 Jul 2016, 11:44

სისხლის მიმოქცევის პრობლემა მაქვს, პერიფერიული სისხლძარღვების სპაზმი (რეინო)

გაკეთებული მაქვს კოაგულოგრამა, კისრის მალების რენტგენი, ზემო კიდურების მაგისტრალური არტერიები დოპლეროგრაფია, და ლაზერულ-დოპლერული ფლუომეტრია.

დიაგნოზი: რეინოს ფენომენი, ვაზოსპატიური ფორმა. განზიდვის სინჯები ორივე მხარეს დადებითია, რენტგენოლოგიურად დამატებითი ნეკნი არ ფიქსირდება. ფიზიოლოგიური ლორდოზი გასწორებულია. ლაზერული ფლურომეტრიით გამოხატულია მცირე კალიბრის სისხლძარღვთა მდგრადი სპაზმი, სისხლძარღვთა ელასტიურობა დაქვეითებულია. სისხლში გამოხატულია ნორმოკოაგულემია.

ვიკეთებდი კლექსანს, აქტოვეგინს. ვსვამდი: დოქსი-ჰემს, აქტოვეგინს, კარდიო მაგნილს. მკურნალობის კურსი ჩავიტარე ზამთარში, მაგრამ შედეგი არ მქონდა.

ახლობელმა მირჩია ცოცხალი სისხლის ანალიზის გაკეთება და იქ მითხრეს, რომ სისხლში პარაზიტებია , რის გამოც შედედებულია სისხლი და სისხლი უნდა გაიწმინდოს. ასევე მითხრეს, რომ ტრადიციული მედიცინა აქ უძლურია ))
ჰემოსკანირების შედეგები: აგრეგაცია, ფინი-7ც, ორთოფოსფორმჟავას კრისტალი, მსუბუქი ლიპიდი, ანთება, ბაქტერიები, თრომბოაგრეგაცია, ფიბრინოგენი, ერითროციტების ლანდები.

მადლობელი ვიქნები, რამეს თუ მირჩევთ

Posted by: მედბიჭუნა 29 Jul 2016, 21:29
სენფორდის ანტიმიკრობული თერაპიის სახელმძღვანელოს სად შეიძლება ვიყიდო ვიცი რომ ქართულად არის ნათარგმნი განათლების სანმინისტრომ თარგმნა ... დაატრენინგნა ზოგნი როგორ გამოიყენონ და მე კიდე ვეძებ .. სად შეიძლება ვიყიდო?

Posted by: avelum 2 Aug 2016, 10:32
ფეხის ნეკი მაქვს შეშუპებული და გაწითლებული. მიზეზი: უცნობი . რა აღარ ვცადე : ფურაცილინი, რივანოლი, მაინც არ მშველის. დამეხმარეთ

Posted by: arut 2 Aug 2016, 12:30
გამარჯობა,ფიზიკური გადატვირთვისგან ფეხზე კანჭის ტყუპი კუნთის ნაწილის ბოჩკო ჩაწყვეტილია და ისახება გემატომა,,,ნემსები ტაბლეტები და მაზი დამინიშნეს დავაკვირდეთ თუ გაიწოვსო და გაგიგიათ ასეთი რამე?,,,,8 ნემსი გავიკეთე,,,თითქოს უკეთ ვარ მარა ბოლომდე მიშველის ვითომ?

Posted by: torckiko 9 Aug 2016, 17:31
გამარჯობა, ეს ანალიზები რამდენად არის ლეიკემიის მაჩვენებელი? გემოგლობინი- 114%, ერითროციტები-3,82, თრომბოციტები-232,8, ჰემატოკრიტი 37%, ლეიკოციტები-4,03, ეოზინოფილები-- 1, ლიმფოციტები-46, მონოციტები-3, როე-18. ჰემატოლოგთან მაგზავნიან და მაგიტო მაინტერესებს.

Posted by: likuna27 10 Aug 2016, 21:42
მოგესალმებით. დედას აქვს მაღალი ედს 60. სვალმს კარდიომაგნილს, ატორისს, კო ამლესას. ჰემოგლობინი აქვს დაბალი -9,3. ასევე, პერიოდულად აწუხებს სახსრები (რევმატიულია) და აქვს დიაბეტი მე-2 ტიპის. შარდის ანალიზში აღმოაჩნდა ოქსალატები დიდი რაოდენობით. ძალიან მეშინია ამხელა როე რომ აქვს, რამე ონკოლოგიური არ იყოს. ხომ იცით, პირველი ამას ფიქრობ. ვინმეს გაქვთ ამ თემაზე გამოცდილება? მადლობა წინასწარ

Posted by: Luka211 11 Aug 2016, 11:13
მოკლედ 1 კვირაა დამეწყო თითქოს კეფიდან მქაჩავს რაღაც და შუბლზე გადმოდის. თითქოს წნევა მაქო. თვალებზეც მაწვებაო თითქოს ვერ გავიგე რის ექიმთან უნდა მივიდე. წინა კვირას კომიუტერთან დამემართა რაღაც სასტიკად არაფრის თავი აღარ მქონდა. თითქოს მეტრომ გაიარა ფეხებქვეშო ეგეთი გრძნობაც მქონდა. დილით რო ვდგებოდი თითქოს თავი დამძიმებული მქონდა. ბოლო დღეებია აღარ მაქ ეგ გრძნობა.
მარჯვენა თვალის წვაც მქონდა და თითქოს მთელი მარჯვენა სახეც მეწვოდა. ნუ კომპიუტერთან ჯდომა მიწევს მთელი დღე user.gif
ხო და იშვიათად კიდე ყურებში უცბად წუილის ხმა გაიჩითება და რამოდენიმე წამში სწორდება ისევ smile.gif

Posted by: blokator 9 Sep 2016, 21:53
ფერიტინი მაქვს მომატებული და ვიტამინი დ ნაკლებობა,(ნორმა <400 და 604მაქვს) დანარჩენი ყველა ანალიზი ნორმაშია.ღვიძლის ფუნქც.თირკმლის ფუნქც და ა.შ. ანალიზები.რა შეიძლება მიზეზი იყოს?არაფერს არ ვუჩივი, ვარ 29 წლის 10> მწეველი .

Posted by: zaza1993 11 Sep 2016, 00:51
ანგინას რა სიმპტომები ახასიათებს ხომ ვერ მეტყვით? smile.gif

Posted by: Mekusha 25 Sep 2016, 23:12
მოგესალმებით, 3 თვის ბავშვისთვის ტანტუმ ვერდეს სპრეი თუ შეიძლება.

Posted by: lila2009 5 Oct 2016, 12:21
შავ დღეში ვარ

თვიური მაქვს ჩვეულებრივ ამიტომ ორსულობა გამოვრიცხე.
მუცელი მეზრდება sad.gif დისკომფორტი და ყრუ ტკივილი.
გაძლიერებული მაქვს მადა, წონაში მოვიმატე ბოლო ორ თვეში.
მაქვს დაბალი სიცხე.

რაზე მქონდეს ეჭვი? ექიმთან მისვლამდე რო შევემზადო sad.gif(((((((((((

Posted by: malaktavus 16 Oct 2016, 11:42
ალლ
კაი უროლოგი მინდა......... ერთი წელი მაქ პრობლემა. და ვერ მივდიოდი . (

ხშირი შარდვა. თურკმელიც მტკივა. sad.gifცოტა


სად მივიდე ჯობია?

Posted by: sandro21 21 Oct 2016, 23:05
. .

Posted by: nakafi 30 Oct 2016, 15:54
მოგესალმებით,
მაქვს კუჭის ტკვილები და ვეძებ კარგ გასტროენტეროლოგს ვის მირეჩევთ ?

Posted by: Naniko1 31 Oct 2016, 13:38
თუ შეიძლება მიპასუხეთ ცეფსუსი რამდენად ძლიერი ანტიბიოტიკია? მაგალითად აზიმაკთან შედარებით

Posted by: niniaa 4 Nov 2016, 16:20
Naniko1
QUOTE
თუ შეიძლება მიპასუხეთ ცეფსუსი რამდენად ძლიერი ანტიბიოტიკია? მაგალითად აზიმაკთან შედარებით


იქიდან გამომდინარე, რომ კონკრეტულ ინფექციას, კონკრეტული ბაქტერიები იწვევენ, აუცილებელია, ზუსტად ის ანტიბიოტიკი მივიღოთ, რომელიც მოცემული ინფექციის გამომწვევ ბაქტერიებს ამარცხებს. ამიტომ ანტიბიოტიკისადმი მგრძნობელობა უნდა იყოს დადგენილი ,რაც სამწუხაროდ არ კეთდება ყოველთვის.
ცეფსუსი არის მესამე თაობის ცეფალოსპორინის ანტიბიოტიკი .მათ გააჩნიათ მაღალი აქტივობა აერობული და ანაერობული გრამუარყოფითი მიკროორგანიზმების მიმართ: ენტერობაქტერიების, გონოკოკების, ჰემოფილური ჩხირების, ზომიერად აერობული და ანაერობული ბაქტერიების მიმართ, აქტიურია ლურჯ-მწვანე დაჩირქების ჩხირების და სტაფილოკოკების მიმართ.
აზიმაკი კი არის მაკროლიდების ჯგუფის აზალიდების ქვეჯგუფის ფართო სპექტრის მოქმედების ანტიბიოტიკი.აზიმაკი აქტიურია გრამდადებითი (სტაფილოკოკი, მათ შორის ბეტა-ლაქტამაზას მაპროდუცირებელი, სტრეპტოკოკი) და გრამუარყოფითი (ენტეროკოკი, ნაწლავისა და ჰემოფილური ჩხირი) ბაქტერიების, მიკოპლაზმის, ლეგიონელას, ბაქტეროიდების, ქლამიდიას და სხვა მიმართ.
* * *
goshte
ენდოკრინოლოგთან არ იყავით კონსულტაციაზე?

Posted by: caucasica 10 Nov 2016, 21:14
სალამი

კამბოჯაში მიწევს წასვლა ამ თვის ბოლოს 10 დღე, ტყეში მომიწევს მუშაობა ----
ერთი ვიცი მალარია საფრთხე და რა პრევენციული ღონისძიებებს მივმარტო?

კამბოჯაში კიდევ რაიმეზე დამჭირდება აცრა?

Posted by: TamunaKr 10 Nov 2016, 23:22
გამარჯობათ. არ ვიცი სად დამეპოსტა და აქ ვარჩიე. ესეიგი მგზავრობის დროს-მანქანაში.მარშუტკაში. . მეწყება საშინელი თავის ტკივილი და გულის რევა. ისე ცუდად ვხდები ზოგჯერ შეგრძნება მაქვს რომ მაჟრიალებს,აი სიცხის დროს რომ იცის. რასაც ქვია ვიტანჯები. მნიშვნელობა არ აქვს პატარა მანძილია თუ დიდი. დრამინა არ მშველის. სხვა რამე საშუალება არსებობს რომ მგზავრობა ჯოჯოხეთად არ მექცეს?ბავშვობიდან მაქვს ეს და ბოლო პერიოდში რაღაც სასტიკად გამიმძაფრდა.შეუძლებელია უკვე მანქანით გადაადგილება . რა ვქნა არ ვიცი.

Posted by: youtubergirl123 16 Nov 2016, 22:49
რატომ ვერ ვპოსტავ მოვაგროვე უკვე ეს პოსტინგი

Posted by: youtubergirl123 16 Nov 2016, 22:49
რატომ ვერ ვპოსტავ მოვაგროვე უკვე ეს პოსტინგი

Posted by: zaza1993 27 Nov 2016, 14:56
ყელისკენ რაღაც წყლულისავით გამიჩნდა, წყლულია თუ რა ჯანდაბაა ვერ გავიგე, მუწუკს ჰგავს :დ იქნებ მირჩიოთ რამე

Posted by: Miria 13 Dec 2016, 00:01
ხალხო, ხომ არ იცით რამ შეიძლება გამოიწვიოს პერიოდულად სიცივის შეგრძნება?
აი, მაგალითად, ჩვეულებრივ, 24-25 გრადუსზე გაცვია თხელი შარვალი, მოკლე სახელოებიანი მაიკა
და ამ ტანსაცმლით კომფორტულად გრძნობ თავს.
მერე, ერთ დღესაც იღვიძებ და გცივა.
ოთახის თერმომეტრს ნახავ - ისევ იგივეს აჩვენებს ან გარეთ გახვალ ამავე ტემპერატურაზე, მაგრამ მაინც გცივა, იყინები.
ჩაიცმევ სქელ შარვალს, სქელ წინდებს, ჯემპრს და ასე ხარ .
და მაინც გცივა . კურტკის ჩაცმა გინდება.

ერთხელ ოთახში 28 გრადუსი იყო და დამეწყო ეს სიცივის შეგრძნება შუადღისით,
ჩავიცვი ნაქსოვი რეიტუზი, შარვალი, ზამთრის ჯემპრი, დუტის კურტკა.
ისევ მციოდა, მაკანკალებდა.
ჩავწექი, დავიფარე 1 საბანი, მცივა.
დავიფარე მე2ე, მცივა.
სულ 3 საბანი მქონდა და 3ვე დავიფარე ბოლო ჯამში.
ასე ვკანკალებდი ზამთრის თანსაცმელში ჩაცმული 3 საბნის ქვეშ ასე 20-30 წუთი.
გავიზომე სხეულის ტემპერატურა , 35,2 მქონდა.
მერე ნელა-ნელა გავთბი და გადავიხადე საბნები სათითაოდ.
მერე ავდექი. ცოტა ხანში დუტის კურტკა გავიხადე.
მაგრამ ღამემდე მეცვა ჯემპრი, რეიტუზი და სპორტული შარვალი.
ღამე მხოლოდ ჯემპრი და სპორტული შარვალი გავიხადე და დავწექი დასაძინებლად რეიტუზით და გრძელ სახელოებიანი მაიკით. და დავიფარე 2 საბანი.

აი, ასე მემაბრთება პერიოდულად.

შეიძლება ეს გაგრძელდეს 1 საათამდე (თუ იმის საშუალება მაქვს იმ მომენტში რომ გავთბე მაშინვე როგორმე).
ან გრძელდება რამოდენიმე საათი.
ან რამოდენიმე დღე.

ვერაფერს ვუკავშირებ.

ეს დამეწყო 2014 წელს. 34 წლის ვიყავი.
.

Posted by: ika_mqvia 20 Dec 2016, 23:12
გამარჯობა
ვერსად ვნახე და აქ ვიკითხავ:

ბებოსთვის ხადულს(ე.წ. ჭოჭინა) ვეძებ ბორბლების გარეშეს,ეგებ ან ვინმეს გქონდეთ რო მათხოვოთ ან მიმასწავლოთ იაფად სად ვიყიდი sad.gif

Posted by: Knjazna 8 Jan 2017, 17:45
ძალიან ბევრს ვეწევი, აქედან დავიწყებ. ვფიქრობ რომ მტკივა მარცხენა ფილტვი. გარედან, ზუგზე ვგრძნობ ტკვილის. ასევე, მიჭირს სუნთქვა და სულ თავბრუ მეხვევა. ხშირად მტკივა მარცხენა მხარი და იღლია. მინდა სასწრაფოდ შევიმოწმო ფილტვები რადგან დავპანიკდი ამ სიმპტომების გამო. ვერ გავარკვიე სად აჯობებს შემოწმებაზე წასვლა. გთხოვთ დამეხმარეთ, თუ იცით სად არის კარგი ფილტვების ექიმი. დიდი მადლობა

Posted by: Kaxab 9 Jan 2017, 21:09
ახლობელს უკბინა ძაღლმა, სამი აცრა გაიკეთა, მაგრამ ცოფზე არა. ნაკბენი ადგილი ფეხზე, აქვს გაწითლებული და შესიებული.
პირველ აცრაზე დახედა მარტო და არც არაფერი დაუმუშავებია. დანარჩენი აცრები გაუკეთა ბეჭში და მკლავზე. ესეც რაღაცატომ არ აჩვენებდა ნაკბენს, მხოლოდ ერთხელ უთხრა მტკივაო და იმ ექიმმა ნუ ხარ ფაქიზოო და ესეც შეეშვა.

რამდენად საშიში სიტუაციაა?

Posted by: marrriam 6 Feb 2017, 03:32
ისეთი შეგრძნება მაქვს თითქოს ყელში ჰაერი მაქვს დაგროვებული სააშინლად მაწუხებს მგონია ვერ ვისუნთქებ და დავიხრჩობი facepalm.gif წუხელ გამეღვიძა მართლა ვიხრჩობოდი drug.gif ეს ჰაერი სუნთქვის საშუალებას არ მაძლევდა.
ამაღამ თუ ისევ დამემართება წავალ ექიმთან
რომ დავწერე ცოტა დავმშვიდდი facepalm.gif
თუ რამე იცით როგორ და რატომ გამაგებინეთ

Posted by: zurinio111 14 Feb 2017, 23:41
ნახველში სისხლი შეიძლება ისე მოხვდეს რომ ტუბთან არ იყოს დაკავშირებული?2 დღეა მახველებს და სისხლი მოყვება,ბევრი არა,სხვას არაფერს არ ვუჩივი,არც სისუსტეს ვგრძნობ,არც უმადობას ვუჩივი,სიცხეებიც არ მაქვს,არც ოფლიანობა,გთხოვთ მიპასუხოთ

Posted by: lexanna 15 Feb 2017, 13:27
აქ მხოლოდ კიტხვებია და პასუხები არა, აქვს აზრი კიტხვის დასმას? ალბატ არა

Posted by: Sofiasoffa 1 Mar 2017, 07:04
გამარჯობა, მეგობარს აქვს შეძენილი ეპილეფსია. იღებს ფინლეფსინს და ეპიქსს. 1 თვეა მარცხენა ფეხი გაუშეშდა, ვერ დადის, აქ შეტევითი ტკივილი, წელის მიდამოდან მთელი ფეხის სიგრძეზე, ამ დროს ფეხი ეჭიმება და ვერ ხრის, მენჯთან ამოებერება ძარღვი, ვერ ბრუნდება. ხო ვერ მეტყვით რა შეიძლება იყოს ან სად გავიარო კონსულტაცია. წინასწარ გიხდით მადლობას!

Posted by: Scoorp 7 Mar 2017, 13:57
მემგონი აქ არავინ იხედება მაგრამ ვიკითხავ მაინც
პარასკევიდან მოყოლებული დაბალი სიცხეები მაქვს, 37,37-2
არაფერი მტკივა,არც მახველებს
სურდო მაქვს ძალიან მსუბუქად
პარასკევი,შაბათი-კვირა სახლში ვიწექი
რამდენჯერმე დავლიე ჰერვექსის (თუ სწორად მახსოვს) ჩაი
გუშინ გამოვედი სამსახურში და შუადღით ნიმესილი დავლიე,სახლში მისულმა გავიზომე სიცხე და 37 მქონდა მაინც
გამიკვირდა ნიმესილის ფონზე რომ არ დაიწია

რამე ვირუსია მსგავსი სიმპტომებით?
რაღაცა ანთებითი პროცესის ნიშნებია,მაგრამ ესე ჰაერზე რა გავიგო sleep.gif
სისხლის საერთოს ავიღებ და კიდე რა შეიძლება გაკეთდეს?

Posted by: Jimi-Gamcxelebeli 31 Mar 2017, 02:50
Scoorp
ეგეც არ გჭირდება

Posted by: Mopirmkrtalo 31 Mar 2017, 03:41
Scoorp
მსგავსი რამ ახლობელს აქვს, ოღონდ უკვე წლებია.
ჩიყვის ბრალიაო როგორც ვიცი, გაესინჯე ექიმს მაინც

Posted by: DON GIO 8 May 2017, 12:41
ერთმა ინტერნეტ მეგობარმა შემცა და შემიკიკინა გინდა თუ არა ეს წაიკითხეო და წამალი Detoxic იყიდეო და თუ პარაზიტები გყავსო ეგ წამალი მიიღე და გარეთ გამოიტანსო ჭიებსაცო და პარაზიტებსაცო. თანაც საკმაოდ ძვირი ფასი მითხდა მხოლოდ ინტერნეტ შეკვეთებია ამ წამალზე. ასე ბრმად არავის ვენდობი და არ ვღებულებ ისეთ წამალს სადაც ოფიციალურად არაა შემოტანილი და არც ექიმები სთავაზობენ პაციენტს.

დავსერჩე გუგლი და Detoxic არაა საქართველოში რეგისტრირებული და არც ერთი ქართული ბრენდირებული აფთიაქი პსპ ავერსი ფარმა დეპო ჯიპისი არ ყიდის ამ წამალს

http://geodetoxic.pro/

ესეიგი თაღლითობასთან გვაქვს საქმე ? სადაა შსს პროკურატურა ჯანდაცვის სამინისტრო ? რატომ არ იძიებენ ამაზე ?
კანონით არარეგისტრირებული წამლის რეალიზაცია იკრძალება !!!

Posted by: Solveig 8 May 2017, 12:55
DON GIO
ჩემი მოკრძალებული აზრით, მორიგი თაღლითობაა ეგ. არადა, აივსო მაგის რეკლამებით ქსელი.

QUOTE
სადაა შსს პროკურატურა ჯანდაცვის სამინისტრო ? რატომ არ იძიებენ ამაზე ?

შსს და პროკურატურა არ ვიცი და, ჯანდაცვის სამინისტრო კი უნდა ინტერესდებოდეს წესით.
QUOTE
http://geodetoxic.pro/

კომენტარები მეტყველებს ყველაფერზე.

მაგათ რომ დაუჯერო, ადამიანთა 100%-ს ნაწლავში, ტვინში, გულში................ სხეულის ყველა ნაწილში პარაზიტული ჭიები უფუთფუთებენ და ერთადერთი საშველია ეგ სასწაულმოქმედი სითხე.

Posted by: Jessica Rabitt 11 May 2017, 14:20
რამდენი ხანი შეიძლება რომ ალერგია გაგრძელდეს?
ვგულისხმობ სიწითლეს და სიმხურვალეს კანზე
და ამ ალერტეკის მიღება რამდენი დღე შეიძლება

Posted by: chanelgirl999 15 May 2017, 11:36
ალერგია და გაციება როგორ უნდა განვასხვავო ერთმანეთისგან? პირველად დამთხვევა მეგონა, მაგრამ უკვე მეორე შემთხვევაა რემონტი როცა არის და იქ ვარ, მაშინ მეწყება სურდო და ცემინება. შეგრძნება ზუსტად ისეთია როგორც გაციებისას. თითქოს ოდნავ ყელიც მტკივა. ალერგიაა თუ გავცივდი ვერ ვხვდები

Posted by: teona1234 9 Jun 2017, 10:28
გლანდების ოპერაციის შემდეგ სიცხე მაქსიმუმ რამდენ ხანს შეიძლება გაგრძელდეს? მერვე დღეა და 37 მაქვს ისევ, სანამ გავიკეთებდი მანამდეც ოთხი თვე მქონდა და ჩირქოვანი გლანდები გაქვს მაგის ბრალიაო მითხრეს კლინიკაში ახლა გავიკეთე მაგრამ მაინც მაქვს სიცხე და როგორ ფიქრობთ სხვა რამის ბრალი იქნებოდა?

Posted by: Solveig 10 Jun 2017, 01:25
teona1234
რევმატოლოგთან ხარ ნამყოფი?

Posted by: teona1234 10 Jun 2017, 16:06
QUOTE (Solveig @ 10 Jun 2017, 01:25 )
teona1234
რევმატოლოგთან ხარ ნამყოფი?

არა, რევმატოლოგთან არა, სახსრები არ მტკივა, თავი მტკივა ხოლმე ყრუდ როცა სიცხე მაქვს. თავიდან იანვარში გავციცდი და მაგის მერე ასე ვარ, შეიძლება გაციებას არ უკავშირდება საერთოდ წარმოდგენა არ მაქ sad.gif(

Posted by: chanelgirl999 16 Jun 2017, 16:34
იქნებ დამაკვალიანოთ, სისხლის საერთო ანალიზი გავიკეთე და ექიმმა მითხრა ყველაფერი ნორმაშიაო, არადა რამდენიმე მონაცემი მაღალია და რანაირად არის ნორმაში? თრომბოკრიტი მაქვს 0,23 (ნორმა 0.1-0,3) და დიდი ზომის თრომბოციტების წილი 36,6 (ნორმა 15-35)

Posted by: BloodElf 18 Jun 2017, 13:09
მოკლედ იქნებ მიშველოთ.. დილით ვიღვიძებ ნორმალურად ვარ ცოტა ხანში მეწყება საშინელი თავის ტკივილი.. განსაკუთრებით მოძრაობაზე ან დახრაზე ლამისაა გასკდეს თავი.. ჩვეულებრივი ანალგინიც აყუჩებს და მერე მთელი დღე კარგად ვარ მეორე დილამდე.. წნევას ვიზომავ მაგრამ ვერ ვიგებ სავარაუოდ აპარატი არ ვარგა.. ნუ სადღაც 90-80 ს აჩვენებს.. ნორმალური წნევა 100-110 მაქვს 80 ზე.. ერთი კვირაა ასე ვარ.. ექიმთან ვერ მივდივარ კაპიკი არ მაქვს sad.gif
ორი კვირის წინ ლაშქრობაში ვიყავი და დავიწვი ცოტა მაგის ბრალია ვითომ? :/

Posted by: teona1234 21 Jun 2017, 01:47
QUOTE (chanelgirl999 @ 16 Jun 2017, 16:34 )
იქნებ დამაკვალიანოთ, სისხლის საერთო ანალიზი გავიკეთე და ექიმმა მითხრა ყველაფერი ნორმაშიაო, არადა რამდენიმე მონაცემი მაღალია და რანაირად არის ნორმაში? თრომბოკრიტი მაქვს 0,23 (ნორმა 0.1-0,3) და დიდი ზომის თრომბოციტების წილი 36,6 (ნორმა 15-35)

0.23 ნაკლებია 0.3 ზე შესაბამისად თრომბოკრიტი ნორმაში გაქვთ, გარდა მაგისა ნორმიდან მცირეოდენი გადახრა არ ითვლება პათოლოგიად და ხშირად ვლინდება, რაც არ შეიძლება იმთავითვე ჩაითვალოს რომელიმე სერიოზული დაავადების სიგნალად ან საფუძვლად. დაუჯერე ექიმს smile.gif
* * *
QUOTE (BloodElf @ 18 Jun 2017, 13:09 )
მოკლედ იქნებ მიშველოთ.. დილით ვიღვიძებ ნორმალურად ვარ ცოტა ხანში მეწყება საშინელი თავის ტკივილი.. განსაკუთრებით მოძრაობაზე ან დახრაზე ლამისაა გასკდეს თავი.. ჩვეულებრივი ანალგინიც აყუჩებს და მერე მთელი დღე კარგად ვარ მეორე დილამდე.. წნევას ვიზომავ მაგრამ ვერ ვიგებ სავარაუოდ აპარატი არ ვარგა.. ნუ სადღაც 90-80 ს აჩვენებს.. ნორმალური წნევა 100-110 მაქვს 80 ზე.. ერთი კვირაა ასე ვარ.. ექიმთან ვერ მივდივარ კაპიკი არ მაქვს sad.gif
ორი კვირის წინ ლაშქრობაში ვიყავი და დავიწვი ცოტა მაგის ბრალია ვითომ? :/

ძნელი სათქმელია რამ გამოიწვია თქვენი თავის ტკივილი და მნიშვნელობა აქვს ტკივილის ლოკაციასაც, თუ მოძრაობისას კისრის ნაწილი და კეფა გტკივათ ძირითადად და სხვა სიმპტომები, ადვილად დაღლა, სისუსტე და მისთ არ გაქვთ და არც რაიმე ქრონიკული დაავადების მატარებელი ბრძანდებით საქმე უნდა გვქონდეს კუნთების დაძაბულობით გამოწვეულ ტკივილთან ართრიტულ ტკივილთან გამოიყენება ანთების საწინააღმდეგო ან კუნთების სარელაქსაციო პრეპარატები უხეშად რომ ვთქვათ. მაინც გაიარეთ კონსულტაცია პროფესიონალ ექიმთან ან მინიმუმ კითხეთ ფარმაცევტს.

Posted by: BloodElf 22 Jun 2017, 23:45
teona1234
დაბალი წნევის ბრალი ყოფილა smile.gif) თურმე კი ვერ ვიზომავდი უბრალოდ ძაან დაბალი მქონდა.. smile.gif) რამდენიმე დღე ყავა მივაშველე და რავიცი გადაიარა


Posted by: F4R2 26 Jun 2017, 14:40
გამარჯობათ

თქვენი რჩევა მჭირდება ვისაც გქონიათ მსგავსი შემთხვევა ან გსმენიათ

ჩემს ათი წლის გოგონას უკვე მერამდენე წელია აქვს თმისა და ფრჩხილების ზრდის პრობლემა
ანუ უბრალოდ არ ეზრდება
ამ პერიოდის განმავლობაში ვიყავით უამრავ ექიმთან და ბავშვს ალბათ ყველანაირი ანალიზი გავუკეთეთ თუ რამე არსებობს
ყველა ანალიზი წესრიგშია, ყველა მაჩვენებელი ნორმაშია, კალცის ან რაიმე ვიტამინის ნაკლებობა გამორიცხულია რადგან ბავშვი სიმაღლეში ასაკთან შესაბამისად და უფრო მეტადაც იზრდება
ერთადერთი, რაზეც პერიოდულად ეჭვი მიაქვთ ექიმებს ესაა ჭიის არსებობა, რადგან ორჯერ შესაბამის ანალიზში რაღაც მსგავსი აღმოჩდა, ჩავუტარეთ შესაბამისი მკურნალობის კურსიც მაგრამ შედეგი ჯერჯერობით დიდად არ გვაქვს

ხომ ვერ მირჩევთ ექიმს ვინც შეიძლება დამეხმაროს და დამაკვალიანოს ან ხომ არავის გქონიათ მსგავსი პრობლემა?

Posted by: ToyotaMR2 28 Jun 2017, 10:14
vis sheudzlia amis targmna ra mdgomareobaa?

Posted by: F4R2 28 Jun 2017, 11:16

ToyotaMR2
ნეიტროფილებია ნორმაზე ოდნავ დაბალი და მონოციტებია ასევე უმნიშვნელოდ მომატებული
დანარჩენს მკურნალი ექიმი გეტყვით, მაგრამ წესით არაფერი ისეთი მნიშვნელოვანი



Posted by: oneman 6 Jul 2017, 22:12
კიდურის ტკივილი, ფეხის. თეძოდან დაწყილი.უფრო თეძოს ტკივილი სიარულის გამო. რომელი ექიმის საქმეა? _ახლობელს უნდა გავურკვიო. "ვარიქსინალმა" თითქოს უშველა, მაგრამ მაინც ტკივა.

Posted by: chanelgirl999 7 Jul 2017, 15:35
სისხლში გლუკოზის დონის მომატება რომელ დაავადებებზე მეტყველებს? მომატება ყველა შემთხვევაში დიაბეტის ნიშანია?

Posted by: una_nuna 30 Jul 2017, 14:46
გამარჯობა მაინტერესესებს ამ პროდუქტზე რა ინფორმაცია გაქვთ თუ მიგიღიათ და რამდენად კმაყოფილი ან უკმაყოფილო ხართ?

user posted image

Posted by: gig@@@ 4 Aug 2017, 22:34
ელექტრო ( ციფრული ) თერმომეტრი ჯობია თუ ჩვეულებრივი უბრალო ვერცხლის წყლიანი ? რომელი ზომავს სიცხეს უფრო ზუსტად ?

ესეთები კარგია ? http://putkari.com/Highly-Accurate-Digital-Medical#.WYS-AklSC70

Posted by: nanuchi_13 10 Aug 2017, 18:05
უცნაური რამე მჭირს და იქნებ გამარკვიოთ რა დამემართა, უკანალზე გამიჩნდა დამწვრობები, თითქოს სიცხისგან ჩაიხუთა, მაგრამ არ გავს ეს ჩახუთვას მარტო, კანი მთლიანად მტყავდება და ვერ ვხვდები რისი ბრალია, გინეკოლგთან ვიყავი , ლევომეკოლის მალამოს ვხმარობ და ანტიბიოტიკიც დამინიშნა , უკეთესობა არის თითქოს, მაგრამ მაინტერესევს რა შეიძლება იყოს

Posted by: MEMORATH 24 Aug 2017, 18:35
ბიცილინ 5 1 500000 სახლის პირობებში ძაან საშიშია? მაშინაც თუ გამოცდილი ადამიანი გიკეთებს??

Posted by: აკა 3 Sep 2017, 16:22
კარგი ნევროპათოლოგი მინდა ვისთან მირჩევთ მისვლას?

Posted by: chanelgirl999 14 Sep 2017, 11:39
ალერგიის წამლის მიღების პერიოდში, ზომიერი რაოდენობით ალკოჰოლის მიღება შეიძლება?

Posted by: temart 18 Sep 2017, 15:24
წყალბადის ზეჟანგი კლავს თუ არა ყველანაერ სოკოვან ინფექციას?


მადლობთ წინასწარ.





Posted by: AnEeEeE=]] 21 Sep 2017, 09:33
ხალხო რამაინტერესებს კონდიციონერს შეუძლია რომ ბრონხიტი გამოიწვიოს? აშკარად ვატყობ რომ ყელი მიშრება ხოლმე შემოდგომაზე მერე ხველება მეწყება და ბრონქიტში გადამდის, სადღაც მეოთხე წელია ესე მომდის და ეხლა დვაკავშირე ამ კონდიციონერს ადრე არ მემართებოდა მსგავსი არაფერი =))

Posted by: Tavtechnics 21 Sep 2017, 11:02
დილით მეღვიძება 6 ზე ვეღარ ვიბრუნებ მერე ძილს, მერე შუადღეს ამის გამო დაღლა მერევა რა ვქნა?

Posted by: gvarjila18 29 Sep 2017, 01:11
მეგობრებო,კოლეგობო გამარჯობა,ხომ არა გაქვთ რამე პათოლოგიის თერაპიული პროფილის რათქმა უნდა საპრეზენტაციო მასალა?

Posted by: piko1989 4 Oct 2017, 21:51
გამარჯობათ იციტ რა მაინტერესებს? როცა 24 წლის გოგონა სიმარლით 147 იწონის 39 კილოგრამს ეს შეიძლება რაინე დაავადების მანიშნებელი იყოს? და რა ანალიზების ჩაბარებას გვირჩევდით?

Posted by: Guzinio 9 Oct 2017, 22:22
user posted image

ნაცხის ანალიზზში ხახიდან ეს ამომეთესა... ადრე საშუალო დონით მქონდა არ ვიმკურნალე და ეხლა გაიზარდა რიცხვი ...ვისთან მივიდე ხომ არ იცით ? თერაპევტს ეხება მგონი ზო ? არვიცი ვისთან მივიდე ამ მხრივ ვინაა ძლიერი რომ დავიწყო მკურნალობა ფაგებით თუ რითიც საჭიროა რა... მირჩიეთ ვინც ამ საქმეში კომპენტეტურია ...

Posted by: oneman 10 Oct 2017, 19:59
აუცილებელი არ არის ექიმმა მიპასუხოს, არაექიმებსაც მოგისმენთ.
ტანზე გამონაყარი, წვრილი წითელი მუწუკები ქავილთან ერთად, რამდენად საშიშია? ალერგია არის თუ რა არის?
წიწაკის დაჭრამ ხომარ იცის?
ჩემ ახლობელს დაემართა მოულოდნელად, ან რამე ბაქტერია ხომარ გამოჩეკა ან შეესია?

Posted by: ogven 4 Nov 2017, 18:54
გამარჯობათ.
მაინტერესებს თქვენი აზრი შემდეგ საკითხზე.
სახსრის პრობლემების გამო (ანთება + სითხე+ და ა.შ.) , ექიმმა დამინიშნა ინექციები კუნთში (ანოდინ-დუალი,შეიცავს დექსამეტაზონს,1Хდღეში), კომპრესები (დიმექსიდი+დექსამეტაზონი,1Хდღეში) და ფერმენტული პრეპარატი ქიმორალ-ფორტეს ტაბლეტები(2Хდღეში). დასაშვებია თუ არა ამ პრეპარატების ერთი და იგივე დღეებში მიღება. ადრე ინტერნეტში ამოვიკითხე ფერმენტული და კორტიკოსტეროიდული პრეპარატების ურთიერთქმედების შესახებ, სადაც ნათქვამი იყო ფერმენტები შლიან კორტიკოსტეროიდებს(თუ პირიქით) და ამიტომ მათი ერთდროული მიღება მიზანშეუწონელია.

Posted by: teona1234 26 Nov 2017, 13:11
QUOTE (piko1989 @ 4 Oct 2017, 21:51 )
გამარჯობათ იციტ რა მაინტერესებს? როცა 24 წლის გოგონა სიმარლით 147 იწონის 39 კილოგრამს ეს შეიძლება რაინე დაავადების მანიშნებელი იყოს? და რა ანალიზების ჩაბარებას გვირჩევდით?

არა, მაგრამ თუ წონაში მკვეთრად ბოლო პერიოდში, არამოტივირებულად დაიკლო მაშინ საშიშია, ისე ჩვეულებრივი წონაა, არაპათოლოგიური, ოდნავ ცდება ნორმას, 147 სმ. გოგონასთვის იდეალურ წონად 41-55 კილოგრამი ითვლება, კარგი იქნება ორ სამ კილოგრამს თუ მოიმატებს.

Posted by: gig@@@ 26 Nov 2017, 21:07
ვიტამინ D3 25 (OH) მაქვს დაქვეითებული, ანალიზის პასუხში მიწერია 9,307
ნორმა უწერიათ => 30 ng/ml რა შეიძლება იყოს მიზეზი დაქვეითების ? ენდოკრინოლოგიურ დარღვევებთან ეს დაქვეითება კავშირშია ?

Posted by: chanelgirl999 28 Nov 2017, 17:10
თავი დავიბანე და 2 საათში გარეთ ვიარე, ნუ გამშრალი კი მქონდა მარა შიში ამეკვიატა მენინგიტი არ დამემართოს თქო, იმდენს ვფიქრობ ამაზე, რომ უკვე ღებინების შეგრძნება მაქვს და ესეც მენინგიტის ერთ-ერთი სიმპტომია. თუ რამეა იმავე დღეს მექნებოდა სიცხე?

Posted by: Ttesa19 30 Dec 2017, 00:09
ანალიზი გავიკეთე ანტისტრეპტოლიზინზე და 550 მაქვს რაუნდა ვქნააა ძაან საშიშია ვისთან მივიდე ხომ არ იცით

Posted by: -XaM- 30 Dec 2017, 09:17
ეს რა შეიძლება იყოს?

გუშინ არ მქონდა. ამ დილით ვნახე. რბილი არაა. ძვალივითაა.

user posted image

Posted by: Ttesa19 3 Jan 2018, 15:25
კარგი თერაპევტი მირჩიეთ რააა ისეთი არ მინდა მხრების ჩეჩვის მეტი არაფერი რომ არ იციან ეგეთები მოვიარე უკვე

Posted by: ferdinandge 4 Jan 2018, 21:52
ერთი კითხვა მაქვს, ჩემი მეგობარია ანუ სამელზე დამოკიდებული არ არის. უბრალოდ როდესაც ალკოჰოლს ღებულობს და იძინებს ღამე დილით ისევ მთვრალი იღვიძებს ვერ ფხიზლდება. ამიტომ არ სვამს სასმელს გამონაკლის შემთხვევაში ნუ ახალი წელი დაბადების დრე და წელიწადში 2 3 ჯერ. მაგრამ როცა სვავს დილით ისევ მთვრალი როა ცუდად გრძნობს თავს და 1 ჭიქა შემდეგ უნდა დალიოს. ესე გრძელდება 2 3 4 დღე მერე ცუდად ხდება ალბათ ინტოქსიკაციააქვს და გადასხმა ჭირდება.

ალბათ მისი ორგანიზმი ალკოჰოლს ვერ იტანს და ვერ ამუშავებს.
არის რამე წამალი ან საშუალება რომ ეს პრობლემა აღარ ქონდეს და გამოფხიზლება შეძლოს ?
კიდევ დავაზუსტებ რომ ალკოჰოლზე დამოკიდებული არარის და არც ბევრს სვავს. საუბარია 3 4 ჭიქა ღვინოზე ან 2 ჭიქა ჭაჭაზე (არაყს არ სვავს).

მე ვფიქრობ რომ რამე მკურნალობის ან რამე მეთოდი იქნება ამის სამკურნალო.

Posted by: zaza1993 6 Jan 2018, 00:37
ალბეზი დამინიშნა ექიმმა. აღწერა რო ვნახე, გვერდით მოვლენებში წერია ჭინრჭის ციება რაც უკვე 1 წელია ისედაც მაქვს. შეიძლება რამე გართულება გამოიწვიოს? ან დექსამეტაზონის გაკეთება თუ შეიძლება ამ წამალთან ერთად?

Posted by: asatiani134 13 Jan 2018, 15:25
გამარჯობა. სისხლის საერთო ანალიზი გავიკეთე და ექიმმა არც გადახედა ბოლომდე, თუმცა მონოციტების პროცენტული რაოდენობა არის 14.40% (ნორმა 3-9), ისე რაოდენობაც მონოციტების 1.15 (ნორმა 0.1-0.8), პლუს ლიმფოციტების პროცენტული რაოდენობაც ნორმაზე დაბალია 17%. (ნორმა იწყება 20%-დან), ერითროციტების დალექვის სიჩქარეც 45 მაქვს , ნორმის მაქსიმალური ოდენობა 20. რამეზე მიანიშნებს? ან ვის უნდა მივმართო? user.gif

Posted by: merciamercia 15 Jan 2018, 19:36
თბილისში Kარგი ნევროპათოლოგი მირჩიეთ რა user.gif

Posted by: s0uL 21 Jan 2018, 02:33
გამარჯობა,
მოკლედ არ ვარ საქართველოში და მიჭირს ექიმთან მისვლა კითხვა მაქვს თქვენთან საერთო ანალიზი გაბიკეთე და ყცელაფერი რომაში მაქვს არაფრი არ გწირსო, მაგრამ ყელის კისრის იღლიის და სასქესო ორგანოს ორივეს მხარეს მაქვს ჯირყცლები და არ გადის დიდიხანია ესეა უკვე 5 -6 თვეა, და ხელს როვიკიდებ ზოგჯერ პტრაა და ზოგჯერ დიდი და ეზლა ერთ სიდიდეზე დარჩა ყველგან ან დიდება, რისი ბრალი შეიძლება იყოს ან ვის მივმართო რის ექიმთან უნდა მივიდე? სხვა ისეთი არაფერი არ მაწუხებს ერთი დაღლილობა და ეს ჯირკვლები დისკომპორტს მიქმნის და ზოგჯერ წქმოდგომისას თავბრურე მეზვევა, მადლობა. იქნებ რამე რჩევა მომცეთ. საერთო უკვე 2 ჯერ გავიკე ამ 6 თვეში და ნორმაშია ყველაფერიო....

Posted by: helloween1995 23 Jan 2018, 19:56
user posted image მითარგმნეთ თუ შეგიძლიათ

Posted by: Perske 24 Jan 2018, 23:05
.....................................

Posted by: საქმიანნი 28 Jan 2018, 12:25
ვის ვკითხო არ ვიცი, აქ დავწერ
სანამ ექიმთან მივალ, თქვენი აზრიც მაინტერესებს

მოკლედ, ძილში მარჯვენა მხარეს როცა ვტრიალდები, თავში მარჯვენა მხარეს საშინლად მტკივა ერთი ადგილი, სადღაც საფეთქლის ცოტა მერე, გეგონება რაღაცას აწვება მანდ ტვინი )
თვეში ასე 2-3 ჯერ ხდება ასე და რისი ნიშანი შეიძლება იყოს?

Posted by: ვიტა ბრევის 7 Feb 2018, 12:18
მოკლედ ორი დღეა,ძალიან გაციებული ვარ,სურდო,თავის ტკივილი,ხველა..ღამის საათებში მეწყება სპაზმები და იმდენს ვახველებ რო ბოლოს ვარწყევ ხოლმე,2 დღეა უკვე..თავის ტკივილები მანამდეც მქონდა,ერთი კვირაა დაახლოებით და იმას ვფიქრობ ,მენინგიტი ხო არაა,თუ ვირუსის ბრალიც შეიძლება იყოს?

Posted by: ninilion 11 Feb 2018, 20:47
სალამი, თუ იცით პროლაქტინის მომატება იწვევს მაღალ წნევას?

Posted by: ciyvi77 28 Feb 2018, 03:03
ანალიზი :H/H 10.7/32.0. Iron level 29, % iron sat 7%. რამდენი უნდა მივიგო რკინა დღეში? მადლობთ.

Posted by: Anubys 26 Mar 2018, 17:57
გამარჯობათ მაქვს სისხლის დენა ცხვირიდან დიდიხანია და რომელ განყოფილებაში შევიდე ვერ ვნახე ვერსად , ალბათ ყელ ყურ ცხვირი მაგრამ ვერ ვნახე ვერც ეს.

Posted by: Perseus 26 Apr 2018, 21:25
ხომ ვერ მეტყვით ხერხემლის ექიმის ვლადიმერ ხოროშუხინის კოორდინატებს?! ვერსად ვერ ვპოულობ, სულ ძველი ნომრებია მითითებული... sad.gif sad.gif sad.gif

Posted by: Zaza123456 12 Jun 2018, 00:39
კითხვის დასმას აზრი აქვს? .

Posted by: polemos0108 4 Jul 2018, 21:46
გამარჯობა.
ყოველი ძილის შემდეგ მექავება გენიტალიები. დაახლოებით 2საათის შემდეგ თავისით მივლის. გაოფლიანებაზეც მექავება. 2 კვირაა რაც ასე დამეწყო.
რისი ბრალია? რაიმე მირჩიეთ?

Posted by: PLAYer_1 9 Jul 2018, 19:52
გამარჯობათ

ცინკის გლუკონატია კარგი თმის ცვენბისთვისო წავიკითხე და გამოვაწერინე მეგობარს..

მოკლედ მე სურათზე ესეთი ვნახე სტატია სადაც წავიკითხე (მეორე სურათი)

მეგობარმა გამომიწერა ესეთი,.,.(პირველი სურათI)

ხომ არ იცით რა განსხვავებაა? დავლიო ვაფშე? :/


მოკლედ პირველი გამომიწერა მეორეს ნაცვლად... ამდენი ინგლისურიც არ ვიცი რო გავიგო რა სხვაობაა და იოქნებ მითხრათ?(((

ეს გამომიწერა
https://www.nowfoods.com/supplements/zinc-glycinate-softgels

ამის ნაცვლად
https://www.nowfoods.com/supplements/zinc-50-mg-tablets

user posted image

Posted by: *m_m* 16 Aug 2018, 22:59
გამარჯობა!

მეოთხე თვეა მაქვს სუბფებრილური ტემპერატურა 36.9 - 37.2 გრადუსი C. იშვიათად 37.5. უფრო ხშირად საღამოს მეტია, ვიდრე დღის სხვა მონაკვეთში. ეს არაა ჩემთვის ბუნებრივი ტემპერატურა. მთელი ცხოვრება 36.5/36.6 მაქვს, თუ ჯანმრთელად ვარ. მაწუხებს და ვგრძნობ როცა 37 და მეტი მაქვს, თვალები მეწვის და შუბლი მიხურს. სამი თვეა გამოკვლევებს ვიკეთებ და არაფერი ხელმოსაჭიდი არ გამოჩნდა, რომ ვიმკურნალო. გამოვრიცხეთ აუტოიმუნური დაავადებები, ტონზილიტი, გინეკოლოგიური ან სასუნთქი სისტემის პრობლემები. ანთება არსად ჩანს, არც სისხლის საერთო ანალიზში, არც ფარისებრი ჯირკვლისა და მუცლის ღრუს ექოსკოპიასა და გულმკერდის რენტგენზე. ლიმფური კვანძები არსად გადიდებული არ მაქვს. ერთადერთი მონაცემი რომელიც არაა ნორმაში, არის ეოზინოფილები - 0.04 მაქვს, როცა ნორმა 0.1-დან 0.4-მდეა. წავიკითხე ინტერნეტში, რომ ეოზინოპენია ასოცირდება მწვავე ინფექციურ (ბაქტერიულ) დაავადებებთან. ჩემმა თერაპევტმა არ ჩათვალა საჭიროდ ამისთვის ყურადღების მიქცევა, რადგანაც სხვა ყველა მონაცემი ნორმის ფარგლებშია და არ მაქვს ჩივილები გარდა სიცხისა. რამდენიმე სხვადასხვა თერაპევტთან და ენდოკრინოლოგთან ვიყავი და ვერ მივაგენი მიზეზს, რომ ვიმკურნალო.

ძალიან გთხოვთ, თუ გაქვთ იდეა, რისი ბრალი შეიძლება იყოს, იქნებ მითხრათ! რომელი ინფექცია შეიძლება მქონდეს, და თუ მაქვს, რატომ სისხლის სხვა უჯრედები არ არის ნორმიდან გადახვეული ან რატომ სხვა სიმპტომები არ მაწუხებს? სად და როგორ გავარკვიო?

Posted by: simebi 25 Aug 2018, 22:55
თუ იცით ააქვს მნიშვნელობა, წამალს ალკოჰოლთან ერთად დალევ თუ წამლის მიღების მერე ეგრევე მიაყოლებ მნიშვნელობა? წესით ერთად ხვდება კუჭში და არანაირი მნიშვნელობა არ უნდა ჰქონდეს ხო?

Posted by: ქართლოსი 11 Oct 2018, 19:33
რომ არ ღამდებოდეს ადამიანს ძილი მაინც მოუნდება ?

Posted by: Lacrimosa_ 1 Nov 2018, 01:28
შიმშილის გრძნობა არა მაქვს უკვე კარგა ხანია.
ანტიბიოტიკი მივიღე დაახლოებით 3 წლის წინ. რის შემდეგაც ხან შეკრულობა მქონდა, ხანაც პირიქით. შემდეგ პილორის ბაქტერია მითხრეს. ვიმკურნალე, მაგრამ ისევ მუდმივად დანაყრების გრძნობა მაქვს.
გურმანი რომ არ ვიყო, ალბათ ჭამა არც გამახსენდებოდა

Posted by: B l a c k J a c k 29 Nov 2018, 19:20
ქიროპრაქტიკოსს იცნობთ ვინმე? ზურგი და კისერი მაქვს მაწუხებს. მასაჟების მერე ჩიტივით ვარ, მაგრამ მერე ისევ ჩაჭიმულობა ბრუნდება (((

აქვე ვიდეო:
https://www.youtube.com/watch?v=n2iF048mTfY

Posted by: VAMPIRE_W 30 Nov 2018, 11:01
გავცივდი ორი კვირის წინ, ჩვეილებრივი უბრალო გაციება იყო, სისველე ცხვირში, ყელის ტკივილი.
დავლიე წამლები 3-4 დღე და თითქმის გამიარა. მხოლოდ ცოტა ცხვირის სისველე მქონდა დარჩენილი.
საღამოს თავი დავიბანე, გავიშრე კარგად და დავწექი და ეგრევე ვიგრძენი რატომღაც თავში რაღაც არაკომფორტული შეგრძნება, ყურები დამიგუბდა, და ცხვირიც ისე გამეჭედა ვეღარ ვსუნთქავდი მთელი ღამე. და თითქოს რაღაც მთელს თავზე წნევა მაწვებოდა.
ვიფიქრე გაივილის მეთქი მაგრამ რამდნეიმე დღე ასე გაგრძელდა, მერე ცხვირში სისველე მეტად გაძლიერდა და გასქელდა. ისევ მივედი ექიმთან და ისევ წამლები დამინიშნა, ვსვამდი მაგრამ არ მიშველა.
3-4 დღეში კიდევ უფრო უარესად გავხდი, თავი საშინლად მტკიოდა, სუსტად ვიყავი ეს ცხვირიც უარესად.
ბოლოს ექიმმა სისხლის ანალიზი ჩამაბარებინა და რაღაც ცვლილებები იყო, ბრონხებშია პრობლემაო, ოდნავ მახველებს და ოდნავ ხიხინი მქონდა.
ისევ დამინიშნა წამლები, რომ ვსვამდი უკეთ ვიგრძენი თავი, მაგრამ ახლა მოვრჩი მეოთხე დღეა და მაინც ცუდად ვარ.

გემოს და სუნის შეგრძნება საერთოდ დაკარგული მაქვს, ყურები ისევ დაგუბებული მაქვს, სისუსტეს ვგრძნიბ და თავი თითქოს მიბრუის.

რისი ბრალი შეძლება იყოს? გუშინ ვკითხე ექიმს და შეიძლება კიდევ უნდა ცოტა დრო რომ გაგიაროსო.
თუ გქონიათ ვინმეს მსგავსი რამე ან თუ იცით რისი რბალი შეიძლება იყოს? sad.gif((

Posted by: taamo 14 Jan 2019, 01:45
სად დავსვა ეს შეკითხვა არ ვიცი. დღეს 2-3 წუთის მანძილზე ჩემს გვერდით იდგა ადამიანი, რომელსაც (მოგვიანებით გავიგე) ტუბერკულოზი ჰქონია. ადვილი შესაძლებელია რომ მეც გადამედოს ინფექცია?

Posted by: ani_k 14 Jan 2019, 02:29
taamo
ინფექცია რომ გადმოგედოთ 50/50, თუმცა ის რომ თქვენ დაავადებით იმის შანსი არაა მაღალი, მითუმეტეს ის. ადამიანი მკურნალობს ალბათ. საერთოდ ტუბერკულოზი საკმაოდ გავრცელებულია და იმის შიშში ცხოვრება რომ დაავადებით სისულელეა , ავადმყოფობაზე ბევრი რამეა დამოკიდებული საცხოვრებელი პირობები, ადამიანის იმუნიტეტი და სხვა მრავალი, დიდი ალბათობით აცრილიც იქნებით ამ ინფექციაზე ასე რომ დამშვიდდით.

Posted by: taamo 14 Jan 2019, 11:02
ani_k

მგონი არ მკურნალობს ის ადამიანი. მადლობა პასუხისთვის smile.gif

Posted by: Likagogo 19 Jan 2019, 11:24
ვარ 31წლის, ბავშვობაში მოვიხადე წითელა ვირუსი, ახლა საჭიროა თუ არა აცრის ჩატარება, ასე გამიგია ვისაც გადატანილი აქვს მთელი ცხოვრება ნარჩუნდება იმუნიტეტიო

Posted by: txamuniko1985 11 Feb 2019, 01:38
გამარჯობა. 3 წლის და 8 თვის ბავშვს ფილტვების ანთება დაუდგინდა. 3 დღეა ვმკურნალობთ. სიცხეები აღარ აქვს 2 დღეა. ანთებასთან ერთად დაიწყო ბავშვმა კოჭლობა. ფეხს არ ხდის, და სიარულის დროს თითქოს კმ იათრევს. გთხოვთ მომწეროთ.
* * *
გამარჯობა. 3 წლის და 8 თვის ბავშვს ფილტვების ანთება დაუდგინდა. 3 დღეა ვმკურნალობთ. სიცხეები აღარ აქვს 2 დღეა. ანთებასთან ერთად დაიწყო ბავშვმა კოჭლობა. ფეხს არ ხდის, და სიარულის დროს თითქოს მიათრევს. გთხოვთ მომწეროთ, ფილტვების ანთებამ იცის კოჭლობა?
* * *
იცის ფილტვების ანთებას კოჭლობა? ქირურგმა გასინჯა. ბავშვს არაფერი სჭირს.

Posted by: txamuniko1985 11 Feb 2019, 01:38
გამარჯობა. 3 წლის და 8 თვის ბავშვს ფილტვების ანთება დაუდგინდა. 3 დღეა ვმკურნალობთ. სიცხეები აღარ აქვს 2 დღეა. ანთებასთან ერთად დაიწყო ბავშვმა კოჭლობა. ფეხს არ ხდის, და სიარულის დროს თითქოს კმ იათრევს. გთხოვთ მომწეროთ, ფილტვების ანთებამ იცის კოჭლობა? ქირურგმა გასინჯა. არაფერი აღმოაჩინა.

Posted by: ani_k 11 Feb 2019, 01:59
თუ ანამნეზში არ გახსენდებათ რაიმე ტრამვა ან არ შეგიმჩნევიათ ,თუ ფეხი მეორე ფეხის სიმეტრიულია, არ არის შეშუპება ან შეწითლებული მითუმეტეს ქირურგმა თუ გასინჯა, სავარაუდოდ რეაქტიული ართრიტია. txamuniko1985

Posted by: txamuniko1985 11 Feb 2019, 08:09
დიდი მადლობა. რამე სერიოზული ხომ არაა? ძალიან ვნერვიულობ.

Posted by: ani_k 12 Feb 2019, 00:42
txamuniko1985
დაავადება შეიძლება საკმაოდ სერიოზულ პრობლემადიქცეს ან სრულიად უკვალოდ ჩაიაროს , გართულებების გარეშე , გირჩევთ რევმატოლოგს მიმართოთ თუ სიმპტომი გახანგრძლივდა ან სხვა სახსრებიც ჩაერთო .

Posted by: nita0607 26 Feb 2019, 16:09
გამარჯობა,

მოკლედ მაქვს ესეთი პრობლემა: დაახლოებით 2 კვირის წინ დავიწყე სახლში ვარჯიში, ბუქნები და სამი დღის შემდეგ დამეწყო სუნთქვი პრობლემა, ზაფხულშIც ვვარჯიშობდი და მაშინაც სუნთვის გაძნელების გამო დავანებე თავი. ისე მაწუხებს, რომ ვიხრჩობი, განსაკუთრებით ჯდომის დროს და ჭამის შემდეგ. გავიკეთე სისხლის საერთო და თიესეიჩი და ნორმაშია, გადავიღე გულის ეხო და არაფერია გულზეც. ყელ-ყურმა ჩამხედა ცხვირში და მითხრა, რომ ნიჟარების ბრალია + ძგიდეც გამრუდებული მაქვს, დამინიშნა ავამისი და არაფერსაც არ შველის. რამე მიშველეთ , დღეს სასწრაფოს გამოძახებას ვფიქრობდი ისე ცუდად ვიყავი, დავლიე ალერგიის წამალი, კორსიზი, არაფერი მშველის

ან რომელ ექიმთან მივიდე რამე მირჩიეთ

Posted by: Unknown232324 23 Mar 2019, 12:34
გამარჯობა! საყოველთაო დაზღვევაში მინდა ჩავერთო და თბილისში რომელი კლინიკს არჩევას მირჩევთ რომელიც ჩართულია ამ პროგრამაში?
ვიცი რომ ევექსი მონაწიელობს და ევექსი ჯობია ვითომ დანარჩენებს?

Posted by: ნაკო 24 Mar 2019, 20:00
არ ვიცი რამდენას სწორად ვწერ თემაში, მაგრამ მაინც ვიკითხავ smile.gif

მეხსიერების პრობლემები პირველად არ მაქვს მაგრამ ასე გამძაფრებულად ჯერ არ მქონია, ვერ ვიმახსოვრობ, ვერ ვსწავლობ, ვერ ვაკვირდები, ვერ ვაანალიზებ და პლუს ჰალუცინაციები. სტრესისი და გადარლის დროს მ/ცვევია თუმცა ჰალუცინაციები აქამდე არ მქონია. რა მდგომარეობაში ვარ იქნებ მითხრათ?

Posted by: Finrod_Felagund 15 May 2019, 13:03
ქრონიკული ფარინგიტი მკურნალობას ექვემდებარება? გუშინ ვიყავი ექიმთან და დაიკიდე პროსტა მზესუმზირას და ეგეთებს მოერიდეო ო.ო

Posted by: lilianna 17 May 2019, 13:52
ანტი სტრეპტო დორნაზა-რამდენია ნორმა თუ იცით ვინმემ?ვერაფრით მოვძებნე ინტერნეტში

Posted by: happy dreamer91 24 May 2019, 12:53
user posted image
ამ პასაუხებზე რას იტყვით? შეიძლება ვიფიქრო რომ ანემია ამქვს? ორშაბათს ჩავეწერე ოჯახის ექიმთან და ვაჩვენებ. ჰემოგლობინი ძაან დაბალია?

Posted by: Ramda 26 May 2019, 02:12
არ ვიცი სად ვიკითხო იქნება ვინმემ მიპასუხით
ბ12 ვიტამინიზე სისხლის ანალიზი გავიკეთე
და მაქვს 526 ნორმალურია?

Posted by: ani_k 26 May 2019, 08:31
happy dreamer91
ყველაფერი ნორმაში გაქვთ.

ჰემოგლობინიც .

Ramda


თქვენს პასუხს ეწერება ნორმის მაჩვენებლები (ლაბორატორიებს თავისი ნორმები აქვთ) , შეადარეთ. (პ.ს მგონი დაბალ ნორმული მაჩვენებელია )

Posted by: Ramda 29 May 2019, 01:36
ani_k

დიახ უწერია ეხლა მივაქციე ყურადღება
225-1000 pg/ml უწერია ნორმალური

ხსენებულ ვიტამინს ვსვავდი და ეხლა კიდევ
ექიმმა კი მითხრა ნორმაშიაო მაგრამ ნორმა ალბათ ნახევარი უნდა იყოს არა?

Posted by: chuvak_lexo 17 Jun 2019, 12:11
გამარჯობათ ერთი თვეა ალბათ უკვე, ყური მექავება, მაცემინებს და მეცრემლება (ყურის/ცხვირის და ყელის ქავილი ჩემი გაციების სიმპტომებია) ასეთი ცემინება და ცრემლვა არ მქონია არასდროს, ხოოდა დღეს დილით მეუღლემაც გამომიცხადა რო მასაც დაეწყო ეს სიმპტომები user.gif

ექიმთან ვერ მივდივართ (15 წუთი რო გავდივარ ხოლმე სამსახურიდან ისეთი სახეებით მიყურებენ ცხოვრებას მოგაძულებენ)

Posted by: Kakasha 17 Jun 2019, 15:33
ვინმემ რამე მირჩიეთ გთხოვთ ... 2 თვის წინ სამსახურში ვჭამე სოკო ხოდა საღამოს ვიფიქრე მაგან მომწამლა თავს ცუდად ვგრძნობდი წნევა ამივარდა თავბრუ დამეხვა პულსი ამივარდა მეგონა ვუთიშებოდი უკვე წამიყვანეს საავადმყოფოში კუჭი ამომირეცხეს გადასხმები გამიკეთეს სისხლის ანალიზი ამიღეს და გამომიშვესს უკან იმის მერე დამეწყო ცუდად გახდომები ერთგვარი შეტევები წნევების თამაში ძირითადად გულის წნევა ამდის ხოლმე 100 110 ზე გული სულ მიბაგაბუგობს . შეტევები მაქვს ხოლმე თითქოს ვითიშები პულსი ამდის 100 ზე ზემოთ მიწითლდება თვალები.. ორი თვეა სულ ჩაწითლებული მაქ მარა შეტევების დროს უფრო მიწითლდება პლიუს თავბრუს ხვევები და თვალებსი გაორება თითქოს კეფაზე რაღაც მძიმე ლოდი მადევს ამ ორი თვის მანძლიზე კუჭში ხან შწკრული ვარ ხან ძაან თხლად გავდივარ ... სისხლის ანალიზი სამჯერ გავაკეთე პლიუს შარდი და არაფერია . გადავირე თავის კატე და მარტო ჰაიმორიტი დამიფიქსირდა და მაგაზე ვიმკურნალე. გადავიღე მუცლის ღრუს ეხოსკოპია და არაფერია მარტო გაზები მაქ ნაწლავებში. კიდევ რა შეილება გავაკეთო ნერვოპათოლოგთან ვიყავი პანიკური შეტევები გაქო და გამომიწერა რაღაცეები მაკარიძესთან ვიყავი... 23 წლის ბიჭი ვარ.. კიდევ რა შეილება ანალიზად ჩავაბარო აღარ შემიძლია ჭიის ანალიზი გაიკეთეო მითხრეს. ხო კიდევ გლანდები მაქვა გადაგვარებული ერთმა ექიმმა დაჩირქებულიაო მეორემ არაა დაჩირქებულიო :/
* * *
კარდიოგრამაც გადავიღე და არაფერია.. მთელი დღეები ძაან სუსტად ვგრძნობ სულ თავს,, კოაგულოგრამაც გადავიღე ყოველი შემთხვევისთვის და ნორმაში მაქ აქაც ყველაფერი ფაბრინოგენის კონცენტრაციას მიჩვენებს 1,90 და ნორმა 2დან 4 ამდეა

Posted by: Babibi 18 Jun 2019, 09:10
გამარჯობათ.კოაგულოგრამაში თრომბოპლასტინის დრო რისი მაჩვრნებელია და მარტო ამ მაჩვენებლის ცვლილება რამე დაავადებაზე მიუთითებს? 9 თვის წინ გაკეთებულ კოაგულოგრამაში თრომბოპლასტინის დრო იყო გახანგრძლოვებული, ორი დღის წინ გაკეთებულში კი პირიქით შემცირებული.ვაპირებ ჰორმონის დალევას.საშიში ხომ არ იქნება ჰორმონის დალევა?სხვა დანარჩენი მაჩვენებლები ნორმაშია,მარტო ეს თრომბოპლასტინი ადის ან ჩამოდის ნორმის ფარგლებიდან

Posted by: tam1 18 Jun 2019, 17:30
ფილტვის ტიპიური კარცინოიდი და კარცინოიდული სინდრომი ერთი და იგივე დაავადებაა?

Posted by: ninikela20 26 Jun 2019, 20:55
გამარჯობათ შეგიძლიათ კარგი თერაპევტი მირჩიოთ თბილისში?

Posted by: Pathetikos 9 Jul 2019, 16:53
მოკლედ, ჯერ ერთი კვირა ვიწექი მაღალი სიცხით (არანაირი ხველა, სურდო ან ყელის ტკივილი, არადა ყელში ანთება გქონდაო, თერაპევტმა მითხრა), მერე დამრჩა საშინელი სისუსტე, რომელიც ახლაც მეტყობა. ბოლო ორი დღეა - ანუ 10 დღეზე მეტი ხსნის მერე სიცხიანობიდან - ისევ შემომეპარა 37-დან 37.3 სიცხემდე. სიცხის დროს თერაპევტმა ჯერ ანტიბიოტიკი მასვა 5 დღე, მერე უბრალოდ ვიტამინების კომპლექსი, და ბევრი წყალი სვიო.
ხოდა რა ვქნა უკვე აღარ ვიცი? რამე ესეთი სპეციფიკურად გრძელი ვირუსია თუ ჯობია, ისევ მივმართო ექიმს თუ გამიგრძელდა? მანამდე თუ ეს დაბალი სიცხე თავს მატკიებდა და მსპობდა, ეხლა ნორმალურად ვარ და მუშაოაბასაც ვახერხებ. უბრალოდ ნერვებს მიშლის, რომ ვიცი, რომ არ მეშვება vik.gif

Posted by: ფაშისტი 4 Oct 2019, 10:27
1 კვირის წინ პოპ კორნს მივირთმევდი, ეტყობა საყლაპავი მილი გამიკაწრა თუ პოპ კორნის ის რაღაცა პრიალა ლებანი მიმეკრო, რა აღარ დავაყოლე, ლამის მთლიანი სუხრები და ნუ მოკლედ რა, არაფერი შველის, 2-3 დღეა დავაკვირდი რომ დაბლა ჩაიწია.
ხოდა ვერ ვხვდები მიკრულია ისვე პოპ კორნი და ასე უჭირს ჩასვლა თუ გაფხაჭნილია საყლაპავი მილი და შეშუპებული და ეს მაწუხებს.
ამდენი ხანი შეიძლება საკვები არ გაიხრწნას და არ ჩავარდეს კუჭში? გადავირიე ..

Posted by: Lacrimosa_ 11 Oct 2019, 15:33
მეგობრებო,
სასწრაფოდ მესაჭიროება ჭკვიანი თერაპევტი, რომელიც შეკრებს ყველა მონაცემს პაციენტის შესახებ, საჭიროების შემთხვევაში გაივლის კონსულტაციებს შესაბამის ექიმებთან და დასვამს დიაგნოზს.
გთხოვთ, თუ ვინმე იცით ასეთი, ძლიერი დიაგნოსტიკოსი, მომწერეთ.
გერმანიაში სანამ წავალ, იქნებ აქ ვიპოვო.

ყველა ექიმი, ვისთანაც ჯერ-ჯერობით მქონდა შეხება, მხოლოდ საკუთარი განხრით უყურებს და სხვა ორგანოები და ფუნქციები მხედველობიდან რჩება.

Posted by: Lacrimosa_ 13 Oct 2019, 11:41
ერთი თერაპევტიც არ გვყავს თბილისში ნორმალური?

Posted by: ninikela20 16 Oct 2019, 18:22
QUOTE (Lacrimosa_ @ 13 Oct 2019, 11:41 )
ერთი თერაპევტიც არ გვყავს თბილისში ნორმალური?

ეგეთი ვერც მე ვიპოვე საქართველოში sad.gif

Posted by: POKERISBLUFF 31 Oct 2019, 04:01
ბლიინნნნ საშინელი ხასიათიდან ფიზიკურ ტკივილამდე არასდროს გადასულა მდგომარეობა
მაქსიმუმ სიცხე მოეცა როცა ძალიან ცუდ ხასიათზე ვიყავი და ამინდიც რაღაცნაირი იყო ეხლა კიდე მგონია რო ვკვდები
უკან წელის არეში რაღაც გაუსაძლის ტკივილს ვგრძნობ 5 წუთი გრძელდება ცოტა ხნით ქრება და ისევ იწყებს
ამ ყველაფერმა კიდე თავი ამატკივა უკვე და თავბრუც მეხვევა


თან მარტო ვარ და ვერც გავდივარ ვერსად თორე რამე ტკივილგამაყუჩებელს ალბათ აფთიაქშიც მომცემდნენ
რო გათენდება კლინიკაში წავიდოდი მარა მანდაც ტყუილად უაზრო ხარჯებს ამკიდებენ და შეიძლება უარესი რაღაც დამემართოს რაღაც სხვა დიაგნოზით , საახლობლოშიც მსმენია ცუდი შემთხვევები
ეს რა ### ქვეყანაა და დაიწყო ამ დედა#####ულმა ტკივილმაც თავს პირდაპირ ძალიან მძიმეს ურტყავს თითქოს გვერდიდან ეს რა ###ობააა ბიჭოოოოო

ცხოვრებაში პირველად ვარ ესე უცნაურად , გაიაროს მაინც რო დავიძინო ამის დ####ს შ#######ი
ეს რა ჭირია არც ვიცი ესეთი გაუგებარი ტკივილის დროს რა გავაკეთო

* * *
akliveeeew janmrtelobaze mnishvnelovani cvetshi araperia adamianistvis sacicocxlod

imdeni gamayuchebeli davitrie agaraperi momklavs
mara gushin dzaan pizdeci ghame iko

arada bolos ro shevmowmdi araperi mchirda vapshe da fizikuri tkivili iset adgilas, sadac arasdros araperi amtkivebia tu nevrozma gamoicvia da kochag magas

karoche asworebs roca ise xar ro araperi gawuxebs fig.gif

Posted by: chuvak_lexo 14 Nov 2019, 15:14
გამარჯობათ, მთელი კვირა მაღებინებდა საჭმელს რო ვჭამდი მალევე, გაციება და შიგნიდან რო ვგრძნობდი რო სიცხე მქონდა და ძვლებში მტეხავდა ეგ უძღვოდა წინ, გუშინ რო ვაღებინე სისხლიც შევამჩნიე, შეიძლება ვირუსი იყოს თუ საყლაპავი მილია შევიწროვებული და რამე კიბოს რისკებიც იყოს sad.gif

Posted by: ნინა 30 Nov 2019, 20:38
http://stumari.ge/mustheraphy/
მუსიკის თერაპია ერთერთი უძლიერესია სხვა თერაპიებს შორის. მუსიკის მოსმენის სიკეთეს ყოველი ადამიანი გრძნობს. ხოლო მუსიკის შესრულება კიდევ უფრო აქტიურად მოქმედებს ადამიანის შინაგან სამყაროზე: აწესრიგებს, აორგანიზებს, უმძაფრებს კოორდინაციას, უხსნის სტრესს, აძლევს თავისუფალი მოქმედების და სწორი გადაწყვეტილებების მიღების უნარს. რაც შეეხება ფიზიკურ ჯანმრთელობას უშუალოდ აძლიერებს სასუნთქ სისტემას, რაც არეგულირებს სისხლის მიმოქცევას, შესაბამისად წნევას. გუნდში სიმღერა ააქტიურებს შემოქმედებით იმპულსებს, ადამიანს მატებს სიმხნევეს და სიცოცხლის სიხარულს.
„საავტორო სტუდია“ სწორედ ასეთ მომსახურებას გპირდებათ. გუნდში სიმღერა შესაძლებელი იქნება ყველა ასაკის ადამიანისთვის, რომელსაც ექნება სიმღერის სურვილი, შესაბამისი მუსიკალური მონაცემების ფონზე.
მეცადინეობა ჩატარდება კვირაში ერთხელ ორი აკადემიური საათი, შესვენებით.
მომსახურების საფასური: 50 ლარი, გადარიცხვით, ხელშეკრულების საფუძველზე.

დაგვიკავშირდით
595 355846
saavtorostudia@gmail.com

Posted by: inga1988 4 Mar 2020, 19:40
გამარჯობა
სახეზე კანქვეშა გამონაყარი გამიჩნდა, რომელიც ღრმა წმენდებით კოსმეტოლოგთან ვერ მოვირჩინე, ვარ 31 წლის, შეიძლება ეს გინეკოლოგიასთან იყოს კავშირში, ჰორმონების დისბალანსთან ?? თუ კი მაშინ რომელი ანალიზია აუცილებელი რომ გავიკეთო? და ადვილად იკურნება თუ რთულ რამესთან მაქვს საქმე??
პ.ს......ასევე ,მაქვს ხოლმე გულისრევის შეგრძნება.
მადლობა წინასსწარ ვინც გამომეხმაურებით კომპეტენტური პირი <3

Posted by: аnderson 6 Mar 2020, 13:26
მოკლედ ორი წელია მტკივა ყელი.


ანუ ძირითადად ძაან ოდნავ, დიდი რომ არაფერი არაა რა. ცოტა ნახველი მიგროვდება ხოლმე. ოდნავ გაციებულის პონტში. მაგრამ წელიწადში ორ-სამჯერ მირთულდება, მაღალი სიცხე მაქ და ისე მტკივდება ყელი რომ რა ვიიც, რომ ვსუნთქავ მტკივა. ეხლაც ეგრე ვარ.

ექიმთან რამდენჯერმე ვიყავი, ხან ამოსახველებელი დამინიშნა, ხან საწუწნი ტაბლეტები. მაგრამ აშკარად არ მშველის. 2 წელია არ მშველის.

ეხლაც ვიიც 1-2 დღეში გამივლის, მარა ცოტათი დამრჩება ისევ ჩაწითლებული და ჩამწვარივით. და მერე 2-3 თვეში ისევ შემახსენებს თავს.

რა წყალში გადავვარდე

Posted by: zzz77 7 Mar 2020, 14:04
аnderson
კარგად გამოაკვლევინე ასე მარტო ზეპირად დანიშნული წამლები არაგრისმომ ემი იქნება







* * *
აქვე ვიკითხავ პროსტამოლ უნო მამაჩემისთვის 75 წლის კაცია და ექიმის დანიშნულების გარეშე შეიძლება მივცე????? კი ვიცი რეცეპტის გარეშეა მაგრამ მაინც

Posted by: аnderson 27 Mar 2020, 14:05
ჰერპესული გამონაყარი მიჩნდება ხოლმე ტუჩზე. გამონაყარი რა, ერთი ცალია ხოლმე პრინციპში. და ცოტა უაზრო. რომ მგონია ჩაცხრა და უნდა გამიაროს ისევ აწითლდება ხოლმე.


წვა და ქავილი არ მაქვს, უბრალოდ გამოსულია შუქნიშანივით. ჰოდა რა ვუშველო?

Posted by: makmanus 27 Mar 2020, 15:28
аnderson
ანალიზი აიღე ჰერპესზე M ფორმა,მწვავე პერიოდი თუა არსებობს მკურნალობა და დაგინიშანვს ექიმი მედიკამენტებს

Posted by: temart 29 Apr 2020, 12:18
გულის ოდნავ მარჯვნივ მცირე სიმძიმის შეგრძნება მაქვს. ოდნავ სველი ხველა მაქვს დღეში 3-4-ჯერ.წამალი ლიზლი დავლიე უკვე 12 ტაბლეტი, რამაც ცოტა მიშველა სრულად კი არა. რას მირჩევთ?
* * *

Posted by: chuvak_lexo 28 May 2020, 13:59
გამარჯობათ ბოლო 2 თვეა შიმშილის გრძნობა დამეკარგა, ყოველთვის დილით შიმშილის გრძნობა მაღვიძებდა და სადღაც 9 ზე ვჭამდი საჭმელს, მერე 12-13 საათზე ვლანჩაობდი, თუ გავუძლებდი მაქამდე, და მერე საღამოს 5-6 ზე, არც ჭარბი წონა მაწუხებს და არც მეტაბოლიზმის პრობლემები მაქვს, 180 - 84 -ზე ვარ მკვრივი აღნაგობით და ალბათ 20% ცხიმით. რაციონიც არ მაქვს ჯანქ ფუდის, სულ სახლიდან დამაქვს საჭმელი, დილით გერკულესი, ხაჭო, მაწონი - ესაა ჩემი რაციონი დანარჩენი ცილა-ნახშირწყლების და პოსტნეულის მონაცვლეობა. მოკლედ ნაგავს არ ვჭამ. სიგარეტს 1-1 ღერს აქამდეც ვეწეოდი და ეხლაც ვეწევი ხოლმე, ამას რომ დავაბრალო. ახლა კიდევ 2 საათი ხდება და აშკარად ვხვდები რომ შაქრის დონე მივარდება და მშია თორემ კუჭი სიგნალს არ აგზავნის, აქამდე კედლებზე ავიდოდი შიმშილის ისეთი ძლიერი შეგრძნება მქონდა ხოლმე sad.gif

Posted by: chotik 29 May 2020, 09:34
მარჯვნივ ცოტა ქვევით ნეკნებთან მაქვს ტკივილი და ვისთან უნდა მივიდე ეხო მჭირდება ალბატ არვიცი ., მოკლედ რისი ექმიმი მჭირდება?

Posted by: lilu777 1 Jul 2020, 23:27
გამარჯობათ,
ვინმე კომპეტენტურს თუ შეუძლია მითხრას პრობიოტიკების თვითნებურად მიღება შეიძლება?
და ძირითადი ჩვენება მაინც რა არის როდის უნდა მიიღო?

Posted by: Domaci 22 Jul 2020, 14:55
გამარჯობა

ამ ბოლო დროს ხშირად მემართება გადაღლა. ლაპარაკის დროს ვიღლები თავბრუ მეხვევა... დღეს ვცადე 1 კმ მონკვეთის ფეხით გავლა და ძაან ვიღები რა.
ადრე არ მჭრდა ესე . ვარ მწეველი მაგრამ ესე ძალიან გადაღლილი არ ვყოფილვარ არასდროს.

თან ვაკვირდები რატო და როდის მემართება ხოლმე ესე მაგრამ ვერ გავიგე რა... წინა კვირას გავათენე მაგრამ ენერგია მაინც მქონად დღისით.

კიდე ერთი რასაც დავაკვირდი არის მსესუმზირა. შეიძლება დამთხვევაა მაგრამ გუშინ საღამოს მსესუმზირა შემომეჭამა საკაიფოდ... და დღეს დილით ვარ გადაღლილი . ადრეც იყო მსგავსი შემთხვევა მსესუმზირაზე..
ექიმებო.. რამდენად შეიძება რომ მსესუმზირამ ნაღველა გააღიზიანოს და ნაღველამ გადაღლით შემომიტიოს??

დღეს პროთრომბინის პასუხიც მივიღე. 84%

ანუ სისხლის მიმოქცევა ნორმ

Posted by: goniashvili 28 Jul 2020, 15:20
გამარჯობათ ვარ 25 წლის კაცი. გამუდმებით პირი მიშრება ამ ზაფხულს განსაკუთრებით მომემატა. დავლევ წყალს, 1 წუთში ისევ პირი მიშრება, კიდევ დავლევ ისევ. დღეში ხანდახან 5 ლიტრ წყალს ვსვამ. არა აქ მნიშვნელობა შევჭამ რამეს თუ არა მუდმივად მაქვს ყელის და პირის სიმშრალე ძლიერი წყურვილის გრძნობა. თან ჰაერის უკმარისობა მაქ. აღარ ვიცი რა ვქნა ბევრ ექიმთან ვიყავი უკვე. ყველანაირი ანალიზი გავიკეთე (სისხლის, შარდის, შაქარზე, ჩიყვზე, ექოსკოპიები გულზე და ა.შ) ყველაფერი ნორმაში მაქ არაფერი არაა მომატებული არც ჰორმონი, არაფერი . არ ვეწევი არ ვსვამ. ნარკოტიკზე ხო ზედმეტია ლაპარაკი. ძალიან გთხოვთ მირჩიეთ რამე ან იქნებ მითხრათ ვის ვიმვართო

Posted by: Kakasha 28 Jul 2020, 17:56
გამარჯობათ მეგობრებო მოკლედ ასეთო პრპბლემა მაქ სისხლჩაქცევები თუ რაღაც გაწითლებული ადგილებიოჩმდება ჯერ ცხვირზ3 გამომივიდა მერე მკერდზე არც მტკივა არც მექავება.. ლეიკემიის ერთერთი სიმპტომია ეს ხო? :/

Posted by: Domaci 30 Jul 2020, 13:39
goniashvili

მეც ვარ ხოლმე ეგრე უფრო ღამე. შაქარი მეგონა, წავედი გავიკეთე ანალიზები და ყველაფერი მწყობრში მაქვს.

მერე დავაკვირდი და ნერვიული აშლილობის ბრალია... მე ცოტა აზარტული ვარ და ვთამაშობ. როცა წაგებას განვიცდი მაშინ მეწყება ხოლმე ...

შენც დააკვირდი აბა როდის გემართება ეგ. მეც ზუსტას ესე წყალს დავლევდი და ცოტახანშ ისევ მიშრებოდა პირი.

Posted by: goniashvili 2 Aug 2020, 18:23
QUOTE (Domaci @ 30 Jul 2020, 13:39 )
goniashvili

მეც ვარ ხოლმე ეგრე უფრო ღამე.  შაქარი მეგონა, წავედი გავიკეთე ანალიზები და ყველაფერი მწყობრში მაქვს. 

მერე დავაკვირდი და ნერვიული აშლილობის ბრალია... მე ცოტა აზარტული ვარ და ვთამაშობ. როცა წაგებას განვიცდი მაშინ მეწყება ხოლმე ...

შენც დააკვირდი აბა როდის გემართება ეგ.  მეც ზუსტას ესე წყალს დავლევდი და ცოტახანშ ისევ მიშრებოდა პირი.

დიდი მადლობა გამოხმაურებისათვის. შეილება არ დაიჯერო მაგრამ თითქმის მთელი დღის განმავლობაში მიშრება პირი. არა აქვს მნიშვნელობა ვნერვიულობ თუ ისე ვარ.

Posted by: temart 8 Aug 2020, 08:10
დიაგნოზი დასვეს :

1.პროსტატა
2.რკინადეფიციტური ანემია, ვიტამინ ბე-12. დეფიციტური ანემია.
3.პილორუსის არხის რეციდიული წყლული. K25 პილორუსის და 12 გ.ნ. ბოლქვის პოსტულცეროზული
ნაწიბუროვანი დეფორმაცია K31 ზედაპირული რეფლუქს-ეზოფაგიტი.K21.0

სულ 8 წამლის მიღება მიწევს.აქედან ყოველ მესამე დღეს ინექცია და ყოველ მეორე დღეს დენოლი.

ასე გავანაწილე 1 დღეზე:
საუზმემდე ერთი საათით ადრე- ულსეპანი.
საუზმის შემდეგ -ომნიკი,ფოლიუმის მჟავა,ფერსინოლ ზ.
12 საათზე -ბ12 ინექცია(ყოველ მესამე დღეს)
სადილამდე 1 საათით ადრე- ლაციდოფილი.
სადილამდე 30 წუთით ადრე - სიმალგელი.
სადილის შემდეგ-ფოლიუმის მჟავა,ფერსინოლ ზ,დენოლი(7 მიღება დღე გამოშვებით)
ვახშმის წინ 1 საათით ადრე-ლაციდოფილი.
30 წუთით ადრე-სიმალგელი.
ვახშმის შემდეგ-ფოლიუმის მჟავა.
----------
ერთობლივი მიღება დასაშვებია?
ნორმალურია ეს?
შემცირება არანაერად არ შეიძლება?

Posted by: IONx64 15 Oct 2020, 16:16
მოკლედ დაახლოებით სამი კვირაა რა მახველებს სველად. ახლა იმდენად არა როგორც წინათ. ცივი წყლით დავიბანე ბიჭმა თავი და მივიღე პასუხიც.

ამ ხნის განმავლობაში 2 ჯერ დავიდგი რინგერი თავისი დექსათი ვიტამინ ცეთ და ბ თი. მივიღე ვიტაგამა დ3 ასევე ტეტესეპტის ჩაი პლიუს ტერა ფლუ. ანტიბიოტიკი დურამოქსი თუ რა ჰქვია მაგას. დღეს ვიყიდე აზიმაკიც.

ჩაის ლიმონით და თაფლით დილა საღამოს ვსვამ.

ტემპერატურა არ მაქვს.

ქოვიდ 19 ზე 3ჯერ გატესტილ და ნეგატიური.

ექიმთან მისვლა რენტგენზე დგომა და ა.შ. არ მინდა.

რას მირჩევთ კიდევ.

ამდენი რამე მივიღე და მაინც ვერ გამოვედი ამის...

ახლა მეშინია ანთება ხო არ ავიკიდო მეთქი.

Posted by: NEVER... 16 Oct 2020, 10:28
QUOTE (IONx64 @ 15 Oct 2020, 16:16 )
მოკლედ დაახლოებით სამი კვირაა რა მახველებს სველად. ახლა იმდენად არა როგორც წინათ. ცივი წყლით დავიბანე ბიჭმა თავი და მივიღე პასუხიც.

ამ ხნის განმავლობაში 2 ჯერ დავიდგი რინგერი თავისი დექსათი ვიტამინ ცეთ და ბ თი. მივიღე ვიტაგამა დ3 ასევე ტეტესეპტის ჩაი პლიუს ტერა ფლუ. ანტიბიოტიკი დურამოქსი თუ რა ჰქვია მაგას. დღეს ვიყიდე აზიმაკიც.

ჩაის ლიმონით და თაფლით დილა საღამოს ვსვამ.

ტემპერატურა არ მაქვს.

ქოვიდ 19 ზე 3ჯერ  გატესტილ და ნეგატიური.

ექიმთან მისვლა რენტგენზე დგომა და ა.შ. არ მინდა.

რას მირჩევთ კიდევ.

ამდენი რამე მივიღე და მაინც ვერ გამოვედი ამის...

ახლა მეშინია ანთება ხო არ ავიკიდო მეთქი.

ასე ანტიბიოტიკების შეცვლა რომ არ შეიძლება იცით?დაჟე ერთი და იგივე ჯგუფის ანტიბიოტიკმაც შეიძლება რეაქცია მოგცეთ...პლევმონია არ იყოს,არ შეიძლება ჯანმრთელობასთან თამაში.დექსამეტაზონი არის ჰორმონი,რომლის ხშირი მიღებაც არ არის რეკომენდირებული!მე როგორც ვიცი რინგერში ვიტამინების შეშვება არ შეიძლება,მხოლოდ ნატრიქლორიდში.

Posted by: temart 19 Oct 2020, 13:24
კორონას დროს სიცხე 38 გრადუსის დაწევა პარაცეტამოლით შესაძლებელია?


Posted by: goniashvili 7 Nov 2020, 08:45
ძალიან გთხოვთ ვინმემ დაწერეთ სრულად რა სიმპტომები აქვს კორონავირუსს, როგორ უნდა მივხვდეთ რო გვაქ ვირუსი და რა უნდა გავაკეთოთ ვის მივმართოთ?

Posted by: temart 7 Nov 2020, 20:35


დაბალ სიცხეზე და თავის ტკივილზე რას ურჩევდით ადამიანს?



* * *
რა გჭირთ ფორუმს, ასეთ დროს ერთი ექიმი ვერ დასვით ამ თემაზე?

ხომ იცით როგორ სჭირდებით ხალხს.


Posted by: V.I.P. 23 Nov 2020, 13:14
temart
პოლიტიკაა დღეს პოპულარული,ვიღას აინტერესებს ჯანმრთელობა?
QUOTE
დაბალ სიცხეზე და თავის ტკივილზე რას ურჩევდით ადამიანს?

დაბალ სიცხეზე მედიკამენტი არ არის რეკომენდირებული,რადგან ორგანიზმი კარგავს იმუნიტეტს და მერე უჭირს მაღალ სიცხესთან გამკლავება...
38 ზევით უნდა იყოს ტემპერატურა,რომ მიიღოთ
სიცხის დამწევი,კორონას დროს ცოტა საფრთხილოა და პარაცეტამოლს ურჩევენ ექიმები...
თავის ტკივილის დროს მე დექსალგინის გარდა არაფერი მშველის,ინდივიდუალურია ალბათ ეგეც...
პ.ს.სად სცალიათ ექიმებს აქ საპოსტად?ცუდი სიტუაციაა ძალიან კლინიკებში,უვიცები კი ამაზე იცინიან და ღადაობენ!

Posted by: lazyadmin 24 Nov 2020, 17:27
37.2 მაქ მუდმიუვად უკვე 2 თვეა. კორონას გამო დავიწყე ტემპერატურის გაზომვა შემოდგომიდან, მანამდე ტემპერატურას არ ვიზომავდი და არ ვიიც ეს მასე უნდა იყოს თუ არა. ყველა თერმოსკანერს გავდივარ biggrin.gif ვკვდები თუ სუპერმენი ვარ რამე?
არაფერი არ მაწუხებს რეალურად არც სიცხეს ვგრძნობ, ანუ კორონას ამბები რომ არა არც გავიზომავდი რეალურად

Posted by: Obey 29 Nov 2020, 19:04
გამარჯობათ
ესეთი საოცარი რაღაცა დამემართე
სიგარეტს გავანებე თავი მესამე კვირა არ ვეწევი და უკვე 4-5 დღეა ფეხების კუნთები ამტკივდა ანუ არაფერი არ შემიცვლია ჩემ ცხოვრების სტილში სიგარეტის განებების გარდა როგორც ინტერნეტში წერენ იცის ხოლმეო ფეხების ტკივილიო მარა ზოგი ცოტა უარეს რაღაცეებსაც წერს და ვინმეს გქონიათ ანალოგიური პრობლემა?

Posted by: Tbilisilovesyou 20 Dec 2020, 01:36
lazyadmin
შესაძლებელია ნერვიული იყოს,გლანდები შეიმოწმეთ მაინც თქვენით
* * *
გამარჯობათ
კოვიდის გადატანიდან 28-ე დღეს სისხლის საერთო ასე გამოიყურება.გთხოვთ მითხრათ თუა მშვიდობა?
მადლობა დიდიuser posted image
* * *
უპს,მგონი მცდარი ფაილი ავტვირთე მცდარ გვერდზე

Posted by: ნაკო 24 Jan 2021, 20:58
დაახლოებითი ინფორმაცია, თუ შეგიძლლათ მომაწოდოთ ამ ორ ინდივიადუალუ შემთხვევაზე.
1.ტკბილის მოთხოვნილება. თუ არ მიიღებს მოთხოვნილებისას ტკბილს, შესაძლია გული წაუვიდეს. (მსუბუქი წონის, არასდროს ყოფილა ტკბილზე დამოკიდებული) (დაბალი ჰემოგლობინი, ქრომის ნაკლებობა თუ სხვა)?

2. ტკბილის ან მწარის მიღების შემთხვევაში თავბრუს ხვევა და წნევის აწევა.

Posted by: temart 28 Apr 2021, 13:57
V.I.P.

მადლობა გამოხმაურებისთვის,გაიხარე



Posted by: Pepper_Crowley 15 May 2021, 15:08
ნეკნებს შორის, ნეკნების შეერთების ადგილზე, ქვემოთ, მუცელი სანამ დაიწყება, რომელი ორგანოა განთავსებული?

Posted by: temart 19 May 2021, 16:56
მიშველეთ,
გულმკერდის ყველაზე უსაფრთხო და ზუსტი მაგნიტურ რეზონანსური ტომოგრაფია იცით სად კეთდება?



Posted by: jarjosini 13 Jun 2021, 21:52
გამარჯობათ. ასთმიანისთვის ეფექტური საშუალება ხომ იცით ამოსახველებელი? ნახველი ვერ ამოაქვს

Posted by: temart 15 Jun 2021, 11:46
ნაღვლის ბუშტის სადინრის კუჭის სიმსივნით (ჯერ არ ვიცი ავთვისებიანია თუ...) ბლოკირების გამო ბილირუბინი მაქვს 6.5.
ქირურგებმა ნაღვლის სითხის სადინრით გარეთ გამოტანა მირჩიეს.
14 თვეში 18 კილო დავიკელი.
თერაპიული საშუალებები არ არსებობს?

თუ ვიყიდო ლობიო სანამ კიდევ არ გაძვირებულა ბოშო.

smile.gif

Posted by: Lacrimosa_ 13 Aug 2021, 11:51
მეგობრებო,
ღამით გამეღვიძა და დრო დამჭირდა, რომ გავრკვეულიყავი, სად ვარ. დასასვენებლად, სოფელში თუ სად. არადა სახლში ვიყავი.
მეხსიერების პრობლემები მაქვს ბოლო რამოდენიმე წელია, მაგრამ გადაღლას ვუკავშირებდი, რთული გონებრივი სამუშაო მაქვს და თანაც ძალიან დატვირთული რეჟიმი.
მეხსიერების პრობლემები როგორც კი შევამჩნიე, ნევროლოგთან ვიყავი, მერე თოდუაში გადავიღე, მაგრამ საინტერესო არაფერი აღმოჩნდა.
ისევ ნევროლოგს მივაკითხო?
* * *
V.I.P.
იქნებ გადახედოთ და დამაკვალიანოთ

Posted by: angelochka_ 8 Sep 2021, 12:34
გამარჯობა!
ლიმფოციტები მაქვს ძალიან დაბალი 3( ნორმა 16-49 უწერია.
აბსოლუტური მოცულობა 0,41 (ნორმა 0,72-3,6)
ნეიტროფილები და ლეიკოციტები მაღალია ამის ფონზე .
ვნერვიულობ ძალიან( ექიმთან ჩავეწერე და სანამ მივალ არ მიძინია ნერვიულობით

Posted by: Hakuhosho 9 Sep 2021, 04:03
გამარჯობა,წლებია მაწუხებს უჰაერობა,სატურაცია მაქვს 95 ანუ ნორმალური,მტკიოდა ზურგი და 3 კვირი წინ მივაკითხე თბილისში ტუბის ცენტრს,გადამიღეს ფილტვებზე და მითხრეს რო მარცხენა ფილტვი "რაღაცნაირად" ნაკლებად იღებს ჰაერს ვიდრე მარჯვენა,თან ზუსტად არ ვიცით რატო,ალბათ გაგიცივდაო.
ახლობელმა გამიშვა წერტილოვან გამოკვლევაზე,ზუსტად არ ვიცი რა ქვია,თითზე აპარატით წერტილოვნად შემამოწმა და რამე დარღვევა თუა გავიგებთო,დიდად არ მჯეროდა მარა რაღაცები მითხრა ისეთი რაც მაწუხებდა,ქრონიკული ალერგია რინიტი და ა.შ.თან მოაყოლა რომ მარცხენა ფილტვზე მაქვს ქრონიკული ბრონქიტი და წვრილი ბრონქები რომ უნდა იტვირთებოდეს 50ით მუშაობს 8%ზე და აქედანაა გამოწვეული ეს უჰაერობა და გაგუდვის შეგრძნება.როგორც წავიკითხე ბრონქიტმა იცის ხველა და გამკვირდა რომ საერთოდ არ მაქვს რატომღაც.
იქნებ მირჩიოთ სად ვისთან მივიდე,თანხაზე არმაქ პრობლემა ამ მარჩიელებთან 150 ლარი მარტო ვიზიტი დამიჯდა და რაღაცნაირი ფილტვის მუშაობის მეტი ვერაფერი მითხრეს.ან თუ გქონიათ მსგავსი შემთხვევა ვინმეს?

Posted by: kriketi-1985 24 Mar 2022, 00:45
სალამი.
წნევას იწვევს სიცივე თჯ პირიქით?

Posted by: TikitaLomos 25 Mar 2022, 19:38
ასეთი შეკითხვა მაავს და თუ შეიძლება, სდრიოზულად რომ მიპასუხოთ:
ნახევარ ლიტრიან ბოთლში მქონდა გახსნილი 250 გრამი მარილი. ოჯახის წევრი არ დააკვირდა და მოიყუდა. საშიშია?

Posted by: MAKSIMUSA 7 Apr 2022, 17:41
მოგესალმებით,ძაან მაწუხებს სახსრების ტკივილი და მერე დღის მეორე ნახევარში საერთო დაღლილობა. დღეს გავიკეთე დ ვიტამინზე ანალიზი და აი პასუხიც და კომპეტენტური ვინც არის ამ საკითხში იქნებ ამიხსნათ შედეგი როგორია? https://ibb.co/7QdjXpG

Posted by: goniashvili 27 Jul 2022, 10:51
27 წლის კაცმა რომელმაც 16 წლის ასაკში გაიკეთა გლანდების ოპერაცია იმ დღიდან დაეწყო საშინელი პირის სიმშრალე და გემოს შეგრძნება თითქმის მთლიანად დაიკარგა (მაგალითად ტკბილს რომ ჭამს ოდნავ გრძნობს გემოს). გთხოვთ გვითხრათ რომელ ექიმთან მივიდეს?

Posted by: Adza 2 Aug 2022, 12:25
გამარჯობა,
შემთხვევით ვინმემ თეა ნამიჭეიშვილის ნომერი ხომ არ იცით?
ოჯახის ექიმია თი ელ მედიქალში (ტატიშვილი)

Posted by: kriketi-1985 13 Feb 2023, 11:24
არის კითხვები, არაა პასუხები:დ
მაინც დავსვამ. მარცხენა მხარეს თუ ვიძინებ, მაღვიძებს რაღაც შუილი და თავის ტკივილი, ისეთი გრძნობა მაქვს, თითქოს ტვინს სისხლი არ მიეწოდება.

Posted by: The DoctoR 17 Feb 2024, 04:19
4 დღეა მაქვს ხველა, სურდო, მაცემინებს, სიცხე ზოგჯერ 37,2-37,4-ის ფარგლებში.
გაცივებული რომ ვარ გასაგებია, მარა ცოტა ექსტრემალურ კლიმატურ პირობებში მოვხვდი, სადაც ქარი და სიცივე იყო და ცოტა ვდარდობ ფილტვების ანთება არ მქონდეს თქო.
სატურაცია სმარტ საათით 98 მაქვს. ღამით ვოფლიანობ...
არ ვარ საქართველოში და ამ ტყეში ექიმსაც იოლად ვერ ვიპოვი ან რენტგენს და იქნებ მითხრათ რამე პროფილაქტიკური მედიკამენტი (ანტიბიოტიკი), თუ ანთებაა, 1 კვირა რომ "გამაქაჩინოს" კიდევ საქართველომდე

Posted by: adam* 17 Feb 2024, 10:18
The DoctoR
QUOTE
იქნებ მითხრათ რამე პროფილაქტიკური მედიკამენტი (ანტიბიოტიკი), თუ ანთებაა, 1 კვირა რომ "გამაქაჩინოს" კიდევ საქართველომდე


ექიმი არ ვარ და ბევრიც არაფერი ვიცი, რაც ვიცი, როზმარინი, იგივე ბექონდარა ან გარეული ქონდარი. (ხმელი უნდა იყოს აფთიაკში, ალბათ ნედლიც ივარგებს თუ სადმე დაკრეფ) ჩაყრი კუშკაში, ძირს დაუფარავ, დაასხავ ადუღებულ წყალს, დააცდი ოცი წუთი, დალევ ნახევარს (ერთ ჭიქას) 12 საათში მეორე ნახევარს. ერთი კვირა.
სულარაობას მგონი აჯობებს

Powered by Invision Power Board (http://www.invisionboard.com)
© Invision Power Services (http://www.invisionpower.com)